Вы находитесь на странице: 1из 417

10 Years Collection of ANZCA Part II Anaesthetic Viva

MK Yuen Albert SW Ku

10 Years viva collection 2009

Content
Preface 2010 ............................................................................................................................. 1 Preface ...................................................................................................................................... 2 ANZCA Anaesthetic Viva April 2000 Day 1................................................................................ 3 ANZCA Anaesthetic Viva April 2000 Day 2................................................................................ 9 ANZCA Anaesthetic Viva August 2000 Day 1 ......................................................................... 15 ANZCA Anaesthetic Viva August 2000 Day 2 ......................................................................... 17 ANZCA Anaesthetic Viva May 2001 Day 1 .............................................................................. 23 ANZCA Anaesthetic Viva May 2001 Day 2 .............................................................................. 28 ANZCA Anaesthetic Viva September 2001 Day 1 ................................................................... 42 ANZCA Anaesthetic Viva September 2001 Day 2 ................................................................... 51 ANZCA Anaesthetic Viva May 2002 Day 1 .............................................................................. 65 ANZCA Anaesthetic Viva May 2002 Day 2 .............................................................................. 83 ANZCA Anaesthetic Viva September 2002 Day 1 ................................................................... 99 ANZCA Anaesthetic Viva September 2002 Day 2 ................................................................. 102 ANZCA Anaesthetic Viva May 2003 Day 1 ............................................................................ 109 ANZCA Anaesthetic Viva May 2003 Day 2 ............................................................................ 133 ANZCA Anaesthetic Viva September 2003 Day 1 ................................................................. 143 ANZCA Anaesthetic Viva September 2003 Day 2 ................................................................. 147 ANZCA Anaesthetic Viva May 2004 Day 1 ............................................................................ 153 ANZCA Anaesthetic Viva May 2004 Day 2 ............................................................................ 156 ANZCA Anaesthetic Viva September 2004 Day 1 ................................................................. 166 ANZCA Anaesthetic Viva September 2004 Day 2 ................................................................. 175 ANZCA Anaesthetic Viva May 2005 Day 1 ............................................................................ 184 ANZCA Anaesthetic Viva May 2005 Day 2 ............................................................................ 204 ANZCA Anaesthetic Viva September 2005 Day 1 ................................................................. 217 ANZCA Anaesthetic Viva September 2005 Day 2 ................................................................. 223 ANZCA Anaesthetic Viva May 2006 Day 1 ............................................................................ 226 ANZCA Anaesthetic Viva May 2006 Day 2 ............................................................................ 231 ANZCA Anaesthetic Viva September 2006 Day 2 ................................................................. 240 ANZCA Anaesthetic Viva September 2006 Day 2 ................................................................. 254 ANZCA Anaesthetic Viva May 2007 Day 1 ............................................................................ 262 ANZCA Anaesthetic Viva May 2007 Day 2 ............................................................................ 270 ANZCA Anaesthetic Viva September 2007 Day 1 ................................................................. 289 ANZCA Anaesthetic Viva September 2007 Day 2 ................................................................. 294 ANZCA Anaesthetic Viva May 2008 Day 1 ............................................................................ 315

10 Years Collection of ANZCA Part II Anaesthetic Viva

MK Yuen & SW Ku

10 Years viva collection 2009

ii

ANZCA Anaesthetic Viva May 2008 Day 2 ............................................................................ 319 ANZCA Anaesthetic Viva October 2008 Day 1 ...................................................................... 326 ANZCA Anaesthetic Viva October 2008 Day 2 ...................................................................... 330 ANZCA Anaesthetic Viva May 2009 Day 1 ............................................................................ 335 ANZCA Anaesthetic Viva May 2009 Day 2 ............................................................................ 344 ANZCA Anaesthetic Viva September 2009 Day 1 ................................................................. 348 ANZCA Anaesthetic Viva September 2009 Day 2 ................................................................. 364 Appendix 1 ............................................................................................................................. 367 Appendix 2 ............................................................................................................................. 369

10 Years Collection of ANZCA Part II Anaesthetic Viva

MK Yuen & SW Ku

10 Years viva collection 2009

Preface 2010
This 10 years viva examination collection used to collect more than 10 years of examination. It is widely circulated within candidates who are going to sit for the ANZCA examination. I have never turned down any candidate who asked for the latest edition. As I believed that they will send me back their viva scenario questions and answers and keep their promise. However, the response come back from those successful candidates are unbelievable disappointing. Therefore, I prepared this version and hope that by circulating within Australia and New Zealand, I can collect more viva scenario from those successful candidates. This version is different from the old version in several ways: some wrong typing was corrected, some graph from original collection was scanned and added into this version, some scanty scenario and questions in medical viva was added as appendix. And unlike some of the older versions already circulating around, it collected the lastest 10 years scenario only. For those who collected the past paper viva scenario that is not available in this collection, it would be my pleasure if you would send it to me so as to make this collection complete. I will be much happy if those successful candidates can send me their viva scenario questions especially when they found this collection help them in passing examination. To, me, the most rewarding moment is receiving a email from a successful candidates who told me this viva collection helped them and a new scenarios were attached. This is more truth when the email is come from someone never asked me for the latest collection. One scenario may change the life of the other. This collection can be updated and more candidates can be benefited. Wish the luck is always with you during the viva examination. MK Yuen 25/3/2010 in Singapore

10 Years Collection of ANZCA Part II Anaesthetic Viva

MK Yuen & SW Ku

10 Years viva collection 2009

Preface
The ANZCA part II anaesthetic viva was the most difficult medical examination I have ever had. I believed it was because seldom anyone can tell you what exactly will be asked and expected for the examination. After passing the examination, I have, as what the other candidates passed their examination have, looking for a way to tell the after coming candidates the way to pass the examination. However, I believed that no one is willing to listen to the advice from someone who need to sit for the examination again and again. Finally, I decided to reorganize my fragile, incomplete anaesthetic viva past paper collected from various source. At the first two times I failed the viva examination, I have tried my every effort to ask my friends who have passed the examination to type down their scenario, the question being asked and the answers they provided. I described that experience as the pain in the ass to the extent even neurolytic block is not going to help. Therefore, I would like to use this collection to pay tribute to those candidates (included myself) who are willing to type down their viva scenario and pass it to other. I have retype all the viva paper I have collected. In order to make it more useful to other, I have to make some edition. I have retyped all the scenario as what have been recorded in the examination reports. However, I have dropped some of the graphs or pictures in the copy I have, due to the fact that I am not families to computer graphic. I also included scenarios that I dont have the questions. So, the user need not to keep too many viva copy while study for the examination. I have some advice for the candidates going to use this collection. The questions and answers provided by the candidates may not be the exact wording they use during their examination. Moreover, I can tell you some of the answers provided are definitely wrong. So, focusing on the questions and the interaction may be more helpful, especially the viva from Dr Anne Leung and Dr Carmen Ng were really useful. For the candidates who want to have a computer copy of this collection, you can send your request to me (a8702563@graduate.hku.hk). After you have passed the examination, it would be my pleasure if you can send me you viva scenario, questions and answers. Even one scenario may help the other. This collection can be updated and more candidates can be benefited. Wish the luck is always with you during the viva examination. MK Yuen

10 Years Collection of ANZCA Part II Anaesthetic Viva

MK Yuen & SW Ku

10 Years viva collection 2009

ANZCA Anaesthetic Viva April 2000 Day 1 Dr Helen Crilly, Gabe File
Scenario 1
A 7 year old falls from a bicycle at 7.00 pm and sustains a compound fracture of the skull near the hairline anteriorly with a 3cm stone embedded in the wound. After transportation to your hospital by air, she requires surgery at 5.00 am to repair the wound. Her history is remarkable only for asthma, for which she takes occasional medication. She appears tired, wanting to sleep, but is co-operative. The CT scan shows about half the stone lodged in the fracture and pushing against the dura. y y y y y y y y y y Pre op assessment of Asthma How would you anaesthetise this patient? Why would you not use STP? Why would you not use sux? Post extubation noisy breathing in recovery, nurse calls you. What would you do? Differential diagnosis? How to manage? You noticed a tooth is missing, How would you manage? ENT surgeon on his way, transfer the child to OR, she goes blue, what would you do? Would you use Heimlech Manoeuvre?

Scenario 2
An 85 year old woman is added to the emergency list for a laparotomy for suspected perforated diverticulum and peritonitis. She is taking unknown medication for "heart problems". On examination she is in considerable pain and is peripherally shut down. Her heart rate is 90 per minute, in atrial fibrillation with a blood pressure 105/80 mmHg and she has a 2.6 ejection systolic murmur over her percordium radiating to both her carotids. There are no beds available in your intensive care unit. y y How would you resuscitate? No ICU beds available, would you transfer to another hospital?

10 Years Collection of ANZCA Part II Anaesthetic Viva

MK Yuen & SW Ku

10 Years viva collection 2009

y y y y y y y y y

She says to you she has heart problems How would you obtain consent in a mentally incompetent elderly? What are the CVS changes in elderly? She is in rapid AF preop (shown ECG), digitalised, slows down How do you anaesthetise her? What monitoring? She goes to rapid AF, hypotensive in recovery Cardiovert? - Why it may not work? Tell me about risks of thromboemboli with AF Causes of unconscious patient postop?

Scenario 3
You are called to assist the on-call registrar in intubating a patient with a neck haematoma following a cervical fusion earlier in the day. The patient is a 66 year old male, Type II Diabetic on Metformin and diet, with cervical myelopathy (Right C5,6) and has had an anterior cervical fusion with iliac bone graft. He was noted as Grade II on earlier intubation, however, after preoxygenation, Thiopentone and Rocuronium, the registrar is now unable to intubate. y y y y y y y y y y y y y y y y How would you proceed? You cannot ventilate what would you do next? Can ventilate with oropharyngeal airway What are you going to do now? Bougie keeps slipping back Saturation dropping, what do you do now? Cannot ventilate, cannot intubate drill Describe how do you do cricothyroidotomy? What partial pressure oxygen do we get brain injury? PO2 50 corresponds to what saturation? You have been successful in orally intubating this Patient. Patient is sent to ICU. Cuff leak in ICU, what do you do? How do you change the tube? Shown a blood gas with high anion gap metabolic acidosis, with high glucose, what are your differential diagnoses? What is type II diabetes? How does metformin work? What does increase BSL, decrease BSL mean?

10 Years Collection of ANZCA Part II Anaesthetic Viva

MK Yuen & SW Ku

10 Years viva collection 2009

Scenario 4
A 24 year old 80 kg male motor accident victim has been admitted to the intensive care unit following surgery for abdominal trauma. His condition is stable, but due to closed chest injuries he remains intubated and ventilated. You are asked to review him in 36 hours postoperatively because he has developed tachycardia and a temperature of 39.0 degrees. y y y y y y y y y y What are the most likely causes and what is its significance? What is SIRS? How do you manage this? What antibiotics would you use and why? Low CVP, tachycardia, decrease urine output, management. When would you start inotropes? Which inotrope? And Why? Do you know any experimental therapy for SIRS? Antibiotics? Cultures? Change of CVLs, What type of CVL? What type of dressing? Ways to decrease sepsis in ICU.

Scenario 5
You are the consultant on call. The registrar phones you on Saturday at 3.00 pm to say that the recipient for the renal transplant has arrived. The operation is scheduled to start at 5.00 pm. y y y y y y y y y What information would you like from the registrar? Any common coexisting problems? Last dialysed 2 hours ago, how would you optimise the patient prior to surgery? Would you transfuse Hb 6.5? What is optimal Hb? How would you give anaesthetics for renal transplant? What monitoring would you use? How to optimise prior to graft? Urine out put is nil after grafting, what would you do? Would you use inotropes? How would you manage this patient post op?

10 Years Collection of ANZCA Part II Anaesthetic Viva

MK Yuen & SW Ku

10 Years viva collection 2009

Scenario 6
You are the anaesthetist on a retrieval term sent to a remote farm where a building accident has occurred 8 hours ago. A 40 year old labourer is trapped under a collapsed wall. He is prone and is pinned by masonry lying on his legs. You are unable to access his head. On your arrival the initial observations are mild confusion, PR 120, BP 100/80 and a respiratory rate of 32/min. y y y y y y y y y (Given signs of class III shock) What class of shock is this? How would you assess this patient? What would your management be? Some one comes with machinery to remove the block, would you let them to do this? Surgeon wants to amputate the leg would you let him? Hyperkalaemia? What would your role as an anaesthetist be for this amputation on the field? How would you provide anaesthesia/analgesia? (no access to the patients airway, but able to communicate and patient is maintaining his own airway) Ketamine? Sciatic nerve block? Infiltration with lignocaine? Would sciatic nerve block be enough? What approach in prone patient? (Raj approach), Cannot do femoral nerve block y y y y Patient is transferred to hospital after successful amputation, two days later comes to OR for amputation of the other leg, how would you give anaesthesia for this? O2 sats decrease to 91%, How to differentiate fat embolism from thromboembolism? Would you anticoagulate with fat embolism? You see this patient two months later with painful leg. CRPII/Phantom limb pain what can you offer this patient?

Scenario 7
You are asked to assess a 60 year old man for a possible laparotomy for a small bowel obstruction. He also has symptoms of double vision, dysphagia, dysarthria and generalised muscle weakness. He has a 30 pack year history of smoking but has not smoked for 2 years. y y y y y What are your differential diagnoses? What drugs help? What are important anaesthetic considerations in this patient? How would you differentiate Myasthenia Gravis from Myasthenic Syndrome? Do EMG, muscle biopsy, how does the neuromuscular junction look like under the

10 Years Collection of ANZCA Part II Anaesthetic Viva

MK Yuen & SW Ku

10 Years viva collection 2009

microscope? y y y y y y y y What other information would you require prior to the surgery? What is the response to sux in Myasthenia Gravis and Eaton Lambert Syndrome? What is phase II block? What type of muscle relaxant would you use? Would you use Mivacurium? What monitoring would you use? What are the Leventhal criteria? What are the features of Cholinergic and Myasthenic Crises? And how would you differentiate between the two? How do you manage post operatively?

Scenario 8
A 45 year old patient presents for endometrial resection of a submucous fibroid. She has had a heavy menstrual blood loss for several months and complains of increasing tiredness, exertional dyspnoea, cough and palpitations. She has a past history of rheumatic fever and has been told she has a mitral heart murmur. Her only treatment is iron replacement. y y y y y y y y y Blood loss Anaemia Mitral stenosis, peripheral signs, Rheumatic fever Hypo-osmolar Hyponatraemia ECG changes with LAH CXR changes with LAH AF synchronised cardioversion

10 Years Collection of ANZCA Part II Anaesthetic Viva

MK Yuen & SW Ku

10 Years viva collection 2009

Scenario 9
A healthy 25 year old multiparous woman had an uncomplicated vaginal delivery three hours ago, but since then has had vaginal bleeding for which the surgeon now plans to perform an examination under anaesthesia and uterine curettage. The student nurse reports the patient has lost about 500ml of blood since delivery. y y y y y y y How would you manage? How would you anaesthetise this patient Patient refuses spinal, and you give GA, Thio, Sux, RSI. Hypotensive, tachycardia after intubation, what would you do? What are likely causes? Patient had anaphylaxis, how would you resuscitate? Patient resuscitated successfully, extubated, post op had more bleeding, now surgeon want to do hysterectomy, How would you anaesthetise this patient now?

10 Years Collection of ANZCA Part II Anaesthetic Viva

MK Yuen & SW Ku

10 Years viva collection 2009

ANZCA Anaesthetic Viva April 2000 Day 2 Dr Helen Crilly, Gabe File
Scenario 1
A 30 year old, 130 kg primaparous woman with a breech presentation presents at term for elective caesarean section. Otherwise the pregnancy has been uneventful. She requests regional anaesthesia for the procedure. y y y y y y y y y y y y y y What difficulties do you anticipate? What equipment is needed? What level would you do your block? What if you cannot get a lumbar block, would you consider doing a thoracic block and why? What if there is a good space at T12-L11 level, would you prodeed with the lock? Will you do a spinal or an epidural? What will you put in the catheter? How much fentanyl would you use? What problems do you anticipate if you do your block at that level? What level do you required for your LSCS? Do you block the sacral level? Shortly after the block, there is hypotension and tachycardia, how would you manage? What do you think is going on? What vasoconstrictor would you use?

Scenario 2
A 60 year old woman with a long history of rheumatoid arthritis presents with increasing headache two weeks after a fall and minor head injury. A CT scan shows that she has a subdural collection with requires surgery. y y y What is of concern to you preoperatively How do you assess her rheumatoid arthritis What x-rays do you want

10 Years Collection of ANZCA Part II Anaesthetic Viva

MK Yuen & SW Ku

10 Years viva collection 2009

10

y y y y y y y

This a cervical x-rays of the patient, what do you think? What other view of the cervical x-ray do you want? How do you know that she has an unstable cervical spine? How does the fact that the patient has rheumatoid arthritis affect your anaesthetic How do yo anaesthetize tha patient for this procedure Why is coughing and barking bad for this patient? How do yo avoid that? Intraoperatively, towards the end of surgery, surgeron is concerned about the brain being a bit collapse?

Scenario 3
A 75 year old 85kg female presents for revision total hip replacement. Her relevant past history includes osteoarthritis, high serum cholesterol, a single episode of classical angina two years ago and an uneventful general anaesthetic one year ago for carpal tunnel release. General anaesthesia for primary hip replacement was uneventful. Her activities are very limited by hip and knee pain and she has experienced significant weight gain. Her only medications are paracetamol and cholestyramine. y y y y y y y y y y y y y y y y What precaution will you take to deal with the bleeding issues? How much blood loss do you anticipate? Do you think predonation is suitable in this lady? What level of hematocrit do you aim for? How much blood can you get? Do you think normovolumic hemodilution suitable? What about cell salvage? You preceed the surgery with general anesthesia and intraoperatively BP 75 HR 120/min, when surgeon is putting in the fermoral prostjesis what will yo do? What do you think is the most likely cause? Following GA, nursein the recovery room said that the patient is slow to wake up how do you deal with that? What could be likely cause if all the investigation are normal what will you do with the patient the ICU physician told you that the patient has hypothyroidism, how will you manage her? What drugs will you give and how do you give it? How do you replace her thyroxine? Do you know any study of how to treat hypothyroidism of late?

10 Years Collection of ANZCA Part II Anaesthetic Viva

MK Yuen & SW Ku

10 Years viva collection 2009

11

Scenario 4
You are working in a large rural base hospital and are called to the emergency room where a 7 year old boy has been brought in by ambulance following a house fire. y y y y y y y y y y y y What aspect of assessment are you concern How is the assessment different from adult How do you replace the fluid? What fluid would you use? Would you use dextrose solution iht the first 24 hours? How do you know that the patient has airway problem? What if the patient presented in accident and emergency department with stridor and hoarseness of voice would you treat it with nebulized adrenaline? What are you concern if the patient require anaesthetic for debridement? What size endotracheal tube would you use? Would you transfer this child? How wyould you deal with this child in ICU? How do you manage his pain therapy? Few days later, the child needs another anaesthetic and your registrar is concerned about giving the anaeesthetic, how wuld you advise him?

Scenario 5
A 65 year old male presents for trans urethral resection of his prostate gland. He is a paraplegic following thoraco-abdominal aortic aneurysm repair. y y y y y y y y y y y y What level of paraplegia do you tnhink yhe has? Why do you think he has a prarplegia? What problems concerns you with the anaesthetic? What anaesthetic will you give? Would you do an epidural? What problems do you anticipate with epidural? How common is epidural hemoatoma? Would you consider spinal? In actual fact he has a paraplegia at T5, how does that affect you preoperative assessment? Why do you think that he is bradycardic? How do you give a general anesthetic? Intraoperatively, there is hypotension, how do yo manage?

10 Years Collection of ANZCA Part II Anaesthetic Viva

MK Yuen & SW Ku

10 Years viva collection 2009

12

y y y

How do you diagnose TURP? Dsodium is 118 mmol/l, how do you manage? Why do you not want to replace sodium quickly?

Scenario 6
There is a 4 year old boy with a history of nocturnal snoring on your day surgery ENT list for adenotonsillectomy. y y y y y y y y y y what in the history affect your preoperative assessment? how do you comfirm the diagnosis of sleep anoea? why are you concern about apnoea syndrome? how do yo assess this child? how do you anaesthetise this child? do you accept him for day surgery? how do you manage the pain relief? how much paracetamol do you use? what opioid would you use? what about the use of a laryngeal mask for the precodure?

Scenario 7
You are asked to transport a 40 year old patient from the Intensive Care Ward (ICU) to the Radiology Department and to manage the patient for a head CAT (Computerised Axial Tomography) Scan. The patient is not intubated, breathing supplemental oxygen, slightly confused but conscious. There is an underwater chest drain in place. y y y y y y y How do you ensure a safe transportation to the CT suite? Would you intubate this patient? How do you check that the chest dreain is alright? Do you apply a clam on the chest drain? What kind of problems do you anticipate in the CT scan sutie? The director of radiology wants you to advise him on how to prepare for a power failure, how would you advise him? Now, the same patient has required an MRI scan of the head, and he expresses claustrophobia, how do you manage

10 Years Collection of ANZCA Part II Anaesthetic Viva

MK Yuen & SW Ku

10 Years viva collection 2009

13

y y

Would you give him any premediacation and if so with what? How do you give the anaesthetic?

Scenario 8
A 54 year old woman who is a recovered alcoholic presents for her fourth resection of recurrent benign laryngeal papillomata. She is very hoarse and develops stridor when trying to breathe quickly or deeply. y y y y y y y y y y y How do you evaluate the patient preoperatively? What investigation would you do? What is a flow volume loop of an extrathoracic obstruction? Intrathoracic obstruction How do you anaesthetize the patient? What ETT would you use for laser excision? Would you use helium? Surgeon proceeded with a tracheostomy, and patient return a few days later for another anaesthetic, after induction, there is difficulty in ventilating patient, how do you respond? How do you know it is pneumothorax? And how will you respond? In actual fact, you discover patient had received 2 litre of iv fluid and has a fluffy white out in CXR, what do you think is going on? How do you differentiate between negative pulmonary edema and fluid overload? How do you treat fluid overload?

Scenario 9
You are an anaesthetist accompanying a surgical team to procure the heart, liver and kidneys from a donor in a large provincial town. On arrival in the Unit, the donor's distraught spouse states that the right foot moved in response to rubbing the heel. You are asked to certify the presence of Brain death y y y y y y What investigation do uou do for brain death? Why do you think there is a movement in his lower limb? If the patient has a facial injury how do you monitor his brain death test? What do you want to see in the EEG? How do you do the anaesthetic? Do you give any muscle relaxant?

10 Years Collection of ANZCA Part II Anaesthetic Viva

MK Yuen & SW Ku

10 Years viva collection 2009

14

y y y y y y y y y y y

Do you give fenatnyl? What if the blood pressure is low? What inotropes do you give and how? What if there is no response to you initial inotrope, what will you do? What about dobutamine? What monitor do you use? What about PAC would you use it for monitoring? How do you monitor the CVS? How do yo use TEE to monitor the CVS? Is there any contraindication to using TEE? How is TEE compared to PAC?

10 Years Collection of ANZCA Part II Anaesthetic Viva

MK Yuen & SW Ku

10 Years viva collection 2009

15

ANZCA Anaesthetic Viva August 2000 Day 1 Scenario only


Scenario 1
A 27 year old male presents to your emergency room after being involved in a motor vehicle accident. He is fully conscious and complains of Left Upper Quadrant abdominal pain. BP is 95/60, PR 110/min, RR 16/min He has no head or neck injury. Pelvic and long bones show no abnormality.

Scenario 2
You are giving an anaesthetic in a rural hospital. Patient is a 69 year old man with urgent laparoscopic cholecystectomy and acute cholecystitis. Past history: hypertension for 10 years (usual BP 130/80), inferior myocardial infarction 11 months ago, 4 episodes of palpitations (30 minutes) in past 10 months. Last night: atrial fibrillation with angina, lasting 30 minutes, treated with digoxin. Now: surgeon is filling the abdomen with CO2 and the vital signs have changed. Prior: HR/min 88, BP 130/85, SpO2 99, EtCO2 39. Now: HR/min 92, BP 95/55, SpO2 98, EtCo2 41.

Scenario 3
A 65 year old man presents with large bowel obstruction (Ca. transverse colon) and is scheduled for a right hemicolectomy. He gives a history of an inguinal hernia repair 25 years ago at a small regional hospital. It was performed under local anaesthesia following a failed attempted intubation under general anaesthesia. He has a 40 pack year smoking history.

Scenario 4
A 68 year old man with non-insulin dependent diabetes mellitus and chronic obstructive airways disease associated with a long smoking history presents with bilateral intermittent claudication. Investigation reveals significant aorto-iliac occlusive disease and he is scheduled for an aorto-bifemoral graft.

10 Years Collection of ANZCA Part II Anaesthetic Viva

MK Yuen & SW Ku

10 Years viva collection 2009

16

Scenario 5
You visit an 81 year old woman who has been admitted with a fractured neck of femur and is booked for internal fixation. She has had no previous surgery, has well controlled essential hypertension and says her daughter has told her that there is a family history of a bleeding disorder. She has a life-long history of bruising, but is usually active and well.

Scenario 6
A 23 year old male is scheduled for a latissimus dorsi free flap to cover a fractured right tibia resulting from a motor vehicle accident seven days previously. External fixateurs were applied at that time because the wound was contaminated.

Scenario 7
A 32 year old woman presents for laparoscopic sterilisation. She has had two previous general anaesthetics, one for a dilatation and curettage and one for an arthroscopic examination of her knee. Both times she had such severe postoperative nausea and vomiting that she required overnight admission. She has no other known health problems, takes no medication, and has no allergies.

Scenario 8
You are asked to review a 46 year old 98kg stockbroker who presents 24 hours post laparoscopic cholecystectomy with severe jaundice.

Scenario 9
You are anaesthetising a 57 year old male half way through an emergency intrabdominal bowel procedure. One hour into the procedure the operating theatre goes totally black. The procedure began at 1700 hours and is expected to finish in another three to four hours. You are using a propofol and vecuronium infusion.

10 Years Collection of ANZCA Part II Anaesthetic Viva

MK Yuen & SW Ku

10 Years viva collection 2009

17

ANZCA Anaesthetic Viva August 2000 Day 2


Scenario 1
A 34 year old farmer presents for re-implantation of an arm which was avulsed above the elbow 4 hours earlier. y y y y Outline the potential benefits of a brachial plexus block. how would you block his brachial plexus? what other issues concern you relating to this mans anaesthetic and surgery? how would you anaesthetise this man?

Two hours into the case you notice smoke arising off the microscope drapes where the light from the microscope has been left on. Suddenly it ignites and flames engulf the microscope. y y Discuss you management. What varieties of fire extinguishes are you familiar with? Which type would you use?

It is now 3 am and you have been awake since 6am the previous day. You feel extremely tired and are struggling to stay awake. y What would you do ? (the other on-call anaesthetist is busy with a laparotomy and no other anaesthetist is available).

Scenario 2
A 42 year old woman for elective anterior resection gives as her only significant history, an episode of embolism and near death during an emergency caesarean delivery four years ago. y y y What are the possible types of embolism? How does an amniotic fluid embolus present? How common are they? How are they diagnosed? How is it treated? What is the outcome? With further questioning about her past embolism she tells you she was pregnant and about to undergo a caesarian section, but she collapsed while being transferred onto the operating table. How would have managed that situation? y y y Back to the current scenario. How would you now manage her with respect to DVT prophylaxis? What are your concerns regarding LMWH and epidurals? What are your guidelines? Which is more effective for thrombo-embolism prophylaxis - LMWH, epidural or both combined?

10 Years Collection of ANZCA Part II Anaesthetic Viva

MK Yuen & SW Ku

10 Years viva collection 2009

18

Scenario 3
An 82 year old lady who lives alone, was found by neighbour in bed with a one week history of nausea and vomiting and abdominal pain. Her surgeon wants to perform an emergency laparotomy for a probable perforated duodenal ulcer. Medications - Indomethacin for arthritis, Diltiazem for hypertension She is a heavy smoker. She presents lethargic, thin, tachycardic (120), hypotensive (90/60) and tachypnoeic with a distended abdomen and air under diaphragm y y y y y y y y y y What pre-operative investigations do you request? What clinical indicators do you use to predict likelyhood of a difficult intubation? Outline technique and grading system for Mallampati. What is sensitivity and specificity? What are positive and negative predictive values? Would you prefer Mallampati to be a sensitive or specific screening test? Justify. This lady has buck teeth, limited mouth opening, a Mallampati 4 and reduced neck extension. How would you anaesthetise her? She has a full stomach. Is a fibre-optic technique safe in view of this? You have trouble visualising the glottis as you pass the fibre-optic scope. What would you do to improve your view? She becomes agitated and extremely uncooperative forcing you to abandon your fibre-optic intubation. How will you preceed?

She has a past history of a difficult intubation.

Scenario 4
A 28 year old male presents with a penetrating eye injury and facial lacerations. He has been drinking and was the driver of a car that was involved in a high speed accident, in which the passenger was killed. The ophthalmologist wishes to examine the eye under anaesthesia and attempt to preserve the sight in that eye. y y y How would you anaesthetise him? What trauma views would you like? What is the sensitivity of C-spine x-rays?

He is haemodynamically stable but has a GCS of 11/15. He has had a CT of his head and abdomen (unintubated) which were normal. Trauma views:- C-spine lateral film, normal to level of C7 Chest fractured clavicle

10 Years Collection of ANZCA Part II Anaesthetic Viva

MK Yuen & SW Ku

10 Years viva collection 2009

19

Pelvis NAD y y y y y y y y y y Would you be happy to proceed to OT? How would you induce him? What are the risks of suxamethonium relating to his eye wound? What is the mechanism for this? What could you do to prevent this? What else could you do to reduce IOP? Induction and intubation proceed without incident, however 15 minute into the case you notice the airway pressures are increasing. What do you do? You eventually detect reduced air entry in the left chest associated with hyper-resonance on that side. The trachea is midline. What do you do? Where do you place your drain? Where does the internal mammary artery course?

Scenario 5
A 76 year old man presents for elective abdominal aortic aneurysm repair by open surgery. He is a smoker, weights 76kg, height 1.75m, and reports getting occasional chest pain which is strongly suggestive of angina, but takes no regular medication. You will be asked to discuss his anaesthetic management; but your first question will be about the investigations you would require in your preparation of him. y y y y y y What investigations do you require pre-operatively? What pre-med would you give? What monitoring would you use? What are the complications of a PAC? Would you place an epidural? What are the benefits? In placing the epidural you catheterise a vein. How would you manage this? The surgeon applies a supra-renal cross clamp and the BP falls, the S-T segments sag and the PA pressure rises. How would you manage this? ( the surgeon has opened the aorta and the cross clamp cannot be released) the surgeon is unable to tell you whether he is above or below the coeliac vessels. y y Change the scenario the cross clamp is applies and the blood pressure rises. What would you do? The BP rises and the S-T segments sag what would you do? How do SNP and GTN differ in their effects on myocardial perfusion?

10 Years Collection of ANZCA Part II Anaesthetic Viva

MK Yuen & SW Ku

10 Years viva collection 2009

20

Scenario 6
A 70 year old man presents to your pre-operative anaesthetic clinic for elective TURP in 6 weeks time. He has a history of ischaemic heart disease with recent blackouts. His blood pressure was 120/80 mm Hg and his pulse rate 38 beats per minute. Medications that he is currently taking are digoxin, propranolol and frusemide. y y y y y y y y y y y y y y ECG given what is the diagnosis? (sick-sinus syndrome) How would you manage this? How are pacemakers classified? What is VVI? What is DDD? You are now in the OR and patient has a pacemaker. What anaesthetic technique would you use? What monitoring would you use? What other equipment would you have available in theatre? How would you manage his pacemaker intra-operatively? The surgeon wishes to use diathermy. Where would you place the ground pad? What direction should the current be flowing? What surgical techniques could be used to avoid the use of diathermy? The patient has a symptomatic bradycardia intra-operatively. How would you manage this? Where would you place the pads for the transcutaneous pacemaker?

10 Years Collection of ANZCA Part II Anaesthetic Viva

MK Yuen & SW Ku

10 Years viva collection 2009

21

Scenario 7
A 75 year old male presents for a left cataract extraction and insertion of an intra-ocular lens. He has a history of angina (6 months duration), non-insulin dependent diabetes mellitus (5 years duration), hypertension (15 years duration) and atrial fibrillation (3 months duration). His medications are warfarin, lisinopril, glibenclamide and amiodarone. His blood sugar level is 6mmol/L, his haemoglobin is 135g/L creatinine 0.11mmol/L and his INR (international normalised ratio) is 2.7. The surgeon requests local anaesthesia with akinesis of the eye as he expects it to be a difficult cataract extraction for anatomical reasons. The patient refuses general anaesthesia. y y y y y What technique would you use? Describe you technique. How would you manage his INR? You (or surgeon) perform a retrobulbar block and the patient becomes unconscious. What are the causes? The same patient returns 6 months later with a lower abdominal mass, weight loss and progressive shortness of breath. What are the possible causes of shortness of breath and how would you investigate? y y Is the pulmonary fibrosis of amiodarone reversible? What are the other side effects of amiodarone?

Scenario 8
A 24 year old woman presents for general anaesthesia for a dilatation and curettage following a miscarriage. Her last menstrual period was 8 weeks ago. She has asthma, but is stable on twice daily ventolin and becotide inhalers. The rest of the assessment is unremarkable. She is 65kg, 167cm in height, and her peak expiratory flow rate (PEFR) is 350. y y y y y y y y You induce her and place an LMA. She regurgitates. How would you manage this? You proceed but her airway pressures rise. How would you manage this? You suspect asthma. How would you manage this? How much salbutamol would you give? What other drugs would you use? Would you extubate her at the end of the procedure? What would be your post-operative management? She returns ten days later with an offensive PV discharge and a temperature of 38.5 degrees. Would you give her a spinal or a GA?

10 Years Collection of ANZCA Part II Anaesthetic Viva

MK Yuen & SW Ku

10 Years viva collection 2009

22

y y

Would you intubate her? How would you induce her?

Scenario 9
A 40 year old alcoholic male is booked for management of active upper GI bleeding. On examination he is vague, exhibits a tremor flap and hyper-reflexia. He has tense ascites and his blood pressure is 95/40 with a sinus tachycardia of 130. y y y y y y y y y y What causes this? How can this be managed? What are the risk factors for patients with liver disease? What risk group would you anticipate this man to be in? The surgeon decides to place a Senstaken-Blackmore tube and asks you to provide sedation. Would you do this? What pre-operative tests would you require? How would you anaesthetise him? What drugs would you use? What drugs would you avoid? How would you treat his ascites? The surgeon elects to drain the ascites. How is this done?

10 Years Collection of ANZCA Part II Anaesthetic Viva

MK Yuen & SW Ku

10 Years viva collection 2009

23

ANZCA Anaesthetic Viva May 2001 Day 1 Dr David Wong, St George Hospital
Scenario 1
A 37 year old male is admitted to the Burns Unit at your hospital with 40% 3rd degree burns to his legs and lower body. You have been asked to advise and assist in the management of his pain. y y y y y y y (Pain question, not interested in acute management of burns) How would you manage his pain initially? How much morphine would you use Some days down the track you are up to 380mg of rnorphine, what else would you do? (neuropathic pain/ opioid rotation) You are now asked to give advise to nurses in OT who are to change his dressings. What is you pain management (without use of GA?) Problems with repeated/prolonged use of nitrous Would you place an epidural?

Scenario 2
An 18 month old boy, 10kgs, with Downs Syndrome (Trisomy 21) presents to your day surgery unit for a right Inguinal Hernia Repair. y y y y y y y y Anaesthetic problems with Downs How would you anaesthetize the child Premed technique Parents are divorced and both would like to attend Cant get venous access, child spasms Child aspirates what would you do? Pain relief

10 Years Collection of ANZCA Part II Anaesthetic Viva

MK Yuen & SW Ku

10 Years viva collection 2009

24

Scenario 3
You are asked to place a labour epidural in a patient who is: o o o Gravida 1 Para 0 at term 4cm dilated receiving a syntocinon infusion

You place the epidural and after a test dose of 3 ml 2% lignocaine with adrenaline 1:200 000 the midwife draws your attention to the CTG trace shown below.

what would you do? Scalp pH 7.22 what do you think? needs LSCS Unilateral block with epd Patient fits what's the DDx Develops coagulopathy

Scenario 4
A 30 year old man is brought in after a high speed motor vehicle accident. He is conscious but restless and smells of alcohol. A cervical collar is in place, he is pale, complaining of pain in his upper chest and has an obvious open fracture of his left femur. Pulse Rate 120, Blood Pressure 95/60 and has been given 4 litres of fluid. y y y y y y y y No O neg blood. X match takes 30 mins what would you give O positive What other investigations # 1st rib and widen mediastinum what's next? Wouid you do the TOE in ED or OT Aortic dissection: where is it most likely? how would you anaesthetise? What organs are at risks? Cross clamp time 30 minutes, what would you say to surgeons? Where does the clamp go

10 Years Collection of ANZCA Part II Anaesthetic Viva

MK Yuen & SW Ku

10 Years viva collection 2009

25

Scenario 5
You are asked to provide anaesthetic assistance for a 20 year old woman with cystic fibrosis who is to have endoscopic retrograde cholangiopancreatography (ERCP). y y y y y y y Anaesthetic issues Draw flow volume loops How would you anaesthetise Premeds Problems with atropine How would you do it without an anesthetic machine Whats the problems with using 100 % oxygen

Scenario 6
A 65 year old man has been admitted to the coronary care unit with angina, his sixth admission in three months. His symptoms are not satisfactorily controlled on maximal medical therapy and surgery is not an option. The cardiologist asks for your help in managing this man and is particularly interested in the role of epidural analgesia. y y y y y Would you do it Benefits? Level? Why not lumbar What would you do if patient on

Aspirin Warfarin Clopidegral Heparin LMWH y Patients presents with AMI requiring thrombolysis: has epi in situ (Other candidate was asked patient was discharged 3 weeks later and developed right thigh numbness, refused to come back to hospital. What will be the management?)

10 Years Collection of ANZCA Part II Anaesthetic Viva

MK Yuen & SW Ku

10 Years viva collection 2009

26

Scenario 7
A 60 year old sheep farmer is referred to your teaching hospital with a diagnosis of hydatid liver disease. He is booked for removal of the cyst (peri-cystectomy) hemi-hepatectomy. He is generally well and currently working. Past Medical History o o o o o o Medications: o o Lisinopril 20mgs daily Salbutamol puffer prn Ischaemic Heart Disease Uncomplicated myocardial infarction 2 years ago No angina now. Hypertension -treated for two years and well controlled Asthma - Usually stable, but reacts to pollens No admissions in last five years

y y y y

Problems with liver surgery in general How would you anaesthetise BP 70 HR 110 DDx Problems with transfusing FFP

Scenario 8
You are asked to provide imminent anaesthesia for a 48 year old male insulin dependent diabetic who is to undergo a renal transplant. The donation is cadaveric. His medications include insulin, metoprolol and lisinopril. He had his last haemodialysis twenty hours previously. y y y y y y y Issues Indications for preop dialysis What investigations Lateral ST depressions on ECG what would you do next? What monitoring Hypotensive during case DDx Post op: delayed emergence DDx

10 Years Collection of ANZCA Part II Anaesthetic Viva

MK Yuen & SW Ku

10 Years viva collection 2009

27

Scenario 9
A 78 year old 60 kg depressed woman presents acutely for her first ECT as a day patient. On pre-assessment you find that she is very withdrawn, and a poor historian. However, you have old hospital notes covering an uneventful abdominal hysterectomy 20 years ago. Physical examination is unremarkable, the BP is 160/90, HR regular. The GP referral letter states that she is on captopril and nitrolingual (glycenyltrinitrate) y y y y y y y y y Issues with ECT How would you assess this lady Would you do it in the Psych unit designed for ECT Heart rate 20 following shock HR now 140 Heart rate still 140 jaw tight Management of MH Diagnosis of MH She needs ECT 2 days later

10 Years Collection of ANZCA Part II Anaesthetic Viva

MK Yuen & SW Ku

10 Years viva collection 2009

28

ANZCA Anaesthetic Viva May 2001 Day 2 Dr MK Yuen, QMH


The candidate has failed the examination, therefore do not believe in the answers provided. It is only aiming at illustrating the interaction between the candidate and the examiner.

Scenario 1
A 30 year old 70kg lady is scheduled for elective laparoscopic splenectomy for idiopathic thrombocytopenia purpura (ITP) not responding adequately to medical management. Allergies - nil, PMH - unremarkable, Medication - prednisone 25mg daily E: How would you prepare this operation? C: I would make sure the threater and the equipment for laparoscopic surgery was available. I would like to assess the condition of ITP and whether there was any associated autoimmune disease. And patient was put on prednisolone, I would like to give steroid cover. E: Would you like to know more about ITP? C: I would like to know the recent platelet count. E: That is 30 C: I would like to arrange platelet for infusion. Because of the small half-life of platelet, I would give it while the patient was transfer to operating threater. E: 1 hour after operation started, BP dropped from 120/70 to 60/50 mmHg. C: COVERABCD approach. As patient is put on steroid, Addison crisis is one of differential diagnosis. Other differential diagnosis included hypovolaemic, cardiogenic, obstructive, distributic causes. E: Which one is most likely? C: I thought it will be Addison crisis. E: So what will be your treatment? C: Hydrocortisone 100 mg ivi E: you mentioned COVERABCD approach, what is V stand for? C: ventilation and vapourizer. E: How about if surgeron told you that there is bleeding inside? C: I would like to know whether it is oozing, arterial or venous bleeding. E: He told you it was arterial bleeding. C: I would like to start fluid resuscitation. And I will ask surgeron how long would it take to stop the bleeding. I would like to bring the SBP to around 100 mmHg instead of 120 mmHg as too

10 Years Collection of ANZCA Part II Anaesthetic Viva

MK Yuen & SW Ku

10 Years viva collection 2009

29

high BP will increase blood loss. E: Would you ask the surgeron to do laparotomy to stop bleeding? C: Yes E: How about if it was oozing everywhere? C: I would ask for platelet replacement. E: Blood bank told you that there is only 8U A+ve platelet available and the patients blood group is O-ve. C: For ABO blood group, it is safe to give blood group A platelet to group O patient. But Rh+ve platelet may induce sensitization in Rh-ve patient, esp she is at age of pregnancy. I would like to give her Rh immunoglobulin to her later. E: Why later? C: because I was busy in doing resuscition. Moreover, it take sometime to have immunoglobulin from pharmacy. E: this is not for other reason. C: it is not for other reason. I would also consider to give DDAVP. E: what is the dosage? C: 30 mcg/kg E: The patient is now in recovery room. You put up a unit of blood for transfusion. She complained loin pain later. C: I thought is may be a incompatibility blood. I would like to check the unit of blood. E: It is B+ve. C: I would stop the blood infusion and change the whole blood giving set. I will re-cross match the blood. And I would like to BELL

Scenario 2
A 54 year old man with a chronic renal failure on peritoneal dialysis is booked for insertion of an AV fistula in his left arm E: how would you assess this patient? C: I would like to know the reason for CRF and whether it is oliguric or polyuric. I would like to know whether there is any coagulation derangement, pH.and GER. E: what else do you want to know? C: whether I can use the other arm for iv and BP measurement? E: what else? C: I want to know the electrolyte and Hb.

10 Years Collection of ANZCA Part II Anaesthetic Viva

MK Yuen & SW Ku

10 Years viva collection 2009

30

E: K 5.6 C: It was high. I would like to lower it down with dextrose insulin infusion. E: what is the dosage? C: D20 200ml + 20 U actrapid over 20 minutes. E: Hb is 6. would you transfuse the patient. C: No. because of decreased erythropoietin, patient will develop anaemia and patient is well compensated for it. Blood transfusion will increase viscosity of blood. E: what is the cause of anaemia? C: besides erythropoietin deficiency, they will have iron deficiency. In the past patient on dialysis are prone to have Al toxicity which cause marrow fibrosis. E: would you give erythropoietin? C: erythropoietin may help but not to increase Hb before operation. E: what is the side effect of erythropoietin? C: there is genetic recombinant and animal extract erythropoietin. Animal extract may cause cross infection. It may cause hyperviscosity and headache. E: How would you anaesthetize this patient? C: I would like to do BPB E: patient developed convulsion after your injection. C: it maybe due to LA toxicity. I would control ABC intubation and hyperventilation. E: Why hyperventilation? C: to decrease cerebral edema after convulsion. E: what else? C: it will decrease the pH of the patient and make LA in ionized form so it would be less toxic. E: what approach would you like to do for BPB? BELL C: axi E: dont answer this question as times up.

Scenario 3
A 54 year old female with severe progressive multiple sclerosis presents to a stand alone day surgery centre for percutaneous endoscopic insertion of a gastrostomy tube (PEG). E: any question you want to ask about this question? C: I would like to know what is the indication for PEG? Is it for feeding or regurgitation. E: it is for feeding. Do you know what is PEG? C: Yes.

10 Years Collection of ANZCA Part II Anaesthetic Viva

MK Yuen & SW Ku

10 Years viva collection 2009

31

E: what would your criteria for day surgery for this operation. C: social, surgical, and medical criteria. As patient has MS. I would like to find out more about this. E: what is assessed in MS? C: I would like to know what is the involvement of MS. Whether it is eye, long tract, bulbar or urinary incontinence. What is the medication the patient is taking. Whether she able to mobile or was wheel-chair bound or bed bound. Is there a bedsore? E: there was bulbar involvement and she was wheel-chair bound. Patient complained recent increased weakness of lower limb. C: it may be a relapse. I would like to look for any precipitating factor e.g. fever, infection, stress. I would like to know whether it is worsen that previous relapse. We can check CSF for some kind of protein to see whether it is relapse. E: you are talking about diagnosis of MS. We come to it later. She said it is not as worse as before. She was taking this medication: dantrolene, amitriptyline, Tramadol, Gabapentin, and prednisolone. C: prednisolone is for and and . E: is there any drug interaction? C: amitriptyline and Tramadol E: patient was having fever 38 degree. C: I would delay the operation and look for the source of infection. I would like to exclude any pneumonia, UTI or infection of bedsore. E: It was due to UTI. She came back after UTI subsided. How would you anaesthetize her? C: I would like to give her GA with intubation as there was bulbar involvement and risk of aspiration. E: Have you ever do case for PEG? C: no, I saw other people doing it? E: how long would it take? C: About 30 40 minutes E: Patient refuse GA despite she understand the risk. C: I will give sedation and I will ask surgeron whether it can be done in lateral position. E: why lateral position? C: to decrease risk of aspiration. E: what is the normal position for PEG? C: supine. E: There is some bile stained fluid come out from mouth. C: patient may have aspirated. I would like to do suction. Control ABC listen to the chest for bronchospasm and consider intubation. E: patient is well and discharged home. She came back 3 weeks later as she slip and fell

10 Years Collection of ANZCA Part II Anaesthetic Viva

MK Yuen & SW Ku

10 Years viva collection 2009

32

causing fracture femur. She refused GA. C: I would like to do, sorry, may I ask which part of femur was fractured? Is it shaft or neck of femur? E: It is NOF. C: I will consider SA or EA. It may affect MS but it do not know in which way. The other choice is femoral + sciatic + lateral cut nerve of thigh block. E: do you heard about how did EA affect MS? C: sorry, I really dont know. I came form HK and there is no MS at all. So I really dont know. Both examiner and observer happy and laugh. BELL

Scenario 4
A lady has been retrieved from a "high-velocity" motor vehicle accident and brought into the casualty department of your hospital. You are contacted by the orthopaedic registrar who requests an anaesthetic for the debridement of a crushed right arm. E: How would you give your anesthetic? C: I would like to assess the patient for other injury. E: This is the only injury. C: besides debridement, did surgeron want to do any re-vacularization? E: No, he only want to do debridement. C: So, I want like to do a brachial plexus block. E: Why did you want to do a BPB? C: it allow the patient to have debridement. It also have post-operative analgesic effect. E: Patient is adequately fasted for 6 hours, why not GA? C: although patient was fasted for 6 hours, trauma cause delay gastric emptying. So, she should be treated as full stomach. E: you do a supraclavicular approach and patient complained of dull chest pain afterward. C: I would like to check CXR to exclude pneumothorax. E: CXR showed pneumothorax with 200 ml air. Would you insert chest drain? C: Did patient have other symptom like SOB, desaturation, tracheal deviation. E: no, only 200 ml air and chest pain. C: I would like to give high % O2 and put patient under close observation. Also discuss with patient for the insertion of chest drain. E: when would recheck CXR? C: I would like to check it 2 hours later or when patient have increased symptom.

10 Years Collection of ANZCA Part II Anaesthetic Viva

MK Yuen & SW Ku

10 Years viva collection 2009

33

E: 3 days later, surgeron want to do amputation of the upper limb. She was worried about preemptive analgesic. C: preemptive analgesic is to provide analgesic before amputation so as to decrease phantom pain. E: so you believed in preemptive analgesic? C: yes. E: what cause this phantom pain? C: it is a neuropathic pain, the pain signal was origin from the nerve and do not involved nociceptor. E: where exactly is it come from? C: it is come from dentron. (attempt to draw a picture to illustrate nociceptor and a nerve, but did not know what exactly was drew) E: any other cause? C: it may be due to a neuroma. E: is it common to have neuroma? C: no, usually the surgeron cut the nerve shorter and avoid the formation of neuroma. E: what else? C: sorry, I dont know. E: how do you provide preemptive analgesic? C: I will do an axillary nerve block and put in a catheter for LA infusion. E: would you use peripheral nerve stimulator? C: Yes, there is commenical kit for this purpose. E: What else? C: stellate ganglion block daily. E: how long would it last? C: it depends on the drug given. It may last up to 6 8 hours. E: so it is not effective. C: I may consider cervical epidural, but it is technically difficult. E: Why is it technically difficult? C: because cervical spine spinous process slope downward. E: how is the cervical spine C6,C7? C: it is more horizontal. E: so why is it more difficult? C: cervical epidural space was smaller. E: what is the size of cervical epidural space? C: cervical epidural space about 3 mm. Lumbar epidural space about 4 mm. BELL

10 Years Collection of ANZCA Part II Anaesthetic Viva

MK Yuen & SW Ku

10 Years viva collection 2009

34

Scenario 5
A 50 year old male, presents to your pre-operative anaesthetic clinic prior to his elective day case inguinal hernia repair. He requests to be awake during surgery. E: patient want to be awake during operation, what is you choice of anaesthesia? C: I will consider SA, EA or ilioinguinal + iliohypogastric + genitofemoral nerve block. But epidural cause longer motor block and is not suitable for day surgery. E: what drug would you use in spinal? C: I would give LA and analgesic. Because it was a day case, I would like to give 0.25% marcain. I would give fentanyl as well. Intrathecal morphine cause delay respiratory depression and is not good for day surgery. E: what is your other choice? C: I read from the book we can use prilocaine but it is not available in HK. I do not know about the dosage. Other choice is 2% lignocaine. But lignocaine may cause neurotoxicity. Usually it was due to the use of 5% lignocaine, but there is report of neurotoxicity of using 2% lignocaine. E: what else cause neurotoxicity? C: perservative of lignocaine. Also use of spinal catheter. E: if prilocaine and marcain was available, what drug would you use? C: I would like to use marcain as I am not familiar with prilocaine. E: patient complained of pain after the operation in recovery. C: I would like to assess whether the block has been regressed. E: I was wear off already. C: I would like to communicate with surgeron to see whether I can do infiltration of LA into the wound. E: What is the other method? C: NSAID if there is no contraindication. E: what is the contraindication for NSAID. C: allergy, bleeding disorder, asthma. E: Which drug you are going to give? C: volteran. E: patient was unable to pass urine on discharge? C: I would like to assess the fluid intake of this patient E: Patient just want to go home. C: pass urine is only a recommended criteria for discharge in day surgery. I would like to make sure the patient is able to come back and contact us if this problem persists after discharge. E: would you do anything else? C: I would like to exclude BPH, may use running water tap to see whether it may help.

10 Years Collection of ANZCA Part II Anaesthetic Viva

MK Yuen & SW Ku

10 Years viva collection 2009

35

E: patient come back 3 days later and complained headache. C: I would like to know the site of headache and whether it is BELL

Scenario 6
A 3 week old infant presents with pyloric stenosis. He was born by uncomplicated vaginal delivery at term. He has been vomiting intermittently for 2 weeks. His weight is now 2.9 kg from a birth weight of 3.1 kg. His Electrolytes are: Measured Na K CI HCO3 Creatinine Glucose 129 mmol/L 3.0 mmol/L 88 mmol/L 36 mmol/L 69 umol/L 3.0 mmol/L Normal Range (133-143 mmol/L) (3.5-5.5 mmol/L) (95-110 mmol/L) (18-24 mmol/L) (20-75 umol/L) (2.5-5.5 mmol/L)

E: surgeron want to add this case at the end of the list, what would you said? C: this is only a medical emergency and not surgical emergency. The child need fluid resuscitation and if patient was adequately resuscitated at that time, we may still proceed to operation. But I cannot compromise at this moment. E: How would you assess this child? C: may I ask what is the gestation age? E: is term as on the question. C: Oh, yes. I would assess the degree of dehydration. E: how? C: there is ~ 10% decreased body weight. Also clinically I want like to look for E: what cause this electrolyte change? C: vomiting caused dehydration and loss of acid E: How would you replace dehydration? C: deficit replaced with Hartmans solution over 6 hours. Maintenance + ongoing loss with solution + KCl 10 mmol/l E: when is the child already? C: when E: how would you anesthetize this child? C: prepare threater, 2 iv line E: why 2 iv?

10 Years Collection of ANZCA Part II Anaesthetic Viva

MK Yuen & SW Ku

10 Years viva collection 2009

36

C: want to make life easy. One for iv fluid and one for drug injection. Patient was covered up with drapping after operation started, not easy to have another one and patient may need fluid resuscitation. E: when will you put in the second one? C: after anesthetize the patient, there should be one already for induction. E: how to put this patient to sleep. C: RSI, fentanyl 1 mcg/kg, thiopental 5 mg/kg, sux 1 mg /kg with cricoid pressure. E: would you pre-O2 first? C: yes. E: what is the problem for pre-O2? C: the child may not cooperate, mask not fit and the airway of neonate make it not easy. E: what will you do if SpO2 dropped to 89% after fasciculation? C: if no airway difficulties, I will immediately intubate the child first and give 100% O2 afterward. E: would you do bagging? C: no, mask ventilation is not effective. If there is no airway difficulty, Intubation will be more effective to give 100% O2. E: what is you maintenance for this patient? C: O2/air/isoflurane, atracurium for muscle relaxation, fentanyl for pain and titrate against effect. IPPV with Penlon ventilator + newton valve. Sorry, I should use servo ventilator. E: it does not matter even you use penlon ventilator with newton valve. After operation, patient did not wake up. C: I would like to review the drug that has been given. E: midazolam, fentanyl, STP. C: I would use peripheral nerve stimulator to use whether there is any residual block. Look at the size of pupil to see whether it is pinpoint. I would consider to use naloxone, flumazenil to reverse opioid and midazolam. E: what is the dosage? C: naloxone 0.1 mg, flumazenil 0.1 mg/kg E: Patient is breathing but not wake up? C: I would like to exclude any hypoxia E: what metabolic cause you are going to look for? C: hypothyroidism. E: (small laugh) What happen if patient was fasted for such a long time? BELL C: hypoglycaemia.

10 Years Collection of ANZCA Part II Anaesthetic Viva

MK Yuen & SW Ku

10 Years viva collection 2009

37

Scenario 7
A primipara now term after a normal pregnancy has been in labour 12 hours with OP vertex presentation and needs Keillands rotation and vacuum extraction. An epidural placed 8 hours ago is not effective. E: is any term in this question you do not understand? C: I am not sure about Keilland rotation. E: It is to turn the position of fetus. What is your mode of anaesthesia? C: I would first discuss with obs to see how much time do I have, would it proceed to emergency OT if failed. E: No, you have a lot of time. C: entonox and imi pethidine were not effective in this case. Doing a spinal on top of epidural may cause total spinal, so I would not consider this. Epidural analgesia would be a choice. But I would like to find out what is meant by not working. E: there is patchy block. C: I will discuss with patient that I can put in another epidural for her. But the risk of patchy block may still present because of septum inside epidural space. If patient agree, I would put in another epidural. Another choice was pudendal block. E: what is the problem of pudendal block? C: the head of fetus may obstruct the site of injection. There is also risk of fetal bradycardia. E: you put in epidural and patient later complained of nausea. C: It may be a sign of hypotension. I would like to check the BP of this patient and to exclude total spinal. E: there is hypotension. C: I would give fluid resuscitation and ephedrine. E: what other drug would you give? C: I will give maloxone for the nausea. E: what is the cause of hypotension? C: it can be due to E: She was delivered and 6 hours later at 1 am, the nurse inform you that patient is unable to move the limb. Moreover, there was a nurse giving patient iv herparin accidentally 4 hours ago. What would you do? C: I would like to do a assessment of this patient. I would like to know the dosage of heparin given. E: the nurse did not know how much was given. C: it is quite unusual as there was 4 hours already. I would like to check the coagulation profile. E: blood test showed normal coagulation.

10 Years Collection of ANZCA Part II Anaesthetic Viva

MK Yuen & SW Ku

10 Years viva collection 2009

38

C: epidural haematoma cannot be excluded, I would like to have a MRI. E: The nurse asked you whether it is necessary as it is 1 am. C: it is necessary. Because the diagnosis was not known and iv heparin was given before. Moreover, if there is epidural haematoma, the earlier the intervention, the better the result. And delivery should be happy, I dont want the patient to be paralysed. E: let said there is numbness and weakness in plantar flexion of ankle. C: it seem it is a complication due to wrong positioning during lithotomy position. E: what nerve has been involved? C: it was innervated by L45S12, so lumbosacral plexus was involved. BELL E: its good. Keep going.

Scenario 8
A 20 year old man presents to your emergency room after being involved in a high speed motor vehicle accident. He is bruised over his sternum and right anterior chest wall. He has no head, abdominal or long bone injury. Blood Pressure 130/80, Pulse Rate 105/min, SaO2 90% on oxygen at 6L/min via Hudson Mask E: how would you assess this patient. C: primary survey E: there is desaturation, tracheal to L and hyperresonance R chest. C: likely to be pneumothorax. I would put in chest drain if the set is available. If not, I would like to needle thoracocentesis. E: After chest drain insertion, saturation improved. CT showed epidural haematoma. Neurosurgeron said it need urgent surgery in prone position. What would you do for transfer. C: Is there any associated injury, is the spine stable? ( I thought it is epidural haematoma of vertebral at that time. Retrospectively, I thought E mean extradural haematoma of the skull) E: there is only epidural haematoma, no other injury. C: I would like to make sure OT is available. The route from A&E, sorry, the route from CT to OT is not occupied. I would like to have monitor to E: But what I mean is the chest drain during transfer. C: I would temporary clamp the chest drain while transfer to other bed and release afterward. E: why? C: there may be chance that underwater seal may lost during transfer to other bed. E: how would you anaesthetize this patient? C: RSI, fentanyl

10 Years Collection of ANZCA Part II Anaesthetic Viva

MK Yuen & SW Ku

10 Years viva collection 2009

39

E: there is desaturation after intubation. C: COVERABCD approachI would like to exclude pneumothorax. E: there is air come out continuously. C: It may be due to bronchial rupture. I would like to use airway exchanger to change for a DLT and comfirm position with FOB. E: how about if there is not DLT and FOB. C: I would consider endobronchial intubation but it is difficult as it usually go to right side as it is more straight. I will also consider using Fogarty catheter to isolate the right lung. E: you still need FOB for positioning in this case. C: yes. E: what else would you do? C: use FOB to look for site of rupture. E: would you like to ask thoracic surgeron to come and have a look? C: yes. E: both surgeron want to have operation, which one should do first? C: as the lung has been isolated, I would like to do the epidural haematoma first. Because the earlier the operation the better the outcome. But I would also like to ask both surgerons about the time needed. If the repair of bronchial rupture was short, I will let him to do it first. So that there is no need to worry for the lung problem during the surgery for epidural haematoma. BELL

Scenario 9
A 47 year old man is scheduled for laparoscopic cholecystectomy. E: what is your criteria for day surgery? C: firstly, the operation has to be accepted by the public as day surgery. E: yes, thats correct. C: there is social, surgical and medical criteria. For social criteriafor surgical criteria, it has to be a short operation, it has to be minimial pain or pain easily controlled with analgesic, should not have severe bleeding, and there should not be any motor weakness or numbness after anaesthesia E: this is laparoscopic cholecystectomy, what are you talking about? C: sorry, for the surgical criteria and I would like to know whether the patient is having any medical illness. E: he has angina. C: I would like to know the severity of angina and the medication the patient was taking.

10 Years Collection of ANZCA Part II Anaesthetic Viva

MK Yuen & SW Ku

10 Years viva collection 2009

40

E: there was no angina attack now, he was on isodril and prn TNG and he never need to take TNG. C: so, he is ASA 2 to 3 patient and it is acceptable for day surgery. E: what is the nature of pain of lap cholecystectomy? C: there is somatic pain due to insertion of port through muscle. There is also visceral pain due to dissection of surgical site. E: what else? C: there is shoulder pain due to inflation of gas inside abdomen. E: how would you control the post-operative pain. C: I will ask surgeron to infiltrate LA into the abdomen before insertion of port. I will consider to use small dose of opioid to control post-operative pain. E: what else? C: I will consider to do a intercostal nerve block for control of pain. E: what will be the problem? C: there is risk of pneumothorax and patient may need to stay in hospital. E: what other method would you use? C: I will ask the surgeron to infiltrate LA into the surgical site. E: would you use ketorolac? C: no, my experience of using ketorolac was not that effective. E: so, what is you criteria for discharge from day surgery. C: I will discharge the patient when he was E: in the middle of operation, the saturation dropped to 95% and airway pressure raised to 30mmHg. C: I would like to go through COVERABCD approach. I would check whether there is feature suggestive of anaphylasis and I would listen to the chest for air entry to look for bronchospasm and pneumothorax. I would also exclude whether there is tube migation due to position change. E: why is that there is position change? C: because in the operation surgeron may put the patient in head up position to increase the exposure. It may cause traction on the ET tube and causing one lung ventilation. E: now the saturation dropped to 85% and ETCO2 dropped to 15mmHg. C: I would like to inform surgeron that I have a problem with the patient and I will ask him to stop operation and deflate the abdomen. I would like to exclude circuit disconnection E: there is no sudden dropped in ETCO2. C: It seem the patient is having a CO2 embolism. I would like to put patient in head down position and L lateral position and put in CVP for aspiration of gas bubble. I will monitor the vital sign of this patient and give 100% O2. E: why 100% O2?

10 Years Collection of ANZCA Part II Anaesthetic Viva

MK Yuen & SW Ku

10 Years viva collection 2009

41

C: it would improve the absorption of CO2. E: now the patient is having non-palpable pulse. HR 40, SBP 60 mmHg. C: I would like to give atropine 1mg ivi. E: the pulse is not palpable. C: I would like to do CPR at the same time. E: the pulse is now 20. C: I would like to give adrenaline 1 mg ivi. E: the patient was successfully resuscitated. What is the other differential diagnosis for the dropped ETCO2. C: it can be due to decreased cardiac output. It can be a cardiogenic cause hypovolaemic E: How to distinguish CO2 embolism from other causes? C: I dont know E: When is the commonest time to have CO2 embolism? C: during gas insufflation.. BELL

10 Years Collection of ANZCA Part II Anaesthetic Viva

MK Yuen & SW Ku

10 Years viva collection 2009

42

ANZCA Anaesthetic Viva September 2001 Day 1 Various source


(The material, obtained from the candidates who passed the examination, was difficult to read and understand. Some of the time it is impossible to distinguish whether it is question or answer provided by examiner or candidates. It has illustrated that a lot of candidates forgot how difficult to find out the past viva questions, immediately after passing examination. The typist has tried all the effort and type down every material that is understandable)

Scenario 1
A 30 year old primagravida, 36 weeks gestation, presents in early labour with meconium stained liquor, BP 190/120, proteinuria, a persistent headache, blurred vision and hyperreflexia. The Obstetrician wants to do an emergency Caesarean Section. How will you manage her blood pressure prior to surgery. E: how would you manage the BP? E: what is the therapeutic level of Mg? How to monitor Mg? C: loss of reflex E: what other drugs would you use? E: how would you monitor this patient? Justify the use of IBP, CVP, urine output. E: what is the BP you are aiming for? E: platelet is 85, would you do it or not? E: what is your option of anaesthesia? (different scenario provided) E: dose of EA, Choice between SA and EA, use of vasoconstrictor. E: when patient returned to ward, she developed convulsion, what is your management? E: what is the differential diagnosis? E: is it likely to be LA toxicity? E: what is your post-op pain management?

10 Years Collection of ANZCA Part II Anaesthetic Viva

MK Yuen & SW Ku

10 Years viva collection 2009

43

Scenario 2
You are on call on a Saturday morning when you are asked to give an anaesthetic for emergency adenotonsillectomy in a 4 year old boy with obstructive sleep apnoea (OSA) who weighs 36kg and has a diagnosis of Prader-Willi syndrome. How could you find out more about this syndrome and its implications? E: how do you assess this syndrome? C: Ill read up before go for assessment. E: how do you get information? C: book, colleague, internet E: what web site? How to assess quality of information? E: what is you concern in this case? E: how do you do pre-operative assessment? C: acute infection afebrile. Any stridor, ditress? E: how to assess OSA? E: what is CVS manifestation of OSA? C: CHF, RVF, loud P2, polycythaemia E: what is your management? C: discuss with surgeon, medical vs surgical treatment E: what medical treatment? C: nocturnal CPAP, nebulized adrenaline, antibiotic E: any premed? E: mother request sedation, what is your response? E: would you let mother accompanied the child? E: how to prepare the patient? C: O2, suction, monitoring, assistance, ENT surgeon, FOB, rigid bronchoscopy standby, inhalational induction with O2 & sevoflurane. E: what monitoring? E: what assistance? E: why sevo or halothane? E: what is advantage and disadvantage of circle: T piece vs Bain circle? E: developed stridor, what is your management? E: what is the most important thing to do? E: what is your management after intubation? C: arrange and transport to PICU E: when to extubate? C: when there is leak E: what is analgesic option for this patient?

10 Years Collection of ANZCA Part II Anaesthetic Viva

MK Yuen & SW Ku

10 Years viva collection 2009

44

Scenario 3
A 75 year old man with advanced prostate cancer with vertebral metastases presents to hospital following a fall. He is complaining of pain in his left side. He has significant chronic obstructive airways disease. A chest X-ray shows fractures of ribs 7, 8 and 9 on the left side. He is currently taking a slow release morphine preparation, 600mg twice daily plus immediate release morphine up to 400mg daily. He is also on dipyridamole, paracetamol, enalapril and aminophylline. What are the main ISSUES when considering the various options of pain relief for this man? C: consult pain management, assessment, option for pain management E: patient on dipyridamole, what is the special management for EA E: PCA on tramadol, ketamine E: patient complained increase pain C: assess pain chart E: decreased BP and increased pulse, SpO2 93%, what is your management? E: what is your differential diagnosis? E: patient was booked for laparotomy and patient request being extubated after operation. What is your anaesthetic technique? C: EA + GA E: after intubation, there is increased PIP. What is your management? E: there is decreased SpO2 and BP, what is your ddx? C: anaphylaxis E: what is the dose for adrenaline? E: ECG HR 50/mins, no p wave, widen QRS, what is your differential diagnosis? C: EMD E: what is the likely cause of this C: latex allergy E: would you proceed for surgery? C: if it is for life saving, I would proceed.

10 Years Collection of ANZCA Part II Anaesthetic Viva

MK Yuen & SW Ku

10 Years viva collection 2009

45

Scenario 4
A previously healthy 55 year old woman has recently been diagnosed with pancreatic carcinoma. She works in a health food shop in the country town where you are the only specialist anaesthetist and is referred to you because of upper abdominal pain. She has a poor appetite but no nausea. What history would you seek? E: how would you assess this patient E: what is your plan of analgesia C: according to analgesic ladder E: what is the drug in level 2? E: what is the potency of oxycodone, conversion ratio, any oral preparation E: what is the action, metabolism of tramadol, any advantage for renal patient? E: patient cannot tolerate oral medication, what would you do? C: fentanyl patch E: how about intervention? C: explain to patient CPB E: how to do it? E: what drug would you use? C: ethanol vs phenol E: what is the complication? E: what is your post-op order? E: where would you put your patient? E: patient still complained pain, what is your management?

Scenario 5
You are preparing to anaesthetise the second case on your neurosurgical list when a recovery room nurse rings you to say your previous patient is convulsing. The 57 year old patient has been in recovery for 5 minutes after resection of a cortical arteriovenous malformation. What will you do? C: inform nurse, cancel list. Give 100% O2, crash cart. ABC. E: convulsion subsided but patient is still drowsy. What is your management? What is your differential diagnosis? C: send for CT scan brain E: how would you prepare the patient? E: what is the drawback of paralyzing patient?

10 Years Collection of ANZCA Part II Anaesthetic Viva

MK Yuen & SW Ku

10 Years viva collection 2009

46

E: what is the monitor you need? E: BP 230/110, what is your management? E: what is your choice of drug: GTN, SNP, Beta blocker E: CT showed intracranial haemorrhage and need crainotomy, but the transport monitor was dropped off. What is your management? C: assess clinically E: intraoperatively, surgeon complained brain swelling. What is your management? E: there is increased airway pressure, what is your management?

Scenario 6
A 58 year old man of average height and weighing 70 kgs, presents for a left pneumonectomy. He has a 40 pack-year history of cigarette smoking. Within the last five years he has undergone a right carotid endarterectomy, bilateral femoral angioplasties and an internal mammary artery graft to his left anterior descending artery. His current medications are: Lisinopril 10mg daily Metroprolol 50mg 12 hourly Ipratropium ("Atrovent") inhaler 2 puffs 6 hourly How would you assess this man's fitness for pneumonectomy? E: what is your concern? E: ABG pH 7.38, PO2 41, PCO2 63, lung function test FEV1 1.7, FEV1/FVC 70%, did patient fit for operation? E: what other test you would order? E: what is your choice of Double lumen tube? E: what size you would use? Left or right sided tube? E: how to check the position? E: how to check if there is no fiberoptic bronchoscope? E: how to check if there is fiberoptic bronchoscope? E: will you see the bronchial cuff? E: what monitoring would you use? E: where do you insert CVP? E: will you insert PA catheter? E: what is the implication of right side CVP? E: there is R sided pneumothorax, what is your management? E: what is implication of insert chest drain?

10 Years Collection of ANZCA Part II Anaesthetic Viva

MK Yuen & SW Ku

10 Years viva collection 2009

47

C: may need to delay operation? E: there is decreased saturation after OLV, what would you do? E: what is the post-op management of chest drain? C: ask nurse to clamp it and release 12 24 hours.

Scenario 7
You are asked for a preoperative consult on a patient having a Tenckhoff catheter placed (a peritoneal catheter for dialysis). The patient is aged 30, weighs 130kg and is 165cm in height. She smokes 10-15 cigarettes per day. Past Medical History Chronic Renal Failure for ten years Hypertension Anxiety Disorder Past Surgical History Perforated appendix - aged 20 years Ovarian Cystectomy Medications Erythropoeitin 2000 I.U. second weekly Omeprazole 20mgs daily Lisinopril 20mgs daily Current Electrolytes Na K Chloride Bicarbonate Urea Creatinine 138 4.8 107 18 36 0.67 135 - 145 mmol/L 3.6 - 5.1 mmol/L 95 - 107 mmol/L 22 - 32 mmol/L 2.9 - 7.1 mmol/L 0.06 - 0.12 mmol/L

What problems does this patient present for the anaesthetist? E: what are the problems? C: obesity: airway, OSA, DVT, aspiration, cardiac and respiratory problem. Smoker: COAD, IHD, PVD, pulmonary complication. HT: control and treatment. Anxiety: comfort patient. CRF: fluid balance, treatment, multi-organ involvement. Any previous GA. E: these are the investigation result: MP 3, difficult intubation at 20 years old for appendectomy. What is your anaesthetic plan?

10 Years Collection of ANZCA Part II Anaesthetic Viva

MK Yuen & SW Ku

10 Years viva collection 2009

48

C: LA vs GA, RSA vs awake FOB E: you are unable to set up iv. What is your management? E: would you set up iv after gas induction? E: would you give sedation? E: when you do awake FOB intubation, there is bleeding from the nose. What is your management? C: apply pressure to nose, change to inhalational induction. E: what will be the risk? C: aspiration E: what is your intubation technique?

Scenario 8
A 77 year old man presents for elective trans-urethral prostatectomy (TURP) for benign prostatic hypertrophy. He has been treated for congestive cardiac failure for 5 years and has a reasonable exercise tolerance. He states that he does not want another spinal anaesthetic because of delayed return of power to one leg after a previous spinal. What discussion would you have with this patient about the anaesthetic options given his past experience? E: how would you discuss with the patient concerning anaesthetic option? E: how would assess the patient for the foot drop after haemorrhoid surgery under SA? E: what will be the likely cause? E: justify the use of SA in TURP? E: what is TURP syndrome? E: what are the risk factors for TURP syndrome? E: how to minimize the incidence? E: what is the different between tonicity and osmolarity? E: what is the composition of the irrigation fluid? E: how would you monitor this patient? C: CVP, IBP, check Na level E: what is the other risk for TURP? C: sepsis, NH3, glycine. E: patient developed post-operative confusion in recovery room. There is associated decreased BP and heart rate. C: check the ECG of patient. E: there is no p wave. C: consider pacing

10 Years Collection of ANZCA Part II Anaesthetic Viva

MK Yuen & SW Ku

10 Years viva collection 2009

49

E: what investigation would you do? C: check Na E: how to replace Na? E: there is no hypertonic NS, what would you do? C: 8.4% NaHCO3

Scenario 9
A 67 year old male is scheduled for laryngectomy, neck dissection and possible radial forearm free flap repair for a supraglottic tumour. You anaesthetised him for an EUA and biopsy 10 days ago using thiopentone and suxamethonium. The intubation was straightforward. What is your anaesthetic plan for this case? E: What is your plan of anaesthesia? C: reassessment E: Reason for reassessment? E: Why did you expect there is change in airway and respiration? E: Concerns for free flap operation? E: Technique for increase survival of free flap? E: What is your aimed BP? E: Your registrar failed to intubate in first attempt with STP and sux, now profuse bleeding, what is your management? C: head down, suction, optimize head position E: How to optimize head position? C: McCoy blade, bougie, suction, another attempt, LMA + FOB standby E: Unable to intubate, SpO2 dropped, management? E: Surgeon not available to do tracheostomy C: cricothyrotomy E: describe how to do it? E: what is the complication? E: can it be use in children? Why increase risk? E: contraindication for cricothyrotomy? E: What other method available? E: 10 days later, registrar show you in the ward that patient caused you a lot of problems, you found that there is profuse bleeding from the wound. What is you management? C: immediately ask nurse to inform surgeon. Ask for IVF, resuscitation trolley, monitoring. ABC. E: what is the likely cause?

10 Years Collection of ANZCA Part II Anaesthetic Viva

MK Yuen & SW Ku

10 Years viva collection 2009

50

E: what vessel?

10 Years Collection of ANZCA Part II Anaesthetic Viva

MK Yuen & SW Ku

10 Years viva collection 2009

51

ANZCA Anaesthetic Viva September 2001 Day 2 Dr Ng Carmen & CP Chau, PWH
At the end of some scenario recorded questions from a failed candidate who are having the same group of examiners, and other successful candidate with different examiners.

Scenario 1
You are the doctor on call for a helicopter retrieval service and are asked to transfer a 30 year old lady, with severe burns, from a small rural hospital to a burns centre. It will take you 90 minutes to reach the rural hospital and a further 90 minutes to transfer her to the burns unit. What advice would you give to the local doctor? E: What will be your advice to the local doctor? C: my main concern are: ABC, % and degree of burns, mechanism of injury, type of burns, any other associated life-threatening injury. I will advice the local doctor to manage the patient according to the ATLS protocolABC, primary and secondary survey. E: The airway seem to be OK. Patient has suffered from 40% flame burns involving face, neck, chest and upper limb and she jumped from 1st floor. How to manage the fluid resuscitation? C: I will make sure the patient has adequate IV access, at least 2 16G angiocatheter. Fluid management according to Parklands formula 4ml/kg/% hartmans solution, 1/2 over 1st 8 hours and 1/2 over 2nd 16 hours. Assess capillary refill, haemodynamic, conscious level, urine output E: When you arrived to the small hospitals, what will you do? C: I will reassess primary and secondary survey, ABC E: You noticed that the patient has stridor and a bit drowsy. C: Since this patient has signs of upper airway obstruction and she becomes drowsy suggesting that she may not protect her own airway. I would like to intubate her early as she may further deteriorate during transport. I would like to propare the difficult intubation trolley, flexible bronchosocpe and ask ENT surgeron to standby. E: No difficult intubation trolley or ENT surgeron is available in this small hopsital. C: I will assess the airway clinically, and do a DL after lignocaine spary to oropharyngeal area. If this is not a difficult airway, I will proceed to do a rapid sequence induction with cervical spine protection. E: suppose you successfully intubate the patient, what will you do next? C: continue primary survey - ? evidence of inhalational injury, secondary survey AMPLE,

10 Years Collection of ANZCA Part II Anaesthetic Viva

MK Yuen & SW Ku

10 Years viva collection 2009

52

trauma series. Arrange transfer monitioring, equipments, staffs, resuscitated drugs etc E: If SpO2 reading > 90%, will you satisfactied about this reading? C: Since she may suffered from CO poisoning which will over-estimate the SpO2, I would like to take ABG for carboxyl Hb level E: How do CO affect the Hb? C: (bookwork) E: How to treat CO poisoning C: FiO2 100% as hyperbaric O2 only decrease T1/2 from 40 mins to 20 mins E: What level of CO considered to be dangers? C: If > 15-20% (guess only) E: If you found there is circumferential burns over upper limb, when will you considered to do fasciotomy? C: Assess severity of burns, peripheral pulse, consult orhtopaedics surgeron to do fasciotomy early as it will compromise blood flow to peripheral limbs. E: What is you plan to ventilate the patient during transport? C: make sure equipment ready portable ventilator, ambu bag, O2, intubation equipment. E: What kind of protable ventilator will you use? C: Oxylog E: If later, he required EOT for debridement. How to assess him? C: my concerns are potential difficult airway, fluid and electrolyte management, potential major blood loss, hypothermia, avoid use of suxamethonium, monitoring may be difficult e.g. ECG Prepare patient: blood available, correct fluid and electrolyte, secure airway Prepare OT: warm up OT, ect monitoring, temp, u/o E: suppose this patient required long-term ventilation, what is your plan of management of his airway? C: I would like to know the indications of long-term ventilation whether he has evidence of ARDS/Chest infection? E: suppose he has ARDS required long term ventilation C: I will consider to change orotracheal to naso tracheal intubation because of better tolerated and less bitting on the tube E: How about tracheostomy? C: Since he has flame burn involving the anterior neck region. Tracheosotmy may be difficult to perform and increased risk of infection. But advantage of trachostomy included decreased dead space more easy tracheal toileting, more easy to wean the patient from ventilator. E: Say this patient eventually has tracheostomy done 3/52 after burns and when just arrived in ICU, there is accidental decannulation of tracheostomy, what will be your approach? C: This is a crisis situation. 100% O2, call for help, difficult intubation trolley, ENT surgeron

10 Years Collection of ANZCA Part II Anaesthetic Viva

MK Yuen & SW Ku

10 Years viva collection 2009

53

standby. Try to put back tracheostomy tube in situ, if failed, proceed with RSI BELL

Scenario 2
A 37 year old insulin dependent diabetic, Gravida 3 Para 2, presents in pre-term labour. Gestational age, by dates and ultrasound, is 32 weeks. You are asked to review and assess her. What features would suggest the presence of an autonomic neuropathy? E: What are the features of autonomic neuropathy C: bookwork E: she has been started salbutamol to suppress her labour. What other drugs has similar effects? C: other Beta agonist such as ritodrine, Mg E: What are the s/e of salbutamol? C: maternal side: hyperglycaemia, hypoK, tremor, palpitation, pulmonary edema E: Why pulmonary edema? E: What are the fetal s/e? C: fetal hyperglycaemia then neonatal hypoglycaemia, CTG tachycardia causing difficult to monitoring fetal well being. E: surgeron wants to proceed emergency C/S because of fetal distress, what will you do? C: I will discuss with surgeron about the urgerncy E: Surgeron says that you can do whatever technique you like. C: pre-assessment: other medical illness, airway assessment, contraindication for regional, discuss with patient. I will proceed to epidural anaesthesia because of advantage of epidural and avoid s/e of GA- difficult airway, aspiration, awareness E: What are your particular concerns in this case C: Because of DM- stiff joint syndrome causing difficult intubation, autonomic neuropathy increase risk of aspiration E: How do you know whether she has stiff joint syndrome? C: assess mobility of hands and wrist prayer sign, head and neck mobility E: Intra-op, surgeron wants to increase relaxation C: check the level of block of epidural, top up if necessary, consider iv GTN E: what is the dosage of GTN C: ~ 50 100 mcg iv E: onset time of iv GTN? C: < 1 minute

10 Years Collection of ANZCA Part II Anaesthetic Viva

MK Yuen & SW Ku

10 Years viva collection 2009

54

E: In RR, patient was found unarousable, management? C: This is a crisis situation, call for help, crash trolley standby. A: airway intubate with RSI, B: 100% O2, C: check pulse for rhythm, volume, fluid boluses, monitoring, ECG, BP, Ddx: LA toxicity, high spinal block, severe haemorrhage, hypoglycaemia, fluid and electrolyte inbalance, DKA not likely, anaphylaxis, other rare cause amniotic embolism E: you found ECG : wide complex tachycardia 180/min, SBP 60mmHg C: This can either be VT or SVT with aberrant conduction. Since haemodynamic is unstable, I will proceed with cardioversion, started with 50 100 J, increase energy if necessary. Correct electrolyte/ hypoglycaemia/ fluid status, continuous ventilate with 100% O2, inform obs E: will you mx the patient differently if patient is still awake C: I will sedate the patient with small dose of iv midazolam, inform what will happen to patient during cardioversion E: What pharmacological tx will you start? C: amiodarone 5mg/kg over 20 min then infusion E: adv and disadv of amiodarone? C: bookwork E: role of adenosine in this case C: differentiate VT and SVT BELL Failed candidate was asked about use of dexamethasone in preterm labour and its onset of action by the same examiner.

Scenario 3
A 20 kg 7 year old male child with mild developmental delay of unknown aetiology presents for open Nissen fundoplication. In the past he has had numerous gastroscopies and orthopaedic procedures to improve his gait, all under general anaesthesia. His parents are most concerned because severe post operative nausea and vomiting has always been a problem and is most distressing for their son. What do you think are the important considerations in determining the anaesthetic technique that you would use in this child? E: What are your main consideration in your choice of anesthetic? C: main concern: paediatric patient, other congenital abnomalities, increase risk of aspiration evidence of chest infection, increase PONV. I will retrieve previous anaesthetic records what kind of anti-emetic has been given to patient. E: What other inforamtion will you look for in previous anaesthetic records? C: ? easy intubation, any complication in peri-op period, whether sux has been given with

10 Years Collection of ANZCA Part II Anaesthetic Viva

MK Yuen & SW Ku

10 Years viva collection 2009

55

problem. E: say you cant retrive anaesthetic record, how to proceed? C: plan: decrease risks of aspiration and PONV, pre-op period: adequate but avoid prolong fasting, maintain good hydration, avoid narcotic pre-med, premed with EMLA and anti-emetic E: what anti-emetic and dosage? C: ondestron 2 mg ivi E: tell me your plan of induction C: decrease aspiration by RSI with cricoid pressure. Propofol and sux E: what are the concern of sux? C: I noticed that in paediatric patient esp < 8 year old may has undiagnosed muscular disease and may has hyperK after sux. But balance risks and benefits, I think sux is still indicated in this case. E: what are your intra-op plan? C: maintain good oxygenation, hydration, good pain relief with minimizing use of opioid, avoid N2O and volatile agent, use TIVA with propofol. E: do you know why propofol is not recommended in children < 3 yr old C: because there are some cases with unexplained metabolic acidosis with use of propofol (not sure) E: have you used any drug when the drug company is not recommend in some patients but you still want to use? C: yes, e.g. in children < 3 yr old, airway surgery e.g. remove FB, supplement anaesthetic with iv propofol E: in that situation, what should you do before proceed? C: ask for other colleagues advise, balance risks and benefits E: what else? C: ? E: will you do some literature reviews? C: yes E: how to management post-op pain? C: after induction, panadol supp 30mg/kg pr, continuous post-op panadol q6h, limit max 90 mg/kg/day. Ask surgeron to infiltrate the wound with LA +/- regional technique e.g. caudal block E: how about intercostal nerve block? C: yes, but disadv including potential LA toxicity because of large dose, pneumothorax E: have you used ropivacaine to do intercostal n block? C: no, but of bupivacaine, ropivacaine decrease LA toxicity E: how to do TIVA intraop? C: start with 10 mg/kg/hr to 8 to 6 or TCI, monitor clinically + BIS 40 60

10 Years Collection of ANZCA Part II Anaesthetic Viva

MK Yuen & SW Ku

10 Years viva collection 2009

56

E: but this regime is decided for adult, how to modify in children? C: since pharmacokinetically children has increase Vd, I think children required larger dose / kg for maintainence. BELL

Scenario 4
Mr. B. is a single 24 year old admitted for excision and drainage of perianal abscess. He is booked for operation within the next 24 hours and reports severe pain over last 6 hours, is agitated and demands something for his pain. The ward staff are not sure what to do. You are the anaesthetist on call for acute pain problems and are asked to see him. What do you advise? E: What is your response? C: my concern: potential major intra-abdominal pathology, causes of severe pain? Whether he is a drug addict, what kinds of analgesic has been given, nature, severity, location, aggravating and relieving factors of pain, inform surgeron whether they can proceed OT earlier. E: yes, he admitted that he is a drug addict now on methadone and 3/7 ago, he consulted GP and was given this bottle of drugs. (I was given a bottle of drugs with label 100 tablets pethidine 100mg bd but only ~ 5 tablets left in bottle) C: I suspect this patient has pethidine overdose. I will monitor the patient very closely and arrange HDU bed. Other concern for iv drug addicts include adopt universal precaution-HIV, hepatitis, difficult iv access, malnutrition/ immunocompromise, tolerance/withdrawal, endocarditis/ poor historian E: why do you think this patient is agitated? C: he may has severe pain or withdrawal symptoms E: what will be pethidine metabolized? C: by liver to norpethidine, s/e including CNS stimulation causing convulsion E: what will happen before convulsion? C: yes, agitation!!! E: surgeron said that he cant do the case earlier, what will be your plan for pain relief? C: after patient was monitored closely in HDU, I will titrate the pain with ivi morphine 1 mg every 5 10 minutes E: this patient said he is allergic to morphine C: I will clarify the allergic history whether previous episode of well documented anaphylaxis and the tx required. If he is really allergic to morphine, I will considered to do regional

10 Years Collection of ANZCA Part II Anaesthetic Viva

MK Yuen & SW Ku

10 Years viva collection 2009

57

technique after rule out contraindication. E: what regional technique? C: epidural at L3/L4 level with 0.5% bupivacaine. Advantage including good quality of pain relief, can top up for intraop period and post-op pain relief. E: How to manage the methadone during intra-op period? C: continuous methadone to decrease withdrawal symptom E: patient refuse to do the operation under regional technique and he requested GA C: I will proceed with combined GA + epidural. The main concern is they are more prone to have awareness. E: How to monitor the depth of anaesthetic? C: avoid use of muscle relaxant, anesthetic technique GA + LMA with propofol. Monitor clinically unexplained hypertension, technique, patients movement Monitor equipment BIS. BELL

Scenario 5
A 60 year old woman presents to the Outpatient Pain Clinic. She has been referred by a neurologist who has been treating her for trigeminal neuralgia for one year. How would you confirm this diagnosis? E:How to confirm the diagnosis of trigeminal neuralgia? C: Dx will be comfirmed by appropriate Hx, PE, Inx. History: nature, site, aggravating/relieving factor/associated allodynia DDx of facial pain including a) norciceptive pain e.g. dental caries, sinusitis, TMJ pain b) neuropathic pain e.g. post-herpetic neuralgia, trigeminal neuralgia, atypical facial pain y y y y y Symptom suggesting trigeminal neuralgia includes Episodic pain with pain free period in between Unilateral Mandibular distribution Associated allodynia

P/E: rule our dental caries, TMJ spasm. For trigeminal neuralgia, P/E usually normal, some may have decreased corneal reflex E: How to assess the function of mandibular nerve C: sensory cotton wool angle of mandible, motor muscle of mastication E: What investigation? C: MRI to rule out aberrant vessel / intracranial pathology

10 Years Collection of ANZCA Part II Anaesthetic Viva

MK Yuen & SW Ku

10 Years viva collection 2009

58

E: If you think he has trigemnial neuralgia, what kind of drugs therapy will you start? C: multidisciplinary approach. Assess functional status, social, family backgroung For durg therapy, I will start anti-convulsant carbamazeping 100 mg BD E: What is the mechanism? C: membrane stabilizer, decrease spontaneous discharge E: What are the S/E of carbamazepine? C: bookwork, required monitoring of CBP/ L/RFT constantly E: other adjunant? C: phenytoin/ valporate. If failed, do nerve block E: How to do mandibular nerve block? E: How to do trigeminal ganglion block for rediofrequency ablation E: What special equipment required in RF E: detail of anatomy of trigeminal nerve and mandibular nerve- branches E: say a patient has a lower lip BCC for excision, What to do? C: mental nerve block detailed required? BELL Other successful candidate was asked coeliac plexus block, stellate ganglion block, zygomatic nerve block. Failed candidate was asked what specific tumour should be excluded in investigation.

10 Years Collection of ANZCA Part II Anaesthetic Viva

MK Yuen & SW Ku

10 Years viva collection 2009

59

Scenario 6
You have responded to a call for assistance from a junior colleague. His patient is 68 years old, weighs 140 kg. He is having a below knee amputation for peripheral vascular disease under epidural anaesthesia. PROBLEM It is 15 minutes since surgery has commenced The patient has become unrousable The anaesthetist is struggling with the airway with a bag and mask MONITORING SpO2 - 88% ECG - 40 beats per minute with ventricular ectopic beats NIBP - unable to read ETCO2 - no waveform Oxygen flow at 6 l/min What do you do to assist? C: Take over airway call for help, crash trolley and difficult intubation trolley standby. Stop OT, difficult airway alogrithm.need circothyroidotomy problems e.g. difficult landmark, barotrauma etc.. Look for possible complication e.g. aspiration. Feel for pulse, PEA ddx E: How to differentiate each of DDx in this situation? E: Respiratory changes in morbid obesity E: physiological effects of PEEP? Balance of risks and benefits between CVS and resp changes E: what will be your anesthetic technique if you are in charge anesthetist? C: epidural because of advantage bookwork esp phamtom limb pain and disadvantage of GA bookwork esp airway manipulation in obese BELL Other successful candidate was asked: What to do if patient complained post-op numbness of the other lower limb? Failed candidate was asked for the risk factors for sleep apnoea.

10 Years Collection of ANZCA Part II Anaesthetic Viva

MK Yuen & SW Ku

10 Years viva collection 2009

60

Scenario 7
A 74 year old man with prostatic hypertrophy and urinary retention presents for transurethral resection of the prostate (TURP) using electrocautery. 10 days ago he was admitted with left leg pain and was treated with heparin. Two ultrasounds have since failed to show any evidence of a DVT and the heparin was ceased two days ago. How would you plan to anaesthetise this man for the TURP? E: what are your anaesthetic concerns? C: elderly, herparin complication, possible DVT, TURP syndrome E: in general, what are your preferred anesthetic technique for TURP C: SA because if no contraindication E: pre-op assessment Hb: 14, plt 145, wcc 8.9, coagulation normal, mild COAD, clinically no coagulopathy C: preferred SA E: would you start DVT prophylaxis in this case? Problems of DVT prophylaxis and regional technique E: Intra-operatively, surgeron found that this is a case of difficult heamostasis. What will you do? C: make sure patient well oxygenated, adequate iv access, blood/ blood products easily available, correct anaemia/ coagulopathy. Discuss role of invasive monitoring and hypotensive anesthesia. E: Assessment of blood loss in TURP? C: clinical assessment E: what do you think of his plt 145? E: what are the mechanism of heparin induced thrombocytopenia E: intra-op, you take some blood for urgent lab. Hb 7.1, plt 65, fibrinogen 0.7 How to differentiale DIC and dilutional coagulopathy? Role of D-dimer/ FDP? E: How to correct coagulopathy E: role of transmanic acid, EACA, aprotinin? E: few days after OT, BP dropped? ECG shown RAD, S1Q3T3 C: likely PE E: How to mx this case? C: discussed resuscitation. Role of anti-coagulation. Role of thrombolytic, embolectomy E: What are the role of heparin in treatment of DIC C: controversial

10 Years Collection of ANZCA Part II Anaesthetic Viva

MK Yuen & SW Ku

10 Years viva collection 2009

61

BELL Failed candidate was asked: Risk factors for DVT, how sensitive is color doppler in diagnosis of DVT, what pre-op investigation would you do (instead of giving the pre-op lab result voluntarily), what is the treatment for Heparin induced thrombocytopenia, what is the indication for platelet replacement. It seem this is all side tracked question that do not give marks.

Scenario 8
A 55 year old male with rheumatoid arthritis presents for a C1-C2 posterior cervical fusion. You are using Remifentanil and Atracurium infusions and ventilating with Sevoflurane, Oxygen and Air. The patient is prone with the head 20 degrees up. In addition to standard monitoring you have an arterial line. The surgery has progressed and bone is being removed. The End Tidal CO2 level falls from 35 to 26 mmHg. How do you manage this? E: what will you do? C: potential crisis situation, required resuscitation, assessment and management simultaneously and urgently. y y y y 100 % O2 hand ventilation inform surgeron to stop OT feel for pulse, BP, ECG

DDx equipment: blocked sampling tube, leakage, patient factors: air embolism, CVS cause, Resp cause E: How to differentiate this DDx clinically, tell me exactly what to do? E: If you suspect this is air embolism, what will you do? C: bookwork E: if you finally found that drop in ETCO2 is due to leakage in ETT cuff. Mx. E: reinflate. If still large leak, change ETT. Inform surgeron, difficult intubation standby, change to supine position if possible, ways to change ETT e.g. bougie, flexible bronchoscope E: during changing ETT, there is dropped SpO2 to 80% for 2 minutes. Would you document this on your anaesthetic records? What are the effects of this duration of desaturation on the body? C: I will document on the record. 2 min of SpO2 80% may cause possible hypoxia brain damage? E: really

10 Years Collection of ANZCA Part II Anaesthetic Viva

MK Yuen & SW Ku

10 Years viva collection 2009

62

C: (finally I change my mind) usually for this transient desaturation will not cause permanent damage to the body (she seem to be satisfied) E: After the OT, you found that the patient is apnoea. Mx C: ddx residual drugs effects, NMB, central causes e.g. CVA, hypoxia brain damage, metabolic causes e.g. hypothermia, surgical causes e.g. cord damage. Important point inform surgeron, required re-exploration. E: suppose if the patient is awake, but still apnoea. Mx E: do you think remifentanil can cause apnoea? Would you use naloxone to reverse effects of remifentanil? C: No. because of very short acting of remifenatnil. Not likely cause post-op apnoea. E: say, due to some surgical complication, the patient eventually has brain death. How to deal with his family? E: what to do if his family members became very angry at you? C: interview with surgeon, department head, social workers. Identify relative, explain situation, question? MDS, documentation BELL Failed candidate was asked how to test the integrity of breathing circle.

Scenario 9
Midway through your morning thoracic surgery list you are asked to see a patient in the ICU who needs an open lung biopsy. He is a previously well 38 year old man who presented with an acute respiratory infection which deteriorated into respiratory failure and ARDS over 48 hours. He is not improving despite multiple antibiotic therapy and while infection seems the likely cause, no diagnosis has been made. The intensivists have recently instituted prone ventilation and want the open lung biopsy to be done as soon as possible. The ICU is one floor up from the operating suite and access is via a dedicated lift. As you walk up the stairs to the ICU what are your main concerns? E: How to approach this case C: anesthetic concerns patient factors : ?immunocompromised, ?FiO2, PEEP, prone position, ventilator setting, inotropes, medication and heamodynamic? Surgical factors : tolerate OLV E: FiO2 0.6, PEEP 10, PA 48/25, mean 28, BP 100/50, noradrenaline 0.1 mcg/kg/min, dobutamine, TV 500 ml, PaCO2 48 mmHg, PaO2 56 mmHg, RR 14/min

10 Years Collection of ANZCA Part II Anaesthetic Viva

MK Yuen & SW Ku

10 Years viva collection 2009

63

C: still decreased SpO2 on high PEEP with FiO2 0.6. Haemodynamic required support by moderate dose of inotrope E: What do you think of his PaCO2 C: clinically due to permissive hypercarbia E: how about his PA value C: some is due to transmitted by PEEP E: How to calculate his true PA value? C: (not sure) ? transiently turn of PEEP to zero E: what will be you plan to transport him to OT? C: before transport, I would like to know how his oxygenation like during supine position. I will turn the patient supine and see whether he can tolerate or not and adjust FiO2 and PEEP to optimize his oxygentaion E: Tell me why he is in prone position? C: intermittent prone postion can improve oxygenation mechanism proposed including better V/Q matching. Proportional larger area of lung will be in better perfused area (actually I am not sure) E: whatelse would you do before transport? C: good communication with surgeron, OT staff and nurse. Prepare equipment portable ventilator, ambu bag, intubation equipment, monitor invasive, drug inortopes, resuscitation drug, staff experience nurse E: tell me what are the ideal features of portable ventilator? C: book work E: you want an ambu bag but the nurse said that she can get only this one (I was shown a diagram showing a Mapleson C circuit) C: I would not accept this one, I would like a ambu bag definitely E: tell me adv and disadv of using a Mapleson C circuit in this situation C: bookwork (very poor answered for this question) E: How to apply PEEP in Mapleson C circuit? C: No idea E: OK, say you are now in the lift during transport because the OT is 1st floor below ICU. BP suddenly dropped to 60/40. What will be you response? C: crisis situation . Confirm by palpate pulse. Increase FiO2, look for causes. Most likely diagnosis tension pneumothorax, inotropes pump failure, look for other causesfluid boluses, increase dose of NE if necessary E: OK you finally noticed that drop in BP is due to disconnection of NE infusion and fixed it already. Tell me what would you do in OT? C: After I prepare OT e.g. machine, monitor. Good communication with surgeron : what would they like patients position, whether OLV is required

10 Years Collection of ANZCA Part II Anaesthetic Viva

MK Yuen & SW Ku

10 Years viva collection 2009

64

E: surgeron said that they definitely want OLV. C: since his FiO2 is only 56 mmHg despite FiO2 0.6 and PEEP 10 mmHg. I dont think he can tolerate OLV. Besides, lung biopsy is not a absolute indication for OLV. I will explain the patients situation to the surgerons and see whether they can do the OT without OLV. E: surgeron finished the operation and inserted a chest drain afterwards. How would you manage chest drain during transport? C: bookwork E: In general, what are the principles to ventilate the patient with ARDS. C: bookwork: BELL Failed candidate was asked how to improve the oxygenation after the pre-op blood gas was shown. What to do if there is no response to increased PEEP. Also asked what to do if the surgeron want a very short period of OLV after explaining that patient cannot tolerate OLV.

10 Years Collection of ANZCA Part II Anaesthetic Viva

MK Yuen & SW Ku

10 Years viva collection 2009

65

ANZCA Anaesthetic Viva May 2002 Day 1 Dr MK Yuen, QMH


Scenario 1
A 44 year old woman presents with headache, dizziness and difficulty swallowing. She is diabetic on protaphane insulin 15 units daily. A CT scan reveals a large tumour in the cerebello - pontine angle. What other pre-operative workup would you require? E: how would you investigate this patient C: For DM, I want to check the fasting sugar and HbA1c for long term control. I also want to know the CBC, electrolyte, ECG and CXR. Because this is a cerebellar angle tumour, I worried about the hearing, I would like to ask ENT audiology to do the hearing test. E: what is your concern? C: my concern are: position and airway control, VAE and cardiac stability and brain stem monitoring. E: yes, surgeon want to do it in sitting position. Why surgeon want to do it in sitting position? C: (bookwork) E: what is the incidence of VAE C: it depend on what kind of surgery. For cervical spine surgery, the incidence is about 20%, for neurosurgical operation the incidence is about 10 15%. E: but how many of them is clinical significance. C: sorry, I dont know. E: what is paradoxical air embolism. C: 25 30% patient has patent foramen ovale, and (bookwork) so I will also ask for a echo for this patient E: Echo showed a patent foramen ovale, what special precaution would you have. C: I would like to avoid valsalva manoeuvre which may reverse pressure between RA and LA. E: how would you induce this patient C: I would use STP and rocuronium to induce this patient. I would like to give lignocaine 1.5mg/kg and fentanyl to blunt the sympathetic response during intubation. I would like to have esmolol on hand and wait until patient is adequately relaxed E: how would you maintain this patient C: I would like to use TIVA with propofol and remifentanil. I would like to know whether surgeon want to monitor facial nerve to decide whether I give muscle relaxant E: how would you monitoring this patient

10 Years Collection of ANZCA Part II Anaesthetic Viva

MK Yuen & SW Ku

10 Years viva collection 2009

66

C: besides standard monitoring, I would like to put in invasive BP monitoring and CVP insertion for aspiration of air if there is VAE. I would like to have a precordial Doppler to detect VAE E: why use precordial Doppler, would you use PAC C: because precordial Doppler can detect air up to 0.2 ml/kg/min. but PAC can only detect air at 4 ml/kg/min E: how about PAC compare with CVP C: CVP is more sensitive and it can used to aspirate air E: how about the use of PAC in high PA pressure C: yes, it can detect high PA pressure and it signified there is air lock in the PA. So it is useful E: there is drop in BP during opening up of dura C: this is a crisis situation. I would like to ask for help, inform surgeon. I would start simultaneous assessment and resuscitation. My most worry is VAE, other ddx included hypovolaemic, cardiogenic and E: yes, patient has VAE C: I would like to put patient in head down position and ask surgeon to flush surgical site with saline. I will give fluid and vasopressor and cardiac message if indicated. I would aspirate the air via CVP. E: would you put on neck compression C: yes, it can decrease the air entrapment E: the BP is still low C: this may be due to obstructed RV outflow, I would put patient into L lateral position. E: turning patient from sitting to lateral position? C: Oh, I would like to put patient in Trendenleburg position E: after resuscitation, would you proceed for the operation C: it depend on the stage of operation. If the operation has just started, I would not proceed. If it is the middle of operation, then I would proceed. BELL

10 Years Collection of ANZCA Part II Anaesthetic Viva

MK Yuen & SW Ku

10 Years viva collection 2009

67

Scenario 2
A 35 year old man is booked for an elective laparotomy. He has a long history of Crohn's disease. His daily medications include sulphasalazine, azothioprine, 30 mg prednisone, 800 mg slow release morphine. Two days prior to scheduled surgery, he complains of thirst and his blood pressure is 90/70. How will you prepare him for anaesthesia E: what is your assessment for this patient? C: my concern was this patient was on prednisolone and require steroid cover. I would like to know the side effect of sulphasazine and azathioprine which may cause bone marrow suppression and liver derangement. I want to know whether there is any association of Crohns disease with other autonomic disease. This patient also has borderline shock and thrist sensation which may indicate dehydration and intraabdominal sepsis e.g. bowel perforation. So I also want to assess the hydration status and do a physical examination to see any sign of acute peritonitis. E: what other thing you want to know with Crohns disease? C: what is the indication and what exact surgeon want to do for operation? E: anything else? C: sorry, I dont know. E: its OK. Would you like to continuous the drug treatment. C: I would like to stop azathioprine because it cause poor wound healing. I would like to continuous sulphasazine and predinsolone before opeartion. I also want to give steroid cover as well. E: what is the dose? C: hydrocortisone 50 mg ivi on induction and 150mg after operation in divided dose. E: what will be your post-operative pain control of this patient. C: I want to give this patient epidural or PCA. E: what is your regime for epidural. C: I will give 0.1% bupivacaine + fentanyl 2 mcg/ml at 4 ml to 8ml per hour. Because this is a patient taking long term opioid, I would also give this patient morphine infusion with he usual dose. E: how much morphine you would give C: the conversion from oral to iv is 3:1 E: what is tolerance? E: what is the mechanism? E: patient want to choice PCA, what you will give to this patient? C: I would give PCA morphine with baseline infusion. I would set the morphine bolus 1.5 mg with lockout 5 to 8 minutes. I would not set my 1 hour max for the first hour and have frequent

10 Years Collection of ANZCA Part II Anaesthetic Viva

MK Yuen & SW Ku

10 Years viva collection 2009

68

assessment of this patient. I want to know the amount of drug used in the first hour to decide the dose of 1 hour max. E: but the patient want to use pethidine, morphine did not give him good effect. C: Pethidine is a nasty drug. In liver and renal impairment, there is accumulation of metabolite norpethidine which cause cardiovascular depression and CNS excitation. E: would you give it to patient? C: I would like to review the case and find out why this patient is on such a large dose of morphine and what is the indication for morphine and how is the control. Because the dose of morphine is very large, it will be a big dose in converted to pethidine. I am a doctor and it is too danger to give it. E: how to convert morphine to pethidine? C: 1:7.5 E: how about tramadol? C: (bookwork included the preparation and conversion ratio) E: how about hydromorphone? C: (bookwork included the conversion ratio, lack of neuroexcitatory metabolite) E: how about ketamine? C: rapid onset, but hallucination and bronchorrhoea. E: how would you give ketamine infusion? C: I am not quite sure about the dose, I will give BELL

Scenario 3
You are called to provide anaesthesia for an urgent caesarean section for a 38 year old G5 P4 at 34 weeks gestation who has had two previous caesarean sections and has an anterior grade 2 placenta praevia with vaginal blood loss. What are your specific concerns about this patient? E: what is your concern? C: This is a premature labour, placenta praevia with anterior position and previous CS may indicate there is placenta arreta. Patient is having vaginal bleeding which may indicate placental abruption. So I want to find out the haemodynamic variable of the pregnancy lady and how urgernt is the operation. Any fetal distress? E: Beside placenta arreta, what will be other complication? C: the placenta can also attach to urinary bladder. E: what is the name of it?

10 Years Collection of ANZCA Part II Anaesthetic Viva

MK Yuen & SW Ku

10 Years viva collection 2009

69

C: sorry, I cant remember the term. E: its OK. Would you transfer this patient to other hospital? C: there are several considerations. Whether the mother is stable to be transfer, any fetal distress and how urgent is the operation. If the surgery is doing here, are we able to look after a premature baby. E: what would be your anaesthetic technique? C: If there is a fetal distree, I would like to do GA with RSI. Because she is having APH, she may not be able to compensate for the sympathectomy effect of regional anaesthesia. E: how about if there is no APH. C: In US, there is review that the time to decide to do hysterectomy and blood loss is the same for regional and GA. But I would lie more on the side of doing GA because the condition will be more under control if under GA. E: what special arrangement would you arrange? C: I would like to arrange acid prophylaxis which included E: no, I want to know any special arrangement for this lady. C: Oh, sorry. I would like to have a large bore iv cannula, blood for transfusion readily available in threater, rapid infusion system, and blood warming system. E: how many unit of blood? C: at least 6 units. E: any special request for the obstetric? C: yes, I would like to have an experience obstetrician consultant who is experience in doing hysterectomy to do the surgery. I also want other experience obstetrician to help out the surgery (I forgot to mention arrange paediatric resuscitation) E: there is drop in BP after surgery started. C: this is a crisis situation, I would like to ask for help, inform surgeron, simultaneous assessment and resuscitation. The differential diagnosis included hypovolaemic, cardiogenic, obstructive, distributive cause. I would like to ask for the blood loss. E: yes, there is blood loss. C: because pregnancy lady can compensate blood loss up to 1.5 litre. So this patient has loss a lot of blood already. I would like to start fluid resuscitation. E: the baby come out and is floppy. The paediatric register did not know what to do. C: my responsibility is toward the mother, so I will not go to help out. But I would like to make sure the nurse has asked the paediatric consultant to come down to help the resuscitation. E: is there anything you can do to decrease the blood loss? C: I would like to give syntocinon 10U iv bolus then 10U per hour infusion. If there is no response, I would like to infuse sulprostone. E: what? C: prostaglandin infusion.

10 Years Collection of ANZCA Part II Anaesthetic Viva

MK Yuen & SW Ku

10 Years viva collection 2009

70

E: Oh, yes. Anything else? C: I would like to ask surgeon to do packing and compression. I would like to ask then to do hysterectomy as soon as possible. E: anything you can give to patient to decrease blood loss? C: I can give transamine but I dont think it is effective because there is no increase fibrinolytic activity. E: At the time you want to put up blood, she told you that she is Jehovahs witness (examiner never said that she is on RA before). What would you do? C: So this is a Jehovahs witness with massive blood loss. This is a nightmare for me. E: Hahaha (examiner cant stop laughing) C: I would like to find out what blood product she is willing to accept. E: She didnt want any blood product. C: in this case, I would not give blood. E: but the husband want you to give blood, he dont want her to die. C: As she is mentally competent, I will not give any blood and ask the surgeon to do hysterectomy to stop bleeding as soon as possible. E: how about if she was drowsy? C: I would like to review the note and the belonging of this lady to find out whether she has declared she is a Jehovahs witness. E: You find nothing C: I would like to ask opinion of other experience colleague. E: you are the only consultant here. C: In this case, as there is no proof she didnt want blood product and she is drowsy. The consent may not be valid. I would give blood but I would wait until the last minute. E: how about cell saver? C: there is risk of infusing amniotic fluid causing amniotic fluid embolism. (I take a pause to wait for the examiners response but he showed no response) It also need to have certain amount of blood loss to be effective. (I forget to mention she is a Jehovahs Witness) E: how about the floppy baby. C: my responsibility is toward the mother, so I will not go for help but maybe give some verbal suggestion. E: what verbal suggestion? C: resuscitation depends on the Apgars score. (bookwork) E: (before I finish all my answer) It is not resuscitation, it is the airway. C: put in a Laryng BELL

10 Years Collection of ANZCA Part II Anaesthetic Viva

MK Yuen & SW Ku

10 Years viva collection 2009

71

Scenario 4
A 3 year old with Down's syndrome presents to a free-standing day surgery unit for myringotomy and insertion of drainage tubes. The parents say that the child is often "chesty", has frequent respiratory tract infections and snores every night. There is no other medical history. The child is very uncooperative, has a clear nasal discharge and is afebrile. There are no obvious venous access sites. On auscultation you detect a systolic murmur. How will you assess the significance of this murmur?

E: How would you assess the heart murmur of this patient? C: (bookwork) E: how about the patient presented with frequent syncope? C: For adult, aortic stenosis presented with syncope. E: but this is a child. C: Sorry, I dont know. E: what investigation would you do? C: I would like to have ECG, echocardiogram. E: (shown a ECG) this is the ECG of this patient C: ECG showed incomplete RBBB and normal axis (I cant detect other abnormalities, but V3 was crossed by other as V4R. ???reason) E: Paediatrican did an echocardiogram and said it is virtually normal. How would you anaesthetize this patient? C: I presumed this patient did not have other abnormalities associated with Downs syndrome e.g. cervical spine instability. I would like to know whether he has any upper respiratory tract infection. E: there is no URTI. C: I would like to give this child syr midazolam for sedation. Oh! Sorry, this patient has mild OSA and I will not give any sedative premed. I would prepare threater, warm up threater, prepare paediatric equipment. I would put on standard anaesthetic monitoring. Give 100% O2 with sevoflurane and CPAP for gas induction. E: what is your standard anaesthetic monitoring? C: (bookwork) E: after induction, you find that patient has HR 60/min. What would you do? C: It is a potential crisis situation. I would like to check the BP, saturation of this patient. E: it is all normal. Only bradycardia. C: So I would like to keep monitoring this child and have atropine on hand.

10 Years Collection of ANZCA Part II Anaesthetic Viva

MK Yuen & SW Ku

10 Years viva collection 2009

72

E: the BP dropped now. C: I would like to check the HR, BP and saturation. I would give atropine imi to raise the heart rate. E: what is your dose? C: 0.1mg E: after that you found this patient has noise breathing. C: It signified upper airway obstruction or laryngospasm. I would deepen anaesthesia, 100% O2, open up airway with CPAP and do gentle suction to see response. E: you found that he has laryngospam. C: beside the above mentioned measure, I would like to apply pressure to Larson point i.e. the posterior condyle of mandible to break the laryngospam. E: it is a technique that usually break the laryngospam, but it didnt work this time. C: I would give sux sublingual injection 0.2mg/kg to break the laryngospasm. E: 0.2mg/kg? C: yes, I just want to break the laryngospasm. E: so after this patient settle and you want to put in the LMA, he has laryngospasm again. C: I would open up airway, give 100% O2 + CPAP, deepen anaesthesia. E: but how would you control the airway? C: I would like to give sux imi 2 4 mg/kg and then intubate this patient E: after intubation, you found frothy sputum come out from ET tube. C: It is likely to be negative pressure pulmonary edema. I would like to give 100% O2 and PEEP. E: so, would you proceed for operation after all this event? C: If patients condition is stable, I think I would proceed. The same condition may happen next time and this operation is relatively non invasive. E: (just begin to say something) BELL

10 Years Collection of ANZCA Part II Anaesthetic Viva

MK Yuen & SW Ku

10 Years viva collection 2009

73

Scenario 5
A septicaemic 52-year-old male with a history of ischaemic heart disease has been admitted to an intensive care unit. You are asked to sedate the patient for endoscopic retrograde cholangiopancreatography (ERCP). He is on IV antibiotics and has a central venous line and arterial line in place. His usual medications are cisapride and aspirin. What do you consider to be the important issues prior to proceeding? E: what is your concern? C: my concern included indication for ERCP, fitness of patient for ERCP and transfer. ERCP suite is a remote location, I would like to know whether it fulfill the college guideline for anaesthesia in remote location. E: what did the college guideline say? C: staff, facility and design, equipment and drug E: what should be the staff? C: The staff included a trained anaesthetist, an assistance experienced in providing anaesthetic assistance. E: (shown a full lead ECG) this is ECG of this patient. C: (I looked at the ECG) E: tell me what you find while you look at it. C: there is q wave V1 to V3. This is old septal MI (I still looked at the ECG) E: OK, so C: I think there is prolong QT as well. I would like to calculate the corrected QT interval. E: its good you can spot it out. 30 mins after the procedure, you see this ECG rhythm (a ECG stripe was shown). C: I think the pronunciation for this condition is Torsade de pointes E: indeed your pronunciation is very correct. What does it mean? C: It is a ballet term meaning twisting around a point. E: (pause and suddenly realized I am not answering what he want) yes, but what is the underlying meaning of this condition? C: Oh! It is polymorphic VT. E: what would you do? C: I presumed that patient is in prone position. I would turn patient back to supine and check ECG, pulse. Declare cardiac arrest, ask for help, e trolley, defibrillator. I would give this patient defibrillation in sack of three, 200J, 300J, 360J and see the response. But this condition is very resistance to defibrillation, I would also give MgSO4 2g to see the response. E: how fast you give MgSO4? C: over 15 mins because bolus Mg also cause arrhythmia.

10 Years Collection of ANZCA Part II Anaesthetic Viva

MK Yuen & SW Ku

10 Years viva collection 2009

74

E: any other drug you would give? C: because this is a cardiac arrest, I would also give adrenaline and amiodarone 150mg, sorry 300mg bolus iv. E: why you give adrenaline? C: (bookwork, but I only focus on effect) E: can you give metaraminol? C: (bookwork and compare with adrenaline) E: this is the arterial line tracing of this patient. (Arterial line tracing was shown.) C: the systolic pressure is only about 40 mmHg, it is not enough for coronary perfusion. E: what is the coronary perfusion pressure? C: (a deep long signs) it is the diastolic pressure minus end left ventricular(thought block) E: its OK if you did not remember. C: it is diastolic pressure minus the left ventricular end diastolic pressure (should be MAP LVEDP) E: what will be the criteria to end resuscitation? C: there is no fixed rule when to stop resuscitation. But it patient develop rigors mortis, resusciation should stopped. E: after resuscitation, patient is back to ICU. And the nurse involved in resuscitation told you she worried about the brain damage in the next day. C: I would like to do examination on this patient and arrange CT scan brain if condition stable. E: but she only worried about resusciation. C: I would like to arrange debriefing to her. I want to see whether she has any problem during resuscitation. E: why you monitor ETCO2 during resuscitation? C: because it reflected the cardiac output to lung and respiration of tissue. I can be a guide when to stop resuscitation. E: what is the level you will stop resuscitaion? C: I am not quite sure, I guess when level less than 10 mmHg. BELL

10 Years Collection of ANZCA Part II Anaesthetic Viva

MK Yuen & SW Ku

10 Years viva collection 2009

75

Scenario 6
A 55-year-old man presents for a laser resection of a squamous cell tumour in his right main bronchus. He has smoked 40 cigarettes per day for over 30 years. His weight is 60 kg, having lost 15 kg over the past 3 months. How does this patient's medical condition affect his perioperative risk? E: how to assess this patient. C: (bookwork, include assessment on smoking and Ca lung) E: what is the life expectance of this patient? E: why do this surgery if the life expectance so short? E: what is your anaesthetic technique? C: I will modifiy my anaesthetic technique for laser surgery. I will us 30% O2 and air, cover non-operative site with wet drapping. I will give TIVA to keep anaesthesia. [I did not do this scenario well and I got lost afterward. I did not exactly remember what examiner asked me. I mentioned I want to know the surgical requirement. I mentioned the way to keep the ventiation of this patient in laser safe environment. My decision depended on the surgical requirement. But he never told me the surgeons requirement.] E: now there is bleeding from the R middle bronchus. What are you going to do? C: I will put in double lumen tube to isolate the left lung to prevent soiling. E: what DLT you are going to put in? C: Left large Robertshaw DLT. E: how to put it in? E: why not put in a right sided tube? C: you can put it in but it need fine adjustment so that it did not obstruct the R upper lobe opening. E: how common it that? C: I think it is common, but I cannot give you a figure. E: can you put in Right sided tube to use the Right side balloon to tamponade the bleeding? C: it seem to be a good idea but I dont know whether it will work. The R upper lobe may not be ventilated. E: surgeon want to put in rigid bronchoscope and insert fiberoptic bronchoscope to apply adrenaline gauze. C: we may not able to ventilate this patient. E: what rigid bronchoscope do you know? C: ventilating bronchoscope with side arm and jet ventilating bronchoscope which use venturi effect. E: so would you let surgeon put in rigid bronchoscope and insert fiberoptic bronchoscope to apply adrenaline gauze.

10 Years Collection of ANZCA Part II Anaesthetic Viva

MK Yuen & SW Ku

10 Years viva collection 2009

76

C: but we cannot isolate the lung to prevent soiling. E: (shown a piece of paper with the following drawing)

This is the left and right opening over carina when you put in fiberoptic bronchoscope. (he drew 2 opening on carina) what will you see when you go into left side? C: (bookwork) E: what will you see when you are on right side? C: (bookwork included Right lower posterior opening) E: this should be at here (the most distal circle). So you can distinguish where are you with FOB. C: yes. BELL

Scenario 7
Trauma: bilateral # femur for internal fixation (exact scenario was not provided in Examination Report) E: what you would like to know from your junior? C: I want to know whether patient has go through a proper ATLS assessment. My concern is whether cervical spine is stable.

10 Years Collection of ANZCA Part II Anaesthetic Viva

MK Yuen & SW Ku

10 Years viva collection 2009

77

E: cervical spine is fine. C: any flail chest, pneumothorax? E: only simple fracture, no pneumothorax. C: any occult bleeding? Because the heart rate is fast. E: there is no occult bleeding, the fast HR is still to pain. How much blood is lost in femur. C: for both side femur #, the blood lost will be at least 2 litre. E: how much fluid will you give? C: the amount of fluid given depends on the BP at admission. E: how about this BP? C: I will give 2 liter crystalloid to this patient and ressess. E: would you happy to send the patient for this BP? C: the BP is acceptable and the fast HR may be due to pain. I would like to make sure there is large bore iv cannula and blood has send for type and screen. E: the blood bank said because they want to save up money and they dont want to do cross match. They can give you blood within ten minutes. Would you accept? C: no, I want blood readily available in OT. E: how many unit? C: 6 unit at least. E: would you send for this patient after 2 liter of crystalloid? C: for this stable BP, I would send this patient to fix the # to stop the bleeding. E: sound sensible. In your unit, do you put in chest drain when there is pneumothorax during OT or do it before? C: Oh! We used to put in prophylactic chest drain. E: you find that this patient has Hb 7.9, would you transfuse him? C: this young patient can survive the Hb 7.9 and I will transfuse him inside the theater. But it mean he lost a lot of blood, I would give a liter of colloid as well. I did not know his Hb before. E: how would you induce this patient? C: I would like to do RSI with etomidate, sux and cricoid pressure. Besides standard monitoring, I would like to put in a arterial line and CVP. E: its good you still have etomidate, you dont have it now. Did you put in arterial line before or after induction? C: before induction. E: during reaming, there is dropped BP, what is yours response? C: this is a crisis situation. I would ask for help, inform surgeon, start stimultaneous assessment and resuscitation. My main concern is fat embolism, I would also like to look for other differential diagnosis including hypovolaemia, cardiogenic, obstructive, distributive cause. E: would you give anything to raise the BP?

10 Years Collection of ANZCA Part II Anaesthetic Viva

MK Yuen & SW Ku

10 Years viva collection 2009

78

C: yes, I will give fluid resuscitation and vasopressor to raise the BP. E: how would you distinguish between each other? C: For fat embolism, the classical triad will be SOB, confusion and petechiae over upper trunk. We can check blood to look for any specific fat globulin. But the result is cannot come back so early. E: yes, it came back late. Did you expect petechiae so early? C: no, usually need 24 hour to show up. I want to ask for amount of blood loss. E: its blood every where. C: because I put in CVP before, I want to know the reading. E: can CVP tell where you are in this situation? C: if the CVP is high or normal, it cannot tell you anything. But if CVP is low, patient is definitely require fluid resuscitation. I would like to listen to the chest for asymmetrical air entry for pneumothorax, any wheeze for bronchospasm. E: could this patient possible to have pneumothorax? C: yes, because the chest drain may not work properly and there may pneumothorax on the contralateral side. I would check the blood gas for hypocarbia. E: hypocarbia? C: hypocarbia. E: blood gas showed acidosis, hypoxia, hypocarbia. Did it give you any further information? C: no. E: could this be due to fat embolism? C: Fat embolism usually do not present immediate after reaming, but it may be the manifestation of previous embolism. E: did you know any treatment for fat embolism. C: I thought the treatment is mainly supportive, I dont know any specific treatment for fat embolism. BELL

Scenario 8
A 65 year old male presents for coronary artery bypass graft surgery with an implanted cardiac pacemaker in situ. What do you need to know about his pacemaker prior to surgery? E: How would you assess the function of pacemaker? E: why you want to know the implant? E: classification of pacemaker E: what will be the mode of pacemaker after put in a magnet?

10 Years Collection of ANZCA Part II Anaesthetic Viva

MK Yuen & SW Ku

10 Years viva collection 2009

79

C: It will become VOO mode. E: During you put in arterial line, you see this ECG rhythm. (showed a poor quality, compressed scaling with change in QRS morphology) C: there is change in HR and tachycardia is not good for IHD. I would check the saturation, BP of this patient and try carotid massage to slow down the heart rate. E: (point to the changed QRS complex) did you think it is a pacing spike? C: yes, it is a pacing spike. E: did you think carotid massage will help? C: no, this is malfunction of pacemaker. I will put a magnet on to it to see the response. E: what is the problem if you put a magnet? C: it obstruct the surgical field. E: anything else? C: sorry, I did not think of other thing. E: indeed you can use sterile magnet. After bypass and cardioplegia, ECG still showed the pacemaker electrical activity. What is your response? C: I would like to make sure there is no ventricular contraction in response to electrical activity. If not, I will keep observe on it. I will also discuss with surgeon whether they want to cut out the pacemaker and put it back later. E: how to cut the pacemaker? C: we can cut the, Oh sorry, we are not doing open heart surgery. I dont know the way to stop the pacemaker. E: now they are taking off the bypass and there is atrial standstill, what will be you response? C: sorry, I dont understand your question. E: BP is low after come off bypass, do you think it will help by put in a atrial pacing? C: yes, E: (shown a arterial line tracing with increase systolic BP) why the BP increase? C: because it increase the atrial emptying of blood into left ventricle. E: when they started to close the chest, there is ventricular fibrillation. C: I will do defibrillation with the paddle peripendicular to the pacing wire i.e. on anterior and posterior surface of the heart. E: what energy? C: I will first give 10J. If there is no response, I will another 10J. If there is no response, I will 20J and look for any acidosis or electrolyte disturbance on the same time. E: now the surgeon is going to do haemostasis on the chest wall and want to use diathermy, what is your worry? C: my worry is the diathermy may affect the function of pacemaker. I will advise surgeon to use bipolar diathermy, but surgeon dont like to use bipolar diathermy. E: why?

10 Years Collection of ANZCA Part II Anaesthetic Viva

MK Yuen & SW Ku

10 Years viva collection 2009

80

C: because the haemostasis is not as good as the monopolar one. E: yes, surgeon want to use monopolar diathermy, what will you do? C: I will let them to use monopolar but in short burst. I will keep observe the ECG tracing and the cardiac output. I will keep inform them the reading and ask them to stop whenever is necessary. BELL

Scenario 9
In the Preadmission Clinic you are asked to see a 70 year old male patient, scheduled for a right carotid endarterectomy on your vascular list next week. He has a past history of hypertension, insulin dependant diabetes, ischaemic heart disease and chronic bronchitis. He has a smoking history of 50 pack years. How will you assess this patient's cardiac risk for this procedure ? E: How to assess cardiac risk C: according to AHA guideline(bookwork) E: (shown a sheet stated past MI 2 years ago with angiogram L main OK, LAD and circumflex 50 60% stenosis, R coronary distal lesion, mild SOB and mild limited exercise tolerance) C: there is old MI and no unstable angina, compensated HF, I would like to ask for a stress echo E: why stress echo C: to assess the severity now and reversibility E: what is the cardiac risk for CEA C: it is intermediate risk surgery, should less than 5 % E: patient said he found out from internet that LA is better than GA, what is your response C: I would like to find out where is it come from, whether it is a professional website or a layman one? I would like to find out whether it mean LA or RA. E: so what would you like to tell patient C: I would like to tell him that there is pro and con for RA. (bookwork) E: what technique would you use C: my locality did not do regional for CEA, I would be more comfortable to do GA E: How would you keep cerebral perfusion C: I would like to check for any involvement of L carotid artery and I will keep the BP of the patient E: what level would you keep C: this is tricky as patient is having a HT and autoregulation will shift to right side. I would like to

10 Years Collection of ANZCA Part II Anaesthetic Viva

MK Yuen & SW Ku

10 Years viva collection 2009

81

review the usual BP of this patient as a norm. MAP less than 50 is definitely bad E: what else can you do C: I would like to do mild hypothermia and avoid hyperglycercemia E: why avoid glucose C: bookwork E: you are talking about cerebral protection, but anything you can do to keep cerebral perfusion C: I would like to keep patient normocarbia and avoid hyperventilation. E: anything else C: we can measure the stump pressure and do shunt. But it is controversial because (bookwork) E: after induction, SBP 70mmHg, pulse 60 C: this is a crisis situation, I would like to ask for help, inform surgeon, start simultaneous assessment and resuscitation. The cause would include cardiogenic cause, obstructive cause, distributive cause, hypovolaemic cause. I would like to check ECG for arrhythmia and ischaemia E: yes, but would you do anything for the BP C: Oh, yes. I would like to give fluid resuscitation and vasopressor E: give vasopressor so early? C: no, I would give fluid first and then see the response E: how much fluid? C: 500ml full rate, if no response I will give vasopressor and ephedrine will be my drug of choice E: how to maintain the BP intraop C: I would prepare antihypertensive and inotrope infusion to control the BP. I would control hypertension with antihypertensive infusion, my drug of choice will be GTN. If there is hypotension, I will give dopamine infusion. E: how would you monitor the cerebral activity C: if patient under RA, I would observe the conscious state E: yes, how about when patient under GA C: There is several monitor can be use for monitoring. For example EEG, but (bookwork). Another way is to use TCD and the advantage is (bookwork) E: after release of clamp, there is rise of BP C: this is a potential crisis situation. I would like to check pulse, ECG, saturation. It can be due to carotid sinus damage, preexisting HT, inadequate anaesthesia, inadequate muscle relaxation, pain, awareness. It can be due to increase perfusion to the brain E: what is the name of it C: it is hyperper..(thought block)

10 Years Collection of ANZCA Part II Anaesthetic Viva

MK Yuen & SW Ku

10 Years viva collection 2009

82

E: hyperperfusion syndrome. C: yes, it is hyperperfusion syndrome causing increased perfusion to previous under-perfused brain. E: so what will you do? C: I will lower it down with GTN infusion E: In recovery, the nurse told you that he has hoarseness of voice C: I will see this patient immediately. I will check the airway, breathing, circulation of this patient. I am worried about the upper airway obstruction. I will give 100% O2, open up airway with CPAP. I will look at the neck for haematoma and I will put patient BELL (I thought he want to discuss recurrent laryngeal nerve palsy instead)

10 Years Collection of ANZCA Part II Anaesthetic Viva

MK Yuen & SW Ku

10 Years viva collection 2009

83

ANZCA Anaesthetic Viva May 2002 Day 2 Dr Amy Poon, PWH


(These are not completed, some of the details have been forgotten. Also the interaction is more complex than mentioned below)

Scenario 1
A 60 year old female with peripheral vascular disease has non-healing foot ulcers and requires a femoro-popliteal bypass graft. She suffers from dementia but is nursed at home by her husband. The surgery is to be performed at a private hospital close to her home. What are the main management problems to consider in the pre-operative assessment of this patient? E: How do you assess the patient? C: Main concerns Dementia, look for causes PVD likely associated with other medical problems (HT, IHD, CHF, DM etc) Dementia, not fit for consent E: How about in a private hospital? Is it appropriate? What is the requirement of the hospital? C: It needs to have HDU/ICU support because patient may develop complications during the OT. E: How about the staff of the hospital? C: The patient has dementia. May need special care. E: How about the other personnel? C: ... E: There may be no doctor at night in this private hospital. C: I work in public institution and there are doctors on call all the time. Certainly there needs to be doctors on call at night. E: What are the causes of dementia? C: May be multiple infarct or Alzheimers disease. E: Any other causes? C: Other reversible causes of confusion: electrolytes, hypoxia, endocrine e.g. Hypothyroidism E: What drugs usually this patient on? C: major traquillers such as haloperiol E: How do you assess the medical problem ? C: HT (severity end-organ damage, control, SE) DM (BW), IHD: BW, high risk surgery and

10 Years Collection of ANZCA Part II Anaesthetic Viva

MK Yuen & SW Ku

10 Years viva collection 2009

84

intermediate risk pt in AHA guidelines, need stress test. E: The cardiologist said that this patient is not for cardiac cath. because of comorbidities. C: In that case stress test is not indicated. I ll explain to her husband the risk of this surgery. He has to accept the risk before OT. E: How do you anaesthesize this patient? C: My plan will be regional if there is no contraindications because..(adv of regional). She is demented. Ill see if she can tolerate. E: She is not cooperative. C: Then I ll do it under GA E: You have mentioned a lot of advantages of RA. Will you do regional after induction of GA? C: No. Because of the increased risk eg. Neurol damage etc E: But you do it in childhood. Why dont you do it in this patient? C: . But in this case the surgeon will give heparin. Then there is increased risk of epidural haematoma. If the paient wakes up with both LL paralysis, I can't differentiate what is the cause e.g. haematoma, sub-arach injection, neurol damage. E: You do the case under GA. Operation is finished and patient is transferred to recovery. The nurse informs you that the patient is very agitated and restless. What would you do? C: I will assess the vital signs and ensure stable. Then ddx (pain, hypothermia, discomfort e.g. Foley pulling, electrolytes/glucose, hypoxia, hypercapnia, MI, ischaemia, any organ failure, drugs) Actions (check drugs given, r/o pain, blood for e-, glucose, ABG, ECG) E: All normal. What will you do? C: After ruling out all organic causes, I will give ?major tranqilizer eg. Halperidol. BELL

10 Years Collection of ANZCA Part II Anaesthetic Viva

MK Yuen & SW Ku

10 Years viva collection 2009

85

Scenario 2
A 28 year old male with multiple tattoos on his arms and chest was stabbed in the left chest during a gang-related fight. His only injury was a left haemo-pneumothorax, treated initially with an intercostal tube. He has been haemodynamically stable and there has not been any compromise to his respiratory system. Due to slow ongoing blood loss and continuous air leak from the intercostal drain, he is taken to the operating room for an exploratory left thoracotomy. Induction of anaesthesia is uneventful but he has been hypertensive and tachycardic from the onset of surgery. What might be the cause of the hypertension and tachycardia? C: The patient is likely a drug addict. DDx (light anaesthesia, drug intoxication e.g. cocaine, drug withdrawal e.g. heroin, anesthetic drug, 1or 2 HT, fluid overload, serious causes e.g. MH, thyrotoxicosis, phaeochromocytoma) E: About light anaesthesia. He wants to ask about awareness. What are the signs? What kind of monitoring can be used? C: (bookwork) No reliable monitoring. BIS is under research. E: What are the signs of MH? C: (bookwork) E: Increased HR/BP or Temperature first? C: Increased HR first. E: Do you think hypoxia can cause HR/BP? C: Yes! ( missed in DDx) E: What are the problems of drug addict? C: infectious risk, difficult iv, drug intoxication, withdrawal, pain management, may be immunocompromised, some have infective endocarditis, lung abscess E: Usually which part of the heart involved? C: Right heart. E: Now isoflurane 2.5%, already given 30mg morphine. What will you do? C: After rule out serious and organic causes, the most likely cause is drug withdrawal. Ill use remifentanil high dose infusion. If the BP and pulse come down, the suspected opioid withdrawal is confirmed. E: Remifentanil is not available. Will you give morphine? C: I will. E: How much? C: I will give boluses and titrated to response. E: Exactly how much will you give? e.g. in the next hour? C: 10mg in divided dose.

10 Years Collection of ANZCA Part II Anaesthetic Viva

MK Yuen & SW Ku

10 Years viva collection 2009

86

E: What are the adjuncts you can give? C: NSAID: ketarolac, panadol, but not in intraop, no iv preparation. E: Any other? C: tramadol (need to mention the dose, route etc) E: Any other? C: . E: How about ketamine? C: yes, can be given if there is no HI (I am not sure if it is good in pt with increase BP/P) My plan of analgesia in this patient will be intercostals nerve block by surgeon and PCA morphine. Review the patient in postop for option of RA if inadequate pain relief. I will not do epidural intraop because no consent and increase risk in GA patient. E: Risk and disadv of intercostals block? C: (bookwork) E: (tell me the total dose of morphine given in OT, forgotten the dose)Details of PCA setting. C: 3mg boluse, lockout time 8min, 4hr limit 40mg, review in postop period. E: Any background infusion? C: yes, 1 mg per hr. review in postop. Someone from the ward informs you that the patient is very agitated and difficult to manage in the ward. What would you do? C: Ill go and assess the patient including vital signs. If it is due to pain, Ill discuss with patient option of regional. E: Ask about the possibility of withdrawal. Ask about signs and symptoms of Delirium Tremens and treatment. C: (bookwork) BELL

Scenario 3
You are on duty in the delivery suite. You are advised that a labouring woman is being brought from an outlying birthing unit with a prolapsed umbilical cord. She is expected to arrive in minutes. Why is a prolapsed umbilical cord an emergency? E: Why does prolapsed cord cause fetal distress? (exact wording forgotten) C: When the fetal head is not a tight fit with the pelvic brim, the cord can prolapse out into the space and compressed by fetal heard and pelvic brim and hence decreases bld supply to fetus and thus distress. E: Ask about immediate assessment and Mx.

10 Years Collection of ANZCA Part II Anaesthetic Viva

MK Yuen & SW Ku

10 Years viva collection 2009

87

C: Head down and lithotomy position. Hand in vagina to push the fetal head from pressing the cord. E: Have you heard of clasp-knife position? C: Yes! This is another option. E: What do you think if you want to transfer the Pt from the delivery suite to OT? (Only then did I realize that the patient is somewhere peripheral) C: It would be difficult for the patient sustained in the clasp knife position with hand in vagina to be transferred. E: How do you assess the patient? C: AN care, fetal problem, any medical problem, previous GA, allergy, medications, fasting, assess airway, vital sign, fetal condition, CTG. Ill explain to patient plan of GA if the airway look not difficult because of the urgent condition of the fetus. Then Ill give acid prophylaxis and prepare machine, equipment, drugs, assistant, difficult intubation trolley. E: The patient now in clasp-knife position in OT and she tell you she refuse GA. C: Ill clarify the reason behind and explain to her the urgent situation of the fetus and RA is slower than GA. It takes at least 10 mins. E: patient still refuse GA. Surgeon said it is OK if it takes 10 mins. Patient is in clasp-knife position. How do you do the SA? C: I would like to move the patient to lat position for the SA. I will DW surgeon whether it is OK. Also I ll explain to the patient if I fail to do the SA on the first goal or the fetal condition deteriorate, I will proceed to GA. E: Surgeon said the lat position is OK. C: Then Ill turn the patient to lat and proceed to SA. E: After turning the patient, the CTG reading is lost. C: Ill suspect fetal distress although there are many possibilities, e.g. Disconnection etc. I will proceed to GA. E: When you do DL, you can only see the post. pharyngeal wall. C: I will optimize intubation conditions, e.g. change blade to McCoy, long blade, and use bougie. E: All fail. C: If starts to desaturate, I would start manual ventilation. E: If found difficult to ventilate C: It is an urgent situation. Ill call for assistance from another anaesthetist. Difficult intubation is available already. Ill optimize the ventilatory conditions. Hold the mask with 2 hands, ask someone to bag. E: Still difficult C: Saturation now? E: Below 90%

10 Years Collection of ANZCA Part II Anaesthetic Viva

MK Yuen & SW Ku

10 Years viva collection 2009

88

C: Put in LMA E: You can ventilate with some difficulties. C: SaO2 now? E: Above 90% , about 92. C: Ill ask the surgeon to deliver the baby quickly and Ill have the cricoidthyroidectomy set standby in case ventilation cannot be maintained by LMA. After the delivery, the ventilation may be easier. E: Will intubate through the LMA? C: It is possible to intubate through the LMA using Fibreoptic endoscope but it is a tedious procedure and is not suitable in this condition. Another option is to use the intubation LMA but it is not avialable in our obs OT. If the saturation can be maintained, Ill preferred the surgeon to proceed and finish quickly. E: What size of ETT you can use if you intubate the patient? C: size 6 ETT through LMA size 3. E: Let say, now you have intubate the patient through the LMA. There is now desaturation. C: Verify the condition, look at colour and feel the pulse , 100% O2 hand ventilate to feel the compliance. Confirm the ETT position by clinical, ET CO2, R/O tube kinking and cuff herniation. Auscultation to the chest: added sound, wheeze, feel for trachea derivation. Eliminate the circuit if in doubt, ventilate by Ambu bag. DDx: esophageal intubation, tube problem, bronchial spasm, peumothorax, circuit problem E: Any other causes? C: Aspiration! E: Yes! Do you think she is at risk? C: Yes, she is in high risk. Also cricoid pressure is removed when difficult to ventilate. E: What would you do? C: (bookwork) BELL

Scenario 4
You are on call for your local hospital. A registrar rings you about a 7 year old with a compound fracture of his right forearm. The child fell from a trampoline at a birthday party. He has no other injuries or major health problems. His mother says he had prolonged nausea and vomiting following tonsillectomy and adenoidectomy when he was aged 4. He also had a very distressing induction of anaesthesia at that time. His mother says he was held down and a mask put over his face and he went to sleep struggling and crying. He had nightmares afterwards.

10 Years Collection of ANZCA Part II Anaesthetic Viva

MK Yuen & SW Ku

10 Years viva collection 2009

89

He and his parents are very anxious about the coming anaesthetic. He is mask and needle phobic. The orthopaedic surgeons want to take him to theatre within the hour. What advice would you give the registrar? E: How do you assess this patient? C: Other injury, fasting/injury time, General; PH, URTI, asthma, family Hx etc, trace record from last GA E: Pt has no other injury. Just had a big meal before injury. Good health, no URTI/asthma. When would you do the operation? C: Ill D/W surgeon the urgency and how long can he wait. E: Surgeon is very keen. It is a compound #, and they want to do it fairly quickly. C: I would explain to the parents about the conditions. There is risk of aspiration. Plan; premed EMLA, RSI E: The child is very irritable. Would you give premed? C: If the child is very agitated despite the presence of mother, I will give premed midazolam 0.5mg/kg E: The pt refuse to take the premed. What else can you give? C: midazolam per rectal, but I have to look up the dose. E: Any other option? C: im ketamine E: What is the dose? C: 5mg/kg E: Good! Now the patient is in the OT. Despite the im ketamine, you cannot put in the iv. C: Ill do gas induction. Ill call an registrar to put in iv after gas induction. E: What precaution you would do in gas induction? C: Ill apply cricoid pressure after the patient LOC. E: What else? In what position? C: Head down. E: Your registar cannot put in iv. C: I ll call the pediatric surgeon to put in the iv. E: They cannot put in either. C: If the registra is experience to hold the airway, I would. E: Your registra is terrible. C: Ill intubate the patient under gas induction. E: Now , lets say, while the assistant applies cricoid pressure, the pt moves and the assistant releases the cricoid to hold the pt, then the pt vomits C: Standard Mx for aspiration E: You cannot maintain saturation after intubation.

10 Years Collection of ANZCA Part II Anaesthetic Viva

MK Yuen & SW Ku

10 Years viva collection 2009

90

C: 100% O2, hand ventilate, r/o machine problem, ETT prob, examine the chest E: No air entry in the Rt side. C: DDx; pneumothorax, bronchspasm (unlikely), endobronchial intubation E: No sign of Pneumo/endobronchial ETT C: It can be obstruction from food particles. Ill check with the FOB. E: You find a piece of sausage obstructing the Rt main bronchus C: I consult ENT/thoracic surgeon. E: What do you think they would do? C: Rigid bronchoscopy to remove the sausage. E: What would you do? C: Check drugs, equipment e.g. Connections. ETT. LA spray to VC and hand over to surgeon. E: What precaution you would do? C: Ill put in a oral gastric tube for suction before extubation. E: What size of OG tube? C: 10 F, the pediatric size. E: Would you like to put in a bigger one? C: Ill put in a larger one but not too large to tramatize the patient. E: What else would you do? C: E: Would you apply cricoid pressure after extubation to hand over to surgeon? C: Yes! E: Now the surgeon has removed the food particle. Would you proceed to the O&T procedure? C: Depends on the vital signs of the patient. If the saturation is not satisfactory, Ill arrange ICU care, try to improve the oxygention before OT. Ill also D/W surgeon how long it can be wait, ?several hours. E: Surgeon says several hours is OK. Now the saturation is 95%. Would you let the surgeon to proceed? C: Ill let them to proceed. BELL Technique: ventilating bronchoscope, let pt breathe spontaneously using volatiles, via ETT. When I think it is deep enough, Ill remove the

10 Years Collection of ANZCA Part II Anaesthetic Viva

MK Yuen & SW Ku

10 Years viva collection 2009

91

Scenario 5
RTA/HI, CT brain shows Rt Extradural H and swollen brain (the exact scenario was not mentioned in Examination Report) E: How do you assess this pt? C: Manage according to ATLS guidelines. Primary survey and resuscitate at the same time. Airway FB?, suction if blood, secretion. Give O2 and cervical protection. Intubate if decrease in oxygenation or conscious level. E: How do you assess conscious level? C: GCS. E: What is GCS? C: (bookwork) <8 GCS, cannot protect the airway. E: Pts GCS is 9. C: Despite being >8, I will intubate him with in-line stabilzation., because I want to maintain good saturation and BP and in acute phase, hyperventilation can decrease ICP. E: How do you assess whether neck is clear? C: Lat Neck XR E: Surgeon wants to proceed with OT. How do you prepare? C: Alert OT, check all machines, equipment, drugs, assistant , ICU , Blood in OT E: Now the patient arrive OT. How would you anaesthetise him? C: Already intubated? Ill put in arterial line.. E: No, not intubated yet. C: Ill intubate him with MILS, despite the neck XR is normal. I cannot clear the neck because he is unconscious. E: Will you do the flexion-extension XR to rule out C-spine instability? C: No, he is unconsciousness. It is not safe. E: Tell me how would you teach the assistant to do MILS? C: (bookwork in details, including RSI) E: What drugs do you use? C: fentanyl, thiopentone, sux E: What dosage of fentanyl? Why use fantanyl? C: 1 ug/kg, to decrease the intubation response. E: Is it enough? C: It helps but not enough. Ill look at the A-line tracing. If it is nit enough, I will give another bolus of thiopentone. E: Why do you want to blunt the pressor response? C: Because if exceed autoregulatory range, lead to increase ICP. E: You have mention hyperventilation. What ETCO2 you would keep?

10 Years Collection of ANZCA Part II Anaesthetic Viva

MK Yuen & SW Ku

10 Years viva collection 2009

92

C: 25-30mmHg. Because too high will increase ICP, <25mmHg will lead to ischaemia. E: The surgeon complains of swollen brain. What can you do? C: (bookwork) E: Would you use Hartman solution? C: No, because it is hypotonic solution. I ll use NS. E: What is the osmolarity of HS solution? C: 135mmol/l, NS is about 150mmol/l E: What is your view of hypertonic saline? C: In some studies, it shows improve outcome in severe HI. But it has problem e.g. Na too high. E: What is the serum value of Na you will target? C: ..not sure, may be at the upper limit of normal value: 150mmol/l E: What is the advantages of hypertonic saline? C (bookwork) E: What is the problem of mannitol? C: (bookwork) E: OT finished and Pt transferred to ICU. How do you monitor Pt? C: Vital signs; BP/P, SaO2, Neuro: GCS, ICP E: How do you monitor ICP? C: Keep CPP >70 mmHg, drain if ICP >20 E: You mean interventricular catheter? Is there any other way? C: Can monitor via epidural, subdural, interparenchymal.. E: What are the disadvantages of ICP (interventricular)? C: Not reflect true ICP, because in different parts of the brain, ICP may be different. E: How about the advantages? C: Can be used as therapeutic method to drain down ICP. Also CPP known. E: Other monitoring? C: Jugular bulb SaO2, measure global cerebral blood flow. E: Ask about IV value (75%) and Abnormal value (<50%). Others? C: TCD. BELL

10 Years Collection of ANZCA Part II Anaesthetic Viva

MK Yuen & SW Ku

10 Years viva collection 2009

93

Scenario 6
A 74-year-old 85kg male has just had coronary artery bypass surgery. Grafts are: Left Internal mammary to the LAD (left anterior descending) Left radial to the circumflex marginal. He has a past history of hypertension and diabetes. 8 hours following surgery (at 3.00 am) you are called back to the intensive care unit after the patient has lost 1500 mls of blood in the last 2 hours into the drains. The Hb is 51g/L Pulmonary artery pressure is 40/25mmHg Heart rate 96 b/min Blood pressure is 60/35 mmHg CVP 22mmHg The patient is ventilated but opens his eyes on command. The surgeon wants to go back to theatre. Outline your immediate management. E: How do you prepare the patient? C: Resuscitate AB (still intubated), C (know BP/P) E: BP 60/40 C: DDx (hypovolaemia/blood loss, pump failure e.g. MI, graft problem ,cardiac tamponade, afterload e.g. vasodilator, anaphylaxis) In this patient, Hb 5 so need to give blood. Then use Swan Ganz catheter to calculate other parameters (CO, SVR) to differentiate cause of shock. E: How do you differentiate cardiac tamponade and pump failure? C: use echo see rim of fluid in tamponade. E: Other information? C: Catheter tamponade equalization of diastolic pressure as in this patient (CVP 22, PCWP 40/25) E: Yes it is cardiac tamponade, what would you do? C: Keep the preload, contractility, prevent decrease in afterload. E: Would you give inotropes? C: Yes if there is still hypotension. E: Which one and why? (then details in dilution and dosage) C: Adrenaline (bookwork) E: How do you prepare for OT? C: Alert OT, prepare blood, rapid infusion device, other blood products: FFP, platelets, prepare vasoactive drugs, check machine, equipment and make sure cardiaopulmonary bypass and perfusionist, assistant available, then transfer the patient to OT. E: In OT, after the surgeon open the thorax, the BP increase to 140/90, there is bleeding from

10 Years Collection of ANZCA Part II Anaesthetic Viva

MK Yuen & SW Ku

10 Years viva collection 2009

94

previous aorta cannula site. What would you do? C: The surgeon would prefer lower BP for repair. Ill lower down SBP to about 100. E: What drugs? C: SNP (details in dilution and dosage) E: Now, there is generalized bleeding. What can you do? C: Give FFP, platelets, blood E: Others? C: aprotinin (detail in mechanism, SE, dosage) E: Others? C: DDAVP (detail in mechanism, SE, dosage) E: Now you found ST elevation in ECG. C: It can be MI, obstruction of the graft. Ill ask the surgeon to check the heart motion and graft. Ill put TEE to look for RWMA. E: (ask which view, which region of LV supplied by RCA) E: You found aortic dissection, retrograde to RCA. Surgeon wants to do total circulatory arrest for 40 min. Can you tell me the principle? C: Cool down the patient to 15-18 C using CPB. Then give IV thiopentone before the arrest. Finally circulatory arrest. Also put ice around the head. E: How to protect the brain? C: IV thiopentone, ice around head, retrograde flushing from SVC to carotid. E: Other drugs C: dont know E: complications of hypothermia? C: (bookwork) E: usually after total circulatory arrest, patient would become swollen. What drugs do you give? C: not sure, steroids? BELL

10 Years Collection of ANZCA Part II Anaesthetic Viva

MK Yuen & SW Ku

10 Years viva collection 2009

95

The following 3 cases, I cannot remember clearly!

Scenario 7
A 65-year-old 105 kg diabetic has come down from the country to have a colonoscopy. He has been booked for sedation first on your afternoon list. The admitting nurse at the private hospital day surgery phones you during your morning session. She wants advice as the patient mentions contact recently with chickenpox and has a vesicular rash consistent with varicella. How do you respond to this request for advice? E: What is your concern? C: Infectious risk to staff, patient immunocompromised, Medical problem (DM, obese), Procedure needs to be cancelled until pox healed (usu 1 week). E: What are the complications of chicken pox? C: immunocomporomised. Other organs not suremay be encephalitis, pneumonia E: One week later, patient is having procedure in the endoscopy suite under sedation. In the middle, there is bradycardia. What would you do? C: Assess conscious level. Feel pulse to verify. Look at colour. Give oxygen to the patient, check BP, ECG DDx: Heart block.. E: Any causes from the procedure? C: Can be due to pumping gas into colon causing bradycardia. E: What else?( keep asking) C: stretching of gut, may be perforation E: What would you do? C: Atropine (detail in dose, upper limit, any effect >2mg?) Prepare transcutanous pacing if still bradycardia and unstable BP. Of course, Ill stop the surgeon, deflat the gut and lie the patient supine. E: Are you going to pace him in endoscopy unit? C: Yes if the BP unstable. E: pacing is not available. C: Ill use drugs e.g. adrenaline, dopamine, isoprenaline E: Which one? C: Dopamine (in detail) E: Finally, on dopamine, you transfer the patient to CCU. C: ( show me ECG ,Dx: MI + heart block) (Another case) Young, fit man. Procedure in peripheral location. Found tachycardia, what would you do?

10 Years Collection of ANZCA Part II Anaesthetic Viva

MK Yuen & SW Ku

10 Years viva collection 2009

96

C: Assess patient, any discomfort/pain, is sedation enough? Verify pulse, colour. Vitals, BP, SaO2, Give Oxygen. E: Patient is on sedation, asleep; cannot answer questions. C: Stop the surgeon, stop sedation. Measure vitals, ECG for ? arrthymias E: Sinus tachycardia (cannot remember details, still needed some more DDx) BELL

Scenario 8
You are asked to assess a patient in Intensive Care. He was admitted following an industrial accident, with a crushed pelvis and leg injuries two days ago, and surgeons wish to amputate his right foot, which is ischaemic and infected. His blood gases are: PO2 46.1 mmHg (6 kPa) PCO2 40.5 mmHg (5.4 kPa) pH 7.43 Bicarb 26.5 mmol/L SatO2 88% He is intubated and ventilated with the following settings: FiO2 0.6 Tidal Volume 15ml/kg Rate 15/min Peak Inspiratory Pressure 45 cmH2O (4.4 kPa). What can be done to optimise his condition prior to his surgery? E: How do you improve oxygenation? C: Need to know the cause of hypoxia. DDx: Chest injury in some accident, ARDS, too early for chest infection. E: no chest injury. C: likely ARDS E: How can you improve oxygenation? C: Increase FiO2 (but < 0.7 to prevent O2 toxicity), give PEEP 10, limit peak pressure to 35 cmH2O, TV too large 5-7 ml/kg. E: Then CO2 will increase. C: I will allow it, permissive hypercapnia. E: What else?

10 Years Collection of ANZCA Part II Anaesthetic Viva

MK Yuen & SW Ku

10 Years viva collection 2009

97

C: Nitric Oxide. (Then details of mechanism, know how to improve the V/Q, side effects. Then ask about the dosage and finally said dont know dosage because not use in my institution) E: Now the oxygenation improved. How do you prepare and transport him to OT? C: (bookwork) E: What do you check before transport? C: (bookwork) E: Now in the lift, the oxygenation saturation decrease from 95 to 75% C: Eliminate ventilator, use Ambu bag, check ETT with suction catheter, r/o cuff herniation, auscultation look for: pneumothorax, endobronchial intubation, bronchospasm E: SaO2 still 75% C: The cylinder run out of O2! E: The oxygen cylinder drop to the floor! BELL

Scenario 9
You are to provide anaesthesia for a 50-year-old male forestry worker who requires urgent exploration of his arm which has been crushed under a log and has no circulation. What other relevant information do you require to help you manage his anaesthetic? E: How do you assess the patient? C: Other injury, fasting time/ time of injury, vital signs, general anaesthetic history and past medical Hx E: The patient ask about whether he will wake up during the OT. What will you tell him? C: It is rare but it can happen(awareness). I cannot guaratee 100% it will not happen but he is not in the high risk group. E: How do you anaesthetise him? C: Prepare OT, machine, equipment, drugs.. Technique: RSI (thio, sux, fentanyl) E: Do you think there are problems with sux? C: Fasiculate to injured arm? (not sure) E: The surgeon is doing the OT, they are nearly finished. They release the vascular clamp and the BP . What is the cause? C: Most likely reperfusion syndrome. Also can be due to bleeding, pump failure (MI, anaesthetic drugs) afterload, obstruction due to emboli. (cannot remember details)

10 Years Collection of ANZCA Part II Anaesthetic Viva

MK Yuen & SW Ku

10 Years viva collection 2009

98

E: In recovery, nurse inform you the BP is 75/40 C: Ill go and assess the patient. Give 100% O2, verify pulse, colour, check BP, ECG E: Found decrease HR. C: Give atropine. If not improving, use pacing (some details forgotten) E: (Show me the ECG) C: (initially cannot found any abnormality besides bradycardia, then she point to me the T wave) Tall T wave in V3 V4 (very subtle one) E: Yes, what is the cause? C: hyperkalamia due to the reperfusion syndrome E: How do you treat the patient? C: (bookwork) E: Asked about reperfusion syndrome. Oliguria causes? C: rhabdomyolysisMx keep hydration and good urine output E: (still not satisfied) What else? C: diuretic (frusemide), also mannitol, alkalinazation of urine E: What else? C: ... E: What about dopamine? C: yes, it can increase renal blood flow but not shown to improve outcome. E: In post op period, patient tells you that he heard conversation during OT.. BELL

10 Years Collection of ANZCA Part II Anaesthetic Viva

MK Yuen & SW Ku

10 Years viva collection 2009

99

ANZCA Anaesthetic Viva September 2002 Day 1 Scenario only


Scenario 1
You are on call for the Delivery Suite at a Private Hospital. At 4 am the midwife rings asking you to insert an epidural in Mrs. B., a 28 year old parturient in labour at 4cm dilation. You go in. On arrival, the midwife presents you with Mrs. B's birth plan - a crushed brown paper document stating, and underlined - "Do not give me an epidural, even if I beg for one." What are the issues relating to consent for this?

Scenario 2
A short four year old boy weighing 28 kilograms with a history of recurrent tonsillitis had his tonsils and adenoids removed six hours ago under general anaesthesia. He is now pale and blood is oozing from his mouth and nose. The surgeon wants to bring the child back to theatre urgently for control of bleeding. What information would you like before anaesthetising this boy?

Scenario 3
You are called to a regional hospital's accident and emergency department to assist with the resuscitation of a trauma patient. The general surgeon on call cannot be located. On arrival the house officer informs you the patient is a 17 year old male who has fallen from a second storey window onto a concrete wall. The findings and management so far are as follows, He is drowsy with no obvious head injury and complains of abdominal pain His BP is 80/40 mmHg and HR is 135/min despite three litres of Normal Saline given via a 16 G IV Screening X Rays are normal and include a CXR [supine], AP Pelvis and lateral cervical spine Bloods have been taken for CBC, U&E and X Match. What is your priority?

10 Years Collection of ANZCA Part II Anaesthetic Viva

MK Yuen & SW Ku

10 Years viva collection 2009

100

Scenario 4
A 42-year-old, 132 kg, 190 cm man presents for a decompressive laminectomy and fusion of his lumbar spine at L4, 5 in the prone position. He has diabetes and hypertension and is treated with glibenclamide 5mg mane and lisinopril 20mg mane. He has been to your hospital's pre-operative clinic and was noted to get occasional reflux and his airway was graded as Mallampati two. He has been admitted as a day of surgery patient for a 08:00 start and was instructed to take his lisinopril but omit his glibenclamide before coming to hospital. Blood count, biochemistry and ECG done at the clinic are within normal limits. How are you going to induce anaesthesia?

Scenario 5
A 50 year old, 97 kg, 165 cm lawyer presents for a laparoscopic cholecystectomy as a day of surgery admission patient. Current medication includes: simvastatin (Zocor) omeprazole (Losec) indapamide (Natrilix) atenolol fluticasone/salbutamol one one one one daily bd daily daily

(Seretide/Ventolin) puffers

What initial assessment issues are you alerted to in this patient?

Scenario 6
A 33-year-old lawyer has his dominant hand crushed, when doing some house renovations. He is admitted to your Accident and Emergency Department with very severe pain and dusky fingers. Surgery has to be delayed, as the surgeon is not available for another three hours. The surgeon asks you to perform a stellate ganglion block, to improve blood flow, and provide some analgesia. What are the key issues that need to be considered for this patient at this time?

10 Years Collection of ANZCA Part II Anaesthetic Viva

MK Yuen & SW Ku

10 Years viva collection 2009

101

Scenario 7
An 80 kg, 57 year old man, with a long standing history of peptic ulcer disease, presents to hospital with a two day history of vomiting and abdominal pain and is booked for a laparotomy. His past history includes hypertension and smoking. He is haemodynamically stable and has a temperature of 37.5. The house surgeon has recorded a systolic murmur in the notes. What are the key issues around the assessment of this patient?

Scenario 8
You are the Consultant Anaesthetist and the surgeon has been delayed. The first patient aged 30 years is scheduled for a laparoscopic cholecystectomy. She has a history of Anxiety Neurosis and although well of late, she is a very anxious person. She requests something to calm her down, as she has had no premedication. She has no other medical history of note, except allergy to Penicillin. You ask your registrar to commence an IV infusion, to prepare the theatre and to give some sedation and that you will return when the surgeon has arrived. The anaesthetic nurse calls you urgently about five minutes later, reporting that the patient looks distressed, is not breathing well and looks blue. She has not been able to contact the registrar. What are the likely causes of the patient's condition?

Scenario 9
10 days post-operatively, a 62 year old woman becomes dyspnoeic and breathless after a coughing episode which produced watery brown sputum. Her operation 10 days ago was a left pneumonectomy for primary lung cancer, and she had been feeling unwell for two days prior to this sudden breathlessness. What diagnoses would you consider, and what ranking would you give them from most likely to least likely, before we discuss how you will assess and help care for this woman?

10 Years Collection of ANZCA Part II Anaesthetic Viva

MK Yuen & SW Ku

10 Years viva collection 2009

102

ANZCA Anaesthetic Viva September 2002 Day 2 Dr Albert SW Ku, QMH


Scenario 1
You are asked to anaesthetise a 64 year old, 70kg lady who is 165cm tall for a right sided pneumonectomy involving excision of mediastinal nodes and pericardium. She has a 30 pack year smoking history, has controlled asthma, and mild hypertension. She completed a course of antibiotics three weeks ago for pneumonia in her right lung. Her medications are: salbutamol (Ventolin), ii puffs 6th hourly Ipratroprium (Atrovent) ii puffs 6th hourly Fluticasone Proprionate (Flixatide) ii puffs BD Ramipril (Tritace) 5mg mane Indapamide (Natrilix) 2.5mg mane How can you ensure that she is maximally fit for her pneumonectomy? y y y y y y y y y y y Your concern Assessment of asthma Assessment of fitness for pneumonectomy Lung function test and ABG value that is fit for pneumonectomy What ETT would you choose Why Left DLT, size, difference of R and L DLT How do you insert and check DLT What would you use if no DLT Advantage and disadvantage of Univent, pressure of bronchial blocker cuff One lung ventilation, desaturation, management Postop, transfer to ICU, develop wide complex tachycardia and hypotension at corridor, crisis management

10 Years Collection of ANZCA Part II Anaesthetic Viva

MK Yuen & SW Ku

10 Years viva collection 2009

103

Scenario 2
You are in the emergency room to assist with the care of a 7-year-old female with Down's Syndrome, who has burns to her face, neck and shoulders. What do you see as the main issues to be addressed? y y y y y y y y Assessment of burn Fluid replacement in burn, Parklands formula, type of fluid Paediatric fluid management Assessment of upper airway obstruction, when to intubate Procedure of gas induction Problems associated with Down syndrome Method of pain control Later the child has to undergo skin graft operation, mother know her childs disease very well and worries about possible suffering, suppose I am the mother, tell me exactly what you will tell the mother about pain control

Scenario 3
You are asked by the recovery room nurse to review your patient. Patient: 25 year old motor cyclist with a fractured femur and tibia and fibula, from a single vehicle accident. No other known injuries. The patient is otherwise fit but smokes 30/day. Operation: Intramedullary nail to femur, and external fixator to tibia. No problems noted during case. Patient woke up well at end of an uneventful general anaesthetic. Problem: Recovery room nurse tells you the patient has become confused and tachypnoeic 35 mins after admission to recovery. On examination: Tachypnoeic, respiratory rate 32/min, SpO2 94% on O2 6 l/min, Temperature 38.5C P 125/min, BP 100/70, Breath sounds - bilateral crackles y y y y How do you manage How to give 100% O2 Differential diagnosis Investigation o o o y A lab result was shown, CXR -> bilateral patchy consolidation ABG -> hypoxaemia, hypercapnia and resp acidosis, low Na and Cl Interpretation of lab result and DDx

Patient was transferred to ICU, subsequent management

10 Years Collection of ANZCA Part II Anaesthetic Viva

MK Yuen & SW Ku

10 Years viva collection 2009

104

y y y

DDx of confusion, some candidate had longer discussion on fat embolism Postop pain control. I said PCA, but examiner said I decided to use epidural, then ask pros and cons of epidural in this situation, drugs used Interaction of epidural and LMWH, time of insertion and removal of catheter

Scenario 4
The neurosurgeons request you review, for surgery today, a 72-year-old male who was admitted overnight, with a headache over the past few months and recent confusion. He had a prostatectomy 5 months ago and was noted to have atrial fibrillation one month ago. His local Doctor had started him on low dose aspirin and warfarin. What conditions could the neurosurgeons be expecting to treat? y y y y y y y y y y y y y y Why do you think the patient is given aspirin and warfarin by surgeon? What are they intended to treat? DDx What investigation will you do A CT brain was shown -> L acute on chronic SDH, midline shift, decreased ventricle size in left side, loss of differentiation of grey and white matter in left What do you think the value of this patients ICP will be Normal ICP value Etiology of SDH. If I tell you he had the prostatectomy done under SA, do you think that these are related? Preop assessment The patient has AF, will you cardiovert him? Under what circumstances you will cardiovert him? Medical treatment of AF Anaesthesia plan (I said TIVA) Reason and advantage of TIVA. Dose of propofol and remifentanil. Will you use N2O? Monitoring Management of CPP and ICP

10 Years Collection of ANZCA Part II Anaesthetic Viva

MK Yuen & SW Ku

10 Years viva collection 2009

105

Scenario 5
A 30 year old primagravida at term presents for elective caesarean section. The pregnancy has been uncomplicated. Your history reveals no health problems and no allergies. Your physical examination reveals no abnormal clinical findings. The patient consents to a Combined Spinal Epidural technique. Weight: 75kg BP 120/80 As you start your anaesthetic, what steps will you be taking to prevent a fall in blood pressure? y y y y y y y y y y y y What measures would you use to prevent hypotension? What vasopressor will you use Pharmacology of ephedrine, metaraminol, phenylephrine, atropine Thoracic segmental sympathetic supply of cardioaccelarator Just after delivery of baby, the patient complaint of SOB, how would you manage? DDx Amniotic fluid embolism. Its etiology, pathology, clinical feature, mortality, complication, treatment Will the baby be affected in AFE? What are the drugs that will be given after baby is delivered? Pharmacology of syntocinon Antibiotic anaphylaxis ..

Scenario 6
A previously self-caring man of 71 years is scheduled for elective right hemi-colectomy for a carcinoma in one week's time. You meet him at the preoperative assessment clinic in a large city hospital new to you when you commence your first specialist post there. You will be his anaesthetist. How do you assess him for this operation, and what preparations and issues would you discuss with him. y y y y y y Preop assessment Drug Hx -> metoprolol + ? What investigation will you do An ECG shown -> old anterior MI, assessment and investigation Will you postpone operation? He had no chest pain before, and surgeon said the removal of tumour cannot be delayed Anaesthetic plan

10 Years Collection of ANZCA Part II Anaesthetic Viva

MK Yuen & SW Ku

10 Years viva collection 2009

106

y y y y y

Intraop hypotension, crisis management Haemocue 8.3(?), your trigger of transfusion Diffuse bleeding, management Postop ICU another ECG -> old anterior MI similar to first ECG, what is your plan ..

Scenario 7
A woman presents for elective knee replacement surgery, to be performed under a combined epidural and general anaesthetic. She is 60 years old, and is in good health apart from arthritis and a history of mild hypertension for which she is not receiving any current treatment. You ask your assistant, the anaesthetic consultant, to administer some sedation (midazolam) and the antibiotics (cephalothin 1g and gentamicin 240mg) as you, the registrar, are establishing the block prior to induction of general anaesthesia. As you complete the catheter dressing, the patient complains of difficulty breathing, feels faint and becomes very distressed. Outline you management of this change in her condition. y y y y y y y y y y y y Management and resuscitation Patients BP 70, pulse 40, bilateral wheezing, DDx You found that your consultant actually gave IV atenolol instead of an antibiotic. He asked you not to report to any body, what will be your response? Will you report this? Quality assurance activities How to reduce the chance of drug administration error ? Are you aware of any article that discuss on how to reduce drug administration error? In aeroplane accident, people usually say that it is the fault of the pilot, but actually most of the time it is not. What is your opinion? Some questions on policy making and administrative procedure Later you find that antibiotic was injected into the epidural catheter, what will you do Will you inject saline into the catheter Will you remove the catheter

10 Years Collection of ANZCA Part II Anaesthetic Viva

MK Yuen & SW Ku

10 Years viva collection 2009

107

Scenario 8
You are on call at a private hospital for the first time. You have never worked there before. At midnight, the shift co-ordinator calls you to tell you that the general surgeon wants to perform an urgent laparotomy on a 60 year old Mr AB; you are requested to attend ASAP. On your arrival you see Mr AB in the holding area. He is sitting up in bed with a cannula and appears to be in no distress. He smokes 20 cigarettes per day. y y y y y y y What will be your initial assessment What and how much fluid will you give The patient had no urine for 10 hours, management Surgeon said that the diagnosis is appendiceal abscess, your anaesthetic plan Will you start anaesthesia right away? You found that the anaesthetic machine in OT is a new electronic anaesthetic machine that you have never seen before, what will you do How do you check anaesthetic machine? Detail description on following was asked o o o o o y y y y Gas supply Flowmeter Leaking Circle system Scavenging system

Your OTA is new, he does not know how to apply cricoid pressure, how do you teach him? Your OTA is too nervous. During induction, he drop the laryngoscope onto the floor and release the cricoid pressure hand to pick up the laryngoscope. What will you do During intubation, you found that there is food residue in oropharynx, management What are the predisposing factor for aspiration pneumonitis

10 Years Collection of ANZCA Part II Anaesthetic Viva

MK Yuen & SW Ku

10 Years viva collection 2009

108

Scenario 9
At the completion of an orthopaedic list, the surgeon requests a lumbar epidural steroid injection for one of his current inpatients. You are told that the patient is 50 years old, has a six month history of lower back pain and has received 2 previous epidural steroid injections that have given good symptomatic relief How would you proceed? y y y y y y y y Indication of epidural injection What is sciatica Neurological signs of nerve root compression What is cauda equina syndrome Contraindication and complication of epidural steroid Drug and dose used, mechanism of action Complication of steroid if dura was punctured and became intrathecal injection .

10 Years Collection of ANZCA Part II Anaesthetic Viva

MK Yuen & SW Ku

10 Years viva collection 2009

109

ANZCA Anaesthetic Viva May 2003 Day 1 Dr. Anna Lee, PYNEH
Scenario 1
A 27 year old female patient presents to your rural hospital with a bowel obstruction. She is 24 weeks pregnant. The hospital has no specialist obstetric service. It is Sunday night. You are asked to assess this woman with a view to possible surgery. What are the major issues in the pre-operative assessment? E: What would be your concern? C: This is a pregnant lady with a potentially viable fetus. My concern include her medical and obstetric history towards her surgical condition, I would like to know the cause and the urgency, the fluid and the electrolyte status towards the fetus, I would like to access the well-being I would also like to know the obstetric and paediatric support in this hospital as there is concern of premature labour in the perioperative period and the premature baby will require PICU care, the mother will also required special obstetric care as tocolytic therapy might be needed, I would like to transfer this lady to a tertiary centre. E: This lady has previous operation for ectopic pregnancy, and this is an IVF baby. Otherwise she enjoys good past health. The pregnancy is also uneventful. The surgeons take an AXR and the caecum is 13cm in diameter. What do you think? C: This is a treasure baby, and the lady also has pending bowel perforation. E: Yes, the surgeons want to have an immediate operation, but the mother worries for the baby and refuses the surgery. What are you going to do? C: I would attend the mother and to find out her worries. I would tell her that the operation is not only to save her life, but also her baby. Maternal well-being would be the best guarantee for her babys survival. E: How are you going to monitor the fetal status. C: With doppler and CTG. E: What else? C: .I couldnt think of others E: Do you think it will affect your management? C: Fetal status would be a reflection of the maternal well-being. This will be part of my monitoring for the mother.

10 Years Collection of ANZCA Part II Anaesthetic Viva

MK Yuen & SW Ku

10 Years viva collection 2009

110

E: How are you going to give your anaesthesia? C: I would alert the tertiary center regarding the condition of this patient. I would then prepare the theater, the equipment and the drugs. I would have the difficult intubation trolley standby, and an experience assistant. I would place my standard monitoring, secure large bore intravenous access, and I would place the mother in a left tilt position. After excluding difficult airway, I would induce the mother with rsi with cricoid pressure using Etomidate and suxamethonium. E: Why dont you use propofol? C: Because of the hypotension, in particularly the patient is dehydrated. E: After intubation, BP 94/50mmHg, HR 120/min, What would you do? C: This is a potential crises situation. I will ensure the airway is patent, increase FiO2 to 100%, feel the pulse to confirm the diagnosis. I would support the circulation by increasing the rate of fluid infusion and administration of vasopressor. I would also increase the left lateral tilt. My . E: What vasopressor will you use? C: I will use ephedrine 3mg . E: Why? C: Because it is safe and effective. But it is not a very potent agent, and I will use phenylephrine if the response is poor. E: Would it be a concern for you? C: Yes, it might impair the uteroplacental circulation. However, my priority would be the mother, and maternal well-being would be the best guarantee for a healthy fetus. E: The mother has an uneventful operation. In the postoperative period, she goes into premature labour. She is very angry and complaint about you. What would be your response? C: I would attend this patient, confirm the diagnosis of preterm labour, I would explain to her that the cause would be multifactorial. At this moment, the things we could do is the use of tocolytic therapy to prolong the pregrancy, and to inform the tertiary centre to prepare for tranfer. If the patient is in pain, I would also discuss with her the type of analgesia available. E: How are you going to confirm the diagnosis of preterm labour? C: I would feel the uterus for contraction. I would also ask the surgeon to perform vaginal examination for cervical effacement. I would perform a CTG as well. E: What sort of tocolytic therapy do you know? C: B agonists such as Salbutamol and Terbutaline, Magnessium, Ca channel blocker and NSAID. E: How are you going to give Salbutamol? C: 3mg Salbutamol in 50ml NS, 100ug bolus followed by infusion at 0.1ug/kg/min. I would monitor for heart rate and blood pressure. E: The mother is well after the operation, 16 weeks later she comes back to the tertiary hospital because she is in labour. You are working in that hospital and you are asked to review

10 Years Collection of ANZCA Part II Anaesthetic Viva

MK Yuen & SW Ku

10 Years viva collection 2009

111

this patient. The midwife gives you this CTG strip C: This is Type I deceleration which could be normal. E: Yes . ( looking at me and waiting for something more) C: However I couldnt exclude fetal distress, I would consult the obstetrian. E: The obstetrian suspects that this is fetal distress, and they want to operate within 1 hour. What would be your plan of anaesthesia? C: As the mother is in pain, I would put in an epidural now. This would improve the uteroplacental circulation. E: Could you describe in details how are you going to conduct your epidural? C: (Bookwork) E: What drugs will you give? C: . BELL

Scenario 2
A 21 year old motor cyclist was involved in an accident. His injuries are: amputation of his right arm just below his shoulder a deep laceration of his left arm a compound fracture of his right tibia He was admitted to the Accident & Emergency Department he has been stabilised (BP 130/70, pulse 100 bpm) there were no other injuries given morphine for pain Morphine 35mg IV has been given over 15 minutes with no relief and he is still screaming in pain. You are asked to give advice re further pain management to the Accident & Emergency Staff. What would your advice be? E: What would be your response? C: I would attend this patient, ensure that he has been evaluated according to ATLS guideline and has been stablized E: Yes, everything is stable except that he is in pain. C: I would access for the cause of failed treatment, such as extravasation of the drug, history of drug abuse. I would also ask the patient for the most painful area. E: He is not a drug addict, and the drip is running well. He says the most painful site is his

10 Years Collection of ANZCA Part II Anaesthetic Viva

MK Yuen & SW Ku

10 Years viva collection 2009

112

amputated right arm. C: I would optimize the medical therapy, and then I would consider the use of regional technique. I would give iv Fentanyl 50ug bolues, titrated with the clinical response. E: Why do you use Fentanyl? C: Because of the rapid onset, and is titratable. E: What would be your concern? C: This is a trauma patient with full stomach, use of opioid associate with risk of respiratory depression, loss of airway and aspiration. I would monitor for the conscious level, haemodynamic states and respiratory rate. E: You give another 100ug fentanyl, 25mg morphine, patient becomes sedated, but still in pain. What else could you give? C: I would give iv ketamine 0.2mg/kg. This has an opioid sparing effect and has minimal respiratory depression. E: What would be the other option? C: The use of regional anaglesia such as interscalene block with catheter technique. E: Why a regional technique? C: It provides effective analgesia, and it might decrease the incidence and severity of phantom limb pain. E: Do you think it is effective in decreasing phantom limb pain. C: Not very effective, but on the other hand, it will do no further harm to the patient if it is properly conducted. E: What other technique do you aware of? C: Axillary block, infraclavicular and supraclavicular block. E: Could you tell me about infraclavicular block? C: It is a block conducted in the infraclavicular region lateral to the rib cage at the level of formation of cords. It is quite a painful block, as the nerve plexus lies deep to the pectoris muscles. I havent done one before, and I am more experience in doing an interscalene block. E: Do you think a cervical epidural will help? C: Yes, however technically it is very difficult because of the very narrow epidural space which is less than 2mm. E: Do you think it is safe? C: It is safe in experience hand, but not for me, as I havent done one before. E: (Smiles) Could you tell me how are you going to do your interscalene block? C: (Bookwork) E: The patient goes for a surgical debridement. In postoperative period, he complaints of a lot of pain in his shoulder. What do you think would be the cause? C: I would exclude surgical cause such as infection and compartment syndrome. My other ddx include neuropathic pain, phantom limb pain and CRPS.

10 Years Collection of ANZCA Part II Anaesthetic Viva

MK Yuen & SW Ku

10 Years viva collection 2009

113

E: How do you treat neuropathic pain? C: Medical therapy. This includes membrane stablizer, TCA, gabapentine. E: Which TCA will you use? C: Amitryptylline, I would start at 10mg nocte. E: How about gabapentine? C: It is a GABA analogue, but the site of action is unknown. It might act on Ca channel and NMDA channel. I would start at 300mg nocte, and the maximum dose is 3600mg per day. E: How are you going to manage the pain in the leg? C: I would put the patient on PCA morphine. Although regional block is an option, the total LA dose will exceed the toxic limit when combine with interscalene block. E: How are you going to set your PCA? C: Morphine bolus 2mg iv, lock out time 5 mins, 1-hour limit 10mg. I would follow up the patient in the ward and readjust the setting accordingly. E: 6 months later, the patient come back to you for phantom limb pain, how are you going to manage? C: Multimodal. Main stay of treatment is physiotherapy. 2nd line treatment - psychotherapy includes copying skill and relaxation exercise. 3rd line treatment - medical therapy includes drugs such as TCA, anticonvulsant, gabapentine, opioid 4th line treatment sympathetic block systemic and regional Last resort would be spinal cord stimulation E: Do you think spinal cord stimulation would be useful ? C: Not very useful, it is only my last resort when all the other treatments fail. E: Have you heard of TENS? C: Yes, this will be included under my physiotherapy. E: Do you think it is useful? C: Yes, some patients find it quite useful. BELL

10 Years Collection of ANZCA Part II Anaesthetic Viva

MK Yuen & SW Ku

10 Years viva collection 2009

114

Scenario 3
A 48 year old patient arrives in the Day Surgery Unit scheduled for Functional Endoscopic Sinus Surgery (FESS - an operation done for conditions like chronic sinusitis or nasal polyps). He is generally well and does regular gym classes. He has not been to the preadmission clinic. Past Surgical History - Nil Past Medical History - Nil Medications - Nil On examination you hear a systolic murmur, which has not been previously noted. BP 125/80 mmHg. Pulse rate 80 bpm. What are the possible aetiologies of this murmur? E: What would be your ddx? C: My ddx includes dynamic and structural. For structural cause, this includes ASD, VSD, MR, MVP, AS and HOCM. For dynamic cause, this includes hyperthyroidism. E: How are you going to access the murmur? C: I will do a history, physical exam and investigation. For my history, I would access for chest pain, SOB, syncope, and exercise tolerance. I would also ask for a family history of sudden cardiac death as this would indicative of HOCM. For my physical exam, I would access for cyanosis, symptoms of heart failure. On examination of pericardium, I would access for displaced apical beat, parasternal heave and thrill. I would also access for abnormal heart sound and nature of murmur. Murmur that is indicative of underlying structural heart disease includes pansystolic murmur, murmur with radiation, and coexisting of diastolic murmur. For my investigaion, I would like to have an ECG for ischemic change and LVH, and CXR for heart size. I would also consult cardiologist for echocardiogram. E: Do you think it will affect your management? C: Yes. Patient with valvular heart disease requires antibiotic cover. Different heart disease also has different haemodynamic goal. Some patients would require further intervention and perioperative intensive monitoring. E: This is the Echo report LVA 1.5cm2, Mean pressure gradient 45mmHg. C: This is moderate to severe aortic stenosis. I would access the patient according to AHA guidelines for major clinical predictors. I would also consult the cardiologist for optimization, and the cardiac surgeons for possibility of surgical intervention. E: The cardiologist said the patient has been optimized, and the cardiac surgeon said the patient wouldnt require an operation for next few years. Are you happy to proceed? C: This patient has been optimized. I would inform the patient of increased perioperative cardiac risk and I would proceed with the operation. E: Tell me how are you going to anaesthetize this patient.

10 Years Collection of ANZCA Part II Anaesthetic Viva

MK Yuen & SW Ku

10 Years viva collection 2009

115

C: I would have antibiotic cover for infective endocarditis. I would prepare my theater, my equipment and drugs. I would apply standard monitoring, with arterial line before induction. My haemodynamic goal would be to avoid tachycardia, maintain preload and avoid sudden decrease in afterload. E: The surgeons want to use cocaine as local infiltration, do you think it is a concern? C: Yes, cocaine causes tachycardia, this would reduce the diastolic filling and the cardiac output. E: What dose are you allowed the surgeon to use? C: I wont allow them to use. What they want from the cocaine is the vasoconstrictive effect. I would ask them to use phenylephrine. E: How are you going to give the phenylephrine? C: 10mg in 1L NS and given as local infiltration. I would monitor with blood pressure. E: The patient undergone an uneventful operation. In the recovery room, your nurse informs you that the BP rises to 180/120mmHg, pulse 110/min. What would be your response? C: I would attend the patient immediately, and ask for any discomfort. I would ensure the airway is patent, administer oxygen supplement, and I would feel for the pulse E: The patient complaints of chest pain. C: This is suggestive of myocardial ischemia. I would confirm the diagnosis by doing a 12lead ECG, and take blood for Troponin T level. I would correct the tachycardia by esmolol boluses. I would also start the patient on aspirin, TNG and morphine. I would consult cardiologist for commencement of anticoagulant such us LMWH. E: What would be the intraventricular pressure for this patient. C: 225mmHg E: Would it be a concern? C: Yes, this would compromise the subendocardial perfusion. E: What other drugs would you use? C: Vasodilators, such as hydralazine E: Do you know the toxic dose of hydralazine? C: ..No, I have to check it up. E: Three years later, this patient comes back for THR. He is worried of contacting HIV infection from transfusion, and he requests for predonation. His cardiac condition has been stable without any changes. What are you going to do? C: I would like to know the details of his worry. I would also have to check if our hospital have the arrangement for predonation. For his cardiac condition, my worry is sudden drop in preload at the time of blood collection. In views of underlying moderate aortic stenosis, he is not a good candidate. I would advise him on the use of other blood conservation technique, such as.(bookwork)

10 Years Collection of ANZCA Part II Anaesthetic Viva

MK Yuen & SW Ku

10 Years viva collection 2009

116

E: He requests to have an epidural, what do you think? C: A well-conducted epidural analgesia is not a contraindication, I would ensure postoperative icu care with continuous arterial line monitoring. E: Intraoperative, the blood loss is 1000ml, are you going to transfuse? C: The decision will be determined by his preoperative Hb, whether there will be ongoing blood loss, and his haemodynamic state. I would access his BP, pulse and Hb level. E: What sort of heart rate do you target at? C: below 80/min BELL

Scenario 4
A 65 year old woman presents with a 36 hour history of Acute Small Bowel Obstruction. She has Chronic Renal Failure and Hypertension. She last had dialysis 2 days ago and her Arterio-Venous Fistula is now blocked. Her medications are: Caltrate+Vitamin D Multivite Sodium HCO3 Omeprazole What do you consider to be the main areas of concern for your anaesthetic for laparotomy? E: What would be the most concern in this patient? C: I have my patient, anaesthetic and surgical concern. For my patient concern, towards his CRF, I would like to know the cause, duration and complication. In particularly, I would access his fluid and electrolyte state, as his AV fistula has blocked and he has bowel obstruction. I would also like to know if he has other associated diseases such as IHD, PVD and carotid artery disease. Towards HT, I would access according the AHA guideline for major clinical predictors, and I would also like to know his preoperative functional level. For my anaesthetic concern, I would access the airway for difficult intubation, and I would discuss with patient the use of PCA morphine as postoperative analgesia. I would also arrange postoperative icu care as patient would require perioperative haemodialysis. For my surgical concern, I would access the fluid state for dehydration, and discuss with surgeon the urgency of operation. For my investigation, I would like to have CBP for anaemia, RFT for electrolyte, ABG for acid-base, ECG for ischemic change, and CXR for heart failure.
10 Years Collection of ANZCA Part II Anaesthetic Viva MK Yuen & SW Ku

Calcitriol Atorvastatin Irbersartan

10 Years viva collection 2009

117

E: What do you think will be his RFT and ABG like? C: For RFT, Na increase or decrease, K increase, urea and creatinine increase. For ABG, there will be metabolic acidosis with respiratory compensation. E: How do you access metabolic acidosis? C: By calculation of the anion gap. E: How to calculate anion gap? C: This will be the difference between Na,K and Cl, HCO3. For patient with CRF, there will be metabolic acidosis with increased anion gap. E: What else? C: There will be decreased base excess and HCO3 as well. E: How do you access his fluid states? C: By accessing BP, pulse rate, and CVP. E: There is no CVP. C: I would access the JVP, mucous membrane and capillary return. As patient has CRF, urine output is not reliable. E: How would you prepare this patient for the surgery? C: I would correct dehydration and electrolyte disturbance. I would also arrange with icu for preoperative haemodialysis if patient have hyperkalaemia or severe metabolic acidosis. E: What fluid would you use? C: I would use non-potassium containing solution such as 0.9% normal saline. E: What potassium level do you think the patient would need a dialysis? C: Above 5.5mmol/l E: How would you proceed with your anaesthesia? C: I would check my theater, equipment and drugs. I would have the fluid warming derives available, and a experience assistant. I would place on standard monitoring and secure large bore iv access. I would then do a modified rapid sequence induction with cricoid pressure using Etomidate and Rocuronium. I. E: What additional monitoring would you use? C: I would also put in arterial line and CVP. E: Why do you want to put in an arterial line? C: For real-time BP monitoring and blood taking for electrolyte. E: Where would you put in your arterial line? C: At the radial artery on the non-fistula arm. E: Would it be a concern? C: Yes, vascular access is valuable for a CRF patient. There is possibility of radial artery thrombosis and prevent the future use of that arm for HD. However, I have to balance the risk and benefit. In this patient, I would anticipate haemodynamic instability and major fluid shift in the perioperative period.

10 Years Collection of ANZCA Part II Anaesthetic Viva

MK Yuen & SW Ku

10 Years viva collection 2009

118

E: Where would you put in your CVP? C: As the patient usually has the internal jugular vein for HD, I would use the subclavian vein for CVP placement. E: Would you use the dialysis catheter as your CVP? C: No. I prefer to have a dedicated line. E: The surgeons have now finished the operation, and they are closing the abdomen. You notice the heart rate increase to 140/min, SBP 86mmHg, MBP 55mmHg, SpO2 86%. What would be your response? C: This is a crises situation. I would inform the surgeons, call for help, call for the resuscitation trolley. I would increase the FiO2 to 1.0, and hand ventilate to access the airway resistance and chest compliance. I would listen the chest for wheezing and crackles. E: You could hear bilateral basal crackles. C: This is indicative of pulmonary oedema. I would confirm the diagnosis by checking the CVP. E: CVP 12mmHg. C: This is not really high. I would look for other causes. I would pass a suction catheter down the ETT to exclude tube blockage. E: You could suck out pink frothy sputum. C: This sounds like pulmonary oedema. I would ensure patient is adequately oxygenated and ventilated, I would limit further fluid infusion, and I would give adrenaline 2ug/kg to support the circulation, titrated with clinical response. I would review the ECG for any ischemic change, and commence vasodilator therapy such as TNG. Frusemide would not be useful as patient has CRF. E: What do you think would be the cause? C: Myocardial ischemia, arrthymia, overhydration. E: Why do you use PCA morphine as analgesia? Do you have any concern? C: There are two methods for pain relief regional vs PCA. Regional technique is not appropriate as the patient could be septic from underlying bowel obstruction, and the patient also has platelet dysfunction from CRF. PCA BELL

10 Years Collection of ANZCA Part II Anaesthetic Viva

MK Yuen & SW Ku

10 Years viva collection 2009

119

Scenario 5
You are anaesthetising a patient for a radical neck dissection. As you pump the table up it catches the soda lime absorber and fractures it, causing a circuit leak. The patient complains of awareness postoperatively. What steps did you take when you detected the leak? E: How do you check for the circuit leak? C: I would disconnect the circuit from the patient to check for leak. Meanwhile, I would ventilate the patient with ambu bag and maintain anaesthesia with propoful infusion. E: You have no ambu bag. How do you check for leak while ventilating the patient. C: With leaking, there will be a fall in tidal volume, and the fresh gas flow has to increase to compensate for the leak. The amount increases indicates the severity of the leaking. The FiO2 and EtAgent will drop, and the capnograph will change its shape. There is risk of hypoxia and patient awareness. E: You increase the fresh gas flow, but you still have difficulty in ventilating this patient, what else could you do? C: I would activate the oxygen flush intermittently. E: What other circuit would you use to maintain ventilation? C: A mapleson D circuit. E: Why a mapleson D circuit? C: Because it is efficient for controlled ventilation. A FGF of 0.8 times the minute ventilation would be sufficient to prevent rebreathing. E: Could you draw me a mapleson D circuit? E: What other variation of mapleson D circuit do you aware of? C: The Bain circuit, which is a co-axial variation of mapleson D. E: Could you draw me a Bain circuit? E: Could you tell me how to check for leak? C: To check for the inner tube, I would occlude the patient end with a pen and observe for a drop in the bobbin. I would also perform the pethick test, by activating the oxygen flush and observe for a collapse in the reservoir bag due to the venturi effect. To check for the outside tube, I will close the APL valve, occlude the patient end, and a pressure of 30cmH20 should be maintained with a FGF of less than 300ml/min. E: What would be the problem with the Bain circuit? C: The inner tube could leak, and result in dilution of the anaesthetic gas. There is also the problem of hypoxia due to increased dead space. E: Postoperatively, the patient says he could hear something during the operation. What would be your response? C: I would attend this patient immediately, and find out what exactly he could remember. I have to distinguish true awareness from dreaming and hallucination. I would confirm his experience

10 Years Collection of ANZCA Part II Anaesthetic Viva

MK Yuen & SW Ku

10 Years viva collection 2009

120

with other OT staff. I would also access for any psychological disturbance, such as anxiety, insomnia and nightmare. I would refer the patient to psychiatrist for assessment and follow-up. E: When would you refer your patient to see a psychiatrist? C: I would ask my patient whether he would like to see one, but I would try to refer all my patients for assessment. E: How could you detect awareness during operation. C: Clinically, and with the use of neurological monitoring. Clinically, I would access for motor and autonomic response at the time of skin incision. E: Are they accurate? C: No. E: What else could you use? C: BIS monitor E: What is it? C: It is processed EEG with a value of 0-100. It monitors the depth of sedation. With a BIS number of less than 60, it predicts loss of consciousness. E: What would be the drawback of BIS? C: It is not useful for anaesthesia based on opioid and ketamine. E: Do you know why? C: The technique developed on healthy person anaesthetized with O2/N20/vapour. The EEG produced by vapour is difference from than of ketamine or opioid. E: What other new technique are you aware of? C: AEP, lower oesophageal motility, frontalis EMG, RR variation on ECG. E: Which of these do you think would be most useful? C: AEP, it measures cortical procession of auditory signal. Most of the time what the patient could recall during the operation is auditory information. E: Back to this patient, at the time of leaking, what could you do to reduce the chance of awareness? C: A bolus of propofol. E: What else would you do when the patient complains of awareness. C: Apart from patient management, I would also have a clear documentation of the event, inform the surgeons and my consultants. I would inform my medical protection in preparing for possible legal proceeding. I would also discuss with the hospital manager for keeping an ambu bag in every theater. E: What type of patients do you think are more likely to proceed for legal action? C: Patient with a lot of pain at the time of awareness. Poor communication and explanation after the incidence. Poor doctor-patient relationship BELL

Scenario 6
10 Years Collection of ANZCA Part II Anaesthetic Viva MK Yuen & SW Ku

10 Years viva collection 2009

121

A 56 year old man presents to preadmission clinic with a referral letter: Thank you for seeing Mr A who is scheduled for a right sided nephrectomy for renal carcinoma next week. He has a long history of smoking 60 a day for 40 years but ceased 3 weeks ago. He has COAD and I have ordered some respiratory function tests. Sincerely, Mr Whilms Spirometry: ACTUAL NORM %NORM FVC (L) FEV1.0 (L) 1.88 0.87 3.43 2.71 79% 2.76 7.48 3.43 55 32 58 18 37 55

FEV1/FVC% 46% FEF25-75% 0.49 PEF (L/s) BEST FVC (L) BEST FEV 1 0.87 2.79 1.88

2.71

32

How do you assess this man's risk due to his respiratory disease for this operation? E: How do you interpret this lung function test? C: This reveals severe obstructive airway disease. E: How is it going to tell you about the risk of operation? C: It implies increase incidence of postoperative respiratory complication, but it doesnt tell you how severe would be the complication. In fact, its use is limited in general surgery other than lung resection surgery. E: Are you aware of any indicator that indicates an increased postoperative respiratory complication? C: For lung resection surgery, I would look at FEV1, but I dont know any indicators for general surgery. E: How are you going to access this patient regardless of his respiratory problem? C: I would access this patient through history, physical examination and investigation. From the history, I would like to know the duration of disease, the adequacy of treatment, history of ICU admission, recent exacerbation and chest infection, and the exercise tolerance. For the physical exam, I would look for signs of exacerbation, chest infection and pulmonary hypertension. For the investigation, I would like to have CBP for polycythemia, ABG for hypoxemia and hypercapnia, ECG and CXR.
10 Years Collection of ANZCA Part II Anaesthetic Viva MK Yuen & SW Ku

10 Years viva collection 2009

122

E: How are you going to prepare this patient for surgery in regardless of his respiratory problem? C: Preoperative optimization for his respiratory disease. This includes maximization of the medical therapy, chest physiotherapy, antibiotics for pre-existing chest infection. It would be desirable to stop smoking for 8 weeks before surgery. However this should be balanced with the risk of delaying a cancer surgery. E: Do you think chest physiotherapy would help? C: Yes. E: Do you support by any evidence? C: Yes, there is good evidence that postoperative chest infection is reduced. E: What would be your plan of anaesthesia? C: My plan is combined GA and epidural. E: Why do you want to put in an epidural? C: Epidural provides good intraoperative and postoperative analgesia. This patient has increased risk of postoperative respiratory complication, there is good evidence that epidural reduces postoperative respiratory failure. Other general benefits include reduced blood loss and homologous transfusion, reduced DVT and PE, and postoperative ileus. E: Do you support by any study? C: Yes, the Master Trial E: What is it about? C: In high risk patient, epidural reduces postoperative respiratory failure. This is the only statistically significant result. E: Does it reduce chest infection? C: No E: After positioning and as the surgeons make the incision, you notice an increase in airway pressure and the SpO2 drops to 85%, what would be your response? C: This is a crisis situation. I would inform the surgeon, stop the surgery, call for help and resuscitation trolley. I would start simultaneous assessment and management. I would ensure the airway is patient, increase the FiO2 to 1.0, and hand ventilate to access the airway resistance and compliance. My different diagnosis includes patient problem and equipment problem. I would listen the chest for asymmetrical air entry for pneumothorax and endobronchial intubation., wheezing for bronchospasm and aspiration, crackles for pulmonary oedema. To exclude equipment. E: You couldnt be really sure about the air entry, it seems to be equal for both side,s what are you going to do? C: To exclude tension pneumothorax, I would feel for the tracheal position, look for dilated neck veins. I would check the marking on the ETT to exclude endobronchial intubation.

10 Years Collection of ANZCA Part II Anaesthetic Viva

MK Yuen & SW Ku

10 Years viva collection 2009

123

I would also exclude equipment problem. I would pass a suction catheter down the ETT to exclude blockade, I E: You find the tube is going too far in, and the SpO2 improves as you pull back the tube. Postoperative, the patient in the recovery complains of a lot of pain. As what you have planned, your trainee puts in an epidural. What are you going to do? C: I would attend this patient immediately, find out where is the pain, and the severity. I would check for the sensory level, review the anaesthetic chart. E: He complaints of wound pain, and you couldnt find the sensory level. Your trainee has given 15ml 0.2% ropivacaine throughout the operation. C: This amount might not be adequate, I would give another 5ml of 2% lignocaine and reassess. If there is no improvement, I would check the marking of the catheter at skin, to ensure it is still in-situ. I would also pull out 2 cm as sometimes the catheter could go too far into paravertebral space. E: Where you aspirate the catheter, you find blood coming out. C: This is indicative of intravascular placement. I would pull out the catheter and monitor for neurological symptoms, as I am worried of epidural haematoma. For analgesia, I would give fentanyl for immediate relief, followed by PCA morphine. E: Are you going to repeat the epidural? C: No, as this would be very difficult since the patient is in distress and is unable to cooperate. This would also mask the sign of epidural haematoma. E: If the bloody tap occurs at the time of insertion, are you going to repeat? C: Yes, as the benefit outweights the risk. The patient is also more likely to cooperate. Epidural haematoma is still a concern, I would give 0.1% ropivacaine with fentanyl. This preserves the motor power and allows for postoperative monitoring. While in the postoperative period, it is very difficult to reinsert as patient is in a lot of pain. At the same time, if I could achieve satisfactory pain relief with iv morhpine, there would be no good indication to repeat the epidural. E: OK. Now the surgeons want to give heparin, would you allow them to do so? C: Yes, but I would explain to them my concern for epidural haematoma, and I would ask them to delay the administration for 2 hours. BELL

10 Years Collection of ANZCA Part II Anaesthetic Viva

MK Yuen & SW Ku

10 Years viva collection 2009

124

Scenario 7
A 4 year old is scheduled for operation in 2 days to remove a Nephroblastoma. Parents indicate a cough and runny nose commenced 2 days ago. You are the scheduled anaesthetist and can see them, together with the surgeon, today at a pre-operative clinic. What further pre-operative assessment and advice about the cough and runny nose will you discuss with the surgeon and parents? E: What would be your concern for the cough and running nose? C: This is a child with suspected URTI, going for a major abdominal operation. I would like to assess the child with a history, physcial exam and investiagion, and I have my patient, anaesthetic and surgical concern. For my patient concern, I would like to know the birth history, congenital abnormality, developmental milestone, and in particular, history of asthma and chronic lung disease. The latter is due to prolong intubation during neonatal period, and would increase perioperative respiratory complication. For the URTI, I would like to know the duration, whether the child has fever, sputum, colour of sputum, and associated constitutional symptoms, such as reduced activity and appetite. On examination, I would look for tachypnoea, use of accessory muscle for breathing, wheezing and crackles on listening of the chest. If clinically indicated, I would obtain a CXR for pneumonic change. For the surgical concern, this is a cancer operation, and the risk of delaying the surgery should be weighted against the risk of perioperative respiratory complication due to recent URTI. I would also like to know if the child has received any chemotherapy or radiotherapy. For my anaesthetic concern, I would discuss with parents concerning the respiratory complication, the possibility of postoperative icu care, and the use of iv morphine infusion as postoperative analgesia. E: This child is a healthy full term baby, with no history of asthma. He gets no fever, and is eating and playing well. Are you happy to proceed? C: Yes, but I would discuss the risk with parents. E: What are you going to tell the parents? C: I would tell them my anaesthetic plan and the risk of anaesthesia. Apart from general anaesthetic risk, I would discuss with them the slightly increase risk of postoperative respiratory complication. Although this is a concern, but the risk is low, and it is not worthwhile to delay the operation. E: What would be your plan of anaesthesia? C: My plan includes preoperative, intraoperative and postoperative. In the preoperative period, I would inform parents of the risk, discuss about parental presence during induction; Fast for 6 hours for food and 2 hours for fluid, EMLA to the venepuncture site

10 Years Collection of ANZCA Part II Anaesthetic Viva

MK Yuen & SW Ku

10 Years viva collection 2009

125

1 hour before the scheduled time. Intraoperative, I would prepare the theater, drugs and equipment as for paediatric patient and for major blood loss. I would warm up the theater, prepare fluid warmer and rapid blood giving set. I would apply standard monitoring, and I would try my best to secure an iv access before induction. As most patients get abdominal distension with risk of aspiration, I would induce the child with rsi using thiopentone and Suxamethonium. I would maintain anaesthesia with O2/air/isflurane and atracurium infusion. I would obtain additional iv access using 18G cannula, and monitor blood pressure with arterial line, urine output, body temperature, and NMJ function using nerve stimulator. E: Could you tell me about your fluid management. C: This is a 4 years old child, expected body weight is16kg. E: Lets takes as 20kg. C: For fluid management, I have to give the maintenance fluid, and to replace the blood loss and the deficit due to 3rd space loss. For the maintenance, I would give D2.5%NS0.45% at 60ml/hour. For the blood loss, I would replace with Hartmans solution in 3:1 ratio, or colloid in 1:1 ratio. For 3rd space loss, the deficit could be up to 15ml/kg/hr, and I would replace with Hartmans solution. I would titrate my fluid management with blood pressure, pulse rate and urine output. My aim is to maintain a urine output of 1ml/kg/hr. E: What would be your plan for postoperative analgesia. C: I would give iv morphine infusion at 15-30ug/kg/hr, and I would monitor the child in icu. E: How are you going to prepare the drug? C: As the child is 20kg, I would mix up 20mg morphine in 50ml NS, and I would give at 1.5-3ml/hr, this is equivalent to 15-30ug/kg/hr. E: What else could you give? C: I could also give paracetamol and NSAID for their opioid sparing effect. For paracetamol, I would give 40mg/kg per rectal at the time of induction as loading dose, and I would limit the daily dose to 90mg/kg. For NSAID, in fact I wouldnt give it for the concern of major blood loss intraoperatively. E: What other methods of analgesia are you aware of? C: Other methods include PCA morphine, nurse controlled analgesia, and the use of regional technique. For PCA morphine, it is often administered to child above aged of 5. For this child, it might not be a good choice. E: Do you mean he is not suitable to use PCA? C: Not really, but then special assessment is required in the preoperative period. In general speaking, it is not a good choice for child below aged of 5. E: What else? C: For nurse controlled analgesia, it is not efficient at all.

10 Years Collection of ANZCA Part II Anaesthetic Viva

MK Yuen & SW Ku

10 Years viva collection 2009

126

For regional technique, these include caudal block with catheter insertion and thoracic epidural, but both are not commonly used in our institution. In fact, I have no experience in putting in an epidural in a child. E: This child has an uneventful operation, and extubated in icu. Half an hour later, the icu nurse informs you that the child develops noisy breathing, what would be your response? C: This is indicative of upper airway obstruction and is a potential crisis situation. I would attend the patient immediately, call for help and the difficult intubation trolley. I would support the airway with triple maneuver, administer 100% oxygen, and assist the ventilation by gently squeezing the ambu bag. At the same time, I would check for the conscious state of the child, and feel the pulse and check the circulation. My ddx included secretion, swelling, aspiration, oversedation and residual muscle block. I would gently apply suction to the pharynx, check for residual block using DBS, and review the drug chart. I would prepare for reintubation if the child remains unresponsive, with poor respiratory effort. E: How would you do the intubation? C: This is a potential difficult intubation because of the oedema at the larynx. I would call for difficult intubation trolley, have a skillful assistant, and have the ENT surgeon standby. If previous intubation is not difficult, I would give a bolus of propofol 4mg/kg and intubate without muscle relaxant. E: You successfully intubate the child, and you could suck out pink frothy sputum from the ETT, what would be your diagnosis? C: This is most likely a case of negative pressure pulmonary oedema. E: Yes, I agree, what would be your management? E: What could be the other ddx? C: Cardiogenic pulmonary oedema. The child could have cardiomyopathy from the chemotherapy, and develop cardiac failure and arrthymia in the perioperative period. E: What sort of chemotherapy are you referring to? C: Drugs such as adrinomycin, bleomycin. E: OK, what else could it be due to? C: Non-cardiogenic pulmonary oedema, due to aspiration, chest infection, transfusion reaction. E: What else? C: . BELL

10 Years Collection of ANZCA Part II Anaesthetic Viva

MK Yuen & SW Ku

10 Years viva collection 2009

127

Scenario 8
A 45 year old male was found naked on the road with multiple fractures. He has fallen 6 metres from a balcony. He has a fractured right femur and tibia and his BP is 85/40 with a heart rate of 115 per minute. He has been given 2000mls of normal saline. CT scan shows cervical spine fractures. His temperature is 34.9 C and his Glasgow Coma Score is 13. What is the differential diagnosis of the hypotension? E: What would be the cause of hypotension? C: I would ensure that this patient has been accessed according to ATLS guideline. Cause of hypotension could be divided into hypovolaemic, cardiogenic, obstructive and distributive. Hypovolaemic due to bleeding into the thorax, abdomen. Fractured tibia could bleed up to 1L. Cardiogenic due to myocardial contusion, arrhythmia. Obstructive tension pneumothorax, cardiac tamponade, embolism of fat Distributive spinal shock due to cervical cord injury E: Good. How are you going to distinguish one from another? C: I would ensure the airway is patent with cervical cord protection, administer 100% oxygen. I would then start simultaneous assessment and management. The most likely cause would be hypovolaemia due to bleeding. I would resuscitate with fluid and arrange for type and screen. I would also apply external pressure to the tibia to stop bleeding. I would investigate the patient with CXR and pelvic Xray. From the CXR, I would look for fracture ribs, pneumothoarx and haemothorax. I would exclude pelvic fracture from the pelvic Xray. To exclude abdominal injury, I would look abdominal distension and tenderness, and ask the surgeon to do a USG or DPL. During the USG, I would ask the surgeon to look at the heart to exclude cardiac tamponade. I would also access for bruises over the sternum, review the ECG for ectopic beat and ST segment change, which might indicative of myocardiac contusion. E: How to diagnose retroperitoneal haematoma? C: It would be difficult. It is suspected if patient gets pelvic fracture or a L1 chance fracture on X-ray. USG or DPL is not diagnostic. Abdomen might or might not be distended. The only way to confirm is CT scan abdomen if the condition of patient allowed. E: How to diagnose thoracic aortic dissection? C: Clinically, patient gets unequal radial pulses. CXR reviews widen mediastinum, with left pleural cap, left haemothorax, left bronchi is depressed, and if patient gets a NG tube, the tube will be displaced to the right.

10 Years Collection of ANZCA Part II Anaesthetic Viva

MK Yuen & SW Ku

10 Years viva collection 2009

128

The diagnosis is confirmed with CT thorax or angiogram. E: This patient gets aortic dissection, how are you going to manage? C: I would continue with my resuscitation, and I would consult the cardiothoracic surgeon for possible surgical intervention. E: What level of blood pressure would you aim at. C: I would like the systolic pressure kept below 100mmHg to prevent further dissection. E: The cardiothoracic surgeons like to operate on this patient, while the neurosurgeon want to fix the cervical spine. Which one do you think should be done first? C: The patient is hypotensive, the dissection should be done first to stop the bleeding. E: Who do you think should make the decision? C: I think this should be made after the discussion involving all the three parties, the two surgeons and the anaesthetists. E: The neurosurgeons insist to operate first. What are you going to do? C: I would explain to them that the patient is unstable, and we should deal with the dissection first. If they insist to operate on the spine, and if the patient becomes unstable and dies during the operation, it wouldnt be very nice for them. E: This patient now goes for thoracic stenting. What would be your concern? C: I have my patient, anaesthetic and surgical concern. For my patient concern, this is a trauma patient with full stomach, and is hypovolaemia from the bleeding. For my anaesthetic concern, this will be a GA in remote area. I would ensure the facility in the radiology suit follow the college guideline. This also involves transport of an unstable patient. I would ensure the patient is monitored during the transport, with appropriate equipment and escorted by qualified person. For my surgical concern, there would a chance of conversion to open procedure. I would ensure cardiopulmonary bypass service is available in this hospital. The procedure is potentially lengthy, with possible complication of rupture and bleeding. I would prepare the area as for major blood transfusion, with warming derives and rapid infusion system available. E: How are you going to anaesthetize this patient. C: I would check my theater, equipment and drug. I would ensure I have an experience assistant, and preferably another anaesthetist to help. I would place on my standard monitoring, plus an arterial line and a CVP line. I would secure large bore iv access, and I would induce this patient with rsi and cricoid pressure with manual-in-line stabilization. I would use Etomidate and Suxmethonium for induction, I would blunt the hypertensive response at the time of intubation, and correct hypotension if any after the induction. I would maintain anaesthesia with Fentanyl, O2/Air/Isoflurane. I would also monitor body temp and urine output.

10 Years Collection of ANZCA Part II Anaesthetic Viva

MK Yuen & SW Ku

10 Years viva collection 2009

129

E: The radiologists failed the procedure, they create more dissection, and now the patient goes for an operation. What sort of monitor are you going to use? C: Apart from standard monitoring of circulaton and ventilation, I would monitor the patient with 5-lead ECG for ischemia, arterial line for continuous BP monitoring and blood taking, CVP for fluid states, urine output, body temperature and neuromuscular function with nerve stimulator. E: On which arm do you want to place your arterial line? C: On the right arm, as sometimes the aortic clamp is applied proximal to the left subclavian artery. Monitoring of right arm allows us to monitor the cerebral perfusion. E: The temperature of this patient is 34.9oC, what do you think? C: This is moderate hypothermia, I would like to maintain the temperature above 35oC. I would increase the theater temperature to 25oC, warm up all the fluid using countercurrent fluid warmer, cover the patient with blanket and force air mattress, use of HME to humidify the inspiratory gas. E: Your colleague said hypothermia is protective to the brain, what do you think? C: Mild hypothermia is equally protective as moderate hypothermia. I would like to keep the temperature above 35oC, so that I could have the advantages of hypothermia while I could avoid the adverse effect. The adverse effect included reduced myocardial contractility, shift of ODC to the left and reduce oxygen carrying capacity of the blood, coagulopathy with increased blood loss and need for transfusion, impaired glucose tolerance, cold diuresis with electrolyte disturbance and dehydration. In postoperative period, there would be delay awakening, poor wound healing, shivering with increased MRO2. E: The surgery involved major blood loss, and the surgeons complain to you about blood oozing everywhere, what are you going to do? C: Since this patient has major blood loss and receive massive transfusion, the bleeding could be secondary to dilutional coagulopathy or disseminated intravascular coagulopathy. I would ensure the patient is adequately ventilated and oxygenated, I would then check the blood for platelet, PT, APTT, fibrinogen and D-dimer level. I would correct thrombocytopenia with platelet transfusion, deranged clotting profile with FFP and cryoprecipitate. E: Have you heard of recombinent factor VII and VIII? C: Y.Yes. this is genetically engineered product free of infections risk, used in the replacement of specific clotting factor deficiency. This is all I know. E: It is OK. The surgeons want to give heparin, what would be your response? C: Why???? I wouldnt allow them to do so as the patient is coagulopathic and is bleeding actively. (I think he wants to ask about the use of heparin in DIC) Extra Question E: In the postoperative period, this patient complains to you of hearing something during the operation. What are you going to do? E: What are you going to tell him when he comes in next time for another operation?

10 Years Collection of ANZCA Part II Anaesthetic Viva

MK Yuen & SW Ku

10 Years viva collection 2009

130

E: 1 year later, this patient comes in for laparoscopic cholecystectomy. After induction, the SBP drops to 60mmHg. What would be the cause specific for this patient?

Scenario 9
You are working in a private metropolitan hospital for the first time. What are the minimum facilities you require for safe anaesthetic practice and how would you determine that the facilities are available? E: What would be the minimal requirement for safe practice of anaesthesia? C: This includes facility, equipment and staff E: How do you know about this? C: From the college guideline. E: Where could you obtain the college guideline? C: From the website, the bulletin, and by post. E: What would you do when you receive the college guideline? C: !!Read it E: (Smiles) At the end of the day when you leave the hospital, you are called back to manage a patient who is arrested intraoperatively. This is a 42 years old lady undergoing laparoscopic cholecystectomy. What do you think would be the most likely cause? C: In a young presumably healthy lady, I think the most likely cause is anaesthetic related incidence such as anaphylatic reaction, arrhythmia, hypoxia due to loss of airway. E: What would be the incidence of anaphylatic reaction? C: The incidence varies from 1 in 7000 to 1 in 50,000. E: What else specially related to this lady? C: Surgical cause such as bleeding and gas embolism. E: Why? C: The surgeons could puncture the blood vessel when putting in the trocer. At the time of gas insufflation, the gas could also enter the blood vessel and cause gas embolism. E: Do you know the incidence of gas embolism during laparoscopy? C: I dont know the exact incidence, but it is not common. E: What else? C: Patient cause. Patient with gallstone could be obese. I am worried of co-morbidity such as coronary artery disease, OSA, pulmonary hypertension. E: Yes, what else? C: I couldnt think of any more. E: OK. When you arrive at the theater, you see the list anaesthetist standing and hand

10 Years Collection of ANZCA Part II Anaesthetic Viva

MK Yuen & SW Ku

10 Years viva collection 2009

131

ventilating the patient. He stares at the monitor and looks blank. The patient is in a reverse trendelenburg position, running a heart rate of 120/min. Blood pressure and SpO2 is unrecordable. What are you going to do? C: There are two concerns, management of patient, and management of staff. I would take over the theater and. E: This is your 1st day in this hospital. C: (???) I would start simultaneous assessment and management. This patient required immediate stabilization. I would call for help, call for the resuscitation trolley, inform the surgeon, put the patient back to supine position. I would ensure the airway is patient, increase FiO2 to 1.0, feel for the pulse and check the circulation. I would review the anaesthetic record, and try to find out what has happened by speaking to the list anaesthelist and other staff. At the same time I would access his functional capacity. If he could not function, then I would take over the theater. E: How do you access the functional capacity of your colleague? C: By speaking to him. If he has no idea of what he is doing, it would be unlikely for him to have the capacity to take care of his patient. E: Yes. Alright, you try to feel for the pulse but you couldnt, what are you going to do? C: This is EMD. I would start external cardiac message, support the circulation by rapid infusion of crystalloid and iv adrenaline 1mg every 3 minutes. I would then look for the underlying cause. My ddx include pneumothorax, cardiac tamponade, hypovolaemia, acidosis and hyperkalaemia. For pneumothorax, I would feel for deviated trachea, percuss for hyperresonance, and listen the chest for asymmetrical air entry. I would check for dilated neck vein and increase JVP for cardiac tamponade. I would then check for blood loss, and skin rashes for anaphylaxis. Finally I would check blood for pH and electrolyte. E: Yes, what else? C: I would apply extra monitoring includes arterial line. E: I mean what other causes should you look for? C: I couldnt think of others. E: OK, this patient has gas embolism, do you know how to make a clinical diagnosis? C: Hypotension, with increase JVP, there is also a sudden drop in ETCO2. E: What would you listen for with stethoscope? C: A mill-wheel murmur. E: What are you going to do with your colleague? C: This would be a very traumatic experience to every staff. For my colleague, I would talk to him, try to find out whether he needs any help. If he is very distress, I would suggest him to seek advice from psychiatrist. E: He refuses to receive your help. What are you going to do?

10 Years Collection of ANZCA Part II Anaesthetic Viva

MK Yuen & SW Ku

10 Years viva collection 2009

132

C: This is understandable as I am a stranger to him, but I would talk to his friends, so that they could take care of him. I would also inform the hospital manager, so that he could do the follow-up. E: A nurse in the theater tells you that this is not his first time. What would be your response? C: I would speak to this nurse to find out the details. If this is the fact, I would inform the hospital manager. E: Do you think it is a matter to you? You only work in this hospital for this time and you wouldnt come back for next few years. C: Yes, I have the responsibility to ensure our patient receive proper care. If I suspect my colleague is not up to standard, I have the responsibility to raise out this concern. E: What else could you do? C: I would report to the Medical Board. BELL

10 Years Collection of ANZCA Part II Anaesthetic Viva

MK Yuen & SW Ku

10 Years viva collection 2009

133

ANZCA Anaesthetic Viva May 2003 Day 2 Dr Vivian Yuen, QMH


Scenario 1
A 40 year old woman is scheduled for laparoscopic hysterectomy. She has moderately severe asthma and weighs 120 kg. 45 minutes into the case you notice the End Tidal CO2 alarming at 60 mmHg (~ 7%). What is your differential diagnosis of this hypercarbia? E: What are the possible causes? C: causes incl hypoventilation, hypermetabolic state, circuit malfunction with rebreathing, early CO2 emblism, Hypoventilation: lap surgery increase intraabdominal pressure and trendelenberg position, decrease FRC, obesity with decreased chest wall compliance, CO2 absorption for abdo insufflation Hypermetabolic state: fever from sepsis, iatrogenic over warming, hyperthyroidism, MH Malfunciton of circle circuit: rebreathing from exhausted soda lime, insp/exp valve problem, Emobolism: early present with increase ETCO2, severe and late with decrease CO, ETCO2 will decrease Asthma with inadequate expiration, bronchospasm E: Can you draw me capnograph of rebreathing E: what happen if valve malfunction E: (Show me diagram of circle system) tell me why if exp valve malfunction can cause increase ETCO2 C: if it gets stuck and close.. E: No, It is stick open. C: OK, then there is rebreathing of dead space gas before the soda lime. E: Good. Now if you figure out the problem of rebreathing is from malfunction of expiratory valve, it takes time to fix the machine, how can you fix the problem? C: I can continue ventilation with a separate breathing and continue anaes with intravenous agent. E: If you are allow to use the machine, what circuit would you use? C: I would use Mapleson A or Mapleson D. Preferably Maplesone D as it is more efficient with control ventilation. E: Say now the patient desaturate from 98% to 85%, can this be due to rise CO2?

10 Years Collection of ANZCA Part II Anaesthetic Viva

MK Yuen & SW Ku

10 Years viva collection 2009

134

C: No E: Why? C: (stuck) E: Any equation? C: Yes the alveolar equation, . Hence the rise of PaCO2 would not explain this extent of this desaturation. E: What can be the cause? C: pt cause, eqt cause and ETT cuase. Pt: asthma, gas embolism, .(I forgot what I have said, but he stopped me before I complete ask me another Q) E: How would you find out the cause? C: I would do simultaneous resuscitation, call for help, inform the surgeon and deflate the abdo, increase FiO2 to 100%, disconnect from ventilator and hand ventilate, at the same time look at the patient and the monitor, check ECG rhythm heart rate BP capnograph, and feel for the compliance as I ventilate, look at chest movement, check the circuit to ensure no kinking/obstruction, assucltate for equal AE, for wheeze, for crep, suction the ETT tube to check patency..(then he stop me before I have finished) E: Can you draw me Mapleson D? E: Can you draw me Mapleson A? E: How do you check the function of exp and insp valve in circle system? BELL

Scenario 2
An 82 year old patient is scheduled for a right above knee amputation in 4 days time. The patient: had a right below knee amputation 6 months ago has ischaemic heart disease and had a coronary angioplasty and stent 4 months ago is currently taking aspirin; clopidogrel was ceased 2 days ago. He has been complaining of very severe pain in his right leg and the surgeon calls you and asks you to advise about pain management. E: What is your response? C: I would go and assess the patient by history, physical examination and investigation. I would like to find out if this is neuropathic pain such as phatom limb pain/stump pain or nocieptive pain such as pain from wound breakdown, infection. E: The patient said she has pain in her right toe, crushing pain.

10 Years Collection of ANZCA Part II Anaesthetic Viva

MK Yuen & SW Ku

10 Years viva collection 2009

135

I would like to know the severity of the pain E: Its 9/10 C: How about duration, is it there all the time, comes and goes, any ppt factors? E: The pain started since 1 month ago. It comes and go occurs about 12 times per day. E: is the sleep and appetite affected? I would also ask if the patient feeling down about the pain. E; The patient said she is not depressed, just fix up the pain. C: Is anything prescribed for the pain, is it helpful? E: The surgeon has prescribed slow release morphine 20mg BD and has no effect. So would you like to give any drugs C: I would use multimodal therapy. Morphine is not a good agent for neurpathic pain. I would give her TCA, start with amitriptyline 10mg nocte and titre to response. I would also start by giving ketamine im E: what dose of ketamine would you use? C: She is a very elderly pain, I would use 5mg to start with Q2H prn review and titrate to clinical response. E What else would you do? C: I would stop the morphine and replace with regular methadone as its NMDA antagonist effect would help with neuropathic pain. E: What is preemptive analgesia? C: administration of analgesia prior to introduction of pain or surfery reduce the chance of development from chronic pain state. This is due to reduction of the wind up phenomenon and central sensitization at the dorsal horn of the pain pathway. E: Is there any evidence support preemptive analgesia? C: evidence so far are conflicting, no definite proof of its effectiveness. E: Can you state some example of method of preemptive analgesic that has been studied so far? C: Yes, use of neuroaxial block few days prior to amputation reduce incidence of phantom limb pain, use of LA infiltration on abdominal wound, use of NSAID E: Would you do neuroaxial block on this patient for pain relief? C: no. she is on antiplatelet agent clopdogrel and aspirin. E: What is clopidogrel. C: It works on ADP receptor of platelet membrane. E: How long does it last? C; Ten days E: Would you do regional block. C: Yes, femoral or sciatic nerve E: Why would you do RA but not epidural

10 Years Collection of ANZCA Part II Anaesthetic Viva

MK Yuen & SW Ku

10 Years viva collection 2009

136

C: Because with peripheral nerve block, even if I cause bleeding, I can apply direct compression, and the effect is not so detrimental comparing to epidural heamatoma or intrathecal haematom. E: Now the pt comes for anaesthesia, she has bad experience with epidural block for BKA, she does not want GA, how would you anaesthetize this lady? C: It is difficult. I would tell her RA is possible, however it may not be adequate, she may need GA or sedation for the procedure. I would perform femoral nerve block with sciatica nerve block. E: What approach would you use for sciatic nerve block. C: I do classic approach ie Labat approach. E: Is there any alternative C: The alternative is ant approach, how it is more difficult and cause more pain. I can also so subgluteal approach. E: What drug would you use? C: I would use max 3mg/kg ropivacaine, she is 70kg, the max dose is 210mg. E: What conc and how much? C: I use 0.375% .. BELL

Scenario 3
A 75 year old male presents with recurrent atrial fibrillation for cardioversion. He has ischaemic cardiomyopathy and has had 2 days of diarrhoea. He is on multiple cardiac drugs. Following cardioversion he has burns from the defibrillation pads. When assessing this patient's readiness for cardioversion, what factors would you be taking into consideration in relation to the atrial fibrillation recurring? y y How do you prepare the patient for cardioversion? When do you stop anticoagulation therapy prior to cardioversion?

(This question doesnt make sense, as I have said to ensure adequate level of anticoag prior to cardioversion, he still asked me when to stop warfarin) y y y y What INR would you keep? What energy level would you use? How do you synchronise cardioversion? Which part of the ECG complex do you synchronise with? Any other defribillator you know of which require less energy level? Yes, bipolar

10 Years Collection of ANZCA Part II Anaesthetic Viva

MK Yuen & SW Ku

10 Years viva collection 2009

137

Show me an ECG, what abnormalities do you see? Any sign of digioxin toxicity?

(sinus rhythm, prolong QT interval, no ST-T abnormalities, I can see no evidence of digioxin toxicity) y y y y Show me another ECG, in AF rate between 50-80, reverse tick appearance consistent with dig effect. What drugs cause prolong QT interval? Do you think this patient is taking these drugs? Are you sure its amiodarone? Droperidol, some nonsedating antihistamine, No, antiarthymic, like amiodarone. AhYes (Just to show you some examiner can be nasty, even if I said the right answer he tried to trick me) y y Okay what other antiarthymic agent? Show me echo report of these patient, what information can you derives from these echo report? Dilated cardiomyopathy.. y y y What is shortening fraction Show me another echo report of another patient, probably with HOCM What abnormalities do you see with posterior myocardial infarct? Class Ia and sotolol

R wave on V1.

Scenario 4
A 65 year old male presents for inguinal hernia repair as a day stay patient. He has had an episode of incarceration with obstructive symptoms that resolved when his GP reduced his hernia. He has a history of Type 2 diabetes. His blood glucose is 15mmol/l on arrival. His medications are: Metformin 500mg tds Glibenclamide 5mg tds

y y

How to assess if he is suitable for day surgery? His wife can look after him throughout the perioperative period, his place is only 15 mins drive away from the hospital. His BP is well controlled, his HbA1c is 7%. He skipped his oral hypoglycaemic agent since yesterday because he misunderstood the information

10 Years Collection of ANZCA Part II Anaesthetic Viva

MK Yuen & SW Ku

10 Years viva collection 2009

138

given from nursing staff. Would you do him as day case? y y y y y y What information can you derive from HbA1c? How would you control his blood glucose? What level of bllod glucose would you keep? In what kind of surgery would you carry out tighter control of blood glucose and why? Postoperatively the patient is confused, how would you management Show me an ECG, acute anteroseptal infarct, what is your management?

Scenario 5
You are called to see a 32 year old woman in the outpatient clinic of the obstetric hospital. She is 16 weeks pregnant and has a history of Rheumatic Fever with known mitral valve disease. She has been asymptomatic except for one episode of atrial tachycardia four years ago which was associated with shortness of breath. She would like to see an anaesthetist to discuss the options for managing labour and any risks from anaesthesia for possible caesarean section. What are the main issues you will address in this consultation? y y y y y y y y y y y y How do you assess the patient when you see her in preanaesthetic clinic? She only have mild MS with pressure gradient ?14mmHg, on no medication. She is concern about peripartum pain control, what would you tell her? She is now 24/40, admitted to emergency dept in rapid AF 120/min, BP 80/50, what is your management? How do you anaes this patient for cardioversion? The cardiologoist said he could perform cardioversion on lateral position, can you avoid intubation if is in late position? The cardiologist asked if you can do this without intubtaion if patient is in left lateral and head down position? How does head down position affect her pulmonary circulation? You have successfully cardiovert this patient. She is now 30/40 and is in labor, she is on enoxaparin, the obs ask you to help in pain relief, what would you offer? How do you set you PCA fentanyl? Her last dose of enoxaparin was 12 hours ago, still in pain with PCA fentanyl, what else can you offer? Her BP is in fact high 140/90. What drug would you give intrathecally?

10 Years Collection of ANZCA Part II Anaesthetic Viva

MK Yuen & SW Ku

10 Years viva collection 2009

139

Scenario 6
A 7 year old child is rescued from a house fire. You have been called down to the Emergency Department to assist with management of this child. He has trunk and upper arm burns. How do you assess this child's airway? y How do you assess this patient?

I would assess this patient according the ATLS guideline, do a primary survey follow by a secondary survey. I would ensure his airway is patent, protective reflexes are intact, look for evidence of upper airway burn or inhalation injuries that would indicate early intubation. I would support his breathing with 100% oxygen, insert two large bore iv and start fluid resuscitation. I would give fluid according to the Parklands formulae, ie4ml/kg/% BSA over a period of 24 hours starting from the time of burn injuries. If patient is stabilized I would also ensure adequate pain relief. y Okay, thats good, we talk about the pain relief later. If his pulse oximtry reading is 98%, would that be reliable? Why would it be not reliable? y y y y y y y y What other substance can be inhaled during household fire? What is the difference in determining the %BSA burn in paediatric and adult patient? How do you give pain relief? How do you set you PCA morphine? How to you monitor patient if he is on PCA? Why would you monitor sedation score? Is resp rate a reliable sign of over sedation with morphine? Now patient is day 1 in ICU, you were called to see the patient because he is desaturated to 85% on 2l/min O2, tachypnoeic, what is you management and ddx. I would see the patient immediately see the patient, enures a patent airway and give 100% oxygen. The cause can be related to patient and drugs. The patient is on morphine PCA. y Would morphine cause tachypnoeic? No, it should not. The other causes lower respiratory tract inahalation injury which usually manifest 24-48 hours later. y y y When would you intubate this patient? Patient is stablilised, extubated on day 3, need change of dressing, what analgesic would you offer for dressing change? What is the dose of Ketamine? I would intubate him early as his condition is likely to deteriorate rather than improve.

Cyanide

10 Years Collection of ANZCA Part II Anaesthetic Viva

MK Yuen & SW Ku

10 Years viva collection 2009

140

Scenario 7
You are asked to assist in the retrieval of a 25 year-old male from a small country hospital. He ran his motorcycle at an estimated speed of 80 km per hour into the back of a car. The referring hospital summary is as follows; GCS 15 Neck and right shoulder pain Fractured 3rd, 4th, and 5th ribs on the left side A small pneumothorax on the left Penetrating left eye injury Facial fractures Fractured left pubic ramus Compound fracture of left femur Haemodynamically stable Able to maintain clear airway Oxygen saturation 97% on 30% oxygen Describe what you would do on arrival at the country hospital? y You are responsible for the retrieval of this patient by ambulance. What do you do when you arrive? I would perform primary survey according to the ATLS guideline. y The other emergency doctor has performed a primary survey already, would you do it again? Yes, as it does not take me long to perform a primary survey. y How to prepare for the transfer? I would prepare equipment, monitor, iv infusion device, fluid and warming device, x-mated blood and resuscitation drugs. y y y y y y y y y y What warming device can you use in ambulance? What monitor? You said invasive blood pressure monitor, would you do it on this patient? What else would you bring? How to prepare he patient? What else would you do prior to transfer? I would also ensure the # left femur is reduced and bleeding is controlled prior to transfer. The patient desaturated to 80% in ambulance, BP 80/50, what is your management? What is your ddx? Do you think you can hear with you stethoscope when the ambulance is moving?

I would take equipment for intercostals chest drain and pericardiocentesis I would ensure his pneumothorax is drained with a CXR performed after ICC inserted.

10 Years Collection of ANZCA Part II Anaesthetic Viva

MK Yuen & SW Ku

10 Years viva collection 2009

141

y y y

So what would you do? I think I can observe for chest movement and palpate. Would you ask the ambulance to stop?

Scenario 8
You are called to the Emergency Department to assess a patient who has been transferred from a nearby freestanding Day Surgery Unit because he was impossible to intubate and there was no help/equipment available. He now has an LMA in place with partial airway obstruction and saturations in the low 90s. The anaesthetist has accompanied him to your hospital. Discuss your initial assessment and management of this situation. y What is your initial response?

I would talk to the anaesthetist to find out more information about the patient. I would like to know the drugs given to the patient. I would assess the patients conscious state. Meanwhile continue support patients airway, oxygenation and ventilation with the laryngeal mask. y The patient is a 70 year old man with know ca of base of tongue awaiting resection. He was admitted to the day surgery centre for dental procedure prior to elective glossectomy. The anaesthetist gave the paitent thiopentone and suxmethonium, and unable to intubate patient with several attempts. He put in the LMA and transfer the patient here. y y What is your management? He is unconscious, not breathing. I would assess the conscious state and breathing effort of the patient I cant to support his airway and breathing with the LMA, ask for difficult intubation trolley, and ENT surgeon to perform a tracheostomy as intubation would be impossible. I would try to railroad a size # 6 ETT thru the LMA. y There is no ENT surgeon, you cannot intubate via the LMA, the patient now desaturated to 80%, what would you do? I would do a cricothyroidotmy with a 14G canula ventilate the patient with Sanders jet thru the 14G canula. y y y y y You notice as you jet, the patient cannot exhale, what would you do. What are the complications of jet ventilation? What else would you do? Do you think the use of LMA is appropriate in this situation? The patient is now in ICU, still does not wake up, what is your management? I would remove the LMA and do chin lift and jaw thrust to achieve patent airway for exhalation.

I would also perform a tracheostomy to achieve a definitive airway..

10 Years Collection of ANZCA Part II Anaesthetic Viva

MK Yuen & SW Ku

10 Years viva collection 2009

142

Ensure ABC, ensure muscle relaxant is fully reversed and check with peripheral nerve stimulator. Check electrolytes and blood glucose, temp, y The patient had thiopentone and sux, can these drugs cause delay awakeing? Patient with atypical acetycholinestase can have prolong action of sux and I would be able to spot this diagnosis with my PNS. y What would you see with PNS if patient is still paralysed from sux?

Scenario 9
A 60 year old woman with long standing rheumatoid arthritis presents for a revision left total hip replacement. Her medications are: Methotrexate Sulfasalzine Folic acid Celecoxib

Hydroxychloroquine Irbesartan Your first contact with this woman is on the day of surgery in the same day admission suite. What are your concerns in relation to provision of anaesthesia for this procedure in this patient? y y y y y y y y y y y y y How do you assess the patient? What cardiovascular complication can this patient have? What blood test would you do? How to assess the stability of her cervical spine? Apart from lateral c-spine x-ray, what other view can you ask for to assess stability of cervical spine. She has unstable c-spine, is it safe to perform c-spine xray with neck flexion? What else can you see from c-spine film? What problem would you anticipate if patient has cricoarytenoid calcification/dislocation? How would you anaesthetize this paitient? How to protect her c-spine intraoperatively? Is soft collar adequate? SpO2 fell to 85% intraop, what is your management? What are the ddx?

10 Years Collection of ANZCA Part II Anaesthetic Viva

MK Yuen & SW Ku

10 Years viva collection 2009

143

ANZCA Anaesthetic Viva September 2003 Day 1 Dr BK Lim, PWH


Scenario 1
An intoxicated 28 years old male is brought to the emergency room following a fight in which he was hit across the head with chair. He has a large haematoma over the left frontal and temporal regions. There are no other obvious injuries. A cervical collar is in place. He is aggressive, abusive, confused and un-cooperative. Blood pressure Heart Rate 160/90 mmHg 120 beats/minute

An urgent CT head scan is deemed necessary. You are asked to help sedate the patient for transport to the x-ray department one floor above the emergency room. How would you proceed?

Scenario 2
You are asked to assess a 65 year old male at the pre-anaesthetic clinic. He has duodenal carcinoma. He was told three years previously at a cardiology review that he had inoperable coronary artery disease. Detail your initial assessment and investigation.

10 Years Collection of ANZCA Part II Anaesthetic Viva

MK Yuen & SW Ku

10 Years viva collection 2009

144

Scenario 3
After starting your next elective case you are called by PACU (Post Anaesthesia Care Unit) to review your previous case, a right upper lobe lobectomy because of the following changes: SpO2 HR Systolic BP Respiratory Rate 96% 88% 85/min 110/min 110/ 80/ mmHg 16 22/min

Anaesthetic: General Anaesthetic with intercostals nerve blocks and would infiltration with local anaesthetic. PCA morphine for postoperative analgesia. Past history: A 68 year old male, ex-smoker with a non-small cell carcinoma. He also has controlled hypertension. Medication: Atenolol What are your priorities? y y what is your priorities in Mx later found the chest drain was clamped

Scenario 4
A 70kg male, aged 58 years is booked for an exploratory laparotomy for trauma to his liver and spleen. He was involved in a high speed motor vehicle accident approximately 2 hours ago. His injuries include a crash injury to lower thorax, left thigh and pelvis with a compound fracture of his left femur. He has a haemo-pneumothorax. Describe your strategy for the management of his intravenous fluids. y y y y y y what is minimal / limited volume resucitation Mx of IVF what kind of solution you used and difference among them what monitoring needed +/- why (pre-intra-post op) effects of blood transfusion later shows ECG - prolong QT (probably due to HypoCa++; and Rx)

10 Years Collection of ANZCA Part II Anaesthetic Viva

MK Yuen & SW Ku

10 Years viva collection 2009

145

Scenario 5
A 6 year old girl presents for major reconstructive surgery on her congenitally deformed right hand, involving a toe to hand transfer. The surgeon requests a brachial plexus catheter to maximize post-operative blood flow. What approach to the brachial plexus block would you choose? Why? y y y y y what approach of BPB you choose + reason + technique Complication arise immediately after the BPB which is done by your registrar and management how long will you resusucitate / end point of CPR surgeon want to continue the OT - what is your response post-event management convulsion for 15min and successfully controlled

Scenario 6
A 30 year old woman is booked on the following days operating list for construction of a vascularised flap to exposed bone in her arm under general anaesthesia (see photo). She suffered burns to 50% of her body 4 months earlier. What are your options in managing her airway? (Picture provided and showed mircostomia + gross neck contracture) y y y problems of airway and sux refuse awake FOB airway management for the OT

10 Years Collection of ANZCA Part II Anaesthetic Viva

MK Yuen & SW Ku

10 Years viva collection 2009

146

Scenario 7
You are the supervising consultant on duty in the operating suite. You are called by a registrar for help. The patient, who is a 55 year old man having bilateral varicose veins stripped, is becoming hypoxic. When you arrive, you notice the registrar concentrating on the monitor, with his back to the patient, and you see that there is yellow fluid in the tube of the laryngeal mask airway (LMA). What is your first action? y y y surgeon complained bluish/dark colored blood from ?arterial bleeding how do you justify for LMA in obese as relative ask what will your tell the relative, with whom will you see together with and will you bring with the trainee

now on 33% O2 SpO2=99% standard practice

Scenario 8
The patient is 38 years old and at term with her first pregnancy. Her blood pressure has become elevated towards term and she has been treated with labetolol for 2 weeks by her obstetrician. Her obstetrician decides to induce her labour. You are asked to insert an epidural for blood pressure control and analgesia during labour. The patient consents to lumbar epidural anaesthesia and you determine there are no contraindications in this patient. Is epidural analgesia useful for blood pressure control in this patient? y y y is EA good for EA control what is your mx for elevated of BP- MgSO4, hydralazine, labetolol later convulse, what is your DDx & how do you mx

Scenario 9
An anxious 12 year old boy is scheduled for an above knee amputation for osteosarcoma of the lower left femur. He has had pain above knee for 3 months and has commenced chemotherapy. He and his parents are anxious that his pain be minimized post-operatively and ask your advice. What will your advice be on the management of this post-operative pain?

10 Years Collection of ANZCA Part II Anaesthetic Viva

MK Yuen & SW Ku

10 Years viva collection 2009

147

ANZCA Anaesthetic Viva September 2003 Day 2 Dr Angel Kwok, KWH


Scenario 1
A 65 year old man taking 8 Panadiene Forte tables per day is scheduled for posterolateral thoracotomy for carcinoma of the lung. He wants to discuss his pain control. Why is good pain management important in this man? E: What is the importance of postoperative pain control in this gentleman? E: Describe the change in FRC, inspiratory reserve vol and expiratory reserve vol after thoracotomy ? Illustrate by drawing the vol-time curve. E: What are the options for pain control for him postoperatively ? E: How do you consider his codeine dose in the PCA regimen? Tell me your PCA dosage. E: How much you give for your EA infusion ? E: Is cryotherapy useful for post-thoracotomy pain ? What is the onset and duration of action of cryotherapy ? How is it done ? E: How about intrapleural analgesia ? E: Few months postop, the patient come back and complaining of severe pain over the thoracotomy site, How would you manage him ? E: You think that he is suffering from neuropathic pain, How can you manage his pain ? E: What is the mechanism of action of TCA, carbamezapine, garbapentin ? E: What is meant by NNT ? What is the NNT for the above mentioned drugs ?

Scenario 2
A 50 year old woman with a history of left mastectomy for breast carcinoma presents with an unsteady gait and nystagmus. She is found to have a midline cerebellar lesion and is scheduled for insertion of an external ventricular drain, followed by a posterior fossa crainiotomy in the prone position. Past Medical History Left mastectomy Medications tamoxifen 20mg daily, dexamethasone 4mg 4 times daily What are the potential problems for the prone position for this patient?

10 Years Collection of ANZCA Part II Anaesthetic Viva

MK Yuen & SW Ku

10 Years viva collection 2009

148

Scenario 3
You are in the pre-anaesthetic clinic and are assessing a 70 year old man with a history of newly diagnosed non-obstructive bowel cancer. He has been scheduled for a left hemicolectomy. From your history you note that his a smoker and has smoked over 1 pack per day for more than 40 years. He claimed he gets mildly breathless after walking 100-200 meters. You observed that he appeared mildly breathless when he entered the room. He has been prescribed tiotopium (Spiriva) b.d., salmeterol/fluticasone (Serotide) b.d. and low dose aspirin (100mg/d) by his GP How would you process with his evaluation? E: How would you assess this patient? E: What will you look for in his preop ECG? E: What advice will you give him concerning his smoking? E: What is your anaesthetic plan? (GA+EA) E: On the day of operation, the anaesthetist responsible for the case refuse to insert a EA. What will you do? E: Eventually, you can find another colleague who is willing to insert an EA, but he is only available to insert the EA at the end of the operation. What is your management? E: What is your opinion concerning the insertion of EA when patient is awake or under GA? E: You put in the EA at the beginning of the operation and continue the EA for postop pain control. In the RR, the SaO2 and BP of the patient drops. How would you manage him?

Scenario 4
You are rung about a 72 year old man who had a fractured neck of femur repaired with pin and plate under General Anaesthesia. He is in the recovery room, and is sweaty, pulse of 120 beats/min and restless. The recovery nurse is requesting sedation to control him. What is your initial management of this situation? E: How would you manage this patient ? E: After you review the anaesthetic record, you discover that the patient is suffering from chronic alcoholism. What do you think he is suffering from ? E: How would you manage his withdrawal symptoms in the RR? E :How would you manage his pain ? E: Are there any classification you know of to assess the severity of liver cirrhosis ? What

10 Years Collection of ANZCA Part II Anaesthetic Viva

MK Yuen & SW Ku

10 Years viva collection 2009

149

parameters are they looking at ? E: Given a set of lab results, What is the grading ? How can it predict the outcome after operation ? E: What is the pathophysiological change in different organs in chronic alcoholism ? E: What is the common cause of death in chronic alcoholism ? E: What is the plasma concentration in a chronic alcoholic ? E: How would you anaesthetize a chronic alcoholic patient for DHS? Are there any drugs you would give before the operation ? E: Okay, Have a nice drink

Scenario 5
You are anaesthetizing a young male for exploration of 3 stab wounds to the right chest and neck following an assault. At your initial assessment you have also noted bruising on the face, chest, abdomen and thighs. He is otherwise fit with no previous hospital admissions. In the emergency department a right pneumothrax was noted on the chest x-ray and an intercostals catheter was placed at 11:40am. Two litres of normal saline was administered in the emergency department. Arrival 11:30am Pulse rate/mon B.P. Resp. rate/min SpO2% GCS 105 115/75 32 95 12 After ICC 11:45am 115 105/65 28 98 14 Pre-induction 12:30pm 110 125/85 28 97 14 After induction 12:45pm 135 70/40 10 93

What actions will you take to diagnose the situation after induction? E: How would you manage this patient ? E: BP not responding with your fluid resuitation and phenylephrine infusion. You notice that this patient had distended neck veins. What do you think he is suffering from ? E: What are signs of cardiac tamponade in our patient ? E What is Kussmauls sign ? E: What are the physiological changes in cardiac tamponade to account for the crash in BP after induction ? E: How would you proceed then ?

10 Years Collection of ANZCA Part II Anaesthetic Viva

MK Yuen & SW Ku

10 Years viva collection 2009

150

E: How is pericardiocentesis done ? E BP improves after pericardiocentesis, but it drops again, What is your management ? E:Surgeon want to perform liver resection as there is laceration noted after laparotomy. Tell me something about Pringles manoeuvre ? E: What is the current guidelines for giving FFP and cryoprecipitate ?

Scenario 6
A 12 year old girl is scheduled for myringoplasty for chronic perforation of her eardrum. She has had one previous anaesthetic for myringotomy and tube drainage six years before. She has no other health problems. She appeared well and calm at pre-operative assessment. In the anaesthetic bay with her mother she refuses to co-operate with induction and says I dont want the operation. What would you do? E: What will you do? E: The mother insist to have the operation done as she is taking leave to bring the child to the day care centre. What will you do? E: When will you re-schedule the operation? E: Okay, lets go to another situation. A 14 year-old suffering from ectopic pregnancy, the obstetricians want to have urgent laparoscopic surgery. The girl refused to have the operation. What will you do? E: After your discussion with her, she agrees to proceed to the operation but does not want her mother to know about this. What will you do? E: She told you she is a Jehovahs witness and refuse blood transfusion under all circumstances. Intraoperatively, the surgeon tear an artery and massive bleeding follows. 4L of blood was lost. What will you do? Will you give any blood for her? E: Her husband want you to give blood so as to save his wife. What will you do?

Scenario 7
A 52 year old male is admitted after colliding his motorcycle with a lamp-post. He has a fractured femur and a fractured humerus and is scheduled for internal fixation of both fractures on this evenings operating list. He is 185cms tall, weighs 120kg, is bearded, heavily tattooed, extremely anxious and keen for a general anaesthetic. What further information do you require?

10 Years Collection of ANZCA Part II Anaesthetic Viva

MK Yuen & SW Ku

10 Years viva collection 2009

151

E: How would you assess this man? E: How do you assesss his airway? E: How predictive is the mallampti score for difficult intubation? E How can you define difficult mask ventilation and difficult intubation? E What is the incidence of difficult mask ventilation and difficult intubation? E: Your junior perform a RSI for this man, fail to intubate after 1st attempt, call you for help. What will you do? E SaO2 drops to 82% and you cant mask ventilate him. What will you do? E: You put in a LMA, SaO2 improves a bit, now is 92%. You noted some whitish sputum coming from the LMA. What will you do? E: ABG results was shown then. E You try to use the FOB to put in a ET tube via the LMA, but you fails to do so. How would you proceed then? E: You decide to postphone the operation and wake the patient up. 2 days later, this man come back, how would you anaesthetize him this time?

Scenario 8
A 50 year old male presents for semi-elective tracheostomy following a period of prolonged oral intubation in ICU. He has suffered a closed head injury and is expected to required ventilation for some time to come. There are no issues to be considered regarding intensive care management of his injury. There is no previous significant surgical or anaesthetic history, and no known allergies. A tracheostomy is planned to be performed in the operating room. How will you prepare for theatre specifically with respect to the airway? E: How would you prepare the equipment for tracheostomy? E: Some smoke was noted after the surgeon cut the trachea, what will you do? E: Where do you place your fire extinguishers in OT? E: What are the contents of the fire extinguishers? E: You document the intraoperative airway fire in the anaesthetic record , the surgeon ask if you can alter the notes before the interview with patients relatives. What will you do? E: The surgeon is blaming you for the fire, as you are giving 100% O2 to pre-oxygenate the patient before the cut. What is your response? were scared and not want to appear in the interview. How can you manage this?

10 Years Collection of ANZCA Part II Anaesthetic Viva

MK Yuen & SW Ku

10 Years viva collection 2009

152

E: Are you going to have the interview with the relatives together with the nurses or surgeon? The nurses E: After the interview with the patients relatives, the surgeon want to sent for the last case on the elective list. What do you think?

Scenario 9
Your are contacted by an obstetric colleague about a 24 year old primigravida who is at 28 weeks gestation and has pregnancy induced hypertension. He things that she will need to be delivered soon. She is not in labour. She has the following test results: Platelet count 75x 109/l Serum bilirubin 1.5gm.dl-1 SGOT 90u/l LDH 1000u/l Haemolytic picture on peripheral blood smear Her blood pressure is 140/100 What concerns you about this patient?

10 Years Collection of ANZCA Part II Anaesthetic Viva

MK Yuen & SW Ku

10 Years viva collection 2009

153

ANZCA Anaesthetic Viva May 2004 Day 1 Scenario only


Scenario 1
Dear Anaesthetist, Thanks for seeing Mr. Patient, aged 60, with rest pain in his legs. He is due to have an aorto-bifemoral graft next week. He has chronic obstructive pulmonary disease (COPD), is treated for hypertension and suffered a small heart attack 10 years ago. His diabetes is well controlled. He weighs 100 kg, his blood pressure today was 140/80 mmHg and his blood sugar level normal. Current medications: Pulmicort Atrovent Enalapril Frusemide Spironolactone Diaformin Yours sincerely, DR. GORTEX GRAFT, FRACS Vascular Surgeon What features do you see as important to elicit in this man's HISTORY to assess his perioperative risk or respiratory complications? Ventolin

Scenario 2
A 58 year old male with a history of chronic liver disease and Childs B cirrhosis presents with melaena and haematemesis. He is now in the endoscopy suite as he requires urgent endoscopy for variceal bleeding. You are asked to provide anaesthesia care. What other aspects of the history and physical examination would you consider important in your anaesthetic assessment?

10 Years Collection of ANZCA Part II Anaesthetic Viva

MK Yuen & SW Ku

10 Years viva collection 2009

154

Scenario 3
PATIENT: 71 year old man for urgent laparotomy for ischemic bowel HISTORY: Hypertension - treated Ischemic heart disease AMI (acute myocardial infarction) 6 months ago AF (atrial fibrillation) longstanding LAST NIGHT: How would you stratify this patient's preoperative cardiac risk? Chest Pain

Scenario 4
You are the retrieval anaesthetist at a building site. A 40 year old male is trapped under a fallen wall of an unstable building. The surgeon wants to amputate the patient's leg. Outline your management of this situation.

Scenario 5
A 19 year old girl has been admitted to the Intensive Care Unit of your hospital in a large provincial town. She has sustained severe head injuries following a fall from a horse. You are the anaesthetist on call for the weekend and are asked to be involved in the certification of brain death. What is your approach?

Scenario 6
An eight year old boy presents with bleeding one week after a tonsillectomy. When you see him in casualty, he is tachypnoeic, pale and cool peripherally, with pulse 135 and BP 80/60. The ENT surgeon wants to take him to theatre to stop the bleeding. Describe your assessment and resuscitation prior to theatre.

10 Years Collection of ANZCA Part II Anaesthetic Viva

MK Yuen & SW Ku

10 Years viva collection 2009

155

Scenario 7
You are consulted about the timing of surgery for a woman who is requesting a post partum sterilisation. She has had a vaginal delivery during the night. What is your advice?

Scenario 8
You are contacted by the neurosurgeon about a 60 year old woman who has a sub-arachnoid haemorrhage due to a leaking cerebral aneurysm, but who also has inoperable ischemic heart disease. She is planning to "coil" (GDC) the aneurysm under general anaesthesia in the radiology procedure room, but is concerned about the risks that her patient is facing. What specific risks and benefits of "coiling" the aneurysm under general anaesthesia will you discuss when obtaining informed consent from the patient?

Scenario 9
A 47 year old man has been referred to your Pain Clinic in a city hospital. He had an ultra-low anterior resection 6 months ago and is known to have metastatic disease. He lives in the country 3 hours from your hospital. He was commenced on a controlled-release oral morphine preparation for pain 3 months ago. He is known to have renal impairment with a creatinine of 0.25 mmol/L As well as the oral controlled-release morphine he says he is taking paroxetine (a selective serotonin reuptake inhibitor). He is complaining of severe pain. What should be your initial approach to this patient?

10 Years Collection of ANZCA Part II Anaesthetic Viva

MK Yuen & SW Ku

10 Years viva collection 2009

156

ANZCA Anaesthetic Viva May 2004 Day 2 Dr. Michelle Cheung, PWH
(Good luck to all ANZCA Part II candidates! These are incomplete pieces of memories which may only serve as an entertainment during your revision. Remember to be confident & TRUST YOURSELF!)

Scenario 1
You are asked to see a 47 year old man from the country who has been diagnosed with lung cancer and is scheduled to have a thoracotomy for a right lower lobectomy. What options are available for post operative pain management in this patient?
(The scenario printed in Examination report is slightly different from the scenario described by candidate. The scenario from candidate stated that patient has history of OSA on home CPAP. He is currently taking morphine slow releasing tablets 200mg BD for his knee pain.)

E: How are you going to manage his pain? C: This is a man with OSA & chronic pain problem going for thoracotomy. My options for his peri-op analgesia include regional analgesia (thoracic epidural, paravertebral block, intercostals block); local wound infiltration + PCA or PCA alone. E: Which one would you choose? C: Given the history of OSA & chronic high dose morphine consumption, Id use epidural analgesia + baseline morphine infusion to prevent withdrawal symptoms. E: Right. How much morphine are you going to give him via infusion? C: Based on his usual consumption. I am going to give 1/3 of his daily oral dose via infusion in 24 hours and titrate against clinical response. E: Where are you going to manage him post-op? C: This patient needs to be managed in ICU post-operatively for close monitoring of his respiratory function and vital signs. As hes prone to develop apnoea/hypopnoea with his underlying OSA. E: What will you do if ICU is full? Are you going to cancel the case? C: No. Id discharge the patient to HDU & let him use his own CPAP there. He could be monitored closely in HDU as well. E: Do you usually put such patients in ICU/HDU post-op? C: No. In my institution, we usually discharge uncomplicated thoracic surgical patients back to surgical ward post-op. But this patient is different coz he has other medical problems. E: OK, you gave the patient thoracic epidural as you planned and hes gone through the

10 Years Collection of ANZCA Part II Anaesthetic Viva

MK Yuen & SW Ku

10 Years viva collection 2009

157

operation uneventfully. Post-op, the ward nurse called you since the patient became difficult to manage. What will you do? C: Id attend the patient immediately. Ill assess his vitals and resuscitate accordingly meanwhile E: His vitals are stable apart from tachycardia. Hes agitated however. C: Id assess him to see whether its due to pain or withdrawal reaction. Pain can be assessed by VAS subjectively and objective signs like tachycardia, sweating, special posture adopted to avoid pain Id check his epidural catheter position and volume of drug consumed E: His catheter is out. C: Then, the patient is receiving inadequate analgesia. Ill change him to PCA, boluses + background infusion E: What else are you going to do? C: E: What about the epidural? C: I am going to continue monitoring his vitals and neurological function for 24 hours. This is standard practice in our institution. E: One month later, the patient who lives 3 hours away from your hospital called you up telling you that hes suffering from persistent pain around the wound area. What are you going to do? C: I am going to arrange an early appointment for him to assess his pain. He might suffer from chronic post-thoracotomy pain. E: Are you going to ask him something via the phone? Hes bit reluctant to go to hospital. C: Ill assess his severity, distribution, character and current treatment of the pain via the phone. However, I insist to have a formal face to face assessment for him as a clinical consultation and plan my treatment accordingly. E: How common is the post-thoracotomy pain? C: It varies with severity and incidence due to different reporting methods used. Roughly, up to 70% patients post-thoracotomy develop some degree of chronic pain problem. E: How do you treat this? C: Starts with prevention: identify patients factors prone to chronic pain development; anaesthetically good analgesia, surgically limit size & extent of wound creation, e.g. use VAT instead of open procedure. Treatment of chronic pain include multi-disciplinary approach and multi-model treatment(book stuff) E: OK, say this patient didnt develop chronic pain. Back to post-op stage, his epidural catheter fell out. You want to replace another one. How are you going to get your consent for epidural? C: I am going to explain the indications/ advantages and risks of thoracic epidural to him. E: Say I am the patient. Doc, what are the risks? C: Complications may be attributed by technique needle placement, catheter insertion or drug effects. There are rare but severe complications (e.g. epidural hematoma/ abscess,

10 Years Collection of ANZCA Part II Anaesthetic Viva

MK Yuen & SW Ku

10 Years viva collection 2009

158

permanent neurological deficit) and common but minor complications (e.g. PDPH, patchy block, shivering, pruritus). E: How common is the epidural hematoma? C: Rare, about 1:150,000.

Scenario 2
75 year old female.

Planned procedure:

revision total hip replacement (due to increasing hip pain).

Treated hypertension - beta blocker - diuretic Outline the potential problems in this patient for this operation? E: What are your concerns about this patient? C: I have my patient, anaesthetic and surgical concerns. Patient: elderly lady with limited physiological reserve; hypertension control, look for CVS risk factors according to AHA guidelines and optimize them; unable to assess functional status due to her hip problem; her Rx and S/E. Anaesthetic: requirement of CVS monitoring intra-op according to her risk factors, EA + GA (with prolonged procedure in lateral position), blood loss, hypothermia. Surgical: prolonged and difficult operation in revision THR, potential risks of fat embolism, cement related hypotension, bleeding, heat loss. E: Why this patient is prone to hypothermia? C: Impaired temp regulation during anaesthesia and surgery. Heat loss (cold theatre environment, exposure of body surface, evaporation) > heat production (lower metabolic rate in elderly and under anaesthesia, loss of shivering, behavioral heat conservation). E: Tell me how is temp regulation impaired during anaesthesia? C: (??!!! Part I stuff all along!) Normal temperation regulation involves afferent pathway (peripheral and central thermo receptors), control unit (hypothalamus) and efferent pathway (metabolic rate, shivering, behavioral change). All of these have been altered by anaesthesia: blunt thermo receptors sensation, increase interthreshold range in hypothalamus, inefficient efferent pathway. E: What is the significance of epidural in which shivering is preserved? C: Peripheral vasodilatation causes heat redistribution and increased heat loss from body surface drop in core temperature. E: How is it compared with GA? C: (not sure) Less hypothermia compared with GA?!
10 Years Collection of ANZCA Part II Anaesthetic Viva MK Yuen & SW Ku

10 Years viva collection 2009

159

E: What is the effect of GA on temp regulation? C: (draw 3 phases curve: temp v.s. time in GA book stuff) E: What are the mechanisms of heat loss in general? C: conduction, convection, evaporation and radiation heat loss. E: Which one is more important during GA? C: radiation >= 40% (due to cold theatre environment usually), evaporation (depends on extent of surgery and exposure of viscera). Conduction and convection are less significant. E: What about blood given to patient? C: Heat loss due to conduction if cold blood given to patient rapidly. But, we always use warming device during transfusion which may minimize the heat loss. E: What is the temp drop if one unit of blood is given to patient without warmer? C: () Fluid like blood has large heat capacity. We may calculate the temp drop based on temp difference between blood from bank (~ 4 degree) and body temp, times heat capacity E: Just make a rough estimation. C: (not sure) 0.5 1 degree drop in body temp. E: 0.5 or 1? C: one I guess. E: So, two units of blood will drop temp by two degrees? C: probably (Bell saved me!)

Scenario 3
A 65 year old male with gross ascites from chronic alcoholic liver disease presents for repair of an umbilical hernia. What are your major concerns about giving this patient an anaesthetic? E: What are the risks? C: Ill divide them into patient, anaesthetic and surgical aspects. Patient:: alcoholic liver cirrhosis with associated complications like coagulopathy, poor nutrition, vitamin deficiency, hypoalbuminaemia, immunocompromisation, encephalopathy; concomitant abuse of other agents and smoking. Anaesthetic: risk of aspiration with ascites and delayed gastric emptying, altered drug Pk with Vd & metabolism, uncooperative patient with difficult pain management. Surgical: portal hypertension, increased bleeding tendency, ileus. E: How do you evaluate this patients liver problem? C: Id classify it according to Child-Pugh classification. E: Tell me about the classification. C: (bookwork) It evaluates the severity of liver dysfunction in cirrhotic patients and gives

10 Years Collection of ANZCA Part II Anaesthetic Viva

MK Yuen & SW Ku

10 Years viva collection 2009

160

peri-operative mortality risk. (draw a table of the classification system with corresponding mortality: <= 5%, 5-10 % and >= 50%). E: OK. Say, this patient has (symptoms & signs pointing to Childs C cirrhosis). You mean this patient has 50% mortality for this operation? C: Not necessarily. The nature and extant of surgery will affect per-op mortality. For this elective hernia repair, Id postpone the surgery and optimize his condition before proceeding. E: Fair enough. This patient has been optimized and operation completed. The recovery nurse called you saying the patient was difficult to manage. What will you do? C: Ill immediately attend this patient. Ill assess his color, pulse & other vital signs and resuscitate him simultaneously according to A-B-C. My differential diagnosis includes patient (drug intoxication, alcohol withdrawal, delirium tremens, e- disturbance, metabolic causes like hypoglycaemia, stroke), anaesthetic (inadequate analgesia, hypoxia, hypercapnia, acute post-op confusion) and surgical causes. E: How do you treat delirium tremens? C: Stabilize the patients vitals, sedation with benzodiazepine group of agents. E: Which one? C: Id use diazepam IV 1-2 mg boluses and titrate against patients clinical response. E: Have you heard of Wernicks encephalopathy? C: Its a reversible cause of CNS dysfunction due to thiamine deficiency. Sorry that I forgot the details about it. (Triad of nystagmus, ophthalmoplegia & ataxia as classical presentation. May also present with acute confusion. Tx: thiamine if left untreated, this condition deteriorates into Korsakoffs syndrome irreversible, unable to retain new memories.) E: Thats fine. What is the other organ involvement in cirrhosis apart from what youve mentioned? C: It may cause renal failure. E: Why? C: The hepatorenal syndrome. Renal dysfunction/ failure due to renal perfusion in cirrhosis. Exact mechanism is yet to known. Systemic hypotension and renal vasoconstriction are the hallmark of this syndrome. Renin-angiotensin-aldosterone system and sympathetic nervous system activation may play a role. E: This patient develops hypoxaemia, do you know why? C: Patient may have pulmonary edema, pleural effusion, or atelectasis due to diaphragmatic splinting in tense ascites. E: CXR is clear. C: E: Have you heard of hepatopulmonary syndrome? C: Yes. Patient develops hypoxaemia with A-a gradient due to V-Q mismatch. (Subpleural telangiectasia similar to spider angiomas may develop in cirrhosis create shunting which is

10 Years Collection of ANZCA Part II Anaesthetic Viva

MK Yuen & SW Ku

10 Years viva collection 2009

161

worse in upright position!) (forgot the rest of viva details. Basically, all medicine stuff hepatopulmonary syndrome, treatment for portal hypertension, etc.)

Scenario 4
A 30 year old woman, who has dense hemiplegia following a stroke 2 weeks ago, presents for a PEG (Percutaneous Endoscopic Gastrostomy) in the endoscopy suite. She has had hypertension for a few years and was also a heavy smoker. What are the main risk factors for this person undergoing this procedure? E: Whats in your mind? C: I have my patient, anaesthetic and surgical concerns. Patient: Id like to know if its a hemorrhagic or ischaemic infarct, her neurological state, GCS, bulbar dysfunction (as she requires PEG for long-term feeding); CVS risk assessment according to AHA guideline, any chest infection. Anaesthetic: remote area anaesthesia, airway management - risk of aspiration, whether patient comes from ICU or NS ward, is she intubated/ tracheostomized, avoid suxamethonium. Surgical: usually straightforward, but may be complicated by bowel perforation, liver damage etc. E: Why remote area? C: The procedure is usually done in endoscopy unit. But this is a complicated patient. Id like to perform GA in theatre. E: Fine. Well talk about the sux. issue later on. The patient is in NS ward. Not intubated.

Scenario 5
You are called in at 0400 hours to anaesthetise a 47 year old male who is acutely paraplegic following a high speed motor vehicle accident 4 hours previously. He has a fractured left humerus, a small encapsulated splenic tear that does not require laparotomy. Cervical spine and head CT are clear. Outline your anaesthetic plan for this surgery. E: How would you manage this patient? C: This is a trauma patient. Ill stabilize this patient first according to ATLS guideline: primary & secondary surveys before proceed.

10 Years Collection of ANZCA Part II Anaesthetic Viva

MK Yuen & SW Ku

10 Years viva collection 2009

162

E: All done. C: This is an emergency operation and its going to be done in a prone position for a prolonged time. My patient concerns include: risk of aspiration, bleeding, hypothermia, nerve injury from prolonged prone position, blindness/ ischaemic optic neuropathy. Anaesthetic concerns: risk of ETT dislodge, difficult airway management and resuscitation (CPR) in prone position, requirement of invasive hemodynamic monitoring and special neuromonitoring techniques, venous air embolism, post-op ICU care. Surgical: prolonged surgical time, bleeding, risk of spinal cord damage and monitoring, requirement for steroid treatment. E: Why steroid? C: Theres evidence shown methylprednisolone (MP) may reduce edema and further cord damage/ inflammation in acute spinal cord injury. Its recommended for use in such patients. E: How to give it? C: MP bolus 30 mg/kg Q1H infusion followed by 5.4 mg/kg/hr for 24 hours if initiated within 3 hours of injury. If initiated within 3-8 hours post-injury like in our patient here, it needs to be continued for 48 hours. (forgot the rest: something on neuro-monitoring seems not necessary in a paraplegic patient undergoing palliative procedure, physiological effects of prone position, etc. pretty standard questions.)

Scenario 6
You respond to a call from a junior colleague. Their morbidly obese (140 kg) 68 year-old patient is having a below knee amputation for peripheral vascular disease under epidural anaesthesia. 15 minutes into the surgery he became unrousable and the anaesthetist is struggling to maintain the airway with a mask and bag. How will you assist your colleague? (difficult airway mx, crisis due to anaphylaxis)

Scenario 7
You are asked to anaesthetise a 2 year old boy for a laparoscopy for bilateral impalpable testes. He looks overweight and weighs 16kg, but is otherwise well. He has been booked as a day-only admission. How do you define obesity in childhood, and what implications does obesity have for his anaesthetic management?

10 Years Collection of ANZCA Part II Anaesthetic Viva

MK Yuen & SW Ku

10 Years viva collection 2009

163

E: How do you define obesity in a child? C: Theres no universally accepted definition. BMI used in adults cant be applied to children. Id use the formula: BW = age x 2 + 8 (kg) which corresponds to 50 percentile BW on growth curve for paediatric patients. BW more than 20% from the formula predicted may indicate obesity. E: What are your concerns in this patient? C: I have my patient, anaesthetic and surgical concerns. Patient: paediatric, birth history, congenital abnormalities esp. in presence of overweight, OSA, URTI. Anaesthetic: chubby paediatric patient, difficult airway management, IV access, difficult landmark for regional anaesthesia, need to consider lean body mass for drug dosage. Surgical: day case, difficult surgical access, prone to wound infection, healing problems. E: How do you calculate your drug dosage for this patient? C: Id use his lean body mass. I predict this as 10-20% less than his current BW and Ill titrate against his clinical response. E: He has uncomplicated past medical history. Will you accept him as day case? C: In presence of his obesity, theres increased risk for intra-op and post-op problems as mentioned above. So, Id accept him as a day case provided theres paediatric service backup in the same institution. Id inform the parents and surgeon about possibility of post-op admission in case of any complication arising. Otherwise, I wouldnt accept him. E: You accepted him as a day patient. What anaesthetic technique will you use? C: After preparing the theatre, equipments, machine, drugs and experienced assistants, Id use general anaesthesia with endotracheal intubation. Id use local wound infiltration by surgeon + supp. paracetamol for analgesia. E: The operation was finished. The recovery nurse called you because the child became very agitated. What will you do? C: Ill immediately attend this patient. Ill assess his vitals and perform simultaneous resuscitation in the sequence of A-B-C. My DDX includes patient, anaesthetic and surgical causes (book work) E: His saturation and pulse were stable. But he started to fit. What will you do? C: This is an emergency situation, Id call for help: assistants, intubation equipment, emergency trolley and defibrillator Id turn the patient into left lateral position, give 100% O2 mask, IV anticonvulsant. E: IV was out. What drug are you going to give? C: Midazolam 0.2 0.3 mg/kg nasally. E: Fitting was stopped after that. What will you do next? C: Continue O2 therapy, secure IV access, monitor his saturation, pulse, BP closely. Investigate the cause of his fitting by reviewing history from his parents, old records,

10 Years Collection of ANZCA Part II Anaesthetic Viva

MK Yuen & SW Ku

10 Years viva collection 2009

164

performing complete P/E, taking blood for e-, Hb, ABG, glucose, septic workup. Arrange paediatric ICU admission. E: His Na+ came back to be 123 mmol/l. Will that explain his fit? C: Its possible. But Id fully investigate to rule out other causes as well. E: (forgot rest of the details)

Scenario 8
A 28 yr old woman is 34 weeks pregnant. She has a history of Systemic Lupus Erythematosis, and Pulmonary Hypertension. A decision has been made to deliver by elective Caesarean Section for obstetric indications. She has presented to Anaesthesia Preoperative Clinic for evaluation. What effect will pregnancy have on her systemic diseases? (interaction between medical diseases and pregnancy, mx of pulmonary hypertension in labour overall a difficult scenario)

Scenario 9
You are asked to provide general anaesthesia for clipping of an acutely bleeding cerebral aneurysm. The patient is a 38 year old woman who presented with a sub-arachnoid haemorrhage 18 hours ago. When you go to review the patient, she appears to be weak in her right arm and keeps drifting off to sleep when you talk to her. What are the issues you face when obtaining consent for anaesthesia in THIS patient. E: How do you get consent for this operation? C: I need to consider clinical, legal and ethical issues when obtaining consent. As the patient is not fit for consent and this is an emergency operation, consent by two consultants is valid for this operation. E: Which two? C: Can be patients neurosurgeon and another consultant of independent opinion, i.e. not involved in clinical care of this patient. E: This patients husband is a sergeant and he wants to be in charge and representing his wife for her clinical decision making. He wants to sign the consent for her. What will you do? C: Its nice to have patients close family informed of whats going on. But, I will courteously tell the husband that a minors consent is not legally valid for an adult patient. A consent by two

10 Years Collection of ANZCA Part II Anaesthetic Viva

MK Yuen & SW Ku

10 Years viva collection 2009

165

consultants is still required in this situation. E: What about patients parents who flew here from Europe insisting the same request as her husband? C: I will manage the same way as I mentioned just now. Have family fully informed is ethically good for rapport and facilitate further management of this patient. Yet, a legally valid consent is a separate issue. E: How do you decide whether to operate on a SAH patient saying that theres no on-going bleeding? C: Few factors need to be considered to weigh the benefit and risk of operation Surgical factors (tight brain makes surgery technically difficult and its decreasing over days), risk of rebleeding (7% in D1-2, approaches 30% till D14, cumulative risk up to 50% overall) and risk of vasospasm (highest if manipulate brain in D7-10 and then decline). (I actually drew a risk v.s. time/days graph showing trends of above 3 risk factors to facilitate discussion) E: Why theres vasospasm? C: Exact mechanism unclear. One postulation is that free Hb in blood clot damage the endothelium of cerebral vasculature and result in NO/EDRF production vasoconstriction. E: What is the management for vasospasm? C: Triple H therapy, i.e. hypervolaemia, hypertension and haemodilution to prevent secondary ischaemic brain damage. E: What else? C: Ca-channel blocker - nimodipine IV or orally 60 mg Q4H, anticonvulsant, steroid. E: Will the operation affect your management? C: Yes! After operative clipping of aneurysm, I can aim a higher CPP without worrying about inducing rebleeding.

10 Years Collection of ANZCA Part II Anaesthetic Viva

MK Yuen & SW Ku

10 Years viva collection 2009

166

ANZCA Anaesthetic Viva September 2004 Day 1 Dr. Annie Chu, PYNEH
Scenario 1
A 26 year old weight lifter presents for a repair of his ruptured anterior cruciate ligament. He is 1.8 metres tall and weights 130kg with minimal body fat. What specific factors relating to this patient's sport may influence the anaesthetic plan? E: What are the special concerns in anaesthesia for this patient? C: Patient require rapid recovery from procedure to resume training Minimal anaesthetic side effect expected: D/N/V, postop pain, backache(if SA) Patients own medication: nutritional supplements, anabolic steroids E: What is your plan of anaesthesia? The case is done under GA+femoral n block E: Technique of femoral nerve block? E: Recovery room acute pain management? E: 4 days later, the surgeon tells you that the patient is complaining of severe pain at home, what is your management? E: What are you suppose to elicit from history? E: What is the reason that the patient may develop neuropathic pain? E: What is the nerve that is likely involved? E: Mechanism of injury? E: How do you treat neuropathic pain?

Scenario 2
An 87 year old woman is admitted with a fractured neck of femur. Past History: Atrial Fibrillation Medications: Warfarin Blood Pressure: 160/90 Pulse: 100, irregular She is confused. Outline your further cardiovascular assessment. E: How to you assess her CVS status?

10 Years Collection of ANZCA Part II Anaesthetic Viva

MK Yuen & SW Ku

10 Years viva collection 2009

167

E: What are your concerns? E: What about the patient is on warfarin? E: What specific investigations? C: INR, K+ E: How do you get consent for this patient? E: How do you anaesthetise this patient? INR 1.9 C: GA+ 3:1 block, LMAinduction drugs, Positioning E: Intraop the patient developes hypotension 70/50 P 130, ECG showed fast AF. What is your management? E: Digoxin dose? E: Which vasopressor for hypotension? E: Patient is stabilized, how do manage intraoperative pain? E: Postop pain management?

Scenario 3
You are called to see a 4 year old boy in the emergency department with a two day history of nausea, vomiting, abdominal pain and lethargy. He has no known co-morbidities. He looks ill and a provisional diagnosis of acute appendicitis has been made. Vital signs: Pulse rate 160, BP 80/40 Resp. rate 38 Temp 37.0c What are the major issues in your pre-operative assessment of this patient that will determine if this patient is ready to go to the operating theatre? E: How do you assess the patient preop? E: How do you optimise him for surgery? E: Estimation of degree of hydration E: How to replace fluid deficit? E: You notice the patient is breathing like deep sighs C: Metabolic acidosis (kussmaul breathing) E: Why? C: Peritionitis, sepsis, other causes E: ABG: PH 7.21 HCO3 12 BE 15

10 Years Collection of ANZCA Part II Anaesthetic Viva

MK Yuen & SW Ku

10 Years viva collection 2009

168

Na 130, K 5.6 Glu 20 Urine ketones +++. Glu ++++ C: Picture of DKA triggered by sepsis E: How do you optimize C: fluids, correct acidosis, insulin, K+ replacement Patient is sent to ped HDU for optimisation E: Patient became more drowsy , complained of headache. What do you suspect? Na 130 135 C: Suspect increased ICP E: What are your special concerns about securing airway for this patient, if all pediatrics equipment are available? C: RSI, drugs, cricoid, preO2 E: What else? E: How about patients BP become 60/20 after u have given the induction drugs? C: Monitor cont arterial pressure, Standby vasopressors, keep CPP E: Patient is stabilized, proceed to surgery. How do you manage intraoperative fluid administration and glucose levels? E: Intraop pain managemen? Postop pain management?

Scenario 4
Yours patient, Mrs S, comes to see you in your rooms. She is due to have her 4th caesarean Section next week. She is 35 years old, G4P3, and is currently at 37 weeks gestation. This pregnancy has been uneventful, apart from worsening asthma, treated with an inhaler. She tells you that the last anaesthetic (a combined spinal and epidural) seemed difficult to perform, and that she had a numbness in her left leg for 2 months after the operation. She is concerned about the possibility of this recurring, as she does not want a general anaesthetic. How will you evaluate this problem, and what recommendation concerning anaesthesia will you make? E: What are your concerns? C: Patient/ anaes/ surgical concerns E: How about her last difficult CSE? C: Review notes, technique E: How about postop L leg numbness? C: Preop s/s, investigations, progress of recovery, treatment

10 Years Collection of ANZCA Part II Anaesthetic Viva

MK Yuen & SW Ku

10 Years viva collection 2009

169

E: What events during CSE insertion may contribute to neural injury? C: Paraesthesia, pain on insertion E: How do you anaethetise the patient this time? C: 1 shot spinal E: After the spinal what would you do? C: supine + L tilt, C: Ice/ pinprick E: Level - T4, Do you check motor block? Lower level of block? E: What to do if the upper level of block is T8 on the left, T10 on the right? C: Try head down tilt, prepare ketamine, fentanyl, GA. Explain, discuss with patient/surgeon E: Intraop during bladder dissection, patient complained of pain, how to manage? C: Stop surgery, Check sensation, Iv ketamine/fentanyl/ LA infiltration E: This settles the pain, after delivery, patient complained of dyspnea & chest pain. What is the DDx? E: Do patient get dyspneic in patient with inadequate block? E: How about embolism phenomenon? Close monitor hemodynamics, IVF, vasopressors E: Check level of block, how?

Scenario 5
You are asked to see a 62 year old man who is presenting for excision of an Squamous Cell Carcinoma of the base of tongue and floor of mouth. He has completed a course of preoperative radiotherapy. What are your principle concerns with anaesthetising this man? E: concerns? C: Patient: premorbid diseases, dysphagia, choking, aspiration risk, malnutrition, Smoker Surgical: standby tracheostomy, prolonged procedure Anaesthetic: airway, bleeding, shared airway, pain mx E: He is a chronic smoker, controlled HT, COAD, stable angina, What do you request for investigations? C: Previous flexible NP scope report, CT scans, Blood/ ECG/ CXR E: What is your plan of anaesthesia? C: Awake FOB, nasal route, #7 reinforce tube, local anaesthesia of airway E: Anything else? C: Nerve blocks: difficult for previous RT E: During FOB, the tumour bleeds, Management?

10 Years Collection of ANZCA Part II Anaesthetic Viva

MK Yuen & SW Ku

10 Years viva collection 2009

170

C: Crisis, call for help, Surgeon standby, help apply external pressure on tongue, try to finish FOB quickly, While prepare for E tracheostomy, 2 suctions systems (one for FOB, another orally) E: any other way to secure the airway? C: awake LA tracheostomy E: but patient is needle phobic, thio-sux-tube? C: Not desirable E: How about inhalational induction? C: It is possible, one can try DL after deep enough, if difficult view, do tracheostomy while patient breathing spontaneously E: How do you monitor the patient for this prolonged procedure? C: Standard monitoring + intra-arterial line (what for?), temp, u/o E: Intraop surgeon complained of poor perfusion of flap, what are you going to do? C: Maintain normothermia, keep MAP 80-90, normal oxygenation, mild hypercarbia, dextran 40 (how does it work?), Hemoglobin >10 E: Postop surgeon complained that the flap is edematous, what is the cause? C: poor perfusion, Vasocontriction, Poor oxygenation, Dependent edema, Hematoma formation E: What can you do to sort this out? C: Maintain oxygenation, improve perfusion (haematocrit, normovolemia), Treat postop pain, elevate the dependent part E: How about postoperative airway? C: Usually surgeon do tracheostomy

Scenario 6
A 25 year old female presents with diagnosis of ectopic pregnancy. The gynaecologist wishes to perform a laparoscopic salpingectomy as soon as possible because she has significant abdominal pain. You go to the emergency department to assess the patient. Her medical history includes: y y A ventricular septal defect that was not diagnosed until adulthood Depression for 12 months, for which she is on fluoxetine (Prozac) 20mg bd

How will you manage this patient's pain preoperatively? E: How are you going to tackle her periop pain? C: iv fentanyl but risk of histamine release or small dose morphine

10 Years Collection of ANZCA Part II Anaesthetic Viva

MK Yuen & SW Ku

10 Years viva collection 2009

171

E: How about pethidine? C: no, iv pethidine increase histamine release E: how about tramadol? C: there is report of serotonin syndrome with SSIR E: Proceed to surgery, what is your concerns? C: Patient (? Eisenmenger syndrome, need antibiotic cover, parasoxical emboli), anaesthesia, surgery. Aim on haemdynamics. E: What is your monitoring? E: What is your special concerns about surgery? C: laparoscopic effect E: postoperatively: desaturation 77%, BP 70/40, pulse 150/min, abdominal swelling. C: preceed to re-open laparotomy for hemostasis E: Intraop surgery need to ligate fallopian tube without consent. C: explain to relative, document, report to department head, MPS

Scenario 7
A 26 year old motorcyclist, wearing an open face helmet, lost control while cornering and collided with a power pole. He has a closed head injury, a mobile maxilla and is in a hard collar. On examination he is restless, moving all limbs, withdraws his arms and opens his eyes to a painful stimulus and mutters words. He is booked for an urgent CT scan of his head and face and cervical spine. He is haemodynamically stable and has a pulse oximetry reading of 99% on simple oxygen mask. Due to his restlessness you have been asked to assist with his management for his CT scan. What is his Glasgow Coma Score and why? E: What is yours GCS? C: E3 (should be E2), M4-5, V3, total 10 E: The neurosurgeon plan to have CT brain, but patient is restless and require your help. C: assess according to ATLS, call trauma team, resuscitation and assessment, look for other injury E: Only head injury and no other injury, how are you going to manage his airway? C: ongoing neurological change and CT scan require cooperative patient. I plan to intubate patient. I will do RSI with manual inline stabilization. (also mentioned the drugs use and details of how to assign assistant) E: Are you going to sedate or paralyse patient E: what are the things to prepare for transfer?

10 Years Collection of ANZCA Part II Anaesthetic Viva

MK Yuen & SW Ku

10 Years viva collection 2009

172

C: Personnel (asked the detail of personnel), equipment, drugs, monitor, O2, suction, communication device. E: What type of portable suctions C: Hand pumped portable suctions? E: what other type do you know of? C: E: Have you heard of suction called (name not remember) which base on principle of venture? C: No E: What problem do you think this type of suction device can give rise to? C: another O2 source, pressure not high enough E: How about O2 cylinder? C: need a full cylinder. E: what is the pressure? C: E: how much O2? C: 10 l/min for 10 hours E: what is your familiar transport ventilator? C: ventiPAC E: What is the features?

Scenario 8
A heavy smoker diagnosed with adenocarcinoma of the lung is scheduled for an (L) upper lobectomy (Video Assisted Thoracoscopy VAT). Preoperative investigations are: FEV1 1.05 (62% of predicted) FVC 1.6 (76% predicted) FEV1 / FVC - 66% PA O2 76mmHg PA CO2 43mm Hg on room air HB 160 g/L WCC 4.8 x109/L Platelets 310 x 109 /L Na 143mmol/L K 4.5 mmol/L Cr 0.08 mmol/L

What other information, beyond a standard anaesthetic assessment, do you need prior to commencing the anaesthetic? (Exclude data you acquire in all patients). E: What other information do you require to assess for fitness to surgery? C: History: co-morbidities, smoking habit, exercise tolerance, chest infection, drug history, allergy. Physical examination: SpO2 at RA, body weight and height, wheeze or crepitation

10 Years Collection of ANZCA Part II Anaesthetic Viva

MK Yuen & SW Ku

10 Years viva collection 2009

173

which indicate chest infection, temperature. Investigation: CXR, CT scan for anatomy, trachea/bronchial distortion. ECG for ischaemia, right heart strain, cor pulmonale, post op predicted FEV1, DL CO2. E: What is your choice of GA technique? E: What type of tube do you use C: Left Bronchocath 39F for average size adult man E: But it is a Left Upper Lobe surgery. C: Discuss with surgeon for likelihood of a pneumoectomy. E: Can you use single lumen tube? E: You proceed to induce patient and DL grade 3 larynx, what are your management? C: continue manual mask ventilation, call for help, difficult intubation trolley, surgeon standbys. Best option: FOB with DLT railroaded, but problem of difficult manipulation and FOB may not long enough, or SLT with bronchial blocker. E: What can you use for bronchial blocker? C: Foley catheter or Fogarty catheter. E: What is the disadvantage? E: You managed to put in DLT but it is leaky? C: keep ventilation 7 ml/kg at rate 15/minutes. E: During surgery, Sp)2 dropped to 92%, what are your management? C: increase FiO2, 100% O2, optimize cardiac output. Add CPAO 5cm H2O to operating lung. E: Why is it useful? E: What is the advantage and disadvantage? C: it may cause surgical difficulties. E: What else can you do? C: PEEP to dependent lung. E: How much? C: 5 cm H2O. E: why not higher? E: What else? C: intermittent insufflation, intermittent 2 lungs, supine position, clamp pulmonary artery not applicable in this case. E: What is the effect of volatiles on hypoxic pulmonary vasoconstriction? E: Postoperatively the patient is to be mechanically ventilated, what are you go to do? C: I plan to change to SLT with Cooks airway exchanger.

10 Years Collection of ANZCA Part II Anaesthetic Viva

MK Yuen & SW Ku

10 Years viva collection 2009

174

Scenario 9
You are the supervising anaesthetist for the operating theatre. You are called by the Recovery Room / PACU about a 32 year old female patient who has just had an excision of breast lump under general anaesthesia and is complaining that she cant breathe. What will you do? E: What is your management? E: What is the differential diagnosis? E: What is your recovery bay management? E: The nurse tells you that the trainee anaesthetist of this case often have problem like this, and this event is not his first, what are your approach to this problems? E: What do you think of this record? (Examiner present a copy of anaesthetic record of this case) There is no date, name of anaesthetist, grade of larynx not mentioned, size of tube not mentioned. Induction with propofol, fentanyl, susmethoium followed immediately by 35mg rocuronium. OT duration 15-20 minutes. Reverse dose given. No NM monitoring. E: As a senior staff of department, how are you going to help this trainee with problems? E: What issue is the SOT supposed to discuss with the trainee? E: How do you approach the trainee?

10 Years Collection of ANZCA Part II Anaesthetic Viva

MK Yuen & SW Ku

10 Years viva collection 2009

175

ANZCA Anaesthetic Viva September 2004 Day 2 Dr. Alexandra Yip, QMH
Scenario 1
You are the duty anaesthetist, and are called to recovery to assist with a 22 year old medical student who has had a knee arthroscopy and is unable to stop vomiting. She is due to be picked up in one hour. What treatments could you recommend to stop the vomiting? E: How would you manage? C: Attend to patient immediately to assess and resuscitate. Talk to patient for level of unconscious. Check vital signs exclude hypertension, hypoxia, arrhythmia. Check glucose level. E: Vital signs stable and awake. C: Antiemetic ondansetron 2mg. Then I would like to look at the anaesthetic record to see drugs given intra-op. E: pancuronium 6mg, morphine 10mg, thiopental, O2/N2O/Halothane. C: Prevention better than treatment. In this case patient has multiple risks factors as young female, opioid, non-smoker, N2O. Other preventive measure by indentifying high risk group. Avoid prolong fasting. Good hydration. Avoid N2O and use propofol, avoid NM blocker so as to avoid neostigmine. Give dexamethasone and ondansetron. E: What dose of dexamethsone? C: 0.5mg/kg E: OK. Let us concentrate on the treatment rather than prevention. This patient still has nausea and vomiting. What else can be given? C: metoclopramide can be tried but not very effective. Further 2mg of Ondansetron. Erythromycin mainly used in promote gastric emptying in ICU. Small bolus of propofol 20mg. E: anything else? C: Hur...... E: have you heard of droperidol? C: oh, yes. It is very effective. But it is not available in my hospital. So I didn't count it as an option. It is blacklisted by FDA. E: Do you know why? C: Because it cause fatal arrhythmia? E: for this patient. How would you anaesthetise her next time?

10 Years Collection of ANZCA Part II Anaesthetic Viva

MK Yuen & SW Ku

10 Years viva collection 2009

176

C: preop good hydration, avoid prolong fasting. Intraop avoid muscle relaxant, avoid N2O, avoid opioid, I would like to use TIVA with propofol, prophylactic medication. E: What is the risk of PONV despite all these measures? C: about 20%. E: This patient is medical student. She says to you that she mentioned to the anaesthetist pre-op that she has PONV. She asked you if the anaesthetist in charge has done something wrong/ inappropiate. Ask for your opinion. C: I don't think it is very appropriate for me to comment on my colleage. I would like to leave it to the in charge anaesthetist. E: The in charge anaesthetist is busy doing a case and ask you to talk to patient and just cross out the morphine on the record. C: I would insist the in charge anaesthetist to talk to the patient. E: He is your ex-boss and refused to come. C: Then I would involve the head of department to deal with him. E: Now say you are the head of department and nursing staff complain to you that your ex-boss has problem at work. C: I would like to collect more evidence by clarify with the staff who makes the complains. The detail of event or what exactly happened/ has it probably documented. Talk to other OT staff and anaesthetic colleague/ trainees who work with him. If there is evidence suggestive of malpractise/fault, I would then arrange an interview with him with myself, witness and a person representing his interest (his spouse/lawyer/friend) to find out his side of story, whether any problem with his social life, finance etc. E: Do you allow him to continue his duty? C: If his practise is doing harm to patient, I would like to ask him to take leave. E: He has no more leave and can't take leave. C: Then I would like to put him in non-clinical post until problem resolved. E: do you think it is realistic? He is 68 years old. C: I am not sure of the age for compulsory retirement. But I would seek advice from medical council and the college. E: let us change this topic completely? Have you done paravertebral block? C: No, I haven't done it. E: From your book work, can you tell me about it. C: (not very sure) extension of epidural to the nerve root. This block provide unilateral block rather than bilateral block as in epidural.

10 Years Collection of ANZCA Part II Anaesthetic Viva

MK Yuen & SW Ku

10 Years viva collection 2009

177

Scenario 2
A 78 year old man from a nursing home presents for day procedure phaecoemulsification of right cataract and insertion of Intra Ocular Lens. Co-morbidities are: coronary artery disease hypertension mild chronic obstructive airways Aspirin (Cartia) Perindopril (Coversyl) Atorvastatin (Lipitor) What is your preferred regional anaesthetic technique and why? E: How would you perform eye block? C: I have not done any eye block as surgeon do all blocks themselves in my locality. But from my reading: peribulbar, retrobulbar, subtenor, topical. E: How would you prepare patient? C: (book work) I would like to exclude any contraindication like cooperation, able to lie supine, stable IHD, continue medication except aspirin, explain to patient. E: During procedure, patient became restlessness. C: Talk to paitent for SOB, chest pain. Assess conscious level, check BP, ECG, SpO2 E: patient vital signs stable, afraid to be under the drape. C: sorry, forget to mention claustophobia as contraindication. E: Why do you think patient may need GA? C: patient uncooperative, unable to lie supine because of COAD, claustophobia. Surgery completely immobile patient, prolong surgery. E: say no claustophobia and surgeon wants to do case under regional block in the regional hospital. He ask for you advice for safety under RA. (forget the rest)

His current medications are:

10 Years Collection of ANZCA Part II Anaesthetic Viva

MK Yuen & SW Ku

10 Years viva collection 2009

178

Scenario 3
A fit and well 45 year old orthodontist presents for a rotator cuff repair following a long standing injury. He requests a regional anaesthetic. What is your advice to this patient? E: What is your management? C: confirm patient refuse GA. Look for contraindication for RA. Explaned long procedure. Potential problem and hypothermia. E: How would you perform block? C: interscalene brachial plexus block. Bolus + catheter infusion with nerve stimulator. Supplemented by superficial cervical plexus block. E: what other block needed to cover this shoulder surgery? (can't remember the details) E: patient complained pain in upper arm few days after operation? C: (book work) E: Neuropathic pain is confirmed. What treatments to start? C: documentations, refer pain team, explained to patient, MPS, follow up

Scenario 4
You are called to the delivery room and asked to see a 26 year old woman in early labour with occiput posterior presentation. She does not want an epidural. She asks you if Transcutaneous Electrical Nerve Stimulation (TENS), acupuncture or psychological methods will reduce her labour pain. What will you tell her? E: How effective are these treatment? C: There is no strong evidence in supporting these therapies are effective but it may help in some patients. E: What is psychotherapy? C: Involve in attending antenatal clinic practising breathing exercise, given explanation on what to expect during labour and moral support from husband. E: What analgesic would you offer for labour? C: Epidural as it has the higher efficacy in relieving labour pain. E: How would you explain to patient? C: Procedure (book knowledge) and risk (book knowledge). Exclude any contraindication. E: What drug would you use?

10 Years Collection of ANZCA Part II Anaesthetic Viva

MK Yuen & SW Ku

10 Years viva collection 2009

179

C: 0.25% bupivacaine to establising the block, followed by 0.1% marcain + 2mcg/ml fentanyl infusion. E: It was quite uneventful labour. Then the next day this patient complained pain over the thigh. What you you do? C: See the patient and review history. Past health pre-existing problem. Obstetric - ? trauma, baby's position epidural diffiuclt in procedure, paraesthesia, durg used pain onset, site, nature, characteristic etc. Examination of patient epidural site for tenderness or discharge, fever, full neurological examination. E: Pain is sharp down the thigh + details (suggestive of neuropathic pain) C: so, patient has neuropathic pain. Given explanation to patient and started treatment. Gabapentine 300mg daily, then bd, then tds. I would like to know if patient is breastfeeding as it might be excreted in breast milk. I would check on the drug information.

Scenario 5
A 6 year old girl wheel chair bound with severe cerebral palsy (spastic quadriplegia) presents with a fractured shaft of femur following a fall from her wheel chair. You are to supervise a junior registrar anaesthetise this patient for open reduction internal fixation of the fractured shaft of femur. What will the junior registrar need to know about severe cerebral palsy to assist you in anaesthetising this patient? E: What is your concern? C: General concern for paediatric patient. Specific: cerebral palsy: difficult airway history, previous OT record, risks of aspiration. Post-op pain management. Associated with any muscular disease (risk of hyperkalaemia) paraplegia: whether associated with cerebral palsy or traumatic reason, duration of paraplegia. Intraoperative may get hypothermic as poor thermoregulation. E: Why do you thick he has risks of aspiration? C: due to cerebral palsy and also pareplegia delayed gastric emptying, bulbar involvement. E: any other reason? C: Hur...... E: patients carer told you he has history of difficult airway at dental treatment. How would you assess?

10 Years Collection of ANZCA Part II Anaesthetic Viva

MK Yuen & SW Ku

10 Years viva collection 2009

180

C: chase previous notes. E: not available. C: call up directly to previous anaesthetist. E: there is limited mouth opening. Why cerebral palsy patient has difficult airway? C: cerebral palsy: posture, poor dentition, uncooperation. (after the examination, I think he wants to know general assessment like TMD/ mallapati etc and cerebral palsy related like contracture.) E: The patient is taking phenytoin and baclofen. C: Baclofen is for muscle spasm. E: How would you anaesthetise this patient? C: Allow parental companion to calm patient at induction as anxiety can induce spastic episode. No sedation. Prepare DAME + specific difficult airway trolley and surgeon standby. Plan for ETT and IPPV with O2/N2O/isoflurane. Small dose of propofol + topicalised airway and awake FOB intubation. E: patient is distressed and stiff as she didn't have her morning medication. C: I forgot to mention that patient should continue her medicaiton. E: Alright, but still distressed and can't do awake fibreoptic. C: Gas down patient with O2/sevoflurane. Ensure deep enough with slowing of HR, regular RR, and pupils small and central. Gentle DL, tropicalise VC and wait, then intubate. E: OK. Now you have intubated the patient and on IPPV, how would you provide analgesia? C: Femoral nerve block bolus + catheter insertion for post-op pain relief. E: Alright, You leave it to your registrar when you come back in 10 mins before the start of OT. What would you like to check? C: Positioning soft padding of pressure point. Bair Hugger poor thermoregulation. Airway secure and no problem with ventilation. Lines secured.

Scenario 6
A 72 year old man is scheduled for laparoscopic cholecystectomy after presentation with acute cholecystitis. He is known to have degenerative calcific aortic stenosis. How do you assess the severity of this man's aortic stenosis? E: How to conduct anaesthesia? C: Assess severity. Clinically SADS, late peak/S4/CHF. Investigation valvular area. Surgery laparoscopic which increase SVR, decrease VR. Aim of haemodynamic in AS increase SVR, maintaine preload and maintain rate.

10 Years Collection of ANZCA Part II Anaesthetic Viva

MK Yuen & SW Ku

10 Years viva collection 2009

181

E: moderate to severe AS but clinically quite stable. C: prefer to do the case in major hospital where cardiothoracic services and ICU support available. Pre-op optimisation and patient warned of risk. Consult cardiothoracic surgeon to notify the case and standby. Invasive monitoring A line before induction. Keep haemodynamic goal. (the rest of details forgotten)

Scenario 7
A 38 year old woman was struck by a car whilst out walking. She sustained a compound fracture of her right tibia with significant (anterior) soft tissue loss. There were no other significant injuries. Her fracture was dealt with during the first 24 hours of her admission to Hospital. It is now 5 days since her admission and she is booked on a theatre list to have a left free latissimus dorsi flap to cover the area of exposed tibia. What are the main considerations for the intra-operative care you deliver to this patient? E: What is your consideration? C: Screen for background medical problem. Trauma patient: any other associated trauma like pneumothorax, head injury. Patient has been admitted for few days, any active infection like pneumonia. Flap: long procedure, prone to hypothermia, flap surivial depends on adequate perfusion, BP, Hb DVT prophylaxis and post op analgesia. I would like to give GA with standard monitoring, ETT intubation, O2/N2O/isoflurane, analgesic with morphine and post op PCA. E: It went smooth with the induction. Patient became hypotensive. What would you do? (couldn't recall whether they have said something about vancomycin) C: may I know the BP? E: 75/40 C: Real hypotensive, crisis situation. Assess and resuscitate patient immediately and simultaneously. 100% O2, fast iv fluid, hand ventilate to fell compliance. Look for wet gauze weight and suciton bottle for blood loss. Look at the I/O balance and check hameocue. Listen to chest for air entry bilaterally to exclude pneumothorax. ECG for arrhythmia. ETCO2 for any sudden drop suggestive of thromboembolic event. E: SpO2 98%, ETCO2 stable, Hb 7.5 g%, blood on drapes. C: likely to be hypovolaemia. To give iv fluid while waiting for packed cell to top up to 10 g%.

10 Years Collection of ANZCA Part II Anaesthetic Viva

MK Yuen & SW Ku

10 Years viva collection 2009

182

E: How much do you expect she needs? C: patient is likely to be hypovolaemia so with replacement of fluid, Hb likely to drop further. In general for an average size of 50kg, 1 unit of pack cell will raise 1g/dl. So she needs around 3 units of packed cell provided no further blood loss. But this needs to be guided by clinical situation. E: How would you assess adequancy of replacement? C: Haemocue. Clinical situations like BP, HR. Urine output concentrated or diluted which give idea of hydration. For setting of CVP, I would not consider now as patient is likely to have hypovolaemia. I think she can tolerate fluid challenge well. E: Now, surgeon says that hte perfusion of flap doesn't look nice. Ask you to push up blood pressure. C: Surgeon to keep the flap warm. Anaesthesia too keep patient warm, turn down volatile, give fluid challenge, correct Hb to 10 g%, good oxygenation, then may consider ephedrine boluses. E: surgeon ask you to give dextran 40. How does it work and how would you give it? C: improve microcirculation to flap. 500ml Dextran 40 over 24 hours. E: Post op analgesia? C: PCA. (can't remember the rest of the scenario)

Scenario 8
A 68 year old patient with a 2 month history of dysphagia requires sedation for an upper GIT endoscopy as a day surgery patient. The patient had a mitral valve replacement and triple vessel coronary artery bypass graft 2 years ago for regurgitation and angina and is currently on warfarin, amiodarone and furosemide. His INR is currently 1.8. How would you manage the anti-coagulation for the procedure? E: What is your management? C: to stop warfarin for 5 days, start enoxaprine 12 hours before OT then stop post op when INR optimised. (I made comment on inadequancy of INR 1.8 for MVR, but i think he wants me to say continue warfarin as minor procedure of OGD) E: patient complained chest pain? C: exclude AMI, check conscious level, ECG, HR, SpO2. Support O2, slow down HR (has been told paitent in AF with HR 100 140/min), relieve pain with morphine and support BP. E: no evidence of AMI.

10 Years Collection of ANZCA Part II Anaesthetic Viva

MK Yuen & SW Ku

10 Years viva collection 2009

183

C: eospheageal perforation, pneumomediastinium...... (patient ended up required emergency OT for thoracotomy. If I didn't stop the warfarin, answer need to include use of FFP and risk of acutely reversing the warfarin. I also remember he asked me something about if stop enoxaprine 12 hours before peocedure, couldn't the patient under the silminar risk of thrombosis?)

Scenario 9
You are called to the emergency room as part of the trauma team to manage a 25 year old lady who was involved in a single car collision with a tree. On initial assessment she is awake and responsive, with a painful right chest with paradoxical movement on the right and a painful and angulated right thigh. How will you assess the extent of her chest injury? E: How will yo assess this patient? C: ATLS approach (book work) particularly pneumothorax, aortic arch injury, hypovolaemia. E: vital signs SpO2 94%, RR 26/min, BP 90/50, HR 120/min C: resusciate as hypovolaemia drips/ fluid/ x-match, chest drain, close monitoring and traction on femur to decrease pain and bleeding. E: drain showed fresh blood about 200ml/hour C: no immediate need for thoracotomy and reassess... E: someone found a pre-directive with patient showed Johovah's witness. C: Oh! Crisis situation. I want to change my management. To operate earlier instead of observing. Found out what blood components acceptable or unacceptable with patient. E: surgeon wants to do thoracotomy and fixation of femur at the same time. Would you allow? C: No. As it will cause too much blood loss. So thoracotomy first and fixation of femur later. Meantime give traction to decrease bleeding. Post-op keep patient sedated with intubation, ventilation and well oxygenation in ICU with close monitoring. Give supplement for Hb: Fe, folate and Vitamin. E: How do you give Fe? C: through oral E: it can be IMI as well. Do you aware of any new change in Johovah's witness belief recently? C: No. E: more relax with their belief of not taking blood. (Details in between questions forgotten, but it shows major flow of scenario)

10 Years Collection of ANZCA Part II Anaesthetic Viva

MK Yuen & SW Ku

10 Years viva collection 2009

184

ANZCA Anaesthetic Viva May 2005 Day 1 Dr. Dr Tang YK, PYNEH & Dr. Ng Denise, PWH
Scenario 1
A 45 year old 112kg, 180cm farmer presents as a day patient for nasal septoplasty and functional endoscopic sinus surgery (FESS). His history includes obstructive sleep apnea (OSA) and he is a smoker; he volunteers no history of other medical problems. How will you assess him with a view to anaesthesia for this procedure? He has not previously undergone a general anaesthetic. E: What are your concerns? C: I am now facing a morbid obesity man with co-morbidity for an ENT day surgery. My concerns are: Morbid obesity apart from OSA he has, I would like to find out if he gets aspiration risk, HT, IHD, cor pulmonale and restrictive lung disease, assess for difficult intubation. Regarding his OSA, I would like to find out its severity by reviewing report of sleep study, look for symptoms of neurobehavioural impairment, complication like HT, cor pulmonale. Is the patient on nocturnal CPAP? Any surgical intervention before? Regarding his smoking hostoruy, I would like to find out his pack year smoking, any associated comorbidities: COAD, HT, IHD, AF, cor pulmonale, his exercise tolerance, is anything that can be done to optimize his condition, like treating chest infection, bronchodilators, physiotherapy and stops smoking. Regards the case was listed as day surgery, I have reservation over this. E: Will you accept him for day surgery? C: I will assess according to patient, social and surgical factors. Regarding his medical condition, I will only accept him as hay surgery if he gets mild OSA and gets no comorbidity associated with smoking and morbid obesity, and with the condition of having the access for hospital admission as backup. Otherwise, Iw ill not accept him as a day surgery patient. E: (shown drug list) natrilix, atenolol, pecidine, statin. C: patient is also having HT and hyperlipidaemia. I would like to know if he gets peptic ulcer disease and make sure electrolyte normalize before operation. E: When the patient lie on the OT table, SpO2 only 94%, what will you do? C: I will confirm the accuracy of the pulse oximetry. Ask if patient get any respiratory symptom. Auscultate chest to look for wheeze and creptiation. Sit up patient to recheck SpO2. E: Pulse oximetry is functioning well. Patient gets no current resp symptom, normal finding on

10 Years Collection of ANZCA Part II Anaesthetic Viva

MK Yuen & SW Ku

10 Years viva collection 2009

185

chest auscultation. SpO2 was around 95% on sittinf uyp. What are you worrying about? C: I am worrying about supine hypoventilation associated with morbid obesity. I would like to take ABG to check for any hypercapnia. Since there is no active respiratory compromise, I will proceed to surgery. E: How are you going to anaesthetize him? C: preoperatively, I will administer acid prophylaxis, continue pepcidine, give Na citrate and metoclopramide premedication. Intraopearatively, Ill prepare theatre with drug, equipment, assistant and difficult intubation trolley. If he has signs suggesting difficult airway, Ill perform awake FOB intubation via oral route. E: your assessment reveals no significant risk for difficult intubation. C: I will still prepare for difficult intubations, with difficult intubation trolley and ENT surgeon standby. After put in iv and monitors, Ill give fentanyl 200mcg, propofol 3mg/kg, test ventilation, give suxamethonium 150mg and intubate with size 9 oral RAE tube. After securing ET tube, I will put in throat pack and position patients head then hand over to surgeon. E: What will surgeon inject ot aid hemostasis? C: In my institution, they give 1% lignocaine in 1:200000 adrenaline for vasoconstriction. E: what is the problem with the vasoconstrictor? C: systemic absorption leading to hypertension and tachycardia. Risk of myocardial ischaemia in patient with pre-existing IHD. E: surgery finish smoothly. Patient now in recovery, nurse inform you that patient is restless, SpO2 91%, BP 190/100mmHg. C: Ill go and asses patient. Support airway, breathing and circulation. Review monitors BP/pulse, SpO2, ECG. Assess consciousness of patient. Ask patient if he gets any discomfort. Since the patient is hypoxic, I would like to find out if it is upper airway or lower airway problem. Ill administer 100% O2, open up airway, assist ventilation if breathing was shallow, look for stridor, ascultate chest for any wheeze or crepitation. E: can you tell more detail about how you assess his airway? C: Ill administer 100% O2 via BMV, open airway by triple manuvre, remove foreign body if any, gentle suction to remove secretion, CPAP if laryngospasm, assist ventilation if breathing shallow. E: You find the throat pack is not removed. What is the practice in your institution to prevent this from happening? C: In my institution, throat pack is inserted by the anaesthetist, nurse will record it on white broad. At end of surgery, surgeon will remove it. Ill cross check by doing DL before reversal of patient, together with suctioning of oropharynx. Moreover, proper documentation in anaesthetic record. E: Why do you think that the BP is high? C: hypoxia and hypercapnia can lead to high BP. Moreover, patient has underlying HT. I would

10 Years Collection of ANZCA Part II Anaesthetic Viva

MK Yuen & SW Ku

10 Years viva collection 2009

186

like to review drug sheet if he had taken morning dose of antihypertensives. Look for other causes like pain, full bladder. E: Patient did has pain. What will you do? C: Ill use multimodal analgesia. Titrate with small doses of fantanyl, sit up patient, give O2 supplement and continue monitor patinets vital sign. Apart from fentanyl, Ill also give paracetamol, NSAID if no contraindication. I would like to rule out peptic ulcer disease in this patient. E: What are your contraindication for NSAID? C: Drug allergy, asthma, peptic ulcer disease, bleeding risk, renal impairment, dehydration, very cautions in elderly. E: patient now gets better and go back to ward. On second day, the nurse inform you that the patient get a crack in his tooth. What will you do? C: Ill first review record to see if any documentation of pre-existing dental problem. Review anaesthetic record for any difficult intubation, trauma to teeth during intubation or extubation period. Ill see the patient to get more information, also assess for any coexisting orophyarynx and facial trauma. I will also ask the surgeon if he had traumatize the teeth during removal of throat pad and inspection of oropharynx. E: what will you do afterward? C: If it is confirmed that the dental damage is from the intraopeative period, Ill refer patient to dentist, not admit fault, explain to patient what had happen, offer dental treatment and follow up patient. Fill in critical incident form, document clearly, inform department head and MPS. E: What can be done to prevent this from happening? C: It starts with identify patient at risk and prevention. Inform risk to high risk patient. Refer for dental treatment preoperatively if time allows. Intraoperatively, avoid GA. If GA is a must, use teeth protector and consider other measns of intubation, like FOB, trachlight. Care during both intubation and extubation period. Check the teeth condition both after intubation and extubation. E: What is your criteria for day surgery patient? C: Ill have patient, social and surgical consideration (bookwork). BELL

10 Years Collection of ANZCA Part II Anaesthetic Viva

MK Yuen & SW Ku

10 Years viva collection 2009

187

Scenario 2
You are part of the trauma team called to resuscitate a 55 year old male farmer who has arrived in the Emergency Department after a tractor accident. There was a delay finding him and transferring him to hospital. He has had one litre of saline and 10 mg of intravenous morphine during transfer. On examination: o He is confused, but moving his upper limbs normally, with reactive pupils. Clinically his chest is clear. o o He has truncal obesity with a suggestion of abdominal fullness. His upper back and spine appear normal, but he has bruising and pain over his lumbar spine and sacrum with pain on examination of his pelvis. o o He has perineal and scrotal haematomas. He has a fractured left femur, as well as the compound fracture of his right lower leg. o o o o o Obervations: Pulse = 120 B/P = 90/60 SaO2 = 98% Temperature = 33.5oC

What are your immediate management priorities? E: How will you assess this patient? C: Ill activate the trauma call. Go and see patient in A&E. Assess, resuscitate and apply monitors according to ATLS. Start with primary survey follow by secondary survey. In primary survey, Ill assess airway and protect cervical spine, check whether patient can talk, remove foreign body if any. Give 100% O2. Assess breathing adequacy and auscultate chest to rule out ;life threatening condition, like tension pneumothorax. For circulation, ensure 2 large bore iv, take blood for routine and cross match. Since patient has already received 1 litre NS, but he is still hypotensive and tachycardia, Ill give another litre of warm HS. Apply external pressure to any active bleeding site. Look for source of bleeding and reassess response to fluid challenge. Assess neurological state by pupil response and look for any major limb disability. After initial stabilization, Ill proceed to secondary survey, take AMPLE history, head to toe examination. E: Good. The patient has no airway and breathing problem, can you tell me more about the circulation problem in this patient? C: This patient is haemodynamic unstable. He has bleeding from # femur and right LL compound fracture. I would like to apply traction and external splintage to decrease further blood loss. Abdominal sign suggest possibility for intraabdominal organ injury and

10 Years Collection of ANZCA Part II Anaesthetic Viva

MK Yuen & SW Ku

10 Years viva collection 2009

188

hemoperitoneum. Ill like to look for any tenderness and ask surgeon to perform USG abdomen. He also has the possibility of pelvic fracutre. Ill rock the pelvis to look for instability. Inform OT for possibility of operation. E: apart from this, any way to assess pelvic #? Will you do any x-ray? C: Ill do the trauma series x-ray lateral neck, AP chest and AP pelvic. E: suppose USG abdomen reveal no abnormality, pelvic x-ray reveal pelvic #. What will you do to reduce bleeding? C: Ill apply external pelvic blinder. Ill discuss with surgeon for external fixation in AED for temporary measures. Discuss with surgeon for more definite treatment like ORIF in OT or embolization in radiological department. E: Will you give blood? C: If patient still doesnt response to further litre of warm HS, Ill start to give blood. Ill give crossmatch blood if it is available. E: How long would it take to get x-match blood in your hospital? C: It depends on whether Ab is present in type and screeen. If Ab is ve, x-match blood can be obtained very quickly within 10 minutes. If Ab is +ve, it may take up to one hour. E: since paitnet just arrive AED, type and screen is not yet available, what will you give? C: O -ve blood. E: What do you think about his temperature? C: Hes hypothermic, likely resulted from delay retrieval. Ill actively warm him up by warming up the resuscitation room, administer warm fluid, use forced air warmer over his body, continue to monitor his temperature. E: Whats the problem with hypothermia? C: It is associated with coagulopathy, which will aggravated the preexisting bleeding from trauma. Moreover, it is associated with delay wound healing, surgical infections, myocardial ishcaemia. E: Orthopaedic surgeon decided to do embloization in radiology department. What are your concerns? C: My concens are 1. Stablilization of the patient before transfer 2. Transfer with monitoring, adequate equipment, drugs and personnel 3. remote anaesthesia with unfamiliar environment and staff, limited access to patinet, hypothermia, radiation risk 4. prepare for massive bleeding, ensure blood and blood product available, set up rapid infusion device and warming device 5. Alert OT for possibility of open surgery if failed embolizaiton. E: How will you prepare the transfer? C: Ill do it according to college guideline. Ensure patient stabilize before transfer. Ensure

10 Years Collection of ANZCA Part II Anaesthetic Viva

MK Yuen & SW Ku

10 Years viva collection 2009

189

adequate personnel, include people to hold up life and have a planned route. Effective communication with radiology department, O2, resuscitation equipment and drugs, monitors. E: How will embolization helps? C: It can embolize the iliac vessels and prevent further bleeding E: What do you think it is more likely to be arterial bleeding or venous bleeding? C: both are possible. Im not sure which one is more likely. Possibly, more likely form venous bleeding in view of the more labile venous vasculature. E: How is the patient anaesthetized for embolization? C: It can be done under LA or GA. In view fo his unstable hemodynamic, Ill perform it under GA. E: Where will you intubate him? C: Ill intubvate hime in AED before transfer. E: How? C: Assess airway difficulty. Prepare equipment, drug and have 2 assistants. PreO2, RSI, cricoid pressure, in-line immobilization. (detail of drug use). Confims ETT position, sedate patient and paralysed him. Portable ventilator for ventilation, ambu bag as backup. E: suppose now the radiologist is doing embolization. Suddenly, there is oozing from the leg. What will you do? C: Ill stop the radiologist. Simultanwously find out cause and resuscitate. I would like to see if there is any displacement of the splint. Look for any evidence of generalized bleeding suggesting coagulaopathy. Review hemodynamic and support circulation with fluid and blood if needed. E: Will you order any investigation? C: Ill take blood for CBC, clotting profile and fibrinogen. E: Blood result: Hb 9, decreased platelet, deranged clotting, normal fibrinogen. C: patient is still anaemic. The thrombocytopenia and coagulopathy can be due to dilutional cause from blood transfusion. E: How will you manage? C: Ill ensure normothermia, give 6 units of platelet and 6 units of FFP. E: will you consider novoseven? C: Ill consider if all the conventional management of coagulopathy fails and there is continue ongoing bleeding. E: What is the mechanism of it? C: (bookwork) BELL

10 Years Collection of ANZCA Part II Anaesthetic Viva

MK Yuen & SW Ku

10 Years viva collection 2009

190

Scenario 3
A healthy 42 year old woman is scheduled for a laparoscopically assisted vaginal hysterectomy. Past History: o o o o o o o 2 uneventful vaginal deliveries, Tonsillectomy age 16, with prolonged PONV D&C age 22 for miscarriage Medications Stopped OCs three months ago Oral iron for anaemia Allergies Amoxicillin caused a rash Smoking Nil

The surgeon tells you this will be a prolonged case. What are your principal concerns in anaesthetising this woman? E: What are your concerns? C: I have my patient concern, anaesthetic concern and surgical concern. Regarding patient concern, she has taken OC pills previously which may increase DVT risk. I would like to know the cause of anaemia, is it from menorrhagia or other cause? Current Hb level and whether she has symptom for anaemia. Regarding anaesthesia concern, I would like to find out any previous anaesthetic problem. Retrieve old anaesthetic notes. Find out more detail about allergic history, was there any evidence of anaphylaxis other than just skin rash. Regarding surgical concern: problem associated with laparoscipic procedure, problem associated with prolonged surgery, like DVT, hypothermia, pressure sores, positioning problem, I would like to discuss with surgeon about surgical durations and diffiuclteis. If very prolohged and difficult operation is anticipated. Ill discuss with surgeon if open operation will be more appropriate. E: What are the problems associated with laparoscopic surgery? C: (bookwork) E: Tell me more detail aobut nerve injury in lithotomy position. C: (bookwork) E: Intraoperatively, patient saturations gradually decreased to 91%. What will you do? C: This is a potential dangerous situation. Ill confirm diagnosis, simultaneous assess and gind out cause. Ddx include equipment problem, ETT problem, lung problem and circulatory problems. Ill administer 100% O2, hand ventilate to exclude ventilation problem and feel for chest compliance. Inform surgeon to deflate abdomen and level table. Suction ETT to exclude kinking or obstruction. Ascultate chest to look for asymmetric air entry suggesting endocroncial intubation or pneumothorax, ascultate for any wheeze or crepitations. Palpate pulse and

10 Years Collection of ANZCA Part II Anaesthetic Viva

MK Yuen & SW Ku

10 Years viva collection 2009

191

review BP for circulatory problem. E: You notice there is decreased air entry on left chest. Tracheal central and no distended neck vein. C: It suggest right endobroncial intubation. Ill check ETT marking, withdraw ETT under direct vision by DL. Ausculate chest to reconfirm tube position. E: Why is there endobroncial intubation? C: Cephalic shifting of carina by pneumoperitoneum and trendelenburg position. E: Patient saturation improved. 4 hours into operation, you notice gross facial and neck swelling. C: Ill like to find out the cause. My ddx includes anaphylaxis look for skin rash and brochospasm, subcutaneous emphysema palpate for surgical emphysema or dependent venous congestion. Ill also inform surgeon and would like to find out when will they finish operation. E: No evidence of anaphylaxis or subcutanwous emphysema. Surgeon is going to finish OT. What will you do? C: Ill inform surgeon about the problem, ask them to finish asap. My concern is there may be associated airway edema. Ill head up patient, administer dexamethasone 8mg iv, perform leak test, assess airway edema by FOB. E: Will you extubate? C: If there is evidence of leak and no evidence of airway edema, Ill extubate him with tube exchanger in situ. Otherwise, Ill not extubate him immediately. E: What are the problem with tube exchanger? C: It may cause patient discomfort. Diffiuclt for proper mask sealing. However, in view of possibility of airway edema and postoperative obstruction, it can act as a guide for reintubation or use for jet ventilation if needed. E: How long do you think the edema will subside? C: not quite sure. May be 30 minutes to 1 hours. E: suppose you eventually extubate her successfully. 2 days later, nurse inform you patient get numbness in her hand. C: Ill review the note to find out if patient has nay preexisting neuropathy or predisposing factors for nerve injury, like DM, uraemia, obesity, carpel tunnel dsyndrome. Ill review anaesthetic and surgical note for intraoperative positioning details. Ill see patient to get detail history physical examinations and relevant investigation. I would like to find out the time course of numbness, perform full neurological examination. E: What neurological exam will you do? C: Ill inspect for any muscle atrophy or edema. Test power, reflex, sensory and coordination. (another examiner request the candidate to voice out the consideration of stroke in the assessment)

10 Years Collection of ANZCA Part II Anaesthetic Viva

MK Yuen & SW Ku

10 Years viva collection 2009

192

E: you find out that there is decrease sensation in her right little finger and medical half of ring finger. What do you think? C: the finding suggest ulnar nerve involvement. E: How can you confirm it in P/E? C: Test for any decrease in sensation in medical side of palm too. Look for claw hand deformity. Ulnar nerve supply all small muscles of hand except thenar muscle and lateral 2 lumbricals. Therefore, Ill test for finger abduction and adduction. E: There is weakness in finger abduction and adduction. C: There is evidence of ulnar nerve palsy. Ill refer him to neurologist for nerve conduction study, documentation, follow up him, inform MPS and department head, fill in critical report, quality assurance, reassure patient. E: what do you look for in nerve conduction study? C: look for evidence of delay conduction and demyelination. E: what else will you do apart from NCS? C: EMG? BELL

Scenario 4
You are called by a cardiologist to anaesthetize a 54 year old, 120kg female who suffered a myocardial infarction 8 months ago and now requires an I.C.D pacemaker for recurrent tachyarrhythmias and episodes of symptomatic bradycardia. What questions would you ask the cardiologist? E: What questions will you ask the cardiologist? C: I would like to ask further detail regarding 1. His previous MI How severe was it? Any medical or interventional treatment? Any complications? Current cardiac status? 2. Arrythmia problem whats the cause? Is it a complication from MI? Its Control? Any SE like embolic phenomena por CVA? SE from amiodarone? Whether patient required temporary pacing? 3. Where will the cardiologist perform the procedure? As it will affect my anaesthetic consideration. 4. Any other co=morbidity of the patient? E: Patients MI problem settled. Temporary pacing inserted 3 days ago. Cardiologist will perform it in cardiac catheterization lab. Have you perform this procedure before? C: No

10 Years Collection of ANZCA Part II Anaesthetic Viva

MK Yuen & SW Ku

10 Years viva collection 2009

193

E: How will you prepare for this procedure? C: Ill prepare the patient and the CC lab. Regarding the patient, Ill optimize patient with input from cardiologist. Continue amiodarone, ensure normal electrolytes, ensure proper functioning of the temporary apacemaker. Transfer patient with monitors. Regarding the CC lab, I would like to make sure it is set up according to college guideline. Prepare equipment, drugs and personnel. In particular, Ill have the percutaneous pacemaker, defibrillatior standby and checked beforehand. Prepare resuscitations drug and trolley. E: What resuscitation drug? C: In particular, Ill have amiodarone for tachyarrhythmia, isoprenaline and adrenaline for bradyarrhythmia. E: Echo report show EF 20%. Cardiologist said no further optimization, what do you think? C: Patient has very poor cardiac function. Having sent hat there is no further room for optimization, Ill accept him for the perocedure as it is intended to help to improve patients cardiac function. But Ill inform patient about the increased perioperative risk. E: Whats your choice of anaesthetic? C: Choices include LA, conscious sedation or GA. Ill discuss with the cardiologist if they anticipate prolonged and difficult procedure. If not, my choice will be conscious sedation in view of the poor cardiac function. E: What drug will you give before putting in the implant? C: Ill give small boluses of midazolam and fentanyl for sedation and analgesia. E: What else will you give? An implant is going to be put in? C: (after some struggle) antibiotic. (another candidate was asked about details in what kind of monitors to be used) E: do you know what will the cardiologist do to test the function of the defibrillator? C: not sure. E: induce VF. How will you prepare for that? C: I anticipate patient may LOC during the testing, the implanted defibrillator may fail. Therefore, Ill ensure the external defibrillator is functioning properly, have close communication with cardiologist, inform patient beforehand, ensure adequate sedation and analgesia, support airway, breathing and circulation when patient lost consciousness. E: Midway into the procedure, patient complain SOB, what will you do? C: Ill assess consciousness of patient. Ask patient if he get any chest pain. Give 100% O2 to patient, support ABG, review monitors for BP/pulse, SpO2 and ECG. Inform cardiologist. Causes include upper airway problem, lung problem or cardiac problem. E: patient is conscious, talking without difficulty. C: I would like to assess if the SOB is from lung or cardiac problem. Ill auscultate chest to look for wheeze suggestive of bronchospasm or anaphyaxis, crepitation for pulmonary edema or aspiration. Also look out for surgical complication leading to SOB.

10 Years Collection of ANZCA Part II Anaesthetic Viva

MK Yuen & SW Ku

10 Years viva collection 2009

194

E: Do you think what surgical complication will happen? Do you know where will the cardiologist insert the wire? C: In great veins. E: They usually insert it via left subclavian vein. C: There is possibility of penumothorax. Therefore, Ill look for asymmetrical air entry and palpate for trachea position. E: Trachea is central. Air entry equal both side. C: pneumothoraix is unlikely. Another surgical complication will be cardiac tamponade (hemopericardium may be a more proper term) from perforation of wire through the cardiac muscle. E: how will you look for it? C: look for distended neck veinm muffled heart sound, ask cardiologist to perform echocardiogram, E: There is distended neck vein. Echocardiogram showed pericardial fluid. BP borderline. What will you do? C: patient has cardiac tamponade which require pericardiocentesis. E: How will you perform? C: explain to patient. Aseptic technique, ensure ECG monitor throughout. LA. Landmark 1cm inferior to left xiphoid chondral angle. Directed towards tip of scapula. Aid by echo, aspirate for blood. E: you aspirate blood. What will you do then? C: Ill put in the catheter via the catheter over needle technique. Connect to 3 way tap. E: any other way to insert the catheter? C: not sure. E: by Seldinger technique. You continue to draw blood. C: Ill inform cardiothoracic surgeon to assess for needle of surgical hemostasis. Inform OT to prepare for emergency operation. Ensure blood and blood product available. Continue to support patients hemodynamic with fluid and blood if needed. E: suppose the procedure eventually finished. 6 months later, patient come back for thyroidectomy. How will you manage his AICD? C: I would like to check its function. Ask cardiologist to switch off the defibrillator function. Regarding the pacemaker. I would like to first find out what mode is it. E: The pacemaker is in AV synchronized mode. C: I would like to have it converted to DOO mode. My concern is the problem of electromagnetic interference. E: where will you put the diathermy pad? C: At least 15cm away fro the AICD. Put in a way that diathermy pathway is away from the AICD. I will put in it right arm.

10 Years Collection of ANZCA Part II Anaesthetic Viva

MK Yuen & SW Ku

10 Years viva collection 2009

195

E: correct BELL

Scenario 5
A 40 yr old woman with long standing Crohns disease presents for Redo Ileostomy. She has severe osteoporosis with an old fracture of 3rd Lumbar vertebral body. She suffers chronic Abdominal and back pain. Rx. Azathioprine, Prednisolone (15mg/Dy), Tramadol (200mg/bd) Describe your peri operative management of this patient's steroids. She will need at least the equivalent of prednisolone 15mg/dy (= 60mg hydrocortisone) daily until oral intake resumes. She may well cope with her normal oral intake throughout. E: what is your perioperative management of the steroid? C: Ill continue her usual oral steroid preoperatively, extra 25mg hydrocortisone on induction. The 100mg/day for 48hours Continue IV steroid till resume oral feeding. E: Whats your rationale for your regime? C: 25mg Hydrocortisone is about the normal daily requirement for steroid. For patient undergoing major surgery, she require 4 times the amount of normal daily requirement to cover the stress response. Therefore, I give the maintenance of 100mg/day. E: Do every patient on steroid require coverage? C: No. If the daily dose is less than 10mg/day, coverage is usually not needed. Moreover, the dose require is also varied with extent of surgery. On the other hand, if patient had long term steroid use stopped within 3 months, she also need steroid cover. E: What other perioperative situation will steroid be useful? C: Dexamethasone to prevent PONV, reduce airway edema, reduce brain edema in patient with brain tumour, for patient with acute spinal cord injury. E: What are the disadvantage of giving steroid cover? C: The advantage is to cover the adrenal suppression. However, steroid use is associated with its side effect like delay wound healing, increase postoperative infection, immunosuppression, DM, HT etc. E: What will be your postop pain management for this patient? C: multimodal. Postop pain management starts from intraoperative period. If there is no contraindication, Ill perform thoracic epidural. E: What are the advantages of thoracic epidural? C: superior analgesia, decreased DVT, enhance bowel function recovery, decrease postop respiratory morbidity and the likelihood of decrease cardiac morbidity.

10 Years Collection of ANZCA Part II Anaesthetic Viva

MK Yuen & SW Ku

10 Years viva collection 2009

196

E: Are the advantage proven? C: According to the MASTER trial, thoracic epidural for at least 2 3 days was shown to decrease postoperative respiratory failure. However, there is no definite evidence to show the decrease in periop cardiac morbidity. E: If patient refuse thoracic epidural, what other means? C: Parenteral analgesia, which will include morphine PCA. Consider to add ketamine infusion. E: What are your setting of PCA morphine. C: 2mg bolus, 5 min lockout. Regular review for SE and efficacy to readjust regime. (another candidate was asked by esaminer if she would like to have background infusion) E: How about ketamine? C: Initial bolus of 10 20mg. Infusion 0.1 0.2 mg/kg/hr. E: Suppose patient accepts thoracic epidural. However, patient complain of pain in the ward few hours later. C: Ill go and assess patient. Find out site and severity of pain. Review record for analgesia given. Check catheter site and check if machine is properly working. E: you find that the catheter is now 4cm at skin. C: review anaesthetic record for initial marking. E: 10cm C: the catheter is slipped out. I would like to remove it and convert to other method of analgesia. E: You find that LMWH was given 2 hours ago. C: I will keep the catheter while continue monitor the neurological function of LL. Ill follow the ASA guideline, only remove the catheter 12 hour after last dose. Ensure the next dose is more than 2 hours after removal. Ill also take blood for CBC and clotting before removal. Explain to patient about risk of epidural haematoma. E: What will you give after stopping the epidural infusion? C: Manage her pain by multimodal mean. E: OK, as youve said. 2 months later, surgeon inform you that the patient still have persistent significant abdominal pain. What will you do? C: Ill like to ask patient to come back and see her in pain clinic. I would like to take a detail history, P/E and relevant investigation. I would like to find out more detail pain history regarding precipitating factor, quality of pain (site, radiation, character, reliefing factor, severity (include functional impairment) and treatment given. Also social history and medical history. E: pain is similar to preoperative condition. Still on tramadol 200mg bd. GP has a letter with her suggesting add antidepressant. What do you think? C: I would like to find out why the GP want to start her on antidepressant. Has she had depression symptom? Is the pain neuropathic in nature? E: How about any drug interaction?

10 Years Collection of ANZCA Part II Anaesthetic Viva

MK Yuen & SW Ku

10 Years viva collection 2009

197

C: Tramadol will have drug interaction with TCA, MAOI, SSRI. I would like to stop tramadol for BELL

Scenario 6
You are administering a general anaesthetic to a woman at 38 weeks gestation for emergency caesarian section. The baby is delivered before the paediatrician arrives and after about one minute, you are asked to help resuscitate the baby. You observe that the baby: o o o o o o Is about 3kg has not cried and has irregular resps is cyanosed all over grimaces during suction of the airway is limp and not moving the heart rate is 90bpm

How would you calculate the Apgar score for this baby? E: What is the apgar score? C: 3 E: what is the use of apgar score? C: AN integrated assessment of different physiological parameters including neurological, cardiac, respiratory symptoms E: What other use of apgar score? C: prognostic for future neurological outcome particularly the score at 5 mins E: How would you resuscitate this baby? C: First of all I want to make sure the mothers condition is stable. If she is stable, I would help by resuscitate according to ABC. Bag and mask ventilation and intubate the child early for 100% O2, IPPV + suction. E: A few days later the baby come back for laparotomy for duodenal atresia. How would you anaesthetize her? C: Preop assessment first. Look particular for any other congential abnormalities. E: The baby has no other problem other than duodenal atresia. C: Optimize first, baby should have NG tube inserted and fluid and electrolyte status optimized beforehand. E: What kind of gastric tube you are going to use? C: sorry, I dont know. E: How are you going to induce this patient? C: attach monitors, iv, skillful assistantRSI with circoid pressure.

10 Years Collection of ANZCA Part II Anaesthetic Viva

MK Yuen & SW Ku

10 Years viva collection 2009

198

E: Do you think cricoid pressure is effective in this age group? C: we are not certain about how effective in this age group. However, since it does not cause harm, I will still apply it. E: Your trainee registra would like to intubate this baby and you let him to do it. Soon after he intubate the baby, SpO2 falls to 88% and continue to fall. And pulse rate falls to 90 bpm. What will you do? C: urgent situation. Ask my trainee to give atropine 20mcg/kg. Turn on 100% O2, hand ventilate with T-piece. Most likely cause endobronchial or endoesophageal intubation. I would like to exclude this and other causes. Is there ETCO2tracing? E: No. C: breath sound? E: cant hear. C: extubate and bag and mask entialtion, reintubate. E: now HR 60 bpm. What would you do? C: give atropine another dose? E: still bradycardia, HR 50 C: recheck airway and breathing, feel pulse E: HR still 50 C: Oh, I need to do external chest compression! (eventfully I remember), roughly 1/3 AP diameter of chest, 100 compressions per minute, at lower part of sternum. (beware it is very easy to forge to do chest compression on bradycadiac small children) BELL

Scenario 7
A labouring healthy, term, nulliparous woman, of body mass index 40, is transferred to the operating room for caesarean section (class 2) because of fetal compromise. She consents to a spinal anaesthetic. Insertion is difficult and despite your subsequent manipulation of posture, after 25 minutes the block has spread cephalad (loss of sensation to cold) to T9. The midwife tells you the fetal heart rate is now 70 bpm. What type of anaesthetic are you now going to suggest to this woman and why? E: What is your management? C: discuss with obs surgeon urgency of surgery. ? sustained fetal braducardia or type I or II deceleration. If still have time, convert to EA, this is the safest route for both mother and fetus. If urgent surgery is needed, management depend on findings of airway assessmen and past anaesthetic history. If no suspicious of difficult airway, RSI. IF suspicious of difficult airway, EA

10 Years Collection of ANZCA Part II Anaesthetic Viva

MK Yuen & SW Ku

10 Years viva collection 2009

199

or FOB awake. E: what is class 2 Caesarean section? C: I dont know. E: surgery is urgent according to obs surgeon, need CS now. Patient told you she has GA a few years ago, no body told her there was any problem. What would you do? C: GA + RSI with adequate preoxygenation and difficult intubation trolley standby. E: How long will you pre-oxygenate? C: 3 minutes with tight fitting O2 mask. E: surgeon with knife at hand and want to cut early, ask you no to pre-oxygenate. What will you do? C: I insist on pre-oxygenation. Because pregnant woman have high O2 consumption + potential difficult airway. E: Any way to achieve quicker pre-oxygenation? C: Ask the patient to take vital capacity breath of 100% O2 for quicker preoxygenation and denitrogenation. E: How do you know preoxygenation is adequate? C: look at expiratory O2 conc if this monitor available. E: Direct laryngoscopy, you see grade 3 larynx. What predispose to diffiuclt intubation in pregnant women? C: airway edema + increased vascularity. E: you past the ETT bindly and find there in no ETCO2 tracing. What would you do? C: 5 points auscultation + esophageal detection bulb if present. E: no breath sound on auscultation. Esophageal detection bulb no inflated. C: esophageal intubation. Remove ETT. What is the SpO2? E: starts to fall. C: bag and mask ventilation for per-oxygenation next intubations attempt. E: you notice patient start to breathe at that time. C: assess whether it is easy to mask ventilate. If yes, sux 50mg for another intuition attempt. If not easy, will not give sux. E: mask ventilation is difficult. SpO2 88% now. C: insert oropharygeal airway, optimize triple maneuver. Check whether assistanct is correctly applying the cricoid. E: Still not easy to ventilate. C: insert LMA, call for extra help. E: what kind of LMA you choose? C: ordinary LMA. I am not familiar with intubating LMA, only use once or twice, not want to use in emergency. E: still difficult and suboptimal ventilation through LM<A.

10 Years Collection of ANZCA Part II Anaesthetic Viva

MK Yuen & SW Ku

10 Years viva collection 2009

200

C: remove cricoid pressure and adjust LMA position. E: Good, you can now have good ventilation. SpO2 gradually rise to > 96%. Surgeon say FHR is 50 bpm, want to start quickly, will you let them start? C: I want to use FOB to guide # 6 cuffed OETT through LMA to secure airway first before let them start. E: FOB not available. C: I would let them start if I have confident the LMA is in good position, and I should reapply cricoid pressure already by now. Assist ventilation before full spontaneous ventilation resume. E: Yes, it is in good position. C: Deepen volatile anaesthesia to avoid light anaesthesia when surgeon cut. O2:N2O 50:50%if no desaturation. E: What would you tell the surgeon? C: finish quickly because I do not have patients airway protected. BELL

Scenario 8
A 45 year old female presents for a laparoscopic cholecystectomy. She was seen in the preoperative assessment clinic by an anaesthetic colleague, who noted the following findings: o o o o Obesity (BMI 32kg/m2) Diabetes mellitus, present for 5 years Medications: metformin, glimepiride (Amaryl), and atorvastatin Blood sugar level (via glucometer during clinic visit): 16mmol/l

The colleague arranged for all her to medications to be omitted on the morning of the day of surgery. You now meet this patient for the first time in the anaesthetic bay just prior to the planned operation. What will be your plan for further blood glucose management in this lady? C: patient has sub-optimal glycaemic control in preop assessment. Likely to have increased sugar now particularly OHA are omitted today. Check Hstix now. Check how long patient has been fasted, ask for hypoglycaemia symptoms. If Hstix > 17, cancel case as it is elective. If Hstix < 10, proceed with DKI drip as maintenance fluid. If Hstix 11 16, insulin infusion, proceed with risk discussed with patient. E: Hstix is now 15. C: start insulin pump, close monitor of Hstix every 1 2 hours to adjust infusion rate. E: How much would you infuse? C: 3 4 units per hour of actrapid.

10 Years Collection of ANZCA Part II Anaesthetic Viva

MK Yuen & SW Ku

10 Years viva collection 2009

201

E: do you want to add some glucose. C: Yes. I would have 1/2 : 1/2 solution infused as maintenance while use NS or HS for volume replacement intra-op. E: How much would you give? C: 1/2 : 1/2 solution 80 90 ml/hr E: Good. What are the different types of OHA. C: There are 3. Sulphonylurea, acarbose and forgot the group name but it includes metformin. E: The metformin belongs to biguanide. What are their mechanism of action? C: sulphonuylurea increase insulin release from pancreatic B cell. Acarbose decrease post prandial blood sugar peak by slow absorption. Sorry, forget about mechanism of biguanide. E: She is a singer. She does not want ETT. She ask about LMA because her friend has used it for surgery a few years ago. What would you tell her? C: In my opinion, KMA is not a safe choice of airway management for this lady. It is because of obesity with aspiration risk and difficult to ventilate through LMA for obese patient. Pneumoperitoneum + abdominal surgery predispose to aspiration. I would use different means To try to decrease risk of post-op hoarseness or sore throat. E.g. smaller size tube, gentle technique, do it myself rather than a junior doing it. Check cuff pressure intermittently + water soluble jelly for lubrication. Hopefully to patient can understand my concern and agree for ETT. But I would also explain the risk of post-op sore throat or hoarseness cannot be eliminated. E: she accepts and agree for ETT. During surgery, SpO2 suddenly falls from 99% to 81%. What would you do? C: Urgent situation. Verify by colour and pulse oximetry position. Inform surgeon and ask them to stop pneumoperitoneum. 100% O2 manual bag ventilation. Check BP, ETCO2, Paw. Any doubt, eliminate machine and circuit cause by using ambu bag. Suction ETT, feel for the trachea, auscultate E: Trachea central. Breath sound decreased over left side. C: what is the percussion note? BP? E: difficult to percuss because of Obesity. BP stable. C: suspect endobronchial intubation, particular in pneumoperitoneum. DL to adjust ETT by direct vision. E: pull out ETT 2cm, still SpO2 <85%. C: any rhonchi, creps? E: No C: Err(dont know the answer). Use FOB to precisely adjust ETT position? E: well, now you find trachea deviated to right. C: tension pneumothorax. Due to low SpO2, needle thoracocentesis followed by chest drain by surgeon. E: How to insert chest drain?

10 Years Collection of ANZCA Part II Anaesthetic Viva

MK Yuen & SW Ku

10 Years viva collection 2009

202

BELL

Scenario 9
You are asked to anaesthetise a 75 yo woman for a closed reduction of her left Colles' Fracture (distal radius). The patient is frail and osteoporotic, weighs 50 kg with Chronic Airways Disease, treated hypertension and depression. She is a known alcoholic. Medications: o o o Budesonide Enalapril Fluoxetine

What are the alternative options for anaesthesia in this patient, and their relative merits? C: IVLA E: Why do you choose this technique? C: short procedure that can be adequately covered by this block. Avoid GA in a COAD patient. E: Why do you want to avoid GA in a COAD patient? C: central depression by anaesthetic impairs respiratory muscle co-ordination and cause resp depression. Stimulate airway by ETT may cause bronchospasm. GA decreased FRC and increased V/Q mismatch etc. E: what other technique can you use? C: e.g axillary nerve block. E: why dont you use this block? C: there are other complications e.g. nerve injury by needle trauma, haematoma due to axillary vessel puncture. LA toxicity if intravascular injection. E: What nerve are often missed in axillary nerve block? C: musculocutaneous and intercostobrachial nerve. E: what are the problems of other alternative of brachial plexus block for this patient? C: other options: interscalene block, but prone to miss C8, T1 lower nerve roots. Supraclavicular block: high incidence of pneumothorax particularly bad for COAD patient. E: what is your choice of drug for this Biers block. C: lignocaine 3mg/kg maximum. E: you want to choose lignocaine? We usually use prilocaine. C: yes, my locality dont have prilocaine. Prilocaine also have the problem of methaemoglobinaemia if overdose. E: If there is no prilocaine or lignocaine, you trainee say should we use levobupivacaine or

10 Years Collection of ANZCA Part II Anaesthetic Viva

MK Yuen & SW Ku

10 Years viva collection 2009

203

ropivacaine? C: No. They have higher risk of cardiotoxicity. If lignocine or prilocaine not available, I will choose another technique e.g. axillary nerve block. E: How to do a Biers block? C: small gauge iv cannula at injured arm. Another large cannula in good arm. Raise the arm. Apply double cuff tourniquet. Use esmarch bandage to exanguinate limb. Then switch on tourniquet. E: You use esmarch bandage? Patient has Colles fracture. C: I will avoid the fracture site. E: How much will you inject? C: 30ml E: why 30ml? C: Err(basically I forget) E: Why use double cuff? C: If tourniquet pain occurs can inflate the lower cuff to increase tourniquet time. E: sequence of cuff inflation? C: (scared, answered in a mess) E: pressure of tourniquet? C: 75 100 mmHg above SBP. E: postop in recovery patient get confusion. DDx? C: (bookwork) E: any other? C: Err E: How about if you notice your trainee give the patient a dose of tramadol sometimes ago? C: then another Ddx is serotonin syndrome. E: what drugs interact with tramadol to give this? C: SSRI, pethidine etc. E: What manifestation? C: CVS and neuro system. E: How to prevent post-op confusion? BELL

10 Years Collection of ANZCA Part II Anaesthetic Viva

MK Yuen & SW Ku

10 Years viva collection 2009

204

ANZCA Anaesthetic Viva May 2005 Day 2 Dr. Janet Wu, YCH & Dr. Katherine Lam, Dr. Andrea Yu, PWH
Scenario 1
An 8-year-old girl with cystic fibrosis (CF) presents with a worsening chest infection. Her physician has requested long-term intravenous access, as venous access has been an increasing problem for her and she has none at present. The surgeon has added her to his list for insertion of a "portacath". She weighs 21kg, has a productive cough and in room air her saturation is 91%. Her sputum is infected with Burkholderia cepacia. She is receiving regular nebulised corticosteroids, bronchodilators and antibiotics and regular physiotherapy. What is CF, and what are the significant anaesthetic issues in this patient with CF who requires a general anaesthetic for insertion of a portacath? E: What is CF? C: (bookwork) E: tell me about the sweat in CF. C: (bookwork) E: how to prepare patient? C: (bookwork) E: What is your anaesthesia plan? C: options are LA + sedation (difficult for 8 years old) or GA. I will do GA with endotracheal tube intubation. E: Why not LMA? C: LAM is acceptable, I generally prefer ETT because(whatever you prefer) E: Would you choose cuff or uncuff tube? C: I choose uncuff tube because E: What are the issues with cuff vs uncuff ETT in Children? C: (bookwork) E: How do you choose the appropriate size of ETT in children? E: How do you make sure you have choose the right size tube? E: How to ensure safe use of cuff ETT in children? E: How do you set your ventilator setting in this case?

10 Years Collection of ANZCA Part II Anaesthetic Viva

MK Yuen & SW Ku

10 Years viva collection 2009

205

E: Intraop, gradually increase in ETCO2, SpO2 97% on 50% O2, BP 98/65. What do you do? C: ABC, vital signs, machine, circuit, ETT, patient factors. Review TV/ventilator setting, temperature. E: There is decreased air entry on one side. C: could be endobronchial intubation, do DL to adjust tube, suction out sputum. E: Still no air entry one side, BP dropping. C: could be pneumothorax, feel for tracheaetc. E: yes, trachea is deviated. C: (management of tension pneumothorax) BELL

Scenario 2
A 54 year old woman presents for a vaginal hysterectomy and repair. She was diagnosed as having multiple sclerosis 12 years ago. Her medications are: o o b interferon 8 million units alternate days Paroxetine

She is a nonsmoker. She has a past history of allergy to sulphurs. She is anxious to talk to you as after her last anaesthetic she vomited for three days. What do you advise her with respect to her anaesthetic choices and management of her post operative nausea and vomiting? C: options are GA or epidural. E: She is concern about the effect of anaesthetic on her MS. C: (bookwork, SA increase risk of relapse) E: what is the level of evidence that SA increase risk of relapse? C: cant remember in details, think its based on case reports? E: so how would you advice on the anaesthetic technique? C: Epidural if not contraindicated. E: What is this patients risk of PONV? C: (Apfels criteria x3: 60%) E: How would you decrease this risk? C: RA, hydration E: Does RA completely eliminate this risk? C: No, but much reduced. E: would you give antiemetic prophylaxis?

10 Years Collection of ANZCA Part II Anaesthetic Viva

MK Yuen & SW Ku

10 Years viva collection 2009

206

C: no if regional technique. E: you said she is high risk for PONV. C: Not my routine if doing regional, cost-benefit reason. E: suppose you decided to give, what would you give? C: Ondansetron, minimial side effect, good efficacy. E: NNT for Ondansetron to prevent PONV? C: cant remember, range of 4 8? E: post-op she vomited in recovery. What do you do? C: assess, ABC, hydration, ondansetron for treatment if not given before. E: what if you have already gie ondansetron as prophylaxis? C: need another agent for treatment. I give dexamethasone (retrospective: wrong answer, not good for treatment) E: What is the evidence for dexamethasone in PONV? C: sorry, not sure. E: in recovery she stopped vomiting. But she is hypertensive, tachycardic, sweating, tremulous, DDx? C: serotonin syndrome, drug withdrawal (use of illict drug or other unknown drug use), sepsis although likely hypotensive etc. E: why would she have serotonin syndrome. C: On SSRI, risk of drug interaction although I havent give nay other those. E: What are the possible drugs to interact with SSRI. C: (bookwork) E: post-op day 2, she develop a foot drop. DDx? C: complication from regional anaesthsia, positioning, MS problem. E: how to distinguish? C: History, P/E etc. E: How can lithotomy position cause foot drop? C: (bookwork)

10 Years Collection of ANZCA Part II Anaesthetic Viva

MK Yuen & SW Ku

10 Years viva collection 2009

207

Scenario 3
A 4 year old with Down's syndrome presents to a day surgery unit for tonsillectomy & adenoidectomy. The parents say that the child is often "chesty", has frequent respiratory tract infections, snores at night and is severely needle phobic. There is no other medical history. The child is very uncooperative, has a clear nasal discharge and is afebrile. There are no obvious venous access sites. On auscultation you detect a systolic murmur. How will you assess the significance of this murmur? E: How will you assess the significance of this murmur? Say child has an echo before which was normal. C: (bookwork) E: How to assess for OSAS in a child? C: How do you prepare for the anaesthesia? E: You cannot establish venous access. C: gas induction E: child lost consciousness but is still struggling, starts to become bradycardic HR 90 anddesaturation 92%. C: laryngospasm or breathholding. CPAP, gentle assist ventilation, atropine imi 20mcg/kg. E: still desaturation. C: im sux, intubation E: you intubate and find pinkish sputum form ETT, DDx? C: trauma form airway manipulation, -ve pressure APO, aspiration, APO from cardiac cause. E: details about ve pressure pul edema. E: Now SpO2 97% on 50% O2, would you allow the case to proceed? C: If oxygenation and ventilation under control then yes. E: What are you criteria for extubation in this case? C: Surgical factor, anaesthesia factor, patient factor, leak test in this case etc. must be admitted +/- HDU/ICU BELL

10 Years Collection of ANZCA Part II Anaesthetic Viva

MK Yuen & SW Ku

10 Years viva collection 2009

208

Scenario 4
A 66 year old male presents to the pre-admission clinic scheduled for laparoscopic gastric banding next week. He takes lipitor and tells you he has sleep apnoea. The nurse provides you with his observations: o o o o Blood Pressure 171/101 mmHg Heart rate 84 b/min Pulse oximetry 96% Wt. 138 Kg

What is the significance of his BP? E: What is the significance of his BP? C: to rule out measurement error. Associated with increased haemodynamic instability and cardiac morbidity. E: What will you do? C: refer medical as it is elective. E: How long will it take to bring down BP? C: about 4 6 weeks. E: What drug? C: beta blocker if no contraindication, because it decrease CVS morbidity. E: If patient is admitted already and surgery is to be done mane, what can you do about BP? C: give betaloc 12.5mg po, reassess pm to see response. No sedative because of OSA. E: What about intraop or preinducation? C: iv metoprolol 0.5 1mg ivi wait for 5 10 minutes for response. E: If your registrar told you he cant put in iv drip, what will you do? C: US guided IJV cnnulation. E: How to differentiate artery and vein? C: compressible and decreased with respiration. E: What about peripheral iv venous choice and what ways to increase chance of success? C: Houseman vein and saphenous vein. Tourniquet, warm towels, lower hand, inhalational volatile for kids but definitely not in this case because of obese and difficult airway. E: Will you try gas down if patient is thin above shoulder but fat with lower body? C: No. Because OSA still present. E: What is the best way to secure airway in this case? C: awake FOB if difficult, may shoot up BP. RSI if good TMD. E: What are the causes of increased BP? C: Primary HT, secondary causes obesity, OSA, RF, Endocrine: acromegaly, Cushing. E: What do you think BP will be like if BP increased with secondary cause? C: higher SBP, increased pulse pressure.

10 Years Collection of ANZCA Part II Anaesthetic Viva

MK Yuen & SW Ku

10 Years viva collection 2009

209

E: What is the time cause of increased BP? Id expect it to be chronic. E: After CO2 pneumoperitoneum, there is increased CO2. DDx? What will you do? C: feel pulse, look at colour, 100% O2, manual ventilation, ABC. Ddx: CO2 absorption, endobronchial intuition, pneumothorax. BELL

Scenario 5
You are asked help resuscitate a 26 year old male in your emergency room. He has been stabbed several times in the chest during a pub fight. Injuries appear to be minor bruises and cuts to face, and three stab wounds to the left chest. One is high on his back; one is in the left anterior axillary line in fifth costal interspace another is just to the left of the sternum in fifth interspace. He smells of alcohol. There has been no loss of consciousness. No other medical history of note. He developed severe respiratory embarrassment in transit to hospital. Current observations; o o o o BP is 85/60 Respiratory rate 40/min, air hunger and Pulse 130. Reduced air entry on Left. Unable to record Sats. Breathing spontaneously.

How will you manage this man? C: patient airway, give 100% O2 via non-breathing mask. He had symptoms of left hemopneumothorax, inserted CD right away. (actually it was a tension pneumothorax in retrospect). E: Chest drain first or intubation first? C: CD first. May be he need intubation for airway protection, but he had pneumothorax, he needs to have a CD. E: Where will you put in the CD? Size of the CD? C: between mid and anterior axillary line, 5th and 6th ICS. # 32Fr or bigger. E: What direction? C: Toward the base. E: You inserted the CD and the patient improved, less respiratory distress. BP was still low, tachycardia. What would you do? C: insert large bore iv cannula, check blood for investigation, X-match, give HS 2 litre, examine for the abdomen. E: Why examine the abdomen?

10 Years Collection of ANZCA Part II Anaesthetic Viva

MK Yuen & SW Ku

10 Years viva collection 2009

210

C: (I hesitate, then he pointed out that one of the stab wound is on 5ht intercostals space, what can be injured?) Spleen. E: What else can be injuried from the 3 stab wounds? C: Injury to the great vessels, cardiac tamponade. E: How can you differentiate cardiac tamponade from tension pneumothorax in this patient. C: (bookwork) Becks triad distended neck vein, muffled heart sound, pulsus paradoxus. E: What is pulsus paradoxus? E: In this case, how are the signs of cardiac tamponade modifiec? C: neck vein distension reduced by hyptension, or pressure in tension pneumothorax. Hemothroax mad the HS muffled, need an echo to differentiate them. E: Youve done an US pericardial fluid, intraperitoneal free fluid. It suggests need to go in for an operation. BP after crystalloid 80/60, tachycardia, SpO2 95% on O2. Would you tap the pericardial effusion before OT? C: Yes, risk of cardiovascular collapse. E: How and where? C: under US guidance by cardiologist ASAP. E: risk of pericardiocentesis? E: Would you intubate the patient going to OT? C: Yes. (in retrospect, probably should not intubate the patient at this junction) E: How would you do it? C: RSI, CP, manual in line stabilization. (In the scenario, there is no sign of neck instabilization) E: Your assistant is new and do not known how to do a cricoid pressure. How would you deal with it? C: ask for skilled assistance. If not possible, teach him to do it. (bookwork) E: How much force? C: 30N, 3 4kg. E: What type of operation would you think the surgeonwill do? C: exploratory thoracotomy and laparotmy, or sternotomy. (after some prompting, but I think it is not a major issue in this scenario) BELL Other candidate was asked more on whether to tap the pericardial effusion in AED or in OT, the pros and cons of doing it in AED vs OT. one candidate mentioned doing in OT, after prepped by surgeon ready for sternotomy, the other 2 insisted on doing it in AED.

10 Years Collection of ANZCA Part II Anaesthetic Viva

MK Yuen & SW Ku

10 Years viva collection 2009

211

Scenario 6
A 37 year old man has sustained a compound tibial fracture in a motor vehicle accident. His past history is that of an IV drug abuser. o o o o Current medications Methadone 80 mg a day Diazepam 5 mg TDS prn Oxycodone 20 mg BD prn

A lumbar epidural was easily placed prior to general anaesthesia and the orthopaedic procedure has been completed uneventfully. The epidural is to be used for postoperative analgesia. What are your instructions to the nursing staff in Recovery regarding the observations required for this epidural? C: vital signs, pain score, conscious level, LL neurology, level of block. E: How would you test eht level of the block? C: use ice. E: 15 minutes later, recovery nurse called you patient has inadequate pain control. What would you do? C: Assess the patient, look at the catheter for dislodgement, position, machine running, drug being delivered to patient. Test the level of block inadequate density or patchy block. Give 0.25% ropivacaine 10ml + fentanyl 50mcg. E: What is your usual drug for postop infusion? C: 0.1% ropivacaine + 25mcg/ml fentanyl. E: Still inadequate pain control after your bolus. What would you do? C: Withdraw the catheter a bit, leave 3cm in space, give another dose, change to epidural morphine. E: What else can you give? C: epidural clonidine or ketamine, E: You have fixed the problem of epidural. But X days later, the nurse called you and tell you the catheter has dislodged. One hour before that, he was given Clexane (examiner said that, not the generic name or LMWH). C: Inspect the site, frequent monitoring of LL neurological function. NS consultation and MRI if symptoms. E: How would you deal with his pain control? C: give him an PCA morphine, resume his usual oral medication. E: any more? C: give him NSAID, tramadol. E: take something about his NSAID. What drug would you use?

10 Years Collection of ANZCA Part II Anaesthetic Viva

MK Yuen & SW Ku

10 Years viva collection 2009

212

C: diclofenic. E: side effect? C: increased risk of bleeding, inhibit bone formation, renal impariement, but probably not a good choice in this case. E: Yes, especially if he had a recent catheter dislodgement and risk of haematoma. What PCA regime would you use? C: morphine 2mg Q5min, 4 hours 40 mg. E: What dose limit do you use? C: 0.3mg/kg q4hr, a bit more for him. No background infusion if he resume his usual medication. If not, I will convert his methadone into iv morphine dose and give 1/3 1/2 over 24 hour as background infusion. E: if still have pain, what else can you give? C: ketamine, as PCA or infusion. A NMDA antagonist, improve opioid tolerance. E: How to give Ketamine? C: ketamine/morphine 1mg/ml PCA, or ketamine 0.05mg/kg/hr infusion, titrate to effect. E: What are the side effect of ketamine? C: (bookwork) E: later, ward nurse called you patient was agitated. What would you do? C: assess the patient, vital signs, neurological deficits, temperature. My ddx: pain, ketaimine side effect, withdrawal of opioid or benzodiazepam, hypoxia, hypercarbia, hypoglycaemia, intracranial pathologies. If vital sign stable, I will give some benzodiazepam to calm him down. E: how much? C: diazepam 5mg, his usual doae. E: still very agitated. What intracranial pathologies he may have. He had an RTA. C: Heamatoma. He need to have an CT brain for investigation. I will intubate him for this procedure, considering the risk and benefits. E: You would intubate him with diazepam alone? C: No, If he is stable after benzo, no need for intuition, but if his condition not improve, then I will give him GA and intubate for the procedure. BELL

10 Years Collection of ANZCA Part II Anaesthetic Viva

MK Yuen & SW Ku

10 Years viva collection 2009

213

Scenario 7
You have been asked to anaesthetise an 83-year-old male with emphysema for a right compound supracondylar fracture of the elbow. You are considering regional blockade. o o o o o o o pH 7.45 PO2 46 O2 58 Fev1 750 mls Weight 80kg PHx CCF Meds Lasix Slow K ventolin Theodur

Which technique would you choose and why? C: Infraclavicular BPB, coracoid approach lower volume required (30ml), low risk of pneumothorax, good anaesthesia of whole limb. E: How about interscalene block? What is the incidence of phrenic nerve palsy? C: up to 100%, not a good choice for this patient who had poor lung function. Also risk of pneumothorax 1%. E: How about supraclavicular block? C: also with risk of pneumothorax. E: can you quantify it? C: up to 5 6% and risk of phrenic nerve palsy. E: OK. Eventually you decided to perform an interscalene block. How would you do it? C: check machine... E: you have done all that. C: supine, towel as pillow, turn head to the other side, aseptic technique, mark the landmark, interscalene groove and C6 level. I will use short bevel insulated needle, nerve stimulator. Durg 30ml 0.5% ropivacaine. Method (bookwork) E: Why this direction? C: towards the transverse process where the nerve root emerge. E: If you hit the transverse process, what would you do? C: redirect more inferiorly. E: diaphragmatic contraction? C: too anterior (phrenic nerve on the anterior surface of scalene anterior). E: you mentioned using nerve stimulator. What is the setting? C: srtart at 2mA, then look for motor response at wrist, elbow, then reduce it to < 0.5mA. E: electrode polarity, frequency, why stimulate motor but not sensory response? E: Why use 0.5% ropivacaine? C: better safety profile.

10 Years Collection of ANZCA Part II Anaesthetic Viva

MK Yuen & SW Ku

10 Years viva collection 2009

214

E: What are the difference between ropivacaine, bupivacaine and levobupivacaine in term of safety profile? C: Bupivacaine most toxic, ropivacaine and levobupivacaine roughly similar. E: How to determine the toxic ddose? C: Intact muscle, in vitro test, conc at which muscle stop contraction. While animal to determine lethal dose. Healthy volunteer infusion of LA until CNS symptom occurs. E: Why 30ml? C: usually 30 35ml in interscalene block (not sure). But in this patient, I will limit the dose. E: Is 40ml better? C: Yes. But higher chance of toxicity and side effect (not sure). E: (given a set of vital signs) 30 min into operation, patient had respiratory distress, increased HR. C: assess the patient, ABC, give him O2. Ddx phrenic nerve palsy, COAD exacerbation, pneumothorax. E: You found that he had paradoxical diagpharmatic movement. ABG pH 7.23, pCO2 50, pO2 66 C: intubate the patient, ventilate for respiratory distress, finish OT ASAP. E: patient was sent to ICU. 3 days later, ward nurse called you because patient had pain in hand. What would you do? C: assess the patient, history, P/E. Ddx: surgical complication tourniquet, nerve injury, compartment syndrome, anaesthesia related. E: paraesthesia over the medical side of the arm and forearm to medical 1and 1/2 finger. C: it is lesion involving the medial cutaneous nerve of forearm and arm, ehcih emerge at same level. Lesion will be in the nerve root, lower trunk or medical cord. E: How would you determine the size of lesion? BELL

Scenario 8
You are a consultant anaesthetist new to the hospital and are phoned by the Gastroenterologist on call to anaesthetize for an urgent Endoscopic Retrograde Cholangio Parcreatography. The patient is a 70 year old diabetic. His only medication is metformin. What specific information do you discuss with the endoscopist about this case? C: indication for ERCP, stability of patient, where to perform ERCP. E: ERCP room for acute cholangitis. What are the problems?

10 Years Collection of ANZCA Part II Anaesthetic Viva

MK Yuen & SW Ku

10 Years viva collection 2009

215

C: remote anaesthesia, patient stability and monitoring. E: What is lacking in ERCP room (remote area)? C: skilled assistant, adequate monitoring, patient in lateral prone position, trolley head not tiltable on X-ray table, no defibrillator, drugs, temperature, radiation hazard. E: What will you do? C: Ill go to inspect ERCP room to see if they comply with college guideline and inform COS. Because Im new to this hospital. E: intraop desaturation 91 to 74%. C: crisis, turn patient supine, intubate, ABC. E: (shown ABG with acute respiratory acidosis) What are the causes? C: aspiration, CHF, ARDS secondary to sepsis. E: (given ABG again) calculate A-a gradient. E: What is the likely cause? C: ARDS. E: Ventilation strategy? C: PEEP 100% O2 E: given clotting increased FDP, decreased fibrinogen, Echis time = 5 C: DIC with uncontrolled bleeding E: How to treat? C: FFP, platelet, cryo if fibrinogen decreased +/- tansamine (not sure) BELL

Scenario 9
A 39 year old has been booked onto the emergency list with a diagnosis of ruptured ectopic pregnancy. She is 15 weeks pregnant. Her current medications include folate, salbutamol inhaler and some natural health supplements. What specific questions will you ask the gynaecologist when she books the case? C: blood loss, fluid or blood given, patient stability. Asthma cannot control, severity, current status. Type of alternative medicine coagulopathy, prolonged awakening, CVSDS. E: give example of drug? C: kava sedation, ginger coagulopathy, Mahuang CVS. E: What to consider if transfer? C: plan lifts, person doctor escort, nurse, potter, patient stabilized, monitor, equipment, OT ready for transfusion, RIS. E: Why do you think patient will have massive transfusion?

10 Years Collection of ANZCA Part II Anaesthetic Viva

MK Yuen & SW Ku

10 Years viva collection 2009

216

C: conceal haemorrhage, young patient have good compensation while crash only till late. E: If I tell you its cornual pregnancy, what do you expect? C: increased bleeding and possible hysterectomy. E: What is cornual pregnancy? E: given 8 units packed cells, what abnormality you would expect? C: give FFP, platelet if clinical uncontrolled bleeding because lab test need 1 hour. Ca supplement, hypothermia. E: given coagulation test compatibile with DIC. E: desatuation, what is your management? E: What investigation you would order? C: CXR bilateral infiltrate. Ddx: fluid overload, ARDS secondary to massive transfusion, anaphylaxis. BELL

10 Years Collection of ANZCA Part II Anaesthetic Viva

MK Yuen & SW Ku

10 Years viva collection 2009

217

ANZCA Anaesthetic Viva September 2005 Day 1


Scenario 1
A 32 year old multigravida woman attends the pre-admission clinic prior to her scheduled elective caesarean section in two weeks time - weight 115kg - seventh pregnancy She has a low lying placenta but felt well throughout the pregnancy. She had a GA for her first caesarean because it was an emergency, but she had a lot of pain afterwards. Her second CS was performed under an epidural, she required a blood transfusion. What obstetric history would you obtain? y y y y y y y y What obstetric history would you like to ask? What other information would you like to know from USG? What is placenta areta? What anaesthetic technique would you choose? Finally, patient insisted on regional technique. How would you anaesthetize her? Massive bleeding occurred after baby out. How would you manage in the initial 30 minutes? What surgeon can do to minimize bleeding? What can you do if persistent bleeding?

Scenario 2
A 52 year old man has presented 4 hours previously to the Emergency Department with worsening anginal chest pain. He is a smoker with a history of treated hypertension. Medications: aspirin, enalapril Coronary angiography has revealed a critically stenosed left anterior descending (LAD) coronary artery. An attempt to perform transluminal angioplasty (PTCA) on this lesion has failed. The patient is being transferred to the Operating Theatres for emergency coronary artery bypass grafting (CABG). What are the anaesthetic priorities / implications of this case? y What is your anaesthetic considerations?

10 Years Collection of ANZCA Part II Anaesthetic Viva

MK Yuen & SW Ku

10 Years viva collection 2009

218

y y y y y y y y y

How would you anaesthetize this gentlement? Hypotension after induction. How would you manage? What are the criteria for weaning off CPB? Hypotension after weaning off bypass, what is the cause? (protamine induced) What is the other side effects of protamine? Apart from hypotension, there is also bronchospasm. How would you manage? (crisis of anaphylaxis) Final Hb around 8, would you transfuse?

Scenario 3
A twelve year old, 45kg girl with idiopathic thoracic scoliosis presents for correction of her scoliosis and instrumented posterior spinal fusion. The curve has progressed rapidly over the last twelve months. The surgery is expected to take 4-6 hrs and will be in the prone position on a padded four-post frame (Relton-Hall, illustrated).

What findings from your preoperative assessment would make you consider elective postoperative ventilation for this patient?

10 Years Collection of ANZCA Part II Anaesthetic Viva

MK Yuen & SW Ku

10 Years viva collection 2009

219

y y y y y y y y y y y y y

What are the factors contributed to post-op IPPV? What pre-op investigations would you order? What is the expected lung function test? How about the changes in lung function test post-op? What are the anticipated problems for prone position? How do you protect the eyes? Which nerves are vulnerable? How aobut cardiovascular, respiratory changes of prone position? Intra-operatively, arterial line suddenly lost reading, how do you manage? (likely a scenario of PEA crisis in prone position, ?VAE) How do you resuscitate in prone position? Suddenly, ECG changes to polymorphic VT, how do you manage? Crisis settle and vital signs stable, surgeon request to continue OT, would you agree?

Scenario 4
A diabetic 65 yr old woman fell 4 days ago, and was admitted to your hospital. She has sustained a fractured right ankle and have fractured right ribs. Since admission medications prescribed are: Paracetamol 1 G 4 hourly PRN Morphine IV PRN as requested Enoxaparin 20 mg sc daily, last given 2 hrs ago Glibenclamide 5 mg po Her only significant history is moderate renal impairment (creatinine = 0.18 mmol/l) She is now complaining of difficult breathing, and chest wall pain. You are phoned by her surgical team for advice on the respiratory and pain issues. What are the important initial issues you would deal with when you see her? y y y y y y How is fracture ribs affect lung function? What serious consequence can arise? What are the available options for his pain management? Patient was given PCA. Few hours later, he became drowsiness with borderline SpO2. How would you manage? Would you consider the alternate options of epidural analgesia? Finally, patient on epidural with satisfactory pain relief. However, 3 days later, he complains of back pain. How would you manage?

10 Years Collection of ANZCA Part II Anaesthetic Viva

MK Yuen & SW Ku

10 Years viva collection 2009

220

Scenario 5
A 25 year old male elite athlete presents for an elective shoulder arthroscopy after recurring dislocations on his dominant side. He is 200 cm tall and 110kg. The surgeon requires the patient to be in the beach chair position. It is anticipated that the patient will be discharged from hospital the same day. What positions are used for shoulder arthroscopies? What problems could you anticipate from these positions? y y y y y y y y What is your anaesthetic concerns? What are the risks with beach chair position? What other position commonly employed for shoulder surgery? What are the options for pain relief? You perform the brachial plexus block before GA. After injection of 20ml LA, patient complains drowsiness. How would you respond? Suddenly, convulsion occurs. How would you manage? Conculsion subside spontaneously, vital signs stable. Surgeon request proceed to OT as there is good analgesic block over upper limb, what is your response? Later patient complain of numbness over right ulcer side of forearm. How would you manage?

Scenario 6
A 43 year old patient is scheduled for a revision mastoidectomy with facial nerve monitoring for a cholesteatoma. She has smoked 20 cigarettes daily for 15 years, drinks socially (two glasses per week) and is currently on moclobemide (aurorix) for a 3 year history of depression. She is otherwise in good health. Her previous operation was the mastoidectomy 4 years ago complicated by vivid memory of the intubation and prolonged nausea experience with this procedure. List the main issues of the anaesthetic. y y y y y y y What are your anaestheic concerns? Would you stop moclobemide before OT? How would you prevent the recurrence of PONV? What is awareness? What is the different from dreaming? How about incidences of each? What is BIS?

10 Years Collection of ANZCA Part II Anaesthetic Viva

MK Yuen & SW Ku

10 Years viva collection 2009

221

y y

This time again a difficult intubation, during prolonged intubation, BIS rises from 50 to 80, what would you do? How would you maintain the anaestheia?

Scenario 7
A 45 year old woman, with a large multinodular goitre that has retrosternal extension, is booked for a subtotal thyroidectomy. Outline your preoperative assessment specifically relevant to this patient having thyroidectomy. y y y y y y y How would you assess her pre-operatively? She has feature of SVCO. CT comfirmed narrow airway because of compression. How would you anaesthetize her? Would you extubate her at the end of operation? What are the criteria for extubation? She was extubated and transport to PACU. She develop stridor with respiratory distress. How would you manage? Failed FOB because of severe airway edema? How would you proceed? What is the complication of thyroidectomy?

Scenario 8
You are a newly appointed consultant when called to assist in the MRI scanner with a 58 year old make having a cranial scan. He is anxious and insists on having sedation for the procedure. When you arrive he is in the MRI scanner. What are you priorities in providing the assistance requested? y y y y y y What is your anaesthetic consideration? What drug would you choose for sedation? Trainee come and help you in administrating the drug. After patient inside MRI, he became restlessness and tachycardia. How would you manage? On examination, you find patient in rapid shallow breathing, bilateral pupils dilated, SpO2 began to drop. What is the possible causes? (a case of wrong drug administration by trainee, who give muscle relaxant) After intubation which is easy, but there is no air entry. How would you manage?

10 Years Collection of ANZCA Part II Anaesthetic Viva

MK Yuen & SW Ku

10 Years viva collection 2009

222

Scenario 9
You are asked to see a twenty six year old woman in the Emergency Department. She has been involved in a high speed motor vehicle accident, initially lost consciousness, and now requires a cranial CT scan. Her GCS is 10 and there is bruising visible over her left anterior chest wall but no other apparent injuries. The Emergency Department staff have inserted a left-sided intercostal tube for a left haemo-pneumothorax. Describe the Glasgow Coma Score.

10 Years Collection of ANZCA Part II Anaesthetic Viva

MK Yuen & SW Ku

10 Years viva collection 2009

223

ANZCA Anaesthetic Viva September 2005 Day 2 Scenario only


Scenario 1
A 25 year old lead singer in a rock band presents to your Emergency Department at 4:00am complaining of sudden onset of a severe headache, neck stiffness and photophobia. Examination reveals the presence of nuchal rigidity. He smokes 30 cigarettes per day. A cranial CT scan reveals the presence of a small amount of subarachnoid blood. A 4 vessel cerebral angiogram performed at 8:00am reveals a right sided anterior communicating artery aneurysm. How do you grade patients who present with aneurysmal subarachnoid haemorrhage?

Scenario 2
A 34 year old primigravida was admitted at 0300 hours in early established labour after an uneventful pregnancy. An epidural catheter was placed. After nine hours of labour, she remains on 5-6cm dilated with an occipito-posterior position. The obstetrician wants to perform an urgent Lower Segment Caesarean Section (LSCS). However, all four operating theatres in the hospital have elective surgery underway at present. As the coordinating anaesthetist in charge of the theatres, how are you going to respond to the obstetrician's request?

Scenario 3
A 65 year old man presents for right fem-pop bypass as he has exercise induced claudication in his right leg. Past history of note is renal failure secondary to diabetes. He is managed with peritoneal dialysis and on twice daily insulin. He is scheduled first on the list. Currently taking metoprolol quinapril and insulin. Allergic to penicillin. BP 160/100 HR 60 regular, No murmurs. ECG reveals ST segment inversion septal leads. Hb 91gm/l, potassium 4.5, urea and creatinine both high. What are your pre-operative concerns in managing this man?

10 Years Collection of ANZCA Part II Anaesthetic Viva

MK Yuen & SW Ku

10 Years viva collection 2009

224

Scenario 4
A 4 year old boy, with a history of progressive weakness and muscle wasting, presents for muscle biopsy for a suspected myopathy. He has had a previous uneventful anaesthetic for neonatal hernia repair. What are your concerns in anaesthetising a child with suspected myopathy?

Scenario 5
A 75 year old man with carcinoma of the oesophagus is scheduled for an oesophagectomy via a right thoracotomy. The surgeon requests the right lung be deflated for the procedure. What are the major issues in the anaesthetic management of this patient?

Scenario 6
A 75 year old overweight man with osteoarthritis (OA) is scheduled for a left total knee replacement (TKR). His part history includes an inferior AMI 5 years earlier but no ongoing angina since. He also has NIDDM (for the last 5 years) and well controlled hypertension. His medications include: Clopidogrel 75 mg daily Metoprolol 50 mg bd Enalapril 20 mg daily Metformin 500mg bd Omeprazol20 mg daily Diclofenac 50 mg bd Simvastatin 40 mg nocte You see him in the pre-admission clinic 4 weeks prior to his scheduled surgery. What specific advise will you give him regarding his medications pre-operatively?

Scenario 7
A 50 year old man presents for emergency repair of a penetrating eye injury, sustained 7 hours ago while eating lunch. The injury is to his good eye, the other eye was blinded in an industrial accident 10 years ago. The rest of the pre-assessment is unremarkable. How will you manage the airway?

10 Years Collection of ANZCA Part II Anaesthetic Viva

MK Yuen & SW Ku

10 Years viva collection 2009

225

Scenario 8
A 65 year old man is scheduled for resection of carcinoma of pharynx and free flap repair. He has a forty pack year history of smoking. He is found to have a systolic murmur during pre-operative assessment in your pre-admission clinic. How would you clinically assess the significance of this murmur?

Scenario 9
It is 22:00hrs. You are the on-site anaesthetic senior registrar, when you are phoned by an intern. You are in the middle of an open appendectomy. You have an anaesthetic senior house officer / Anaesthetic RMO with you in the Operating Room. She tells you that a 25 year old patient admitted to the ward for neurological observation after a period of concussion is difficult to rouse with noisy breathing. What further information do you require from the intern?

10 Years Collection of ANZCA Part II Anaesthetic Viva

MK Yuen & SW Ku

10 Years viva collection 2009

226

ANZCA Anaesthetic Viva May 2006 Day 1


Scenario 1
You are contacted urgently by the Medical Emergency Team regarding a 55 year old female patient who has become agitated during a coronary artery angioplasty Vital Signs: Blood Pressure = 90/40 mmHg Pulse Rate of 50/min The cardiologist suspects that the right coronary artery has been ruptured. He is requesting that you provide intravenous sedation to the patient to allow deployment of a stent to control the rupture. How would you approach this problem? y y Questions re initial assessment & management Diagnosis of Tamponade y y y y Becks triad Other Features - Pulsus paradoxus, Electric altemant

(Explanation of each is required) Initial Management how to perform pericardiocentesis Induction of patient to Operating Theatre Awareness: Dealing with patient Detailed questioning about diagnostic features & Dx of post traumatic stress.

BELL

Scenario 2
An uncooperative 14 year old boy with Cerebral Palsy and developmental delay presents for multiple dental extractions under general anaesthesia. He has had previous surgery for tendon releases and bilateral suction myringotomy with drain tubes and he is extremely anxious. What further information would you seek about this child's cerebral palsy? y y y y y Cerebral palsy Management of unco-operative child pre-induction What if wont accept PO midazolam 0.5mg/kg PO Weight is 75kg would you still give 0.5mg/kg PO What is the dose of IM ketamine for sedation?

10 Years Collection of ANZCA Part II Anaesthetic Viva

MK Yuen & SW Ku

10 Years viva collection 2009

227

y y y y y

Cant get IV access pre-induction options for induction Assessment Turns out to be intraoral blood loss (significant) What are the issues? How would you induce the patient this time?

Child very agitated in recovery

BELL

Scenario 3
A 65 year old man is added acutely to the list for biopsy of a right sided supraglottic carcinoma. How would you evaluate the patient's airway? y y y y y y y y Airway assessment Details about: What stridor means, Why inability to use flat significant Induction of choice & why & explain in detail issues related to other options eg fiberoptic & IV Other questions such as bag mask ventilation & CPAP will this prevent upper airway collapse? Shown nasendoscopy picture asked to identify supraglottic structures. Management of obstruction during gas induction Crisis desaturation post induction (turned out to be a circuit problem) Possible sites of leaks in circuits.

BELL

10 Years Collection of ANZCA Part II Anaesthetic Viva

MK Yuen & SW Ku

10 Years viva collection 2009

228

Scenario 4
You are scheduled to anaesthetise a 50 year old woman with chronic renal failure who presents for repair of an acutely obstructed inguinal hernia. She is usually dialysed via peritoneal catheter twice a week and passes small amounts of urine. She has been unable to be peritoneally dialysed for 4 days and has signs of fluid overload. What are you priorities in assessing this patient? Issues related to CRF y y y Initial assessment & Management Shown full set of bloods Causes of anaemia, causes of hypocalcaemia, causes of increased urea apart from renal failure Patient wants a regional options & discuss y y y y Neuraxial Ilioinguinal & iliohypogastric - n block asked in details Discuss method of general anaesthesia Crisis seizure in recovery DDx management.

BELL

Scenario 5
A 65 year old woman presents at the pre-admission clinic prior to a planned laparotomy for ovarian malignancy. She has a history of sever rheumatoid arthritis which is currently quiescent. She is mobile only with assistance. As she enters the room, you notice that she is short of breath. What is the significance of the shortness of breath? Discussion regarding RA and malignant causes of SOB y y y y y y Cardiac & respiratory What Ix shown restrictive spirometry How would you discuss this with the patient? What do you think the patient means by life support How would you manage this? Would you give her an epidural risk/benefit?

Patient does not want to be on life support

Despite epidural 2/7 post-op found hypoxic on ward. Shown ABG PO2 ~ 50, PCO2 ~ 60 on 60% O2

10 Years Collection of ANZCA Part II Anaesthetic Viva

MK Yuen & SW Ku

10 Years viva collection 2009

229

Patient refuses to go to ICU or be intubated, family insisting that she be intubated. y How would you manage the situation? BELL

Scenario 6
A 40 year old nurse has sustained a fracture of her distal radius and ulna following a fall while at work. She has undergone surgery for open reduction and internal fixation under general anaesthesia. As the consultant for acute pain, you are called to see her on the first day post-operatively because she is still complaining of pain. She has received a total of 30mg of morphine IV over the past hour. How will you assess the cause of her pain?

Scenario 7
A 60 year old woman is listed for left upper lobectomy for lung malignancy, You see her in pre-anaesthesia clinic one week prior to surgery. She is 130kg. Her BMI is 48kg/m2. She stopped smoking one week prior. She has a post history of old inferior infarct. A persantin-thallium shows EF 50% and a fixed lesion consistent with prior infarction. She has been seen by a cardiologist who thinks she is optimal from a cardiac point of view. What are the implications of morbid obesity to anaesthesia? Features of obesity y y y y y y y y Detailed pharmacokinetic changes Metabolic syndrome Apart from diabetes other endocrine changes. Techniques for lung Isolations Advantages & disadvantages for both Questions regarding technique of insertion of bronchial blocker (Some people asked how to clean a bronchoscope) Desaturation during OLV Benefits of smoking cessation

BELL

10 Years Collection of ANZCA Part II Anaesthetic Viva

MK Yuen & SW Ku

10 Years viva collection 2009

230

Scenario 8
A 19 year old male has ridden a trail bike into a wire fence, hi your accident and emergency department he is sitting forward, has stridor and appears distressed, and is intermittently spitting out blood stained saliva. Outline your immediate management. Assessment & management in DEM y y y y y y Appropriateness of C-spine collar & Xrays Patient refuses awake trachea Other options Mx at conclusion Discussion about weaning patient in ICU with trachea Management of anaesthesia Management in OT

Patient develops caninal stenosis trachea 5mm wide. BELL

Scenario 9
A health 32 year old woman is being induced at 41 weeks. You are called to assist urgently in the delivery unit where you are told she has collapsed shortly after her membranes were ruptured. What are you first priorities when you arrive? y y y y y y y Patient arrested management of arrest Details on protocol one vs. 2 operations etc Description of CPR How long until foetus should be delivered? DDx & how to differentiate possible causes. Transport to operating theatres. Management of massive blood loss due to DIC in operating theatres & discussion regarding pathophysiology / mortality of AFE. BELL

10 Years Collection of ANZCA Part II Anaesthetic Viva

MK Yuen & SW Ku

10 Years viva collection 2009

231

ANZCA Anaesthetic Viva May 2006 Day 2


Scenario 1
A 65 year old retired businessman presents to a private free standing day surgery unit for a flexible colonoscopy. He has a history of PR bleeding for the past 6 months. He has a history of hypertension and is on ditiazem and bendrofluazide. Outline your priorities for providing sedation for the colonoscopy. E: Outline your priorities for providing sedation for the colonoscopy. C: Assessment patient, personnel (include recovery staff; anaesthetic assistance), location (equipment, monitoring, resusc facilities) Patient HT, antihypertensives, PR bleed (may be anaemic), bowel prep. E: What are the anaesthetic implications of bowel prep? E: Different types of bowel prep; side effects. E: How would you provide sedation for this procedure? C: MADE E: What about patient-controlled sedation? E: During the procedure, you noticed this trace on the pulse oximetry irregularly irregular rhythm with variable amplitude, rate ~120. What would you do? E: In the recovery room, the patient complains of chest pain, what would you do? C: Hx, PE, Ix. E: ECG showed STEMIFurther Mx? C: Optimise ABC; aspirin, GTN, analgesia, anticoagulation. Tranfer to hospital with cardiology services (incl interventional cardiology) for further Mx. E: How would you organise the transfer; what escort? E: What if youve got more pts on your scope list & youre the only one there? E: The patient underwent PCI + stent. He is booked to have a hemicolectomy next week for bowel Ca diagnosed on colonoscopy. What are your concerns? Perioperative MI/stent thrombosis/surgical bleeding. E: What anti-thrombotic agent(s) is the patient likely to be on? E: What is clopidogrel? E: How would you manage perioperative anticoagulation? C: Discuss with cardiologist! E: How would you anaesthetise this patient? C: Multidisciplinary approach involvement of surgeon, cardiologist. Location hospital with interventional cardiology services & ICU.

10 Years Collection of ANZCA Part II Anaesthetic Viva

MK Yuen & SW Ku

10 Years viva collection 2009

232

Thorough pt assessment echo post-MI? Preop optimization. MADE what drugs for induction, maintenance etc. ?Intraoperative TOE. (Probably not for epidural given likely requirement for aggressive periop anticoagulation). Postop ICU care. BELL

Scenario 2
A 10 year old girl presents to the emergency department of a rural hospital with an acute exacerbation of asthma. She has not other significant medical history and no allergies. She weighs 55kg. Despite medical treatment with salbutamol, adrenaline and steroids she continues to deteriorate and the decision to intubate is made. What are you going to do now? E: What are you going to do now? C: Preparation/assessment. MADE Plan for intubation what drugs? RSI. Plan for sedation. Would you paralyse the patient how long for? Plan for transfer to centre with ICU facilities. E: Following intubation, what ventilator settings would you choose to provide ventilatory support for this patient? C: (Ventilation strategies for acute exacerbation of asthma) E: How to manage high airway pressure? E: How would you transfer this patient to the Tertiary referral centre? C: Planning mode of transport (1hr ambo by road), communication with receiving hospital. Equipment, monitoring, personnel, drugs(which ones?) E: During the transfer, the SpO2 decreased to 92%, what would you do now? C: Stop the ambo. Pt assessment crisis Mx of hypoxiaDDx. E: Findings decreased AE on left side? C: PTX Mx. E: Would you insert an ICC? (Do you have any ICC/chest drain bottle in the ambulance?) BELL

10 Years Collection of ANZCA Part II Anaesthetic Viva

MK Yuen & SW Ku

10 Years viva collection 2009

233

Scenario 3
You are the anaesthetist on-duty rostered to your hospital trauma response team. You have been given 10 minutes warning that a 30 year old male motorcycle rider is due to present to the Emergency Department. He has been involved in a high speed collision with a 4WD vehicle. The man was wearing a helmet and full protective riding gear. He was found 20 metres from his motorcycle. Paramedics promptly attend to the man. They observed moderate damage to the front of his helmet. At the accident scene he was un-cooperative and combative. His initial observations were Pulse 110/min sinus rhythm, BP 110/60 mm/Hg. What will you do in the time before the patient arrives in the Hospital? E: What will you do in the time before the patient arrives in the Hospital? C: Planning: Personnel trauma team; surgeon; radiographer, anaesthetic assistance; ?blood bank. Alert OT. Preparation of resuscitation area in ED. Equipment resusc airway equipment, IV, level 1 fluid infuser, ICC etc. E: The patient arrives in ED what do you do now? C: ATLS primary survey & resusc. E: Findings HR 120 BP 90/50 SpO2 90% on 6L/min O2 RR 32/min. Decresed AE & movement left side of Chest. Bruising & tenderness on L side of chest + surgical emphysema. What would be your concerns? What would be your Mx? C: concerns possible flail segment, PTX, HTX, pulmonary contusion, tracheobronchial injury +/- BPF. High flow O2, Fluid resusc, CXR if not already done; ICC. E: ICC drained 1000ml of blood. Source of bleeding in haemothorax? E: What are the indications for thoracotomy in this case? E: Further 600ml of blood drained from ICC what would you do now? E: Surgeon like to perform thoracotomy in ED, what is your response? E: Role of resuscitative thoracotomy in ED? E: Thoracotomy in OT Describe your anaesthetic techniques. C: MADE, Cx spine precaution, RSI. E: Techniques for lung isolation pros & cons of each; which one would you choose? E: ICU requested epidural to be inserted to facilitate weaning from ventilator what will you do? C: Consent issues; probable coagulopathy/sepsis; likely need for prolonged ventilation from underlying lung injury/pulmonary contusion. E: What other methods of analgesia? E: Describe how you would perform a thoracic paravertebral block? E: How many segments does a single shot TPB cover? E: What are the drawbacks of a TPB?

10 Years Collection of ANZCA Part II Anaesthetic Viva

MK Yuen & SW Ku

10 Years viva collection 2009

234

E: What are the possible complications? BELL

Scenario 4
A 16 year old girl is about to arrive in the Emergency Department following a fall and trampling by a horse. Information transferred by ambulance is: GCS initially 15, now 11 Facial bruising Tender abdomen You are asked to meet her on arrival and form part of the trauma team. From this information, what is the most important aspect of this girl's management? E: From this information, what is the most important aspect of this girls management? C: Prioritise Primary survey + resusc: ABCDE, FAST scan. Main issues suspected head & abdo trauma. Exclude other life-threatening injuries in primary survey. Need for OT for head or abdo? need rapid Dx & decision making. Need to stabilise haemodynamics (prevent secondary brain injury). Inform surgeon, neurosurgeon, OT, radiographer E: Fluid Mx? What & how much? BP goal? (active bleeding vs head injury). BP 120/50 HR 110 GCS still 11 C: Need CT scan E: what would you do? C: Monitors/equipments/drugs/personnels for transfer E: would you intubate? E: Decision made to go to scanner without intubation. Blown R pupil in CT. Mx ? C: secure airway with MILNS, measures to decrease ICP, alert surgeon & OT. E: Whilst this is happening, patient became hypotensive with increase abdomen distension what to do now? C: Need urgent laparotomy. E: Laparotomy completed; need craniotomy. Neurosurgeon turns up smelling of alcohol; also had recent marital breakup; what would you do now? C: assess competency; urgency of surgery (urgent!); general surgeon to perform the burr hole. E:Neurosurgeon became belligerent. What to do now? BELL

10 Years Collection of ANZCA Part II Anaesthetic Viva

MK Yuen & SW Ku

10 Years viva collection 2009

235

Scenario 5
A 30 year old female is a passenger in a car involved in a high speed MVA; the car ran off the road, rolled and hit a tree with impact on her side of the car with moderate intrusion. She was wearing a seat belt, and the car had dual air-bags, and as a result did not lose consciousness. The driver who was not wearing a seatbelt was ejected from the vehicle and killed. At the scene she was conscious, complaining of pain down the left side of her body. Her initial BP was 100/70, heart rate 120, and she was tachypnoeic. En route she has been given a total of lOmg morphine in divided doses, and 500ml of crystalloid. The patient has been airlifted to the emergency department and you are part of the receiving team. On arrival she is restless, but conscious, with a heart rate of 125, BP 100/60, RR 30, and SpO2 of 92% on 6L/min of oxygen via mask. What are your priorities in assessing this patient? C: ATLS primary survey & resusc. Adjuncts to primary survey XR, FAST scan. Possible blunt chest & abdo trauma with hypovolemic shock. Assess need for OT; notify surgeon. E: What is the estimated blood loss? E: What class of haemorrhagic shock does this pt presents with? E: What are the potential sources of blood loss in this patient? C: Thoracic, abdominal, retroperitoneal, fractures, pelvic haematoma. E: FAST scan showed fluid ++ in abdomen. HR 120 BP 90/60 after 2L of crystalloids. What would you do now? E: Indications for surgery? E: Describe how you would anaesthetise this patient for laparotomy. C: Resusc/optimise. Surgeon scrubbed & ready; abdo prep & draped. MADE, lines, blood products in OT. Cx spine protection; RSI. E: Laparotomy ruptured spleen. Describe your management of massive transfusion. C: Blood products PCs, FFP, platelets (warm blood bank). Pharmacological adjuncts: rFVIIa. Role of blood salvage in this situation (possible contamination with bowel contents). Monitoring haemodynamic, blood tests ABG/Hemocue/coagsMethods to maintain normothermia, electrolytes (Ca, K), acid-base. Warming devices, rapid fluid infusors. E: What are the criteria for giving recombinant FVIIa? Dose/formulation/cost of rFVIIa. E: SpO2 drops intraoperatively Mx? C: Usual crisis Mx protocol. DDx Equipment, PTX, HTX, pulmonary contusion, APO, ARDS, TRALI. E: What is TRALI? How is it managed? BELL

10 Years Collection of ANZCA Part II Anaesthetic Viva

MK Yuen & SW Ku

10 Years viva collection 2009

236

Scenario 6
You are the pain consultant on for the day. You are called to see a patient in recovery. He is an 80 year old, 70kg diabetic male who has had a below knee amputation under spinal anaesthesia by a colleague. The spinal has worn off and he is complaining of a severe burning pain felt in his stump. He has been given 20mg of morphine in total by the recovery staff and while very drowsy, is still complaining of "terrible" pain. What would you do about this man's pain? C: Patient assessment Hx, PE pain history (site, nature, onset, duration, severity etc), usu pain meds (?opioid tolerance), preoperative pain (?element of neuropathic pain from underlying PVD). Chart R/V consider other causes of burning pain ? neurologic injury from spinal (unlikely), surgical cause. E: Finding pt is normally takes oxycontin 40mg bd. RR 6/min. What is your Mx plan? C: Multimodal analgesia PO, IV, RA; adjuvants (clonidine, ketamine, tramadol) ? feasibility if patient already quite drowsy. RA: Femoral nerve, fascia iliaca block, Sciatic nerve block; E: technique? USG guidance? E: consent issues with epidural given drowsy pt in pain. C: Referral to MPS. E: Describe your technique of sciatic nerve block. What drugs? E: How do you confirm needle placement? (remember pt just had a BKA!) E: Would USG guidance be of benefit? E: What suggestions are the chronic pain team likely to make? E: Are there any effective way to provide pre-emptive analgesia to decrease post-amputation pain? E: How will you give an anaesthetic for similar operation in the future? E: What would you tell the patient about the risks associated with an epidural? BELL

10 Years Collection of ANZCA Part II Anaesthetic Viva

MK Yuen & SW Ku

10 Years viva collection 2009

237

Scenario 7
A 26 year old POG1 presents at 38 weeks complaining of increasing shortness of breath. What would be your initial differential diagnosis? E: What would be your initial differential diagnosis? C: DDx CVS (CHD/VHD, IHD/MI, aortic dissection, PE/AFE, PPCM, PIH, SBE), RESP (asthma, COAD, infection), anaemia (?APH), Sepsis. E: How would you assess the patient? C: Assessment Hx, PE, Ix. What to look for? What investigations would you order? Also assess fetal well-being. E: Findings patient is previously well, 2/52 Hx of inc SOB, now dyspnoeic at rest. PE HR 115 BP 100/40 SpO2 92% on RA, RR 33, inc JVP, basal creps, gallop rhythm, Bloods Hb 120, Na 135, K 5.0, Cr 140 ABG pH 7.35, CO2 38, O2 85, HCO3 22 on RA Echo EF 20%, biventricular dilatation. E: What is your diagnosis? E: What is your management? C: ABCMultidisciplinary approach obstetrician, cardiologist, ICU (Tertiary centre!). Haemodynamic monitoring. Optimise pt with medical therapy. Plan for delivery timing & mode E: What drugs are likely to be used to treat the pts condition? C: Diuretics, vasodilators (not ACEI), digoxin, +/- inotropes. E: Would you consider an epidural for labour analgesia? E: Decision made to deliver baby by C-section. Outline your anaesthetic management. C: Repeat assessment make sure pt is optimised. Usual obstetric precautions aspiration prophylaxis, avoid aortocaval compression, ABx. MADE - ?CVP/TOE/inotropes & pressors ready. RA vs GA. Caution regard use of uterotonics. Postop management. BELL

10 Years Collection of ANZCA Part II Anaesthetic Viva

MK Yuen & SW Ku

10 Years viva collection 2009

238

Scenario 8
A 25 year old male presents for electroconvulsive therapy (ECT). He has a life threatening depressive illness which has not responded adequately to medication, however, he is till taking Parnate (tranylcypramine). The first year psychiatry registrar contacts you to find out what if any work up you require prior to his ECT. C: Assessment: Co-existing disease; substance use; previous anaesthetics Hx. Contraindications to ECT. Fasting status as for any anaesthetics. E: When & where. E: What is Parnate? What implications does this have? Would you stop it pre-procedure? E: What other drugs may a psych patient be on that you would be concerned about? C: TCA, SSRI, Lithium E: What is serotonin syndrome? How is it managed? E: What are the implications if patient is on lithium? E: What are the physiological effects of ECT? E: What is your preferred anaesthetic technique for this procedure? What drugs & why? E: On the day of the procedure, you discovered that the pt had previously had a muscle biopsy because certain family member had problems with anaesthetics. Result of the muscle biopsy is not available; what would you do? What is your anaesthetic plan now? C: Treat as MHS; in main OT area (not stand-alone DSU) & appropriate recovery monitoring; monitor for 4 hrs post-procedure. Equipment & drugs for Rx MH. Anaesthetic technique avoid MH triggers; paralyse with non-depolarizer; ETT vs LMA. E: After the procedure, youre told by the recovery staff that the pt now has a HR of 160; what would you do? C: ABC, assessment, DDx, Mx. ?MH, drug interactions MAOIs/drug errors, arrhythmias E: The patient is to have ECT 3x times/week, what would be your subsequent anaesthetic Mx of this patient. C: ?Same as above. BELL

10 Years Collection of ANZCA Part II Anaesthetic Viva

MK Yuen & SW Ku

10 Years viva collection 2009

239

Scenario 9
An older colleague seeks your advice regarding anaesthesia for a 45 year old, 112kg, 180cm fanner presenting as a day patient for nasal septoplasty and functional endoscopic sinus surgery (FESS). His history includes obstructive sleep apnoea (OSA) and he is a smoker; he volunteers no other medical problems. What advice will you give your colleague regarding his assessment of this patient with a view to anaesthesia for this procedure? Your colleague has no previous experience in this area of anaesthesia. C: Issues Suitability for DSU; Patient Fx obesity, smoker, OSA implications & assessment of each; Anaesthetic Fx shared airway, prior anaesthetic esp intubation Hx. Surgical Fx Shared airway, throat pack, nasal vasoconstrictors, nasal pack postop. Postop care. Feasibilility of CPAP postop. E: What nasal vasoconstrictors are likely to be used for this surgery? Implications? What is in co-phenylcaine? E: Preop BP 150/85 what would you do? Implications? Would you cancel surgery? E: Describe your anaesthetic technique for this situation. E: The patient is now in recovery; youre the duty anaesthetist on the day & youre asked by the recovery staff to see this pt as they are concerned about his noisy breathing. The colleague who had given the anaesthetic is nowhere to be found. Describe your Mx. C: ABC, assessment, DDx, & Mx. Remember throat pack! E: Pts SpO2 dec to 60%; as you performed direct laryngoscopy, you saw & removed the throat pack; unfortunately the pt proceeded to asystolic arrest. What is your Mx now? C: ACLS protocol. E: After 15mins, the patient was resuscitated & transferred to ICU. What do you do now? C: Post-critical incident Mx. E: Your colleague asked you if you could not tell anyone that he left the throat pack in the pt. What would be your response? E: What strategies are in place in your hospital to prevent such adverse event from recurring? BELL

10 Years Collection of ANZCA Part II Anaesthetic Viva

MK Yuen & SW Ku

10 Years viva collection 2009

240

ANZCA Anaesthetic Viva September 2006 Day 2 Dr Assad Hussain, QMH


Scenario 1
You are requested to anaesthetize a woman who is 32 weeks pregnant with a suspect acute appendicitis. She is febrile 38.2C, tachycardic, tachypnea and looks unwell. The surgeon plans to perform a laparoscopic appendenctomy. What are the anaesthetic implications of the physiologic changes of pregnancy in this lady? Aims of the Viva: The aim of this viva is to ensure that candidates can: Discuss the induction and management of general anaesthesia in pregnancy Discuss the maintenance of uteroplacental flow and foetal monitoring. Manage foetal bradycardia intraoperatively Describe neonatal resuscitation C: patient concern: mother ACC, aspiration, difficult intubation, fetus teratogenicity, threatened labour. Surgical concern: lap vs open, Anaesthesic concern. I will take a history , do P/E and look at appropriate investigation, make sure obs/paed service available. E: How will you anaestheise her and what are your concerns. C: preop preparation antacid prophylaxis, left lateral tilt. Intraop RSI with cricoid pressure, prepare for difficult intubation, plan postop monitor fetus/ labour/inform obs colleagues. E: will you allow surgeon to do it laparoscopically? C: no, because we dont have to surgical expertise to do it laparoscopically in my institiution. E: surgeon says he is very skilled at lap technique, will you allow him to proceed? C: No E: can you think of any advantages of doing it laparoscopically C: minimal physiological disturbace, less pain, shorter hospital stay. E: after induction, there is a drop in BP, what is your management? C: urgent situation, simultaneous assessment and resuscitation. Casuse likelyleft lateral tilt, IVF, vasopressor. E: Now the ETCO2 suddenly drops. What can this be due to? C: (bookwork) E: can you tell me about treatment of amniotic fluid embolism. C: ABC + supportive E: will you allow surgeon to continue operation? C: yes, if she is stable enough to proceed.
10 Years Collection of ANZCA Part II Anaesthetic Viva MK Yuen & SW Ku

10 Years viva collection 2009

241

E: do you know how to confirm AFE? C: (bookwork) E: How can you monitor the fetus intraopeartively. C: pre and postop by FHR and CTG. Intraop it is difficult to directly monitor fetus, but can indirectly monitor the fetus by monitoring and mastering the health childs mother. E: at the end of the operation, the HR falls to 50, what is your management. C: simultaneous resuscitation and assessment, causes included E: what is your postop management and your post op concerns. E: post op the women thinks she is in labour and complaints about you. What is your response? C: confirm labour. Inform obs colleague, FHR monitoring E: OK, now she really is in labour. It is decided to proceed with a emergency CS. How will you anaesthetize her? C: talk with surgeon about need for urgency of CS. If time allows, SA. If not then GA. E: Baby is finally delivered, mother is stable. Baby needs resuscitation. Tell me how you will resuscitate the baby. C: new neonate resuscitation guidelines. E: does she need DVT prophylaxis for lap surgery? C: Yes E: apart from lap surgery, what other risk factors are there for DVT. C: (bookwork) E: what will you give her for DVT prophylaxis? C: preop hydration, mobility. Intraop LMWH, PCD, TED stocking. Postop mobilization. BELL

10 Years Collection of ANZCA Part II Anaesthetic Viva

MK Yuen & SW Ku

10 Years viva collection 2009

242

Scenario 2
63 year old male presents with headache, sweating, chest pain, palpatations. BP 220/120 and pulse of 65. Past history of hypertension controlled on enalapril, myocardial infarction 3 yrs ago. Smokes 20 cigarettes/day. A left adrenal phaeochromocytoma is confirmed by 24 hour urine catecholamine levels and MRI. The patient is scheduled for laparoscopic adrenalectomy in 2 weeks. What are the management goals for this patient before they come to theatre? Aims of the Viva: The aim of this viva is to ensure that candidates can: Discuss the peri operative management of Phaeochromocytoma. Describe and manage the problems with laparoscopic surgery Discuss the DD and management of tension pneumothorax intra op

E: What is your concern? E: How would you optimize preop? C: IVF management and blocker phentolamine. E: Phentolamine? C: sorry, I meant phenoxybenzamine. E: what else will you give to the patient. C: clocker. E: why? C: prevent reflex tachycardia and prevent actions of catecholamines on adrenoreceptors. E: Now the patient is optimized. How will you anaesthetize him? C: fasting adequate, consent, prepare theatreGA + ETT. E: How about your monitoring? C: Apart from the monitoring stated by the college guidelines, I will also insert an A line and CVP. E: what drugs will you prepare or use intraop. C: remifentanil, O2/air/isoflurane E: why not use nitrous oxide C: (bookwork) E: what else? C: dexmedetomidine E: why? C: analgesia, intraop cardiovascular stability. E: what else? C: magnesium.

10 Years Collection of ANZCA Part II Anaesthetic Viva

MK Yuen & SW Ku

10 Years viva collection 2009

243

E: what doseage? C: Infusion 1-2g/hr, but needs hourly plasma level monitoring. E: How is magnesium beneficial and what is the therapeutic range. C: blocks catecgholamine release, blocks receptors and is a direct vasodilation. Normal range 2 4 mmol/l. E: itnraop, sudden increase in an pressure and desatruation. What is your response? C: crisis situation. Ask surgeon to stop surgery, call for help. Deflate abdomen E: why do you need to call for help. C: 2 pairs of hands better than one. E: what can the extra person do? C: depends on the cause but as patient will need simultaneous resuscitation and assessment, an extra pair of hands is always useful. E: OK, so what would be the causes? C: equipment, patient, surgical causes E: how to differentiate pneumothorax form endobronchial intubation without percussion. C: using ETCO2 and checking ETT marking and adjusting tube if necessary. E: now there is sudden drop in BP. C: urgent situation, simultaneous resuscitation and assessment. Causes: cardiogenic100% O2, IVF, vasopressor. E: at what points during surgery do you expect BP to be labile? C: induction and intubation, penumoperitoneum, tumour manipulation, after tumour removal E: what will you check for after tumour is out? C: CVP level and BP maintenancin with IVF and vasopressors. E: what else? C: sorry, Im not too sure. (I guess he was asking blood glucose level) BELL.

10 Years Collection of ANZCA Part II Anaesthetic Viva

MK Yuen & SW Ku

10 Years viva collection 2009

244

Scenario 3
A 35 year old female with metastatic breast cancer is admitted with a pathological femur fracture and is booked for a femoral nail. She is significantly short of breath. Her last course of chemo was 4 months ago, and she is currently on various 'alternative therapies' under the guidance of her naturopath. You assess her the night before her scheduled surgery. What specific issues will you focus on at the pre-operative visit? Aims of the Viva: The aim of this viva is to ensure that candidates can: Describe the preoperative assessment and management of dyspnoea Discuss the implications of Alternative therapies for anaesthesia Describe the DD and management of Fat/Tumour embolism

E: what are your concerns? C: patient ca primary and metastasis, what chemotherapy agent and side effect, what alternate therapy. Surgical fat embolism, blood loss etc. anaesthesia type of anaesthesia (because of alternate therapies) E: why does she have SOB? C: I will do History, P/E, Investigation. Could it be primary or secondary lung Ca or metastatic pleural effusion or pneumonia or ascites. E: she has a pleural effusion. Will you tap before OT? C: yes, as she is symptomatic, I would tap it to optimize her for OT. E: how much will you tap out? C: at the most 1 litre because dont want to overtap. E: why? C: Risk of re-expansion pulmonary edema. E: can you explain that? E: will you put in a drain before OT? C: no, as the tapping is a temporary symptomatic relief to get her through operation. Postop I will consult the oncologist for further management to the pleural effusion and the need for chest drain or pleurodesis E: how much fluid needs to accumulate before it can be seen on CXR? C: Im not sure for the exact figure, but in the range of 500ml to 1 litre. E: will you do under GA or RA? C: depends on the further prop assessment. Before chosing RA, I need to rule out any coagulopathy or contraindication due to either her cancer or alternate therapies. E: She is on Kava, ginger and ginseng.

10 Years Collection of ANZCA Part II Anaesthetic Viva

MK Yuen & SW Ku

10 Years viva collection 2009

245

C: Kava can potentate Parkinsons disease. Ginger and Ginseng can casue coagulopathy and bleeding risk. E: OK, so you decide to do OT under GA. Ther eis a sudden drop in BP intraop. Management? C: crisiscauses E: Now there is a rise in CO2, the SpO2 dropped to 60 and BP 80/50. (cant remember rest of scenario, but I think it went into diagnosis and management of fat embolism) BELL

Scenario 4
A 3yr old girl presents for resection of a Wilms tumour. She has no other significant medical history and no allergies. She weighs 15 kg. She has been receiving vincristine and doxorubicin prior to her planned nephrectomy. How would you assess the potential side effects of this chemotherapy? Aims of the Viva: The aim of this viva is to ensure that candidates can: Deal with anxious child and family for major surgery. Describe the impact of preoperative chemotherapy on anaesthesia management. List the DD and describe the management of intraoperative hypotension. Discuss major blood loss and peri operative fluid management in children, including

allowable blood loss, transfusion thresholds etc E: What is your concern about these agents? C: can cause bone marrow suppression and cardiotoxicity. E: the child odes have a history of cardiotoxicity. How would you assess? C: History, P/E, investigation E: The child has had multiple hospital admissions. Both he and his parents very anxious. How will you deal with them? C: Involve parents/ child in quiet area for discussion. Ask about what she fears. Try to allay them. Play with child and talk at eye level with child etc. E: how to decrease childs anxiety? C: parental presence, rapport and premedication. E: what premedication? C: I will use oral midazolam 0.5mg/kg E: child usually scared of injections. Apart from oral, How else can you give premed.

10 Years Collection of ANZCA Part II Anaesthetic Viva

MK Yuen & SW Ku

10 Years viva collection 2009

246

C: PR, IN E: surgery now proceeds. I hours into OT, sudden decreased BP 90/50 to 65/35. Management? C: crisis, stop surgery, simultaneous assessment and resuscitation. Causes.100% O2, IVF, vasopressor. E: what surgical causes can cause this? C: compression of vessels i.e. IVC or aorta. E: which is more likely? C: IVC. E: OK, nowhe stops pressing on IVC, 10 minutes later sudden decreased BP, Management? C: same E: this time is hypovolaemia. How to assess blood loss? C: suction bottle, surgical field, ask surgeon, weight gauze, check Hb. E: 150ml in sucker and 80ml in swabs, will you transfuse? C: depends on what is percentage of total blood loss and whether can still maintain organ O2 perfusion. What is Hcue? (need to calculate blood volume from age) E: Hcue dropped from 10 8 g/dl. C: If further bleeding anticipated, Id prepare fur massive bleeding and transfusion. Blood warmer, check blood, large bore iv etc. E: how much blood will you transfuse? C: start with 10ml/kg packed cells and reassess after. E: How much will this raise Hb by? C: 1g/dl E: what are the complications of massive transfusion? C: (bookwork) BELL

Scenario 5
A 55 year old man presents to the Emergency Department following a 1 metre fall from a ladder whilst painting his house. He is complaining of pain in his lower neck and his right wrist. He has a fractured right radius which the ED staff would like reduced under an intravenous regional anaesthetic technique. He was diagnosed with Ankylosing Spondylitis at the age of 35. His current medications include prednisone 10mg, methotrexate and diclofenac. A lateral spine x-ray was taken because of the pain.

10 Years Collection of ANZCA Part II Anaesthetic Viva

MK Yuen & SW Ku

10 Years viva collection 2009

247

How does this x ray affect your preoperative assessment? Aims of the Viva: The aim of this viva is to ensure that candidates can: Plan anaesthesia for C spine stabilization and fusion including intubation issues Discuss the issues for prone surgery positioning especially in patient with unstable C spine Discuss the role of spinal cord monitoring Discuss a plan for safe extubation of this patient

E: how will you manage? C: activate trauma call, assess according to ATLS guidelines, ABCDE etc. E: this is the lateral C spine X ray. What does it show? C: anterior subluxation of C6 or C7, bamboo spine E: how to stabilize his C spine in ER? C: neck collar E: if the neck collar does not fit his C Spine? C: can use sandbags on each side of head. E: what are your concerns about AS? C: heart AR, Pulmonary fibrosis, Bamboo spine difficult intubation, limited mouth opening, drugs steroid, deformity positioning issues. E: How will you prepare for OT? C: prepare theatre, equipmentsteroid cover. E: tell me how youd give steroid coverage and when? E: How to secure airway? C: awake FOB. E: why awake? C: difficult intubation and unstable C spine E: how to do? E: what are problems with prone position? E: can you explain principles of SSEP and MEP and how it affect your anaesthetic technique. E: what will you tell patient about wake up tes and how will you do it? E: OT finished. Will you extubate? C: unlikely, send to ICY for stabilization first. E: No ICU bed available. What will you do? C: try to arrange it. In mean time I will observe in recovery room with trained nurse for unconscious patient. E: What are your criteria for extubation? C: awake patient and warm. Fully reversed, check with NM monitoring. Good TV and good

10 Years Collection of ANZCA Part II Anaesthetic Viva

MK Yuen & SW Ku

10 Years viva collection 2009

248

negative inspiratory pressure. Not difficult airway if reintubation. E: if he meets all these will you extubate? C: yes, if really not a difficult intubation patient. E: How? C: extubate with Cooks airway exchanger in situ. Prepare for reintubation with difficult intubation trolley and FOB. BELL

Scenario 6
You are on call.It is 2200 hrs. A surgeon calls and advises that he has a patient on the ward who has just been transferred from a local nursing home. He feels she needs urgent laparotomy. He tells you she's not in good shape and he's worried she may have dead gut. He has a theatre organized for a half hour from now. What would you ask the surgeon? Aims of the Viva: The aim of this viva is to ensure that candidates can: Interpret ABG's (severe metabolic acidosis) and manage fluid resuscitation Discuss ethical issues around operation or not. Describe the management of rapid AF.

E: What do you want to ask surgeon? C: urgency of operation and condition of patient including current condition, past medical and surgical history, allergic history, functional capacity. E: What are your concerns? C: elderly lady with AF for major intrabdominal emergency surgery. Patient elderly, AF, HT, assess according to AHA guideline, any dehydration, septic, acidosis, renal failure. Surgical long OT, major fluid shift etc. Anaesthesia her pre op optimized, RSI with cricoid pressure, post op ICU, prepare CVP, A line, inotorpes. E: How will you assess her preop? E: She is 85 year old, frail, ADL dependent, early dementia, controlled CCF, AF, IHD for 6 years but no active angina, can walk on flat ground only. Now what will you do? C: investigation blood, CXR, ECG etc E: what does her ECG do? (ECG was shown) E: what does her ABG show?

10 Years Collection of ANZCA Part II Anaesthetic Viva

MK Yuen & SW Ku

10 Years viva collection 2009

249

E: No bed available in ICU, will you proceed with OT or wait? Why? C: yes, as it is dead gut, it is reversible and she may recover. I will observe her in recovery room till ICU bed available. E: do you think it is fair not to proceed with her OT as her is old, frail and many comorbidities. C: yes it is fair, but in my institution we would still proceed because it is a reversible casue. E: OK, you anaesthetize her, sudden drop in BP, what is your management? C: crisiscauses E: ECG shows this? (ECG was shown) C: fast AF and ST depression. E: how to treat? C: amiodarone 150mg in 100ml D5 over 15 minutes. E: what is your postop management? BELL

Scenario 7
A 57 year old man presented to the emergency department with 4 hour onset of severe back pain, leg weakness and faecal incontinence. What is the differential diagnosis for the cause of his symptoms? Aims of the Viva: The aim of this viva is to ensure that candidates can: Discuss the diagnosis and management of a spontaneous epidural haematoma Describe the peri operative management of anti coagulation in emergency spinal cord Discuss the risks of ceasing anti coagulation in this patient. Can discuss the issues anaesthesia for this patient including maintenance of spinal cord

decompression.

perfusion in this context E: what will be the causes? E: how to diagnosis epidural haematoma? C: History, P/E, Investigation. Coagulopathy history, antiplatelets drug? Back pain, fever, leg weakness with new onset paresis. P/E: full neurological exam and examine back. Investigation: coagulopathy, MRI. E: what is the time limit for favourable results after a surgical decompression. C: 8 hours E: 2/52 ago she had a coronary stent inserted, now on metoprolol, clopedogrel and aspirin.

10 Years Collection of ANZCA Part II Anaesthetic Viva

MK Yuen & SW Ku

10 Years viva collection 2009

250

Why is she on these medications? C: metoprolol IHD, clopidogrel and aspirin for 6 months if drug eluting stent to prevent stent thrombosis. E: will you continue before OT or stop medication? C: as the surgery is urgent, no point to stop them because of long half life. E: what are the risks? C: increased intraop bleeding. E: how to prevent this? C: preop, intraop and postop methods. Preop, can give platelets. E: how much platelets? C: I will give 10 units. E: when? C: just before induction. E: is that enough? Will you need to give any more. C: also will give postop to prevent for the risk of bleeding due to antiplatelets. E: she comes in for a surgical decompression. What are your concerns about the operation? C: patient IHD, and coronary stent, prone position, long OT, massive blood loss, postop concerns. E: What are your concerns regarding prone position? C: (bookworks) E: How to minimize bleeding? C: (bookworks) E: How do you prepare for massive blood loss? C: cell saver, warm patient, 2 large bore iv, hotline, rapid infusion systems, blood checked and ready to give, Hcue available, blood products readily available. E: good, suddenly she develops ST elevation in II and V5. What is your management? C: 100% O2, morphine for pain and tachycardia, nitrates. E: But she is hypotensive. (Hypotensive technique has been mentioned earlier on) C: Oh, sorry, I forget, I will then raise up her blood pressure using bolus and infusion of phenylephrine to see if this helps her ST elevation. E: OK, her ST elevation settles. Total blood loss 3 litres and 5 litres of crystalloid given. She develops facial edema. How will you manage this? C: put her supine, head up, dexamethasone, assess her for extubation or not. E: what is your criteria for extubation? BELL

10 Years Collection of ANZCA Part II Anaesthetic Viva

MK Yuen & SW Ku

10 Years viva collection 2009

251

Scenario 8
You are called to the Emergency Department to assist the medical staff there who are struggling to analgese a 54 year old male involved in a pedestrian accident leaving him with a compound fracture of his left tibia and fibula. He has been given 45mg intravenous morphine and as you approach the ED he is still screaming in pain. What are your immediate plans? Aims of the Viva: The aim of this viva is to ensure that candidates can: Discuss the Anaesthetic management of patient on naltrexone Manage the issues for Alcoholism and anaesthesia Diagnose and manage compartment syndrome in a patient with regional anaesthesia

E: How will you manage this? C: immediately assess the patient according to ATLS guideline for associated injuries, primary survey E: OK, he has no associated injuries. What are your concerns? C: patient - ? opioid tolerated patient, past health, allergy. Surgical fat embolism, tourniquet. Anaesthesia full stomach and RSI, sux use, compartment syndrome. E: How will you provide analgesic? C: Firstly, I will assess his pain and his history. Is he an opioid user, assess his pain, look our for compartment syndrome. Then I will us multimodal analgesia and explain to him the aims and expectations. I will use ketamine around 0.1 0.5mg/kg bolus and then low dose infusion. E: What else? C: I will give some fentanyl. E: is it useful? After 45 mg morphine? C: no, probably not useful. I will then consider regional anaesthesia. E: describe your technique. C: (bookworks) E: why may the surgeon not be very happy about the nerve block? C: there is a risk of compartment syndrome and it is a relative contraindication in this situation to put in a block because it mask the signs. E: tell me about compartment syndrome. C: (bookworks) E: how to measure intracompartmental pressure? C: (bookworks) a valve above 35 40 mmHg is diagnostic. E: What is the management? C: remove constricting bands on limb and urgent fasciotomy.

10 Years Collection of ANZCA Part II Anaesthetic Viva

MK Yuen & SW Ku

10 Years viva collection 2009

252

E: why is myoglobin harmful? C: renal ATN. E: how to treat it? C: IVF keep urine output high, forced alkaline diuresis. E: How doe forced alkaline diuresis help the renal function? E: OK, so he is a alcoholic. What are your concerns? C: intoxicated? Withdrawal symptoms, full stomach, complications of alcohol. E: What are they? C: cirrhosis, AF, IHD, cardiomyopathy, thiamine deficiency Wernickes encephalopathy, Korsakoff psychosis. E: How to treat withdrawal? E: OK, so how will you manage his pos op analgesia? BELL

Scenario 9
You are the retrieval anaesthetist at a factory where a 30 yr old man is trapped by his partially severed and obviously fractured Right forearm under heavy machinery. He is screaming in pain and continues to bleed freely from his arm wound. His head is just under the machine giving you no access to his airway other than to deliver oxygen via nasal cannula. Amputation may be the only option to free him. How will you manage this situation? Aims of the Viva: The aim of this viva is to ensure that candidates can: Describe a safe plan for Anaesthesia/analgesia for safe extrication from machine Discuss issue involved in anaesthesia for reimplantation of forearm Describe the safe use/risks/benefits of Continuous brachial plexus block.

E: How will you manage this? C: on the way to scene get as much information as possible and bring along equipment for securing airway, oxygen cylinder for transfer and equipment for giving IVF. Also will need equipment for monitoring and drugs for resuscitation, warming device and reflective blanket. E: When you first arrive at the scene what will you do? C: Check my surrounding and the scene to make sure it is safe. E: How will you assess him? You cant reach his airway. C: According to the ATLS guidelineand also keep him warm.

10 Years Collection of ANZCA Part II Anaesthetic Viva

MK Yuen & SW Ku

10 Years viva collection 2009

253

E: But you cant reach his airway. C: Then I will provide analgesic for the amputation. E: How? C: multimodal, fenatnyl, ketamine, regional. E: you cant do RA because no access to arm and neck. C: OK, then just fentanyl and ketamine. E: after freeing him, what is your management? C: 5 Ps of retrieval. E: Will you intubate him before transfer? C: Yes, because he would have suffered blood loss and it will be a more controlled situation for transfer. I have a low threshold for transfer. E: can you not intubate in an ambulance if you needed to? C: yes, you can, just ask the ambulance to stop temporary. E: you are back in TO and the surgeon want to reimplant the limb. How to manage him? C: perop preparation, intraop E: How will you anaesthetize him? C: RA + GA (RSI) E: what RA? C: infraclavicular block. E: please describe? What are complication? E: surgeon want s you to improve perfusion to the limbs, what will you do? C: (bookworks) E: the BP is 90, perfusion to other limbs good. Will you use a vasopressor? C: no, because it may worsen perfusion by vasoconstriction. E: you mentioned dextran. What type? C: dextran 70. E: what are the problems with dextran 70? C: coagulopathy. E: what else? C: sorry. Im not sure. E: its OK, How will you provide postop analgesic? C: multimodal BELL

10 Years Collection of ANZCA Part II Anaesthetic Viva

MK Yuen & SW Ku

10 Years viva collection 2009

254

ANZCA Anaesthetic Viva September 2006 Day 2 Dr. Vincent Kong, QMH, HK & Dr. Sanjiv Sawhney, Brisbane, Australia
Scenario 1
A 2 year-old presents for bilateral ueteric reimplantation, she weighs 14 kg and is otherwise well. Her parents are very anxious about pain post operatively. The parents ask what the options are for post-operative pain management for this child for this procedure. Explain their risks and benefits. Aims of the Viva: The aim of this viva is to ensure that candidates can: y y y y y y y y y y y Describe a sensitive approach to parental presence or absence at induction Describe the safe use of caudals Detect and manage LA toxicity Describe the use of desflurane and a circle system in children Discuss with parents about post op analgesic plan Options available. Pros and cons. How to perform caudal analgesia, what agent or volume to be given Discuss supplement that can increase duration and efficiency of caudal block Discuss continuous caudal infusion. Pros and cons, how to set up How about epidural when compared to caudal Discuss about parents present at induction. How about parents refusal. What would you do? How long normally can a single shot bolus caudal block work? What to do next? At the time of putting the epidural catheter, patient developed hypotension. What would you like to do? Dose of adrenaline given? Devore injection of LA via caudal catheter, aspiration of blood positive, what to do next? How to set up a morphine infusion for post op analgesia.

10 Years Collection of ANZCA Part II Anaesthetic Viva

MK Yuen & SW Ku

10 Years viva collection 2009

255

Scenario 2
A 52 year old female presents for coiling of a left anterior communicating artery aneurysm. She is a smoker with a known history of hypertension and ischaemic heart disease. BP HR ECG: 180/110 100 ST elevation, T wave inversion, QT prolongation.

What are the issues in the anaesthetic management of this patient? Aims of the Viva: The aim of this viva is to ensure that candidates can: y y y y y y y y Discuss the issues of out-of-theatre/off the floor neurovascular procedures Conduct safe anaesthesia in a patient with ischaemic heart disease Demonstrate an understanding of the effects of rebleeding What are your concerns? Comment on the successful rate for this procedure? Why ECH changes and increased BP? How would you further investigate? How to induce anaesthesia? Maintenance? How to set up your remifenatnil infusion? What range would you normally use? What would you do if the radiologist told you that he punctured an artery. What BP level you aim to keep? How would you prepare for failed radiological intervention? What if Neurosurgeon ask you to keep temperature at 32C? Any evidence about the usefulness of decrease temperature for this operation? Which study? And what is the results? Uneventful operation. Post op D1, complained left upper limb weakness. What would you do? Discuss potential causes? Likely to be vasospasm? How to prevent vasospasm? Why Nimodipine useful and what is the mechanism? y How to differentiate vasospasm from total occlusion? How to investigate? Pros and cons?

10 Years Collection of ANZCA Part II Anaesthetic Viva

MK Yuen & SW Ku

10 Years viva collection 2009

256

Scenario 3
You are telephoned and asked to urgently attend the emergency department to assist with the management of a 25 year old woman who has been brought in following a high speed motor vehicle accident. She is confused and hypotensive. Her husband, who has suffered minor injuries in the crash, has informed staff that his wife is 28 weeks pregnant with their first child. On arrival in the resuscitation room how will you conduct you initial assessment? Aims of the Viva: The aim of this viva is to ensure that candidates can: y y y y y y y y Conduct the initial assessment and resuscitation of pregnant trauma patient Discuss management of hypotension, utero-placental blood flow and foetal protection in Describe a safe anaesthesia plan for laparotomy/LSCS Manage major blood loss and discuss the use of cell saver in this situation How would you assess this patient? Sudden decreased BP, what would you do? Discuss causes. How would you manage her in A&E? PV bleeding noted. What is your priority in terms of management? Stabilized to CT suite, what would you do? After imaging, she developed VF on arrival to OT. What would you do? Discuss methods to intraoperative blood loss. Discuss clinical uses of cell salvage intra-op. Discuss its contraindication. Discuss blood products transfusion for massive blood loss.

off the floor/out of theatre environment

Scenario 4
You are the consultant anaesthetist on call. The hospital anaesthesia registrar is an Emergency Medicine Trainee with 7 months anaesthesia experience. He rings you for advice regarding the induction of anaesthesia for the following case. An intoxicated 18 year old male has had his right leg run over after a push bike accident. He has sustained compound fractures to both tibia and fibula with significant soft tissue loss. There are no other injuries. He is cardiovascularly stable. He has no apparent head injury and his cervical spine has been cleared. He refuses any regional techniques. His initial orthopaedic surgery will take 2 hours. What advice do you give the registrar regarding the induction of anaesthesia? Aims of the Viva:

10 Years Collection of ANZCA Part II Anaesthetic Viva

MK Yuen & SW Ku

10 Years viva collection 2009

257

The aim of this viva is to ensure that candidates can y y y y y y y y y Describe a safe plan for anaesthesia in this patient for prolonged initial surgery Discuss the safe use of tourniquet Discuss principles of maintaining graft perfusion perioperatively Describe how they would teach trainee RSI and safely manage failed intubation You are a consultant at home. You got a call from a resident with 6 months of anaesthetic experience, who is in A&E. What would you do? How would you assess this patient? Your resident tried intubation for one but failed, able to ventilate. What would you do? You are 30 minutes travel time from the hospital. Upon arrival, how would you manage this patient? How about if the patient insisted refusing RA? How would you intubate this patient? How would you maintain the position of the patient during operation? Uneventful operation. He is scheduled another operation for skin graft. What are your concerns? How to increase graft survival? How to decrease incidence of dextran related anaphylactic reaction?

Scenario 5
A 47 year old female is booked on your next morning list for resection of liver tumour. The biopsy reveals carcinoid tumour. What issues will you look for? Aims of the Viva: The aim of this viva is to ensure that candidates can: Discuss the Perioperative Management of carcinoid Discuss the perioperative management of patient for maj or liver resection Discuss the management of a patient with tricuspid incompetence in this context including

appropriate cardiovascular monitoring. CVP vs Swan vs TOE y y y What are your concerns? How would you assess this patient? Discuss pathophysiology of carcionid tumour. What organs and systems are affected? What investigation would you order preoperatively and why? Why echocardiogram. What are you looking for? What would be your anaesthetic management if valvular

10 Years Collection of ANZCA Part II Anaesthetic Viva

MK Yuen & SW Ku

10 Years viva collection 2009

258

involvement positive. y y y y How would you anaesthetic this patient? What monitor would you use and why? During the operation, the patient developed hypotension. What would you do? Let say you successfully managed the hypotension, but now the paitent developed bradycardia. What would you do? From ECG, there is complete heart block. What would you do? How would you prepare an isoprenaline infusion?

Scenario 6
You are the anaesthetist working on a retrieval team in a major metropolitan hospital. You take a call from a GP in a remote town 60 min away by helicopter. She requests retrieval of an 18 year old male who has been trapped in a house fire. The patient is estimated to have 50% burns, including airway and facial burns. The GP states that the patient has stridor. What further information do you want from the GP? Aims of the Viva: The aim of this viva is to ensure that candidates can: Describe the safe management of airway and intubation issues to GP and issues for safe Describe the other acute management of 50%burns: analgesia and fluids Discuss the management of change of dressings in the ward some time after acute retrieval of patient

episode y y What are your concerns? How would you assess this patient? Do you think this patient needs intubation and why? Would you allow the GP to do the intubation? Apart from intubation, what can be done to manage the upper airway obstruction? Now the patient desaturates to SpO2 85%, what would you do? y y y y y y How would you manage the patient before transfer? Tell me your preparation? What drugs would you carry with you during transportation via helicopter? How would you resuscitate this patient including fluid management? When would the surgeons normally do debridement after burns? Tell me the pathophysiology to burns to different organs: cardiovascular, respiratory, gastrointenstinal, renal system? How would you manage her analgesia if surgeons scheduled daily wound debridement in ICU? What methods agent would you use and why?

10 Years Collection of ANZCA Part II Anaesthetic Viva

MK Yuen & SW Ku

10 Years viva collection 2009

259

Scenario 7
A 77 year old from a nursing home presents with an acute abdomen; the surgeon are concerned she has a perforated viscus and want to perform a laparotomy. She has a past history of PVD [fem-pop bypass] and CVD [mild CVA] COAD and long history of smoking Episodes of heart failure Diverticular disease R renal staghorn calculus osteoarthritis

What are your priorities in perioperative assessment? Aims of the Viva: The aim of this viva is to ensure that candidates can: y y y y y y y Describe appropriate Preoperative resuscitation Discuss a safe choice for anaesthesia Discuss the possible causes for inadequate reversal in this patient including the role of Formulate a plan for situation of no available ICU bed Assessment Investigations Optimisation Disoriented how you get history, consent Per-op tachycardia ECG shown Atrial flutter. D/D, Management Where to keep post-op No ICU beds available in area.

monitoring neuromuscular response

Scenario 8
A 26 year old male presents for laparotomy and bowel resection for Crohn's disease. The patient takes prednisone 20 mg and oxycontin 120 mgs daily. He has no other co-morbidities. The patient requests an epidural for post-operative analgesia. What are the advantages and disadvantages of an epidural for post-operative analgesia in this patient ? Aims of the Viva:

10 Years Collection of ANZCA Part II Anaesthetic Viva

MK Yuen & SW Ku

10 Years viva collection 2009

260

The aim of this viva is to ensure that candidates can: Discuss the perioperative management of patient on steroids Discuss the advantages and disadvantages of an Epidural for post op analgesia Manage epidural problems/failure Describe the risk management required for an institutional policy on minimising epidural

risk/consequences of epidural haematoma and infection y y y y y y y y y y Management of prednisolone and why Pain control plan Epidural what in epidural and why Epidural not working troubleshoot Changed to PCA what/ how much/ why Benefits/ problems of epidural in this patient How you consent for epidural Oxychodone issues, management What are the considerations in formulating pain protocols? Measures to minimise complications from epidural.

Scenario 9
A 65 yr old lady requires surgery for revision hip arthroplasty. She has longstanding atrial fibrillation and a history of transient surgery. What further details of her history are relevant to planning your anaesthetic? Aims of the Viva: The aim of this viva is to ensure that candidates can: y y y Identify the issues with respect to re-do THR in this patient Discuss the perioperative management of anticoagulant therapy in the context of atrial Describe appropriate perioperative blood conservation strategies Describe the management of a Post-operative nerve palsy Assessment Perioperative Management of anticoagulation Blood conservation techniques ischaemic attacks prior to being put on warfarin therapy. Currently she is on warfarin 3 mg daily. You are consulted one week prior to the planned

fibrillation and potential blood loss

10 Years Collection of ANZCA Part II Anaesthetic Viva

MK Yuen & SW Ku

10 Years viva collection 2009

261

y y y

Pain management Risk of DVT When to place/ remove epidural

10 Years Collection of ANZCA Part II Anaesthetic Viva

MK Yuen & SW Ku

10 Years viva collection 2009

262

ANZCA Anaesthetic Viva May 2007 Day 1 Dr. Lewis Fung & Dr. Timmy Chan, QMH
Scenario 1
A 75 year old man, who already is on your Hospitals waiting list to undergo Aortic valve replacement and Coronary artery bypass grafting, is admitted at midday with a diagnosis of acute pulmonary oedema. You are contacted at 5.30 pm by the Head of Cardiac Surgery who wishes to operate on this man immediately? What will be your response to the Surgeon? The aim of this viva is to ensure candidates can: y y y y y Discuss the pathophysiology of aortic stenosis, indications for valve replacement, and optimum timing for surgery in this patient. Outline a safe anaesthetic technique for induction and management of hypotension and AF in the pre-bypass period. Discuss treatment options for post bypass bleeding including role if any of Factor VII What question would you like to ask the surgeon? (Give you the vital sign that patient is continuous deteriorating after admission as pulse increased from 90 to 110/min, sinus rhythm, BP dropped from 110to 90 systolic. SpO2 decreased to 90%) How to optimize patient before OT? y y y (Echo result for interpretation: AVA 0.7cm2, pressure gradient ~ 50mmHg, Coro result showed ? 2 or 3 vessel disease) How would you anaesthetize this patient? Do you thik patient will got better or worse after induction? Why? Crisis after induction, BP drops. Management?

Other candidate was asked management of intraoperative AF. Also was asked the management of hypotension during wound suturing.

Scenario 2
You are a Specialist at a Base Hospital 4 hours by road for a tertiary centre. The Base Hospital has a 2 bedded Intensive Care Unit and full surgical and laboratory services. The Anaesthetic Department consists of 5 specialists and 1 trainee. There is a Neonatal Unit level 2.

10 Years Collection of ANZCA Part II Anaesthetic Viva

MK Yuen & SW Ku

10 Years viva collection 2009

263

The next patient at the preassessment clinic is a 30 year old primigravida with twins. She is 20 weeks pregnant. An elective Caesarean section is planned at 36 weeks. Demographics: Age: 30 Weight: 220kg Height: 1.75m Medications: Folate, vitamins, sulbutamol inhaler What are the implications of a twin pregnancy for her anaesthetic management? The aim of this viva is to ensure that candidates can: y Discuss both the anaesthetic assessment of a morbidly obese parturient and the issues involved in deciding on optimal place and timing of delivery, including place of multidisciplinary team meeting about these issues. y y y y y y Describe a management plan for elective section in a morbidly obese parturient. Discuss anaesthesia for urgent Caesarean section in this patient. How would you assess this patient? Would you allow her to do in your hospital or transfer her to other hospital? Suppose the obstetrician would like to do this case in your hospital. What is you response? At 34 weeks of gestation, patient had preterm labour and CTG showed type II deceleration, fetal scalp blood showed pH 7.2. Obstetrician request emergency CS. What would you do? y y Suppose you successfully intubate the patient and surgery started. After baby delivered, the peak pressure suddenly increased from 25 to 45cm. What is your response? Somethings about Amionic fluid embolism. Other candidate was asked about failed intubation drill in GA.

Scenario 3
You see a 40 year old obese woman presenting for an ankle reconstruction on the day of surgery. She has a severe needle phobia, takes the oral contraceptive pill and smokes 20 cigarettes a day.

10 Years Collection of ANZCA Part II Anaesthetic Viva

MK Yuen & SW Ku

10 Years viva collection 2009

264

What do you understand by the term needle phobia? The aim of this viva is to ensure candidates can: y y Discuss alternative methods of dealing with anaesthetic induction in a patient with needle Describe gaseous induction in this patient using a circle system and placement of phobia. laryngeal mask and show knowledge of the relationship between flow rates and changing gas concentrations in the circuit during induction. y y y y y y y y y Discuss a plan for postoperative analgesia including appropriate local anaesthetic Discuss issues around DVT prophylaxis for this patient. What is the meaning of needle phobia? How would you assess this patient? How would you anaesthetize this patient? Suppose patient had needle phobia and do not allow you to insert iv access before induction. How would you induce her? Postop pain relief method? What do you think of about the risk of DVT in this patient? What is your management on DVT prophylaxis? techniques.

Scenario 4
You are the anaesthetists in the preoperative clinic. You are assessing a 65 year old man proposed for C5-6, C6-7 anterior disc fusion in two days time. He has a history of hypertension and hypertrophic obstructive cardiomyopathy. His only medication is verapamil. What are the important aspects of your neurological assessment of this patient? The aim of this viva is to ensure candidates can: y y y y Discuss the perioperative management principles when anaesthetising a patient with Discuss appropriate management of post-operative airway obstruction in recovery. Outline the signs expected with C5/6, C6/7 disc disease. If I told you that patient had left C6-C7 radiculopathy. What is the sign? HOCM including assessment of severity.

10 Years Collection of ANZCA Part II Anaesthetic Viva

MK Yuen & SW Ku

10 Years viva collection 2009

265

y y y y y

can you draw me the dermatome of upper limb? Which responsible for the tricep reflex? What is the features of HCOM and what do you expect to see in Echo? What is the anaesthetic implications of HOCM will you give any premedication? Post op Stridor Mx

Scenario 5
A 75 year old male presents for a left cataract extraction and insertion of an intra-ocular lens. He has a history of stable angina (12 months duration), non-insulin dependant diabetes mellitus (5 years duration), hypertension (15 years duration) and atrial fibrillation (3 months duration). The surgeon says that the operation cannot be done under topical anaesthesia (eye drops). How will you assess the cardiovascular risk of this patient having cataract extraction done? The aim of this viva is to ensure candidates can: y y y y Assess cardiovascular risk for this patient having this procedure and decide on any Discuss management of his anticoagulation. Discuss and describe appropriate eye block for this patient including description and Appropriately manage complication of blocking the wrong eye including how it may be further IX needed.

relative merits of Subtenons and peribulbar block including management of complications. prevented. y y y y y y y y y y How would you assess his cardiac risk? Surgeons would like to do under RA, how do you tackle the problem with warfarin? What block will you do? What is the advantage and disadvantage of peribulbar block over retrobulbar Whats the advantages of subtenon block? How to do a subtenon block? (ask quite details including the dosage of LA and hyaluronidase?) Few minutes after your block, patient suddently colloapse, Mx and Ddx (LA toxicity, drug allergy, brainsteam anaesthesia) How do you know that it is brainsteam anaesthesia Assume there is no complications after the block, After you block, you discovered that you had performed the block in the non operative eye. What is your Management Will you do the block on the other side if patient agreed

10 Years Collection of ANZCA Part II Anaesthetic Viva

MK Yuen & SW Ku

10 Years viva collection 2009

266

How do you prevent this to be happened again

Scenario 6
You are a senior registrar on call at a tertiary hospital. A 70 year old women with mild Alzheimers disease presents with a large bowel obstruction and peritonitis. The surgical registrar requests an urgent laparotomy. At pre-operative assessment the patient is confused with a temperature of 38.5deg. pulse of 120, BP110/70mmHg and saturations of 90% on 6 litres of oxygen. How would you go about ensuring the patient was consented? The aim of this viva is to ensure candidates can: y y y Describe an appropriate plan for preoperative investigation and resuscitation of this Outline the implications for anaesthesia of Alzheimers disease including consent issues. Discuss a plan for anaesthesia for this patient with appropriate choice of technique and patient and post-operative care.

management of regurgitation during induction. Manage intraoperative haemodynamic instability with appropriate choice of fluid resus and inotrope support. y y y y y y y How to get consent? How would you assess her? What investigation would you do? (ECG showed ischaemia pattern, RBBB, AF. CXR showed aspiration pneumonia. ABG showed severe metabolic acidosis) How would you optimize her? Suppose no ICU bed available, what would you do? Suppose OT 1 hour later. How would you optimize her in OT? Intraop hypotension. Management?

Scenario 7
A 20 year old male is brought into the emergency department of your trauma centre after a fight in a city laneway. Ambulance officers report that he has sustained a gunshot would to the chest. Initial assessment reveals that he has a Glasgow Coma Score of 15 but is distressed by

10 Years Collection of ANZCA Part II Anaesthetic Viva

MK Yuen & SW Ku

10 Years viva collection 2009

267

dyspnoea and cannot speak a sentence. He has a SaO2 of 94% on oxygen via a Hudson mask, BP 130/80 and HR 130. Examination confirms a single gunshot would with entry in the right midthorax, midaxillary line and an exit would at the base of the left side of the neck above the clavicle. He has a subcutaneous emphysema and diminished air entry on the right. A chest x-ray reveals right sided opacity and pneumomdeiastinum. How would you initially assess and resuscitate this patient? The Aim of this viva is to ensure that candidates can: y y Discuss initial assessment and resuscitation of patient with penetrating chest/mediastinal Outline plan for evaluating and securing the airway in a patient with a potential tracheal injury and using knowledge of anatomy outline potentially injured structures. injury including the place of fibreoptic or rigid bronchoscopy and the need for formulating detailed plan with surgeons. Discuss an anaesthetic plan and options for surgical repair of injury to thoracic trachea. y y y y y y y How would you assess this patient? Suppose your put a chest drain on right side but right lung just partially expanded. Chest drain drained 800ml. What is the cause? What other vital organ do yo think that has been injuired by that injury? Suppose you suspect patient had trachea injury. How would you assess? Patient was agitated and do not allow awake FOB assessment. What do you do next? How would you anaesthetize him for rigid bronchoscopy? Suppose cardiothoracic surgeon found that the injury 2 cm above carina. How would you anaesthetize him?

Scenario 8
A 40 year old man who is Hepatitis C positive, presents for debridement of his hip for chronic osteomyelitis. He has been stable for 10 years on Methadone 100mg daily, for his heroin addiction. He is known to have very difficult i.v. access. How would you assess this patient in light of his Hepatitis C status? Aim of this viva is to ensure candidates can:

10 Years Collection of ANZCA Part II Anaesthetic Viva

MK Yuen & SW Ku

10 Years viva collection 2009

268

y y y y

Discuss principles of perioperative care of patient with opioid addiction. Describe an appropriate plan for anaesthesia and postoperative pain management in this Discuss issues around PCA usage in patients absenting themselves from the ward. Describe appropriate strategies to resolve this problem including using community

patient.

resources e.g. drug and alcohol workers and patients methadone prescriber. y y y y y y y How would you assess this patient? What is Childs Classification? How would you anaesthetize this patient? What is your pain relief plan? Suppose you plan to give GA + EA for pain relief. Postop patient complain of pain, how to deal with this problem? 2 days later, the epidural dislodge and patient complained pain. How would you management? Before discharge, durgeon ask you to assess this patient and provide you opinion on pain relief for this patient. What is you plan?

Scenario 9
A 6 year old boy presents as a day stay patient for conservative dentistry and multiple extractions or primary teeth under general anaesthesia. He has a history of asthma and lives 50 kilometres from hospital. He has a runny nose and a cough but on examination he is aferbrile with a clear chest. How would you evaluate the severity of this respiratory tract infection? The aim of this viva is to ensure candidates can: y not. y y y y y y Outline an appropriate technique for dental anaesthesia in children including nasal Discuss appropriate discharge criteria for day stay anaesthesia in this child. Has DD and management plan for intraoperative hypoxia. Would you accept this patient for surgery? How would you assess his URTI? Suppose you accept this patient for surgery. How would you anaesthetize this patient? intubation. Discuss preoperative assessment of URTI with practical decision criteria for cancelling or

10 Years Collection of ANZCA Part II Anaesthetic Viva

MK Yuen & SW Ku

10 Years viva collection 2009

269

y y y y

Uneventful surgery, postop the nurse asked you to assess his agitation. How would you assess him? Cause of agitation? Intra-oral bleeding. What is your management? Post op discharge criteria? Suppose post op patient complained nausea and vomiting. What is you management?

Other candidate was specificly asked whether throat pack will be put in. Also was asked about management of intraoperative desatuartion.

10 Years Collection of ANZCA Part II Anaesthetic Viva

MK Yuen & SW Ku

10 Years viva collection 2009

270

ANZCA Anaesthetic Viva May 2007 Day 2 Dr. G Wong, QMH


Scenario 1
You are the Consultant Anaesthetist at an Obstetric Hospital. You are asked to come urgently to assist in the management of a 35 year old multipara (G3P2), who has had borderline hypertension, but has presented at 34 weeks with severe headache. Her BP is 200/110. There are concerns about the CTG. What other information would you require? The aim of this viva is to ensure that candidates can: y y y y Discuss the assessment, differential diagnosis and management of severe unstable Safely manage general anaesthesia in this situation and investigate and manage haemodynamics intraoperatively.. Discuss the management alternatives had patient been known to have hypertension in pregnancy. unstable

phaeochromocytoma before caesarean section. E: What is your anaesthetic concern for this lady? C: Medical concern includes HT, its cause, other complication and need urgent reduction; third trimester pregnancy and physiology changes. Obs concern includes: She is multigravida and may increase risk of PPH, uterine atony; for the fetus: he is premature at 34 week, nonreassuring CTG may need urgent LSCS. Anaesthetic concern: RSI, difficult intubation, high risk of PPH E: What is the cause of HT for her? C: Pregnancy related e.g. preclampsia, labour pain, placental abruptio or nonrelated like chronic idiopathic HT, HT secondary to thyrotoxicosis, cushing disease, Conns syndrome, phaeochromocytoma. E: How would you assess her HT? C: Ill reveal antenatal and past medical notes to look for preexisting HT and treatment, the most likely diagnosis is preeclampsia, so Ill search for other manifestation of this multisystem disorder: In CNS: headache, increased ICP, convulsion; in CVS: HT, contracted iv volume; in lung: they are prone to pulmonary edema; also elevated liver enzymes and low platelet count, coagulopathy. Then, Ill start MgSO4 to lower the BP

10 Years Collection of ANZCA Part II Anaesthetic Viva

MK Yuen & SW Ku

10 Years viva collection 2009

271

E: Before you give MgSO4, what else will you do? C: Ill confirm eclampsia with urine test, inform obstetrician, continue CTG monitoring, transfer the patient to Obs HDU for close monitoring of complication of eclapmsia, insert A-line under LA, start MgSO4 infusion and use labetolol 5mg iv increment to lower her BP E: What dose of MgSO4? C: 4g iv bolus, then 1-2g/hr to keep MgSO4 at 2-3mmol/L E: What is the side effect of Labetolol to fetus? C: Fetal hypoglycaemia and fetal bradycardia secondary to B Blockage E: What is the criteria of diagnosing PET? C: beyond 20 week gestation, proteinuria >300mg in 24hr, BP >140/90 in two separate occasion of 4hr apart E: The CTG become late deceleration, obstetrician want to perform urgent LSCS. What is your anaesthetic plan? C: Ill have quick preop assessment of the patient, make sure she is adequately resuscitated and BP is under control, continue MgSO4 infusion, obtained informed consent, give antacid prophylaxis, inform paediatrician to standby for a PREM fetus. Ill ask obstetrician for their urgency, If time is not allowed, Ill perform GA for LSCS E: What factors determine whether youll perform a SA or GA? C: Obstetrical urgency, anaethetists experience, how quick I can perform a reliable block E: What is the advantage of SA over GA in this patient? C: A quick and reliable SA avoid the complication related to GA like regurgitation, difficult intubation and orodental trauma, allow mother to participate in child birth, less drugs from intrathecal space being transferred to fetus and better APGAR score. E: Suddenly, fetal HR drops to 80/min, obstetrician need an urgent LSCS. How would you induce the patient? C: Ill give antacid prophylaxis, transfer in L lateral tilt 15 degree, make sure T&S, Paediatrician is informed, Ill preoxygenate and prepare difficult intubation trolley, RSI using thiopentone 4mg/kg and suxamethonium 2mg/kg with cricoid pressure using sized 9 COETT, maintenance with O2/N2O/Iso E: Just after the Baby is delivered, BP dropped to 80/40, ETCO2 dropped to 30mmHg, what is your response? C: Ill confirm the situation for a reliable ETCO2 tracing. Inform surgeon, resuscitate while simultaneously look for the cause. Give 100% O2 via ETT, increase IVF replacement, quick check on surgical blood loss and catch up with loss. Ddx includes: hypovolaemia secondary to blood loss; cardiogenic like AMI or arrhythmia; Obstructive cause like DVT with PE, amniotic fluid embolism, sepsis or anaphylaxis. Ill have a quick look of the ECG tracing, look for rash and fever, auscultate the chest to rule out pneumothorax, examine LL for DVT. E: What is the treatment of amniotic fluid embolism?

10 Years Collection of ANZCA Part II Anaesthetic Viva

MK Yuen & SW Ku

10 Years viva collection 2009

272

C: Mainly supportive of ABC. E: The patient survive the operation and come back for elective intraabdominal surgery 3 month later, her high blood pressure is being diagnosed as phaeochromocytoma. How would you do your preop preparation? C: Ill optimize her phaeochromocytoma by giving alpha blocker phenoxybenzamine; replace IV fluid, then beta blocker like metoprolol or propanolol. Ill allow for elective surgery when she can fit the RoiZen criteria: includes: BP below 160/90; nasal stuffiness, BP above 80/40, No PVC and no ST and T wave changes in last 24 hour. Bell

Scenario 2
A 70 year old man, with cervical canal stenosis. Presents for posterior cervical decompressive laminectomy. He has mild hypertension, controlled with medication. What causes the symptoms and signs in patients with cervical spine disease? The aim of this viva is to ensure candidates can: y y y y y Recognise significance of this patients neurological signs. Describe preoperative airway assessment and discuss factors guiding choice of Discuss the advantages and disadvantages of prone Vs sitting position for this surgery. Outline a safe plan for anaesthesia in the sitting position for surgery in this man. ? if time Describe a systematic approach to diagnosis and management of fall

intubation technique.

intraoperative fall in ET CO2. E: What is the cause for sign and symptom of him? C: Tumour, trauma, vascular incident, infection, inflammation, prolapsed disc on either spinal cord or nerve roots leading to upper and lower motor neuron lesion. E: if the spinal stenosis is on C4-C6, what is the expected sign and symptom? C: Sensory changes over lateral aspect of shoulder, upper arm, forearm and down to middle finger; motor changes: loss of shoulder adduction, bicep flexion; maybe pain in C4-6 dermatome. E: Can you compare the pros and cons of sitting vs prone position for this surgery? C: Prone position problem: difficult to access the patients head during procedure, easy dislodgement or kinking of ETT, facial or cerebral edema after prolong head dependent position, pressure on abdoman will impede VR from LL, increase epidural venous preesure hence increase surgical blood loss, no way for CPR in prone. Sitting position has the

10 Years Collection of ANZCA Part II Anaesthetic Viva

MK Yuen & SW Ku

10 Years viva collection 2009

273

advantage over prone for easier access to patient s airway, less abdominal compression, however, it has difficulty in fixing the head in position, prone for falling, traction of brachial plexus, reduce venous return from LL, increase DVT, stretching sciatic nerve, compression on femoral nerve and common peroneal nerve, need careful padding, increased risk of VAE or postop blindness compare to prone, it is also difficult to perform CPR if sitting. E: What monitoring will you use? C: Monitoring according to college guideline like NIBP, SPO2, ECG, ETCO2; Ill also use Aline for blood pressure monitoring, PNS, temperature probe, foley for urine output monitoring; SSEP and MEP for cord function monitoring E: Where will you put youre A-line? C: Either side radial artery, if proved impossible, Ill choose dorsalis pedis, with the transducer at heart level E: Why do you want to put in heart level? C: So, it reflect accurate BP E: There is some suggestion of using CVP, what is your opinion? C: The purpose of CVP can be: assessing fluid status and guide replacement; infusion of inotrope, aspirate air in VAE. I wont put a central line for this patient because I dont expect large surgical blood loss or inotrope usage; besides, if VAE is very likely, I need a special CVP with multiple lumen and side holes which aid aspiration of air & the tip should be inside RV instead of RA as in ordinary CVP line E: What is the exact location where the tip of that catheter should be? C: At junction between RA & RV. E: You have mentioned youll use SSEP and MEP, where do you think the cord will be susceptible for injury? C: Anterior 2/3 as they are supplied only by a single anterior spinal artery, this includes anterior motor tract and lateral spinothalamic tract for pain and temperature sensation E: What is the origin of anterior spinal artery? C: both vertebral artery E: How about posterior cord blood supply? C: By 2 posterior spinal artery E: What is the origin for posterior spinal artery? C: posterior inferior cerebellar artery E: The surgeon decided on sitting position, pins and extenal fixator applied to head, You have put in your monitoring, 30 minutes into surgery, you noticed BP become 90/70, what will you do? C: Ill confirm a reliable A-line BP tracing, correct position of transducer of A-LIne, look at patient color, feel for pulse, quick look at surgical field for blood loss. Ill correlate the patients normal BP reading and see whether it is low compare with normal setting. Ill resuscitate while

10 Years Collection of ANZCA Part II Anaesthetic Viva

MK Yuen & SW Ku

10 Years viva collection 2009

274

simultaneously look for cause. Ill give 100% O2, readjust anaesthetic depth with volatile, increase fluid replacement and prepare alpha agonist like phenylephrine 100mcg iv bolus increment and titrate with BP response. The DDX could be: hypovolaemia due to blood loss or dehydration; controlled hypotensive technique; cardiogenic like AMI, arrthymia, Obstructive: Pneumothorax, VAE or DVT complicated with PE, Distributive: anaphylaxis, sepsis E: Would you consider 90/70 is low BP? C: It depends on patients normal blood pressure, since the patient has hypertensive disease, the brain autoregulation range will be shifted to right, 90/70 may be lower than normal for him E: You noticed also a gradual decrease in ETCO2? C: Ill confirm the reading and reliable ETCO2 waveform, the reason could be leaking from machine or circuit or patient; stuck opened or closed valve; hyperventilation; reducing cardiac output like PE or arrhythmia; blocked sampling line. Ill check integrity of circuit and valve, adjust ventilator setting. Bell

Scenario 3
A 4 week old infant born at 38 weeks gestation presents with an inguinal hernia. His birth weight was 2.4 kg Apgars 5 and 9; He now weight 2.9kg is breastfed and is otherwise well having been home from day 5. This is the parents second child; they live 15 minutes drive from the hospital and are both teachers and are very keen for same day surgery. Is this infant suitable for same day surgery? The aim of this viva is to ensure candidates can: y y Discuss criteria for day surgery in infants and its appropriateness in this case. Discuss risks and benefits of spinal vs general anaesthesia for this case.

Outline a management plan for failed intubation in this case E: Would you allow them as day case? C: No, because he only has PGA of 42 week, there is high chance of postop apnoea and need close observation in ward. Current guideline suggest full term neonates less than 46 week or preterm neonates less than 60 week PGA should not be accepted as day case due to high incidence of postop apnoea. E: How about the parents are very keen for leaving on surgery day? C: Ill explain the reason is for the good and safety of their baby due to higher incidence of postop apnoea or complication. E: What lab investigation will you do to reduce postop apnoea?

10 Years Collection of ANZCA Part II Anaesthetic Viva

MK Yuen & SW Ku

10 Years viva collection 2009

275

C: Usually I wont order investigation for a baby with good antenatal and postnatal health for inguinal surgery. However, to look for risk factor for postop apnoea, Ill check Hb, electrolyte, temperature, glucose level to rule out anaemia, hypo/hypercalcaemia, hypo/hyperkalaemia, hypo/hyperglycaemia, acidosis which could precipitate postop apnoea E: If the parents express their interest on regional technique, what can you offer? C: Peripheral nerve block: inguinal field block or central neuraxial block like spinal or caudal block E: How long will you fast you patient? C: 4hr for formula milk; 3hr for breast milk; 2hr for clear fluid E: How would you anaesthetize this baby? C: After preop assessment and optimization, proper fasting and obtained consent, give EMLA premed, Ill assess parents anxiety level, hopefully I can ask the parents to accompany the baby to OT and help with induction. I prefer iv induction, but if IV access is proved impossible, Ill use gas induction with 100% O2 and sevoflurane, intubate with sized 3 noncuff ETT using sized 0 miller blade, taped 9cm at lip, confirm position with presence of bilateral breath sound, no gurgling sound in epigastium, maintenance with O2/Air/Iso, give panadol 20mg/kg PR E: If you cant intubate during your first attempt, what will you do? C: Ill have a second attempt after ensuring the induction agent has been flushed in, readjustment of head and neck position, use of one size smaller tube, adjust external laryngeal pressure; at the same time Ill assist patient ventilation with T piece and 100% O2, ask for a difficult intubation trolley and prepare difficult intubation equipment like sized 0 LMA and other airway adjunct. E: What is the problem with using LMA for neonate? C: Neonate has variable body weight and highly variable upper airway anatomy, LMA sized 0 may not fit for all these anatomical variation and make insertion difficult or easy malposition during procedure; besides, Neonates has high closing volume compare with FRC, high basal atelectasis and prone for hypoxaemia; Neonates breathes with their diaphragm, lack type 1 muscle fiber in diaphragm and prone for fatigue if use LMA with spontaneous respiration method. E: You have mask ventilate the patient during the struggle, what would happen to the patient? C: gastric distension with O2. So Ill pass a orogastric tube to deflate stomach. E: Just the stomach or the lower intestine as well? C: Mainly stomach, as a working pyloric sphincter will reduce gas being insufflating into small intestine. E: Tell me how you would perform a spinal anaesthesia in this baby? C: Ill check indication, rule out contraindication like sepsis, coagulopathy or parents refusal, obtained informed consent from parents. E: Just tell me how you would do the block?

10 Years Collection of ANZCA Part II Anaesthetic Viva

MK Yuen & SW Ku

10 Years viva collection 2009

276

C: Ill perform spinal after induction of GA as neonates wont cooperate, fast as if for GA, prepare theatre, staff, drugs and equipment for neonatal anaesthesia. After establishing monitoring according to college guideline, and a reliable iv access. Ill ask an assistant to turn patient lie lateral, surgically prepared the patients back. Prepare instrument: neonatal spinal needle,22G 26mm long needle, 1ml syringe, 0.5% plain bupivacaine at 0.1ml/kg, give extra 0.1-0.2ml as needle dead space, for this baby, Ill give 0.5ml in total. Since neonate ends their spinal cord lower than adult, can down to L3 level, so Ill choose L4/5 space, midline E: How long will you expect the spinal would work? C: 1-2hr E: Do you expect a drop in BP after the spinal? C: No, since neonates has poorly developed sympathetic system E: What risk will you tell the parents about the spinal? C: Expectancy, duration, common but reversible complication: fail block, drop in BP, urine retention or uncommon but significant complications like infection, nerve injury, SA can help reducing postop apnoea but not 100% guarantee of this. E: What is you postop pain management? C: Ill use multimodal technique based on Panadol 20mg/kg po up to 60mg/kg/day, supplement with LA infiltration, titrate with up to 0.1mg/kg iv morphine Bell

Scenario 4
You are a Specialist at a tertiary hospital who has participated in the cardiac anaesthesia service for 12 months. At 6pm you are asked to anaesthetise Mr John Smith for emergency CABG and mitral valve repair/replacement. Demographics Male aged 40, previously well. Admitted yesterday with acute inferior myocardial infarction. Now hypotensive (80/50) and breathless at rest in the CCU with a new heart murmur. Treated with docutamine 5mcg/kg/min, tirofiban and GTN infusions Medications. Nil prior to this episode, no allergies Uneventful GA for arthroscopy in past. How would you optimise Mr Smiths condition preoperatively?

10 Years Collection of ANZCA Part II Anaesthetic Viva

MK Yuen & SW Ku

10 Years viva collection 2009

277

The aim of this viva is to ensure candidates can: y y Describe how this patients condition may be optimised preoperatively including the place Describe placement, optimal positioning and timing of IABP. of IABP. Describe approach to patient with anticipated and actual difficulty weaning from bypass. May include use of milronone and/or levosimendan E: How will you optimize this patient before emergent CABG? C: Since this patient is suffering form acute inferior AMI, Ill optimise by increase myocardial O2 supply and reduce demand. Increasing supply by giving 100% O2 via nonrebreathing mask, check SPO2 and respiratory effort, if patient is fatigue, Ill intubate to reduce respiratory stress, give GTN, morphine 5mg iv bolus to relieve anginal pain, check Hb level, increasing O2 carrying capacity by topping up Hb to 10g.dL, maintain DBP for coronary perfusion by increasing dobutamine infusion, reduce myocardial demand by treating arrhymia or tachycardia E: You have increased your dobutamine to 10mcg/kg/min, O2 100%, however, BP still around 90/60, Is there anything you can do to further optimizing him? C: Ill make sure the patient is adequately monitored in CCU with NIBP, Aline, SPO2, ETCOs if intubated, add alpha agonist like phenylephrine infusion to increase DBP, ask cardiologist for TTE to check for complication of AMI like MVP, MR, acute heart failure, cardiac effusion or even cardiac tamponade E: ECHO result: global hypokinesia, EF 25%, no tamponade. Coronary catheterization: severe stenosed RCA and other coronary vessel. Any other drugs that you can use to optimize him? C: alpha agonist like noradrenaline. I dont use adrenaline until last resort as I worry about tachycardia from B agonist effect could further increase heart demand. Dopamine is not useful because it start with dopaminergic and beta effect at low and moderate dose. New drug like levosimendan, mirinone to push up contractility E: What is Mirinone, how do they work? C: Mirinone is a phosphodiesterase inhibitor, it work by stimulating cAMP and increase myocardium Ca level in a mechanism other than B agonist, so increasing myocardium contractility E: Do you know any device could help? C: Yes, I can put in an IABP. E: How do IABP help? C: It is a balloon device that is put via femoral artery cannulation, just distal to L subclavian artery origin and distal end above renal artery origin, inflate at early diastole to increase DBP, deflate at end of diastole, just before systole to reduce afterload of heart

10 Years Collection of ANZCA Part II Anaesthetic Viva

MK Yuen & SW Ku

10 Years viva collection 2009

278

E: How can IABP reduce afterload of heart? C: It reduce LVEDP, so reduce afterload E: Can you draw me a normal aortic pressure waveform? E: On top of it, can you draw the new aortic pressure wave form after IABP started? E: What is this? (Pointing to a diacrotic notch), what does it signify? C: Aortic backflow during systole after aortic valve closure E: The patient BP is optimized to 120/80, HR 80/min, taken into theatre and CPB started, before the surgery is finish, what will you prepare before coming off bypass? C: Ill make sure patient is warm, no hypothermia, check electrolyte and correct if any disturbance and acidosis, check ACT and prepare protamine to reverse heparin action, prepare vasopressor like phenylephrine or noradrenaline in case BP crash after offing bypass and vasodilator like GTN, SNP if BP surge up, prepare epicardial pacing device, use TOE, make sure the surgeon has adequately deair before offing the bypass E: Anything else? C: Since the patient has very poor EF and acute heart failure, Ill prepare defibrillator, epicardial and transthoracic pacing E: Surgeon cant off bypass as BP is only 60/40, what will you do? C: Ill optimize the patient and try to reverse any reversible factor, ddx could be: preexisting heart failure, inadequate revascularization, mechanical factor like air in coronaries, direct pressure and kinking of coronaries, severe metabolic disturbance. Get a more experience surgeon, use TEE to look for new complication. Bell

Scenario 5
You are called to the emergency department as the pain consultant for the day. A 75 year old man with advanced prostate cancer with vertebral metastases has presented to hospital following a fall. He is complaining of pain in his left side. He has significant chronic obstructive airways disease with limited exercise tolerance. A chest x-ray shows fractures of ribs 7, 8 and 9 on the left side. He is currently taking oral sustained release morphine, 600mg twice daily, plus oral immediate release morphine up to 400mg daily. He is also on paracetamol, enalapril, aspirin and inhalational puffers. How do you decide on his initial management?

10 Years Collection of ANZCA Part II Anaesthetic Viva

MK Yuen & SW Ku

10 Years viva collection 2009

279

The aim of this viva is to ensure candidate: y y y Have an understanding of pain management in patients on chronic opioids. Can discuss options for appropriate pain management for this man with risks and Can demonstrate an understanding of treatment options for his neuropathic pain

benefits of regional procedures. including systemic treatments and the risks and benefits of intrathecal therapy. E: How will you offer your pain management? C: Ill ensure the patient has been assessed and managed according to ATLS protocol, ensure ABC safety, give 100% O2 via nonrebreathing mask and attach monitoring. Then, Ill assess the patient via history, PE and Ix and formulate my pain management plan. Ask a detail pain history about: character, site, any radiation, any precipitating and relieving factor, other coexisting injury causing the pain. Perform a proper physical exam, check for other complication of rib fracture like pneumothorax, haemothorax, flail segment compromising ventilation. Ill give iv morphine 5-10mg bolus and ketamine 5-10mg iv bolus and titrate response E: The patient does not suffered from any other injury or no pneumothorax or haemothorax. But he still complain of pain after your morphine. How much do you think you need to give to relieve his pain? C: Since he is taking almost 500mg iv morphine equivalent daily, Ill expect he will need at least 50-100mg iv to relieve the pain. E: So, what will you do for his pain? C: Ill make sure he has receive his normal daily morphine maintenance via infusion, Ill manage his new injury which might contain both nociceptive and neuropathic component with multimodal technique: besides morphine, Ill give tramadol, panadol, ketamine, antidepressant, anticonvulsant, however, we have to beware that antidepressant need 7- 14 days to work, other choice like membrane stabliser and discuss with patient on choice of regional block E: What regional block will you do? C: Ill perform thoracic epidural on T8 level, give continuous infusion of 0.125% ropivacaine and 2mcg/ml fentanyl at 8ml per hour. It has advantage over multiple intercostal nerve block as it can provide prolong analgesia via catheter technique, less systemic absorpion and fewer systemic toxicity; also has advantages over interpleural or paravertebral nerve block Besides, TEA is good for a COAD cripple as it give best analgesia and reduce respiratory complications. E: After your epidural and run at 6ml/hr, nurse complain of low BP of 90/70, what is your response? C: Ill confirm the BP and compare with patients normal BP level. Resuscitate while simultaneously look for cause: Ill make sure ABC is alright, rule out other causes for drop in

10 Years Collection of ANZCA Part II Anaesthetic Viva

MK Yuen & SW Ku

10 Years viva collection 2009

280

BP like bleeding somewhere, haemothorax, tension pneumothorax or intrathecal migration of epidural catheter E: Do you expect an epidural of your concentration will responsible for a drop in BP? C: No. E: How long will you keep your epidural infusion? C: In our institution, well normally keep EA catheter for up to 5days as more complication like epidural abscess, infection or catheter migration usually higher after 5 days. However, Ill opt for keeping the catheter longer than 5 days for this patient, up to 7days E: You have mentioned ketamine, how effective is ketamine in treating neuropathic pain? C: Ketamine is an NMDA receptor antagonist, it reduce wind up phenomenon, reduce central and peripheral hypersensitivity, improve pain, allodynia and hyperalgesia and decrease requirement of other analgesia, can act as adjuvant to opioid in chronic opioid user E: How will you give antidepressant? C: Ill give amitryptiline, start with 10mg, up to 25mg po nocte E: What is their side effect? C: Slow onset, need 7-14 days to have analgesic action, anticholinergic effect: drying up, tachycardia, sedation; antiadrenergic action: lower BP; antihistamine: sedation; and antidopaminergic effect E: How about anticonvulsant, what will you give? C: Ill choose gabapentin start with 300mg, some patient may need up to 1800mg daily po E: What membrane stabilizer will you use? C: IV lignocaine infusion, Na valproprate, carbamezepine Bell

Scenario 6
Your registrar calls you from the endoscopy suite where she has been sedating a 60 year old woman for upper GI endoscopy and biopsy of an oesophageal lesion. She is very upset as she gave atracurium instead of midazolam and was forced to intubate and ventilate the patient. She is now giving sevoflurane anaesthesia. How will you and the registrar manage this adverse event? The aim of this viva is to ensure candidates can: y y y Describe an appropriate professional and ethical approach to management of the initial Discuss reasons for drug error including system errors. Plan anaesthesia for subsequent oesophageal surgery taking into account patients event with both follow-up for the patient and support for the trainee.

10 Years Collection of ANZCA Part II Anaesthetic Viva

MK Yuen & SW Ku

10 Years viva collection 2009

281

previous experience. E: How will you manage this situation? C: Ill go and attend the patient immediately. Ill obtain more history and information of the patient on my way to the UE suite. I ll make sure the patient is adequately anaesthetized and ventilated, ABC is alright. Assess surgical procedure and any other anaesthetic or surgical complication. E: The UE has finished. What will you do? C: Ill reverse the patient after ensuring no other complications, no hypothermia, check degree of paralysis with PNS and reverse with 50mcg/kg neoostigmine. Ask my registrar to document everything clearly in patients note. Arrange interview and follow up with patient to detect any complication related to this incident. E: She is a day case, while you follow her up later in the afternoon, she told you that halfway through the operation, suddenly, I feel very weak and cant move, then, somebody cover my face with something and pumping me with some air, then I feel a cold thing in my mouth and my tongue being pulled to one side, then something stucked into my throat and then I lost consciousness. How will you response? C: This is a serious complication. I worry that the incident has lead to awareness. Ill detailing the patients experience, express empathy and assess her psychological damage related to this incident. Ill ask a nurse to closely look after and accompany the patient as patient having severe post traumatic stress experience may harm themselves or even commit suicide. Ill document this in patients notes. Refer patient for psychological counseling. E: The patient is a day case, will you allow her to go home? C: No, Because there is anaesthetic complication happened, I need close monitoring of patient. Ill explain my reason to patient and her family E: To monitor what? C: Her psychological reaction after a traumatic experience. E: How about your registrar? C: For my registrar, she is definitely getting upsetted. Ill arrange counseling with her ASAP in a quiet and private room. Tell her that it is common for every anaesthetist to commit some form of complication in our career life. Ill discuss with her and check with the reason of the incident like: inadequate training, education, insufficient supervision, tired anaesthetist or miscommunication or some acute problem affecting the registrar like life event. Ill offered help and refer psychologist counseling if needed. Ill relieve her from her afternoon duty and get more social support from her family, colleague and friends. E: Who else will you inform? C: Ill also apologise to the surgeon for any delay in their list. Report to department head and arrange departmental education after investigating the reason causing this incident, report also

10 Years Collection of ANZCA Part II Anaesthetic Viva

MK Yuen & SW Ku

10 Years viva collection 2009

282

to insurance company. E: The patient need to come back for oesophageal surgery as last UE found out she has oesophageal perforation. The patient is anxious and her family is anger, they ask whether the experience will happen again in this anaesthetic. How will you response? C: Ill arrange the preop interview in a private room with nurse as my witness. Ill show empathy of her awareness experience, tell her that awareness is a known complication of incidence around 3:1000 that could happen in any anaesthetic. I will do every means to reduce this to happen again like using midazolam as premedication to reduce preop stress level, besides using basic monitoring as suggested by college guideline, Ill use extra monitoring to check anaesthetic depth like physiological parameters: tachycardia, HT, lacrimation and BIS monitor, lll promise her that Ill put extra care for her but I cannot guarantee that awareness would not happen again. During the anaesthetic, Ill document in details of what intervention or drugs being given. Arrange postop visit. Bell

Scenario 7
A 24 year old female with severe, poorly controlled asthma presents with acute peritonitis for ruptured appendix. She has been booked for urgent surgery. What are the major issues for you anaesthetising this patient? The aim of this viva is to ensure candidates can: y y y Assess asthma severity and optimise treatment of asthma in this situation. Discuss merits of Open vs laparoscopic procedure for this patient. Discuss DD and manage hypotension and difficulty with ventilation intraoperatively (due

to anaphylaxis). E: What is your anaesthetic concern? C: Ill divide it into medical, surgical and anaesthetic concern Medical concern: Causes of asthma, severity, complications, treatment, need of steroid? How frequent does she need ventolin puff, any previous ICU admission and intubation? Smoker and recent URTI may increase severity. In PE: want to check for sign of severe asthmatic attack like tachypnoea, use of accessory muscle, intercostal insucking, hyperinflated chest, generalized wheeze, Ix I want to check ABG for hyperventilation pattern, PFR and compare with patients normal value, make sure T&S is ready as patient is going for surgery, retrieve past medical notes from her GP Surgical concern: She is suffering from intraabdominal sepsis, I worry about dehydration and

10 Years Collection of ANZCA Part II Anaesthetic Viva

MK Yuen & SW Ku

10 Years viva collection 2009

283

whether she has been adequately resuscitated with antibiotics and rehydration Anaesthetic concern: Emergency surgery does not give me much time for optimization of patient, laparoscopic surgery need GA with endotracheal intubation & IPPV which may precipitate perioperative bronchospasm and respiratory complication, RSI for possibility of full stomach, I need to check her steroid treatment and consider steroid replacement periopoeratively E: Sheet of Ix result: SPO2 92% on 6L/O2, hypercapnoea, hypoxaemia, RR 24/min, generalized wheeze. What will you do next? C: Ill resuscitate the patient right away in ward, make sure ABC is alright, give 100% O2 via nonrebreathing mask, discuss with surgeon the need to optimize her asthmatic attack before intraabdominal surgery E: How will you optimize the patient? C: Rule out other reversible condition like tension pneumothorax or pneumothorax due to rupture of bullae, Treat precipitating factors like URTI with antibiotics, give O2 supplement, make sure her usual asthmatic medication has been taken correctly . and give extra ventolin puff. Ill give ventolin via nebulizer since her attack may hinder her from getting ventolin via volumetric device E: What dose of ventolin nebulizer? C: Ill mix 2.5mg ventolin in 1:1 ratio with 0.9%NS and administer via nebulizer device E: What else can you give to treat asthma? C: B2 agonist, anticholinergic, ketamine, steroid, mast cell stablizer, Na montaleuklast Intraoperatively with volatile agent like halothane E: The surgeon said they want to do laparoscopic appendicectomy, would you allow them to? C: No, because laparoscopy can further increased intraabdominal pressure and reduced lung compliance and impede ventilation E: What ventilatory strategy would you do? C: Lung protection management: lowest FIO2 to maintain SPO2 above 95%, increased RR to 15/min, lowered TV to 6-8ml/min, permissive hypercapnoea, IE ration1:2.5 or 3 E: Your patient has been treated by what you have described and been taken into theatre, 30minutes into surgery, BP suddenly dropped from 110/70 to 70/40, IAP increased, dropped SPO2 to 89%, what is your action? C: Ill confirm the situation by checking for a reliable wave form, look at patient for color and pulse checking, increased FIO2 to 100%, and IVF to full rate, tell surgeon to stop surgery Ill resuscitate while simultaneously look for the cause of hypotension and desatuation Ddx: Tension pneumothorax, anaphylaxis, Asthmatic attack with increasing bronchospasm For desatuation, it could be machine cause, circuit or patient Ill isolate the patient from machine, hand ventilate, feel compliance and pass suction catheter

10 Years Collection of ANZCA Part II Anaesthetic Viva

MK Yuen & SW Ku

10 Years viva collection 2009

284

into ETT, aspirate any sputum, confirm there is no ETT kinking, quick check on the depth of ETT marking, auscultate for unilateral BS for endobronchial intubation, pneumothorax, generalized wheeze for bronchospasm, rash for anaphylaxis, crepitation for pulmonary oedema or aspiration, Quick look into surgical field to estimate blood loss E: You found that patient is flushy, what is your action? C: I worry patient is having anaphylaxis. Ill stop giving any drugs that I am suspecting for the anaphylaxis. Resuscitate with 100% O2, adrenaline 10ug iv bolus increment, H1, H2 blocker, 100mg hydrocortisone, further B2 agonist but mind not to limit toxic dose, take blood for mast cell tryptase, review anaesthetic notes and past medical history for any suspicious drugs causing anaphylaxis, book ICU bed for monitoring while asking the surgeon to finish the operation ASAP. E: How does adrenaline work in anaphylaxis? C: Adrenaline has alpha agonist to counteract the effect of histamine vasodilation and beta agonist for bronchodilation. Bell

Scenario 8
A 50 year old, 100kg builder in on your list for an arthroscopic acriomoplasty. His only medication is Enalapril 10mg daily. The surgeon prefers an interscalene block for postoperative analgesia. Please obtain informed consent for this block from me as if I were the patient The aim of this viva is to ensure candidates can: y y y y Outline problems with positioning this patient for surgery. Outline problems with ACE inhibitors and anaesthesia and have appropriate Discuss postoperative analgesia including inter-scalene block. Discuss consent issues for block.

interventions for hypotension.

E: Imagine I am the patient, how would you obtain the consent from me? C: Good morning, Tony, I am your anaesthetist Dr W for your shoulder operation. E: Hello Dr W. C: I am here to assess your well being and discuss varies anaesthetic option for this operation. Your orthopaedic surgeon has asked us to perform an interscalene block. Have you heard of this? E: No

10 Years Collection of ANZCA Part II Anaesthetic Viva

MK Yuen & SW Ku

10 Years viva collection 2009

285

C: This block can be done as your sole anaesthetic technique or combine with GA as postop analgesia. Ill discuss with you the following aspect: your expectation about the block, the procedure, advantage and both common and significant complications. If you decide for the block, we will fast you as if for GA, youll be positioned on OT table, attached monitoring and iv access. Ill clean your neck skin with some alcohol, use a small needle to locate the nerve in your neck. You need to keep still during the procedure. A small current will pass through the needle to guide us to locate the correct nerve. Once we inject the LA, youll feel numbness and loss motor power of your arm until the effect of the drugs wear off. Complication includes: fail block, bleeding and infection, difficulty breathing due to anaesthetizing nearby structure and even nerve injury. I know that you are a builder, so, it might has more meaning to you if there is a nerve injury. E: Can I do it after I get sleep? C: Because of all that worries that I have just described with you, most anaesthetist will perform this block while you are awake so that we can detect reversible complication earlier and prevent devastating complications. Its for your own safety. E: OK, I am the examiner now. You have obtained the consent, surgeon told you that the operation will be done in beach chair. What problem can you expect for beach chair? C: Head position need to be fixed, worry of fall, traction on brachial plexus, stretching of sciatic nerve, compression of femoral nerve and common peroneal nerve. Reduced VR, increased risk of DVT due to immobility, risk of VAE, risk during transferring the patient includes lines dislodgment. E: So, you have done a GA and interscalene block with catheter, surgeon started in beach chair position, suddenly the patient BP dropped to 85/50, HR 38/min, what will you do? C: Ill confirm the BP tracing and HR. Resuscitate while simultaneously look for cause Ensure ABC, give 100% O2 via ETT, check ETT position by auscultation, increase IVF rate and quick look at surgical field to catch up surgical blood loss. Make sure T&S ready in case blood loss. DDx includes: Interscalene block: Intravascular injection of LA lead to LA toxicity, Epidural or spinal injection of LA lead to brainstem anaesthesia, pneumothorax secondary to BPB and complicating to tension pneumothorax. Other causes includes hypovolaemia due to bleeding or dehydration, cardiogenic like AMI or arrhythmia, obstructive: tension pneumothorax, tamponade, DVT complication PE and VAE from arthroscopy, sepsis or anaphylaxis. So, Ill check ECG tracing for possible arrthymia or AMI, auscultate chest for loss of breath sound to one side which indicate pneumothorax and endobronchial intubation; crepitation as aspiration or pulmonary edema; wheeze as in bronchospasm and anaphylaxis with rash. E: What do you think will cause both BP dropped and HR down to 38/min? C: Total spinal or brainstem anaesthesia secondary to interscalene block. So, Ill use IVF, vasopressor, phenylephrine infusion to top up BP, atropine to improve HR, stop further infusion

10 Years Collection of ANZCA Part II Anaesthetic Viva

MK Yuen & SW Ku

10 Years viva collection 2009

286

of LA via interscalene catheter, document, inform surgeon to finish operation ASAP and book ICU bed for postop monitoring. Ill arrange interview and FU with patient Bell

Scenario 9
40 years old male, victim of motor traffic accident that bumped into a tree. He wore seat belt. Driver was dead. He complained of SOB, chest pain and neck bruises. PE: GCS 15, SBP 90, HR 105/min, SPO2 95% on 6l/min O2
(this scenario was not mentioned in the examination report)

E: What is your priorities in managing this man? C: Ill assess the patient via ATLS protocol, make sure trauma team is alerted, A&E room is warmed up. In primary survey, airway: make sure airway is patent, open up airway, clear up any foreign bodies and blood clot. Breathing: expose the chest for physical exam: look for bruises, fracture ribs, any tachypnoea, use of accessory muscle and intercostals in-sucking, deviated trachea and hyper-resonance for tension pneumothorax or haemothorax, insert chest drain if indicated. Circulation: Ill attach monitoring , check BP, HR, ECG rhythm, color of patient, established 2 14G iv access and run 20ml/kg warm lactate ringer solution, cross match, look for any external bleeding and stop it by direct pressure Intermittent checking on ABC, check GCS E: What will you order in A&E? C: Ill order primary survey adjunct like trauma series XR on CXR, lateral cervical XR and pelvis XR, Foley, ABG and Hb level, cross match, NGT. E: Cervical XR show fracture C6, 7. Do you think a neck collar is needed? C: Yes, fracture spine may lead to cervical subluxation and neck collar is needed to stabilized the neck temporary E: CXR show widen mediastinum, what do you think is the cause? C: rupture aorta, tracheobronchial rupture, rupture oesophagus E: What other sign in CXR if it is a ruptured aorta? C: Haemothorax with white out lung field, mass effect in pushing trachea and mediastinum to R side, widen carinal angle and L bronchus being depressed. Haemodynamic changes as hypotension and tachycardia secondary to massive bleeding in rupture. E: Can it be a concealed rupture? C: Sure E: In fact the patient has a concealed haemorrhage. What investigation can confirm? C: CT thorax or angiogram using radiocontrast

10 Years Collection of ANZCA Part II Anaesthetic Viva

MK Yuen & SW Ku

10 Years viva collection 2009

287

E: Which one is better? C: Radiocontrast angiogram is a gold standard; however, for this unstable patient with unstable spine and haemodynamics, I prefer CT thorax to angiogram because it can be quicker and also use simultaneously for diagnosis of neck fracture and other intrathoracic and intraabdominal injury that require intervention E: What level do you think the rupture will be in aorta? C: Distal to origin of L subclavian artery, around T2-T6 level E: You are right, do you know why? C: Because it is attached to pulmonary artery with ductus arteriosus at that level E: How would you prepare for repair of rupture aorta? C: Ill resuscitate the patient, make sure ABC is alright, ensure big IV access, insert Aline and CVP if time allowed, T&S ready and prepare 6 units of blood, Use vasopressor or vasodilator to keep MAP around 60-70, avoid oversurging of SBP which might convert a concealed rupture to open rupture; Make sure our hospital has cardiothoracic surgeon and facilities for CPB back up. Prepare theatre, staff, equipment and drugs; equipment for OLV and thoracotomy like DLT and blocker, Hotline, even cellsaver E: Which side will you put youre A-line? C: R radial artery because if rupture involved subclavian artery, there will be loss of L radial artery reading E: How will you anaesthetize this man in theatre? C: After those preop assessment and optimization, prepare theatre, staff, equipment and drugs, establish the monitoring like A-line, prepare short acting B Blocker and vasodilator or vasopressor to blunt intubation stress. Ask surgeon to scrubbed up first .Ill induce with RSI with manual-in-line immobolization using cricoid pressure, using sized 39 L side DLT with careful and tender technique during insertion of tube. E: Why do you need careful and tender technique? C: Because I dont want any trauma to the ruptured aorta by the DLT E: Thoracotomy started, where do you think the surgeon will clamp the aorta? C: Distal to L subclavian artery and proximal to renal artery origin to preserve cerebral blood flow. E: Do you think L subclavian artery contribute to cerebral blood flow? C: No. E: How do the L arm get their blood supply during cross clamp? C: By collateral circulation. E: What is the consequence of aortic cross clamp? C: reduce spinal cord blood flow and also renal and splanchic arterial supply E: What is the incidence of spinal ischaemia in aortic rupture surgery? C: It depends on institution, in our institution, it range from 1 in 300-500

10 Years Collection of ANZCA Part II Anaesthetic Viva

MK Yuen & SW Ku

10 Years viva collection 2009

288

E: What can be done to reduce spinal cord ischaemia? C: Surgically: get an experience surgeon, reduce cross clamping time to less than 30min, Anaesthetically, preserve spinal cord perfusion pressure which is MAP-ICP or epidural pressure, Ill maintain MAP with vasopressor and blood, reduce ICP with CSF drain and locally cool the spinal cord to reduce metabolism. Bell

10 Years Collection of ANZCA Part II Anaesthetic Viva

MK Yuen & SW Ku

10 Years viva collection 2009

289

ANZCA Anaesthetic Viva September 2007 Day 1 Scenario only


Scenario 1
You have a 61 year old male for a left total hip replacement. He has a body mass index (BMI) of 39. How would you assess this man for his total hip replacement? The aim of this viva is to ensure candidates: y y Describe appropriate preoperative assessment of the morbidly obese for this surgery and a plan for anaesthesia including thromboprophylaxis. Discuss the management of intraoperative hypotension following reaming and cementing Evaluate alternatives for pain management and choose an appropriate regime.

Scenario 2
You are called urgently to the Emergency Department to manage the airway of a 23-year-old female cyclist who has just arrived via helicopter from a rural hospital. She has a head injury and fractured femur. Her Glasgow Coma Score (GCS) at the rural hospital was 13, but she is now minimally responsive with a GCS of 6, BP 80/40, pulse 120, SaO2 95%. She has equal pupils, abnormal flexing to painful stimuli, and no lateralising signs. Temp 35 deg C. It is now 3 hours since her injury. No abnormalities are seen on x-rays of chest, abdomen, pelvis or lateral cervical spine. What will be your initial management of this patient? The aim of this viva is to ensure candidates: y y y Describe the appropriate management of a trauma patient with rapidly deteriorating GCS and discuss the principles of cerebral protection Describe the safe management of the c/spine in a patient with a severe head injury and discuss clearing the cervical spine. Discuss the timing of surgery for femoral fixation, the role of ICP monitoring and the management of changes in ICP during surgery.

10 Years Collection of ANZCA Part II Anaesthetic Viva

MK Yuen & SW Ku

10 Years viva collection 2009

290

Scenario 3
You are called by the Emergency Department nurse because an anaesthetic registrar is struggling to obtain adequate analgesia in a 24 year old male with a fractured ankle. This patient is a frequent attendee to the Emergency Department, with sickle cell disease, always difficult to manage, often hostile and aggressive. His initial medications include: Slow release morphine 90mg bd Amitriptyline 25mg nocte What would be your initial approach to these issues? The aim of this viva is to ensure candidates: y y y Discuss pain management in opioid tolerant patient with poor response to prescribed analgesia. Discuss appropriate management of suspected opioid misuse by a colleague (ie administering to self rather than patient) Describe principles of anaesthetic management for sickle cell disease when patient comes to surgery.

Scenario 4
A 39 year old intellectually disabled woman with a large retrosternal goitre presents to your pre-anaesthetic clinic for proposed total thyroidectomy. Which particular issues would you consider to be most important in the pre-anaesthetic assessment of this patient? The aim of this viva is to ensure candidates: y y y Describe appropriate preoperative assessment of this patient with interpretation of relevant investigations. Discuss issues of consent in this patient. Describe safe anaesthetic management of this case with discussion of alternative techniques. Discuss with appropriate decision making, anaesthetic management of urgent surgery for airway obstruction due to bleeding 3 hours postoperatively.

10 Years Collection of ANZCA Part II Anaesthetic Viva

MK Yuen & SW Ku

10 Years viva collection 2009

291

Scenario 5
You are on call for Obstetric Anaesthesia at tertiary referral hospital. You are in the hospital when the duty Obstetrician rings you, wanting to take a patient to theatre for an emergency Caesarean Section. The Patient, a 32 year old, G3P2 at 34 weeks, has been an inpatient for 4 weeks due to intermittent small ante-partum haemorrhages from a known placenta praevia. She has now had a large bleed. What further information would you like from the Obstetrician? The aim of this viva is to ensure candidates: y y y Describe a safe management plan for anaesthesia for emergency LSCS in this situation. Identify the management options for major intraoperative haemorrhage due to placenta accreta and describe appropriate management of massive transfusion and its sequelae. Discuss appropriate handling of an adverse event - patient awareness.

Scenario 6
A 50 year old male chronic smoker underwent left pneumonectomy for carcinoma of the lung. His recovery was complicated and two weeks after his initial surgery, he is listed for operative drainage of empyema thoracis and repair of broncho-pleural fistula. A chest drain is in place, draining purulent fluid and air bubbles. How is broncho-pleural fistula diagnosed in this post-pneumonectomy patient? The aim of this viva is to ensure candidates: y y y Discuss the problems associated with anaesthesia for this procedure and formulate an appropriate anaesthesia plan. Recognise AF and manage intra-operative rapid AF Evaluate alternative pain management options.

10 Years Collection of ANZCA Part II Anaesthetic Viva

MK Yuen & SW Ku

10 Years viva collection 2009

292

Scenario 7
70 year old male with rest pain in his foot presents for femoropopliteal bypass grafting. Past History: y y y Myocardial infarction 7 years ago stable angina transient ischaemic attack 3 years ago

What is this man's risk of suffering a major cardiac event in the perioperative period? The aim of this viva is to ensure candidates: y y y Discuss the appropriate preoperative assessment of a patient with IHD including the management of unstable angina in a patient requiring semi urgent surgery. Discuss appropriate anaesthetic techniques for this surgery in this patient. Recognise and describe the appropriate management of perioperative myocardial infarction.

Scenario 8
You are asked to assess a 20 month old child with a 6 day history of a cough and worsening respiratory distress. He has: y y y y y Respiratory rate 60 per minute HR 150 per minute Temperature 37.7 C Reduced air entry on the left side and widespread bilateral wheeze Oxygen saturation = 88% on room air

Please comment on the chest x-ray taken on admission? The aim of this viva is to ensure candidates: y y y Describe initial safe airway management in the emergency department Discuss the anaesthetic management of LBO for inhaled foreign body. Describe with appropriate decision making their management of Intraoperative hypoxia due to bronchospasm or pneumothorax

10 Years Collection of ANZCA Part II Anaesthetic Viva

MK Yuen & SW Ku

10 Years viva collection 2009

293

Scenario 9
As part of an "Emergency Rescue Team" you have been called to the scene of an accident. Two hours ago, a 28 year old male was crushed by a forklift that fell from a platform. The Fire Service is almost ready to remove the forklift. What are your immediate considerations? The aim of this viva is to ensure candidates: y y y Discuss the management of crush injury and demonstrate appropriate decision making for management of cardiac arrest during reperfusion. Discuss the problem of antibodies on cross matching in a patient who needs urgent transfusion. Discuss the problems of and describe safe anaesthetic management during transportation, embolisation in angiography suite and application of external fixateurs.

10 Years Collection of ANZCA Part II Anaesthetic Viva

MK Yuen & SW Ku

10 Years viva collection 2009

294

ANZCA Anaesthetic Viva September 2007 Day 2 Dr. Tony Wei, Melbourne & Dr. Patricia Kan, PWH
Scenario 1
A twenty (20) kg two (2) year old girl presents with an acute abdomen. She has been unwell for two (2) days. The surgeon believes she has torsion of an ovarian cyst and wants to perform the surgery laparoscopically. What further information do you require? The aim of this viva is to ensure candidates: y y y Discuss anaesthesia for laparoscopy in children and the problems posed by of obesity and devise an appropriate plan for anaesthesia Describe the appropriate management of intra-operative hypercarbia. Describe the management of post-operative fitting due to hyponatremia with discussion of post-operative fluid management in children. E: What further information do you require? C: This is a paediatric patient going to have an urgent abdominal procedure, I would like to know the detailed history, do my examination and look for relevant investigation. I would talk to parents ask for general medical, surgical and anaesthetic history. I am concern of significant hypovolaemia in this child due to decreased oral intake and nausea and vomiting if it existing. This patient may need optimize before the operation. I will talk to surgeon about the procedure, can it be done by open procedure? I am not comfortable with it. E: How do you assess hypovolaemia in children? C: Decrease body weight, dry skin and membrane, lose of skin turgor, cold peripheral, lethargic, decreased urine output. E: How do you assess urine output? C: Ask the parent what the last time this child passing urine. E: The nappy is wet, what does that mean? C: That means this child has passing urine. E: What investigations do you want? C: Full blood count, look for haemoglobin. E: How about electrolyte, can it predict hypovolaemia? C: Yes, vomiting can cause hypokalaemia, and dehydration can cause hyper or

10 Years Collection of ANZCA Part II Anaesthetic Viva

MK Yuen & SW Ku

10 Years viva collection 2009

295

hyponatraemia. E: Can she have difficult airway? C: Normally not at this age group, but she is overweight, normal weight around 12kg, so airway could be difficult. E: How do you do the anaesthesia? C: I would do my routine paediatric pre-anaesthetic assessment, including fasting and premed if necessary, I would prefer to put drip in before induction, I would discuss parent present with parents. I would prepare theatre as per requirement of paediatric anaesthesia, paediatric airway trolley.. E: You can not get drip in, what to do? C: I would do gas induction. E: What monitor will you choose? C: I would choose monitor as per college guideline, SpO2, ECG, NIBP and Temperature for this patient. E: What size of ETT ? C: 4.5 and prepare half size bigger and smaller. E: How do you do the gas induction? C: I would use 100%O2 and Sevoflurane, after adequate depth of anaesthesia, I will spray 1m of 2% lignocaine to the cords and do a direct laryngoscopy. E: How do you assess adequate depth of anaesthesia? C: Eye signs and end tidal of sevoflurane, respiratory rate slow and regular and heart rate. E: How much fluid are you going to give? C: I will give maintenance fluid and replace the lost. 60ml/hr plus lost, E: Patients ETCO2 up from 40 to 50, whats the cause? C: due to the absorption of CO2, inadequate ventilation, MH if ETCO2 continue climb up with hyperventilation. E: What analgesia are you going to give? C: Paracetamol 20mg/kg intro-op and QID post-op with daily limits to 60mg/kg/day. Morphine 0.1mg/kg intro-op, ask surgeon to infiltrate wound with local anaesthesia 0.2% ropivacaine 10mls. Continuous morphine infusion post-op. E: How do you make morphine infusion? C: put 1mg/kg of Morphine in 50ml syringe and run 1ml per hour that is 20mcg/kg/hr. E: Ok, patients pain been adequately controlled. Unfortunately patient has seizure in the ward, what to do? C: Emergency situation, call for help, simultaneous resuscitate and assess the cause of seizure. A B C, left lateral position, suction airway, supply 100%O2 with face mask, check vital signs. Secure airway with ETT if can not have adequate ventilation and oxygenation. E: What are the DDx?

10 Years Collection of ANZCA Part II Anaesthetic Viva

MK Yuen & SW Ku

10 Years viva collection 2009

296

C: Pre-existing epilepsy, hyperthermia. Hypoxia, hypotension. Electrolyte, Hypoglycaemia. Drug related E: What durg? C: E: Will heart rate give you an idea? C: Bradycardia can cause by hypoxia, tachycardia can cause by hyperthermia, hypovolaemia, hypoglycaemia. E: Are you going to stop seizure? C: Yes, I will give midazolam intravenously. E: What test are you going to do? C: ABG, including electrolyte and Glucose. E: Shows ABG, (Na 120) whats the cause? C: Hyponatraemia BELL

Scenario 2
A 55 year old male with severe osteoarthritis presents for left total knee replacement. Four months ago he developed an acute coronary syndrome treated by percutaneous coronary intervention and coronary stent insertion. He is currently well with no cardiac symptoms. In view of his history of coronary stenting how would you assess this man for his surgery? The aim of this viva is to ensure candidates: y y y Discuss the preoperative assessment and management of patients with coronary stents coming for surgery. Discuss problems associated with knee replacement surgery and alternative techniques of anaesthesia for this patient having this surgery. Describe the appropriate recognition and management of complete heart block occurring intra-operatively and may include perioperative MI. E: How do you assess this patient for surgery? C: Hx, Ex, and Ix. Apart from my rontine anaesthesia assessment. My main concern is the coronary stent, needs more information about that. How many stent, what type of stent, what anticoagulation management this pt has. Details related with the stent put in, severity of acute coronary syndrome and function reserve. E: What kind coronary stent is available? C: Bare metal and drug eluting stent is the two main types cardiologists put in.

10 Years Collection of ANZCA Part II Anaesthetic Viva

MK Yuen & SW Ku

10 Years viva collection 2009

297

E: Cardiologists said it is a drug eluting stent. What is drug eluting stent? How does it work. C: bookwork E: How do you manage anticoagulation for the stent. C: Depends on different stent, I would consult cardiologist. E: How long does it needs to be anticoagulated? C: 6-12 month, but will ask the cardiologist. E: What drug they put in the DES? Aspirin? C: Can not remember the name. Aspirin is combine with the anticoagulation therapy. E: What is the risk if stop anticoagulation? How high the risk is? C: Stent thrombosis. Above 50% E: How high the mortality will be? C: Very high, patient will have MI. E: How do you assess this patients cardiac function? C: This patient can not move properly, I will arrange a thallium scan. E: How do you arrange this patient for surgery? C: Talk to Orthopaedic surgeon and Cardiologist, decide the appropriate surgery time. E: What anaesthesia you will give to this patient? C: I will give GA for this patient, in the light of the risk of bleeding with regional technique. E: You give GA to this patient, where are you going to give anaesthesia for this patient? C: I would do this case in a tertiary centre with cardiologist service and CCU available. E: What are the post op issues related with this patient. C: Cardiac monitoring, pain management E: What else, what specific for orthopaedic surgery? (examiner prepare to turn over the page) C: DVT prophylasis!!!(examiner turned over the page with a relief) E: intro-op, this patient developed hypotension, Bp 80/50, and HR 45. What are you going to do? C: Potential crisis situation, I would simultaneous management and assessment. Make sure airway patent, ventilate with 100% O2, check ECG rate and rhythm, BP, feel pulse, watch patients color, check SpO2 and ETCO2. Give fluid bolus and vasopressor, metaraminol 0.5mg bolus E: Do you think give metaraminol is appropriate in this situation. C: Oh, I forgot bradycardia, should give ephedrine 10mg, see respond, if no respond then give adrenaline 10mcg. E: Not much respond for that. As you mentioned the rhythm. (examiner shows ECG trace) (Monitor ECG trace shows 2 degree heart block with slow ventricular rate.) What are you going to do? C: This patient has cardiovascular compromise need pacing, but before pacing I would give 50mcg of adrenaline to this patient.

10 Years Collection of ANZCA Part II Anaesthetic Viva

MK Yuen & SW Ku

10 Years viva collection 2009

298

E: Ok, the Bp and HR come back, what pacing you are going to give to this patient? C: There are transcutaneous pacing and intravenous pacing. E: What pacing are you going to use in your institute? C: I am going to use intravenous pacing, because its more reliable. E: Patient develops chest pain in recovery, what are you going to do? C: Potential crisis, attend immediately, supply O2, check vital signs (BP, SpO2, ECG-5 leads), ask nurse do a 12 lead ECG, take blood for ABG and troponin. E: Shows ECG, anterior infarct. Whats the principles for management AMI? C: O2, analgesia, aspirin, ACEI, thrombolysis, cardiologist referral. BELL (other candidate comment the first ECG as complete heart block)

Scenario 3
A 35 year old man is referred for preanaesthetic assessment at your hospitals Preadmission Clinic. He is scheduled for LASER surgery for papilloma of his larynx and trachea. The clinic nurse asks you to see him first because he is very anxious and becoming agitated in the waiting room. She tells you that he has a history of amphetamine abuse. He smokes and is otherwise well. He has noticed a change in his voice and some difficulty breathing when he walks up the 3 flights of stairs to his flat. You ask to look at his medical records before you see him. What specific information would you look for before seeing this patient? The aim of this viva is to ensure candidates: y y y Describe an appropriate pre-operative assessment and plan for anaesthesia for this patient. Discuss with appropriate decision making the issues relevant to laser surgery involving the larynx and trachea Discuss appropriate management of an airway fire.

E: What specific information would you look for before seeing this patient? C: I would look for documentation of signs and symptoms of upper airway obstruction, e.g. stridor, hoarseness of voice, respiratory distress, ability to lie flat. I would look for the ENT nasodoscopy report which includes the location, extent and severity of the lesions, the adequacy of the airway and mobility of the vocal cords. Review the CT report again for the extent of the lesion and can also allow me to assess the size of the airway and estimate the

10 Years Collection of ANZCA Part II Anaesthetic Viva

MK Yuen & SW Ku

10 Years viva collection 2009

299

size of ETT needed. See if lung function test with flow volume loop has been done to assess the airway obstruction. Check for previous anaesthetic record and the airway management. I would also look for problems associated with smoking, e.g. IHD, COAD, etc. As he has history of amphetamine abuse, I need to screen for complications, e.g hypertension, arrhythmia, CHF, ICH, pulmonary hemorrhage, etc. Also, look for presence of other substance abuse. E: The patient has no signs of upper airway obstruction. But he has needle phobia. What would you tell him about your anaesthesia? C: I would explain to him about the procedure and risk of general anaesthesia. For the specific anaesthetic technique, I would need to discuss with the ENT surgeon before hand as it depends very much on the location, severity and extent of the lesion. As laser is used, the technique could be with tube or without tube. With tube, the options include laser resistant tube or ML tube protected with Al foil or apneic anaesthesia with intermittent intubation or tracheostomy. Without tube, the options include spontaneous breathing with nasopharyngeal airway or jet ventilation. I would explain to the patient that it would be safer to have an iv access before anaesthetic is given esp he has airway problem. I would prescribe EMLA to him so as to minimize the pain associated with establishment of iv access. However, I would avoid sedative premed. As the patient has history of substance abuse, he would have increased risk of awareness. But I would assure him that every effort would be made to reduce the risk including the use of BIS to monitor the depth of anaesthesia. Also, he may suffer from the side effects of amphetamine and would have increased CVS risks, e.g. MI, CHF, arrhythmia. He may also develop withdrawal symptoms, including hypertension, arrhythmia, seizure, etc. As laser is used, the risk of laser would also be explained. E: What are the risks of laser? C: The risks of laser include burn injury to both patient and staff. It can cause injury to tissue surrounding the lesion, skin and eye including retina. There would be risk of airway fire and the plume would cause irritation of airway leading to bronchospasm, pulmonary oedema and transmission of infective material. E: How do you protect your patient from injury? C: I would use moist cotton pad to cover patients eyes and the surrounding tissue. I would try to ensure patient being immobile during the surgery. Careful alignment should be made before laser is activated and there should be audible signal when laser is in use. To avoid airway fire, I would use either tubeless technique to avoid presence of combustive material or laser resistant tube with double cuff so that methylene blue can indicate cuff perforation and saline can put off fire. A bucket of water should be available during the surgery. N2O is avoided and minimal O2 should be used. E: Patient understands the risks but still refuses cannulation. The surgeon would like to have a laser resistant tube. How would you anaesthetize him?

10 Years Collection of ANZCA Part II Anaesthetic Viva

MK Yuen & SW Ku

10 Years viva collection 2009

300

C: My choice would be gas down with sevoflurane or awake fibreoptic intubation. If the patient has features of difficult airway, I would use awake fibreoptic intubation as it is safest. E: The patient has no features of difficult airway and he refuses cannulation. So your choice would be C: inhalational induction E: Tell me how you would perform inhalational induction. C: I would prepare patient and ensure that he is adequately fasted. I would then prepare theatre with equipment, drugs and skilled assistants. In particular, I would ask for difficult intubation trolley and fibreoptic bronchoscope standby and make sure ENT surgeon is present during induction and be ready to perform rigid bronchoscopy or tracheostomy. After applying monitoring according to college guideline, I would preoxygenate the patient with 100% O2 and then add sevoflurane gradually to 8% and maintain spontaneous breathing. I would try to establish iv access after patient is asleep. When the anaesthesia is deep enough, I would spray 10% xylocaine to the vocal cord and intubate. E: Before intubation, the patient develops stridor and increased BP. SpO2 is 97%. What would you do? C: I would try to maintain the airway by optimizing the head position, holding the mask with both hands and using oropharyngeal airway. I would continue 100% oxygen, deepen the anaesthesia and apply CPAP as it is likely that the patient may have laryngospasm. E: If the patient still has stridor, what would you do? C: It depends on the cause. If it is related to laryngospasm, I would use suxamethonium and intubate the patient. E: So, you use sux? C: Em.Yes, if it is larygospasm. E: After suxamethonium is given, you cannot intubate the patient. What would you do? C: It depends on the reason why I cannot intubate the patient. I would try to optimize the intubation condition by optimizing the head position, using McCoy or long blade, using a smaller size ETT, using stylet or Bougie, appling laryngeal pressure. E: You still cant intubate. C: I would declare that it is a crisis situation and call for help. E: Who will you call? C: I will call the more senior anaesthetist and skilled assistants. I will also tell the ENT surgeon who should be present in the OT to get ready to perform rigid bronchoscopy and tracheostomy as stridor could be due to upper airway obstruction by the lesion and rigid bronchoscopy is required to bypass the lesion. E:. BELL

10 Years Collection of ANZCA Part II Anaesthetic Viva

MK Yuen & SW Ku

10 Years viva collection 2009

301

Scenario 4
You are called from home to put an epidural in a 28 year old primip in established labour who is 4 cm dilated. She is in considerable distress and has thus far had no analgesia. You are 30 minutes away. What will you suggest to the midwife that can be done prior to your arrival? The aim of this viva is to ensure candidates: y Describe an appropriate approach to obtaining consent for epidural analgesia in labour; demonstrate knowledge of the difficulties of consent in this situation and of alternatives to epidural analgesia. y y Describe with appropriate decision making the safe management of high spinal block, including maternal and foetal resus. Evaluate different management strategies for post dural puncture headache and discuss appropriate handling of this adverse event. E: Whats you suggest to the midwife? C: This patient in pain, needs analgesia before my arrival, needs to prepare the epidural and also needs to monitor the wellbeing of the mother and the fetus. Analgesia: nonpharmacological environment, experienced midwife, massage, warm patch. Pharmacological entonox, ptheidine 25mg iv bolus Relevent investigations platelets and exclude other contraindication. Prepare epidural equipment, local anaesthetic, fentanyl. Resuscitation trolley outside the door. Monitor vital signs of mother and CTG of fetus. Intravenous catheter functioning(16G) E: Patient in pain and distress, and ask you to put in the bloody epidural now. What to do? C: I would talk to the partner and the parents, also ask if they have attended the antenatal class. Its a difficult situation, but I would try to make the consent brief and precisely and make sure the patient understand. E: What else you can do to make consent? C: E: How about timing with the contraction. C: Oh, yes, I would consent patient after the contraction gone. E: What do you say to the patient? C: I would say there are benefits and risks of epidural.

10 Years Collection of ANZCA Part II Anaesthetic Viva

MK Yuen & SW Ku

10 Years viva collection 2009

302

Benefits: good pain relief during contraction, patient comfortable, less mentally and physically stress. Risks: May not working or not working well, may take time to put in due to difficulty, other risks book work E: How do you perform epidural in details. C: Environment, monitor, assistant, position, landmark, aseptic technique, needle size and style, local to skin, loss of resistance, identify catheter position, depth, 1st dose (drug, volume and why) Following epidural management. E: You noticed fluid come out, what happened? C: maybe dural tape or the saline used for LOR. E: How to know it is CSF? C: Normally the CSF leak through epidural needle is quite obviously, other things include temperature, glucose and protein by stick paper. Also the amount of fluid come out compare with the saline put in as loss of resistance. E: any other way? C: E: How about use air for LOR? C: Oh, yes, if use air, any clear fluid come out would likely be CSF. E: You put epidural successfully in another level, how do you give the first dose? C: 0.2% ropivacaine with 50mcg of fentanyl E: 30 min later, patient distressed and difficult breathing, desaturation, what happened? C: likely high or total spinal, local anaesthetic toxicity, other causes, PE, AFE, Cardiogenic, CNS, anaphylaxis E: What to do? C: Crisis situation, call for help, get resuscitation trolley in, A.B.C. 100% O2, bag mask ventilation, left lateral tilt patient. Ask patient any numbness of mouth any tingling to confirm local anaesthetic toxicity. Ask patient to move toes and limbs, do dermatome test to confirm high spinal block. While resuscitation, monitoring wellbeing of the fetus. E: Will you intubate the patient if it is total spinal? C: I would because patient can not maintain ventilation, oxygenation and great risk of aspiration. E: Will you call the obstetrician? C: Yes I would and actually this patient needs to go to the theatre to have emergency LSCS, I would ask theatre to prepare for that. E: Surgery successful, patient c/o headache post-op, what to do? C: Most likely PDPH, other causes could be pre-existing headache, migraine, pre-eclampsia,

10 Years Collection of ANZCA Part II Anaesthetic Viva

MK Yuen & SW Ku

10 Years viva collection 2009

303

intracranial haemorrhage E: How do you make sure it is PDPH? C: Exclude other causes, feature of the headache, frontal or occipital, relieve with lying down, exacerbate with stand and mobile, associated symptoms like nausea and vomiting, history of dural tape E: What are the reasons for a patient developing headache after dural puncture? C: Decrease volume of CSF, sink of brain, stretching meninges. E: Thats the old theory, do you know new theories? C: Not sure. E: What is the treatment? C: Conservative, bed rest, hydration, paracetamol, caffeine. Epidural blood patch. E: How does caffeine work? C: E: vasoconstriction E: What the successful rate for blood patch? C: Above 80% E: When will you give the blood patch? C: after the block wears off and post 24 hours. E: How much blood you put in? C: around 20mls, up to 30mls. BELL

Scenario 5
You are in a surgical ward of a large public hospital reviewing the case notes of a patient for your list tomorrow. A nurse rushes out from another patients room shouting Come quickly. The patient is not breathing! You run to the room and find an obese, 75 year old man unconscious and cyanosed 24 hours after an antero-posterior resection for bowel cancer. What are your immediate actions? The aim of this viva is to ensure candidates: y y y Demonstrate knowledge of basic and advanced life support in a patient with a difficult airway. Discuss an appropriate approach to dealing with an impaired colleague in this situation. Describe a logical strategy to deal with the problem of pain management in a patient with sleep apnoea.

10 Years Collection of ANZCA Part II Anaesthetic Viva

MK Yuen & SW Ku

10 Years viva collection 2009

304

C: It is a crisis situation. I would call for help, call for crash cart and start resuscitation and assessment simultaneously. I would check BP/P, SpO2 and put on cardiac monitor to check the rhythm. I would maintain the airway patency initially with face mask +/- oropharyngeal airway. Give 100% O2 and assess breathing. I would intubate the patient if needed. Get prepared for difficult intubation which is likely in this case. I would insert LMA if needed. I would support the BP with fluid resuscitation +/- vasopressor and assess the patient for the cause of collapse. E: The patient is unconscious and this is his ECG (VF). C: The patient has VF. I would manage the patient according to ACLS guideline and immediately start CPR and defibrillate the patient as soon as possible. E: Tell me what ACLS decribes? C: To start CPR immediately with chest compression to ventilation at 30:2. Give adrenaline 1mg every 3min. Defibrillate the patient as soon as possible at 360J followed by CPR for 2 min and then reassess the patient. If still VF, I would defibrillate again at 360J. If still not responsive, give 300mg amiodarone iv push. E: What defibrillator do you use? C: Monophasic. E: After resuscitation, the patient was sent to ICU. You found that the patient was prescribed morphine 15mg sc Q3hr with no O2 supplement by a senior consultant anaesthetist post op. The patient is 100kg. C: The dose is too high even for a 100kg patient. The arrest could be secondary to opioid overdose. Of course, other causes have to be detected, e.g. hypoxia, hypercarbia, electrolyte disturbance (hyperK), hypoglycaemia, pneumothorax, MI, PE, septic or anaphylactic shock, etc. E: What would you do afterwards? C: The whole event would be clearly documented in the medical record. As it is a serious incident and I need to write the incident report and report to the director of OT for subsequent review of the case. E: What would you tell the consultant anaesthetist? C: I would be frank to him and tell him about the event possible opioid overdose leading to the collapse of the patient. I would like to understand the reason why such a large dose of opioid is prescribed. Tell him that I have reported the event to the director of OT. E: The consultant is very angry and thinks that the dose is reasonable. He complains to the medical board about your behaviour. What would you do? C: I would contact the MPS and my lawyer to prepare for the possible legal proceedings. I would get the medical record ready for review or the listening. If possible, I would seek second opinion from some other senior staff or the director of OT or COS. E: How would you manage the postop pain in this patient?

10 Years Collection of ANZCA Part II Anaesthetic Viva

MK Yuen & SW Ku

10 Years viva collection 2009

305

C: As the patient has obesity and may have OSAS, he would be very sensitive to opioid and prone to hypoxia. Multimodal analgesia would be needed. I would give simple analgesic like paracetamol, tramadol, NSAID if no contraindication. On top of that I would perform epidural analgesia if no contraindication. If epidural not possible, I would give PCA, starting with a lower dose and titrate it according to patients response. Close monitoring and O2 supplement is needed. BELL (other candidate was asked differently in the middle of examination) E: Nursing staff worry about the colleague and noticed his patients seem to have more problems then other anaesthetists, how to you approach? C: This is a difficult and sensitive situation, I am concern the wellbeing of the colleague and the safety of patients. Confidential is very important throughout the investigation. First, I will pass the concern to the head of the department and seek more evidence of impropriate behavior of the colleague, like previous M&M meeting, patient history Second, a small meeting with the director and experienced senior staff should be held to discuss this issue, also should surveillance the clinical practice and personal and social change of the colleague. Third, if there is any positive finding, a meeting involve the colleague and his supporter should be held, and discuss the positive finding, angry and deny is expected from the colleague, also the wellbeing of the colleague should be supported by his friend, family member, his GP, psychologists. E: Are there any other resources will you consider helpful? C: Yes, doctor welfare organization, college, hospital administration, medical legal officer BELL

Scenario 6
A 72 year old woman with a background history of NIDDM and 30 pack-year history of cigarette smoking presents for Carotid Endarterectomy under Local Anaesthesia. Describe your technique for regional blockade for Carotid Endarterectomy. The aim of this viva is to ensure candidates: y y y Describe the technique of regional blockade for CEA with an evaluation of the advantages and disadvantages compared to GA. Describe appropriate management of intraoperative seizure during carotid clamping. Describe management with appropriate investigation and decision making of failure to wake postoperatively (assuming patient was anaesthetised in the management of the

10 Years Collection of ANZCA Part II Anaesthetic Viva

MK Yuen & SW Ku

10 Years viva collection 2009

306

seizure) C: CEA can be performed under superficial and deep cervical plexus block. E: Tell me how you wound do it. C: Book work: prepare patient prepare theatre with equipment, drugs, skilled assistants, monitoring including A-line before the procedure and 5-lead ECG E: What are the complications associated with deep and superficial cervical plexus block? C: Book work E: What are the advantages and disadvantages of regional anaesthesia? C: The advantages include being able to monitor the neurological function clinically, decreasing the need for selective shunt, reducing the respiratory complications and avoiding the haemodynamic stress associated with intubation and emergence. The disadvantages include the fact that not all patient can tolerate the operation under RA. It is not suitable for patients who are not cooperative, with dementia, claustrophobia, cant maintain the airway patency, cant lie flat or lie still for long time. It also requires experienced surgeon and the operation cant be too long. E: What are the advantages and disadvantages of GA? C: GA offer better flexibility on duration of surgery and has less demand on the surgeons technique. It is suitable for more complicated cases and for patients who are not cooperative and cant maintain their airway or lie flat. However, it requires some neuorphysiological monitoring during the operation e.g. TCD. It may also increase the need for selective shunt and complication associated. E: Is there any evidence that which one technique is better? C: As far as I know, there is no evidence about which particular technique is superior. E: Have you heard about GALA trial? C: Yes, it is a RCT comparing GA vs RA in CEA. But Im not sure if the result is released or not. E: Yes, the result is not released yet. C: ?!?!? (I guess he may be one of the investigators ?!?!?) E: You have done the block. During the operation, the patient developed seizure, what would you do? C: I would declare crisis and call for help and immediately resuscitate the patient with ABC. I would immediately check other vitals (BP/P, SpO2, ECG) and maintain the airway patency and give 100% O2. Start fluid resuscitation +/- vasopressor if BP is low. Abort the seizure with Valium 5mg iv and then load the patient with phenyltoin. I would try to correct the underlying causes e.g. taking ABG to check the gas exchange, acid base balance, electrolyte, Hstix to exclude problems like hypoxia, hyper/hypocarbia, electrolyte disturbance and hypoglycaemia. E: Would you just hold the airway with mask? C: No. If the patients consciousness remains impaired, I would give GA and intubate the

10 Years Collection of ANZCA Part II Anaesthetic Viva

MK Yuen & SW Ku

10 Years viva collection 2009

307

patient with thiopentone/suxamethonium and maintain the anaesthesia with sevoflurane/ O2/Air. Close haemodynamic monitoring and speical neurophysiological monitoring is needed. E: The patient is 70 kg. How much thiopentone and sux would you give? C: I would give 200mg thiopentone and 100mg sux and of course I would monitor the patients haemodynamic closely. E: Would you extubate the patient after the surgery? C: Ideally, I would extubate the patient to facilitate the neurological assessment postop. However, it also depends on the situation, e.g. there is no major intraoperative event, no hypoxia, hyper/hypocarbia, hypoglycaemia, hypothermia or acid base disturbance. Before extubation, I would make sure that the patient is awake with the return of protective reflex, can obey command and have the muscle relaxant adequately reversed. E: Where would you place the patient postop? C: I would monitor the patient in HDU or ICU. E: What would you expect the patients BP post-op? C: The BP is very labile, could be high or low. E: Do you know why? C: It is because of carotid sinus dysfunction. E: What would you do if the patient fails to wake up at the end of the operation? C: I would make sure that the patient is adequately oxygenated and ventilated. Maintain BP and give fluid resuscitation and vasopressor if needed. Check ABG to exclude acid base disturbance, hypoglycaemia, electrolyte disturbance. Make sure that muscle relaxant is adequately reversed with peripheral nerve stimulator. I would also inform the surgeon as it may be related to intracranial pathology e.g. ICH, ischaemia stroke, seizure. After ensuring the patients stability, I would then proceed with Ix like CT brain +/- EEG.. E: What have you given may cause the problem? C: Em As I use sevoflurance which has rapid onset and offset, I presume that the patient would wake up quickly after the gas is adequately washed out. E: What about opioid? C: Well, as I have done RA at the beginning, usually I would not give further dose of opioid as the RA should provide adequate analgesia for the procedure. But, of course I would review the drug chart to make sure that opioid is not given accidentally and causing opioid overdose. E. Bell

Scenario 7
A 58 year old male presents to pre-admission clinic one week prior to oesophagectomy via

10 Years Collection of ANZCA Part II Anaesthetic Viva

MK Yuen & SW Ku

10 Years viva collection 2009

308

laparotomy and right thoracoscopic dissection. He has been experiencing dysphagia to solids. He has been a smoker for 40 years and admits to drinking 8 standard drinks of beer a day for a number of years. Other past history: Hypertension on irbestartan and spironolactone. On examination he is 68kg and 175cm tall. Examination of the airway reveals Mallampati one, normal thyromental distance, top denture. How may a history of heavy chronic alcohol consumption influence your anaesthetic? The aim of this viva is to ensure candidates: y y y Discuss an appropriate plan for the optimization of patient and the perioperative management of alcoholic patient. Discuss anaesthesia for thoracoscopic surgery and the management of intra-operative hypotension and problems with ventilation. Evaluate alternative post operative pain management options

C: Alcoholism can cause problems in multiple systems. It can cause cardiomyopathy, arrhythmia, CHF, etc. The patient is prone to GIB. It can also cause impaired liver function, liver cirrhosis and its associated problems, e.g. coagulopathy, thrombocytopenia, portal hypertension, ascites, hepatic encephalopathy, pleural effusion, hepatic pulmonary syndrome, hepatorenal syndrome, etc. For the CNS system, it can cause cerebellar atrophy, thiamine deficiency leading to Wernicke encephalopathy, Korsarkoff psychosis, withdrawal symptoms, delirium tremens. The patient is more likely to develop electrolyte disturbance, e.g. hypoMg, hypoNa, hypoK. They are prone to hypoglycaemia. E: What causes GIB in patients with alcoholism? C: It can be caused by peptic ulcer or oesophageal varices, E: Can you comment on the blood result? C: (impaired LFT due to alcoholism) E: What other investigation would you order? C: I would need ECG, CXR, +/- Echocardiogram, +/- lung function test depending on patients condition. I would also review the CT to assess the extent of the lesion and estimate the size of DLT needed or other methods of OLV. E: Other blood test? C: Em Clotting profile E: How do you provide analgesia for the patient? C: There are many options. Multimodal analgesia is needed. I would give paracetamol and tramadol. NSAID may not be suitable if patient has GIB or bleeding tendency. On top of that I would give epidural analgesia if there is no contraindication. If it is contraindicated, I would give iv PCA morphine. E: What are the advantages of epidural analgesia?

10 Years Collection of ANZCA Part II Anaesthetic Viva

MK Yuen & SW Ku

10 Years viva collection 2009

309

C: It can provide satisfactory intraop and postop analgesia. It can reduce respiratory complications and reduce the chronic pain associated with throacotomy. Side effects of morphine can be avoided. It can also reduce the risk of DVT and paralytic ileus. E: At what level would you perform the epidural analgesia? C: T7/8 to cover the thoracotomy wound. E: How would you anaesthetize the patient? C: I would optimize the patient and prepare him by ensuring adequate fasting, making sure T&S is done with blood readily available before OT, arranging post-op ICU care. I would prepare the theatre with equipment, drugs and skilled assistant. I would establish iv access and monitoring. In addition, I would insertion A line and CVP after the patient is asleep. I would also prepare equipment for OLV. I would put in EA before GA. I would then perform RSI after preoxygenation with propofol 2mg/kg, suxamethonium 1.5mg/kg and then insert DLT. E: How do you choose DLT? C: I would choose L sided DLT unless it is contraindicated. I would estimate the size with the use of CT. E: What do you measure in the CT film? C: I would measure the LMB width. If it is >12mm, I would choose 41 Fr; >11mm , 39 Fr; >10mm, 37 Fr; >9.5mm, 35 Fr. E: 10 mins after OLV, patients saturation drops to 88%, airway pressure rises up from 28 to 40 cmH2O, BP: 85/40, P: 90. What would you do? C: I would confirm the finding by checking patients colour, pulse, NIBP, transducer level. I would declare crisis, call for help and start resuscitation. I would give 100% O2 and hand ventilate the patient to feel for the compliance. Eliminate patient from the machine to rule out machine problem. Quickly exam the chest for wheeze or crepitation or trachea deviation. Perform suction and check the position of DLT with FOB. If the hypoxia is related to OLV, I would then give CPAP 5cm H2O to non-ventilating lung. I would inform surgeon before that. If not responsive, I would give PEEP to ventilating lung. If still not responsive, I would need to inform surgeon and start intermittent 2 lung ventilation. The patient also has second problem: the drop in BP. So I would also start fluid resuscitation with vasopressor to support BP. I would look for causes, e.g. checking the surgical field for bleeding E: What are the causes of hypotension? C: It could be due to hypovolaemia due to bleeding; cardiogenic shock due to MI, APO, arrhythmia, electrolyte disturbance; analphylaxis, septic shock; obstructive causes like PE, tension pneumothorax. E: What else? C: Em E: What have you done?

10 Years Collection of ANZCA Part II Anaesthetic Viva

MK Yuen & SW Ku

10 Years viva collection 2009

310

C: Em. E: You have put in epidural. C: Ah! Yes, high epidural block can lead to hypotension. Bell (other candidate was asked the post-op extubation criteria)

Scenario 8
A 15 year old, developmentally delayed girl has been transferred to your tertiary-referral hospital from a peripheral hospital after falling 4m from a tree onto her vertex 10 hours ago. She is distressed, indicating to her mother that she has pain in her neck. A hard collar is in place. She moves all limbs. Glasgow Coma Score has been 15 since the incident and her vital signs are all stable. Primary and secondary surveys are normal except for cervical spine X-rays suggesting an unstable fracture-dislocation of C1 and C2. The neurosurgeon requests an anaesthetic to permit CT imaging to further assess her cervical spine injury, followed by application of a halo vest under anaesthetic. How will you assess this patient preoperatively? The aim of this viva is to ensure candidates: y y y Describe a safe anaesthetic technique for CT and application of halo traction in this patient. Discuss the principles of spinal cord protection and monitoring. Discuss anaesthesia for urgent spinal cord decompression when she is found to be quadriplegic on awakening. E: How do you assess patient pre-op? C: This is a paediatric trauma patient, needs a multidisciplinary trauma team simultaneous assessment and management according to EMST guidelines. I will do primary survey, A B C D E, and secondary survey. I will obtain history from parents, previous medical, surgical and anaesthetic history. Allergy, Medication, do airway assessment, also prepare airway equipment and drugs. Patient is likely very distressed, and needs general anaesthesia for CT and Halo vest procedure. E: What are you going to say to parents? C: There is a potential high spinal cord injury, despite our best effort it is likely paraplegia will happen afterwards. E: How do you give anaesthesia in CT room?

10 Years Collection of ANZCA Part II Anaesthetic Viva

MK Yuen & SW Ku

10 Years viva collection 2009

311

C: Remote area and problems associated with it, I will prepare the CT room according to College guideline suitable to give general anaesthesia. I would ask for difficult intubation trolley in CT room, I need experienced assistant E: What anaesthesia you will give to this patient? C: I would do RSI with manual inline stabilization to protect C-spine. E: Patient has sandwich 6 hrs prior to injury, will you still do RSI? C: I would still do the RSI, because there is gastric delayed emptying and increased risk of aspiration. E: You have grade 3 view, what are you going to do? C: I would try to pass gum elastic bougie in under direct vision, if failed, I would use proseal largngeal mask airway to provide ventilation and oxygenation and proceed my second plan to intubate the patient by fibroptic bronchoscopy via PLMA. E: While you do the airway, patient agitated, what happened, what to do? C: Most likely light anaesthesia, but I would check the monitor and color of the patient make sure there is no hypoxia, I would give a 30mg bolus of propofol iv. E: What is the risk to C-spine with laryngoscope? C: C-spine is likely to have extension during laryngoscopy, increased the risk of spinal cord injury. E: Which part of the C-spine is likely to extension? C: Upper cervical area, C1-C3 which is the injury area. E: What should be careful intro-op? C: Adequate anaesthesia and muscle relaxant, make sure no spontaneous movement, no coughing and strain. Maintain MAP above 70mmHg, maintain CBF and spinal cord perfusion. E: How do you extubate? C: apart fulfill the extubation criteria, I would perform leak test to make sure no airway oedema, if in doubt, leave the tube in and send the patient to ICU. E: Patient in now in ICU, suddenly developed hypoxia, SpO2 88, what to do? C: Significant hypoxia, need simultaneous assessment and treatment. Review ABC, supply 100%O2, help breathing with face mask if inadequate ventilation, consider reestablish definitive airway E: What can be the problem? C: Most likely cause is the bleeding or swelling of # site, cause spinal cord compression, respiratory depression and hypoxia, or swelling and bleeding compress the airway. Other likely causes, excessive sedation, residual muscle relaxant, airway obstruction E: How do you manage the airway? C: I would use PLMA and rail load ETT through FOB. E: What other airway can patient have in ICU? C: Patient can have a tracheostomy, because with halo, its very difficult to access the airway.

10 Years Collection of ANZCA Part II Anaesthetic Viva

MK Yuen & SW Ku

10 Years viva collection 2009

312

ENT surgeon or ICU colleague can do this. Forget the rest of viva BELL

Scenario 9
A 56 year old male pedestrian arrives in the Emergency Department shortly after being hit by a car. You are called to Emergency as a part of the trauma team to manage this patient. He is hypotensive (80/60 mm/Hg), tachycardic (120/min) and tachypnoeic (25/min). His GCS is 15 and he is answering the questions in short sentences. How will you assess and resuscitate this patient in the Emergency Department? The aim of this viva is to ensure candidates: y y y Discuss the Early (ED) management of exanguinating blunt abdo trauma. Discuss with appropriate decision making massive transfusion management and anaesthesia for damage control surgery Describe appropriate anaesthetic management of patient with multiple organ failure (ARDS/ARF/inotrope dependent etc) returning to theatre for change of abdominal packs who develops intraoperative hypoxia. E: How do you assess the patient? C: This is a trauma patient, I will assess the patient according to EMST guideline, do primary survey, ABCDE. This patient is hypotensive and tachycardia, likely to have grade III shock, even the GCS is 15, but it can drop with the deteriarate of the condition. I will form a trauma team simultaneous resuscitate and assessment the patient. E: He has 8-11 right side rib #, what the potential injury? C: Likely to have liver injury can cause massive introabdominal haemorrhage. Respiratory pulmonary contusion, pneumothorax, haemothorax. Exclude other injury, second survey if patient relative stable. E: How do you do the DDx? What investigation? C: I would look for distended abdominal, decreased bowel sound, shifting dullness. Listen to chest, look for decreased air entry, hyperresonance, trachea shift. I will do CXR and C-spine X-ray, pelvic X-ray, FAST scan to detect introabdominal bleeding. +/CT scan. E: Apart from resuscitation, patient still hypotensive. Surgical registrar wants to do a CT, will you agree?

10 Years Collection of ANZCA Part II Anaesthetic Viva

MK Yuen & SW Ku

10 Years viva collection 2009

313

C: This is an unstable patient, if the CT is just next door to ED, and can do it very quickly I will agree and stay with the patient, otherwise not, this patient will need to go to theatre immediately, I will call theatre to prepare and arrange transfer the patient to theatre. E: How do you manage the patient now? C: A B C, supply 100% O2, send blood for test and cross match, fluid resuscitation with 2L of hartmanns solution, setup monitor. Call theatre to prepare emergency laparotomy, cellsaver, rapid fluid infusion device, send Packed Cell to theatre. Ask for 2nd pair of hand if available. E: Patient in theatre, still hypotensive, whats you hemodynamic goal for this patient and why? C: I will continue fluid resuscitation, and maintain MAP above 65mmHg to maintain CPP, Systolic BP <100mmHg, avoid increase bleeding. E: The cross match will take some time, what will you give to this patient. C: I will give O negative blood before the cross matched blood available. E: What other fluid you can give. C: I can give colloid, Gelofusion or Album. E: What monitors are you going to use? C: Apart from standard monitor, I will use intro arterial line, BIS monitor, urinary catheter, temperature and central line if have chance and time. E: How do you induce the patient? C: I will give ketamine 1.5mg/kg, suxamethonium 2mg/kg, and do RSI with cricoid pressure once patient prep and dripped and surgeon is ready to operate. E: Intro-op, massive bleeding, youve given FFP, Platelets, Cryoprecipitate, patient still bleeding, what can you do? C: I will maintain fluid resuscitation and vasopressor to ensure adequate persufion pressure, I will check clotting and give clotting factor if need, I will prevent hypothermia, give fluid and blood through warming device and apply active warming device to patient. Surgeon can pack the wound with gauze. E: Can you use Nove7, Recombinant factor VIIa (rFVIIa) C: Yes, I can. There are good case reports to support its use. E: How does it work? C: Form tissue factor and rFVIIa combination and activates the coagulation cascade on the site of injury and minimum systemic effects. E: What dose are you going to give? C: 60 to 120 units/kg, every 4 hours. E: Whats the problem with Factor VII? C: may not available, very expensive, anaphylaxis. E: Patients abdomen been packed with gauze and sent to ICU, developed ARDS and on

10 Years Collection of ANZCA Part II Anaesthetic Viva

MK Yuen & SW Ku

10 Years viva collection 2009

314

ventilator with inspiratory pressure 46cm H2O, and PEEP 10 mmHg. How do you manage the respiratory intro-op? C: Aim is to maintain oxygenation, prevent hypoxia and barotrauma, allow hypercapnia. I would start tidal volume from 6ml/kg, maintain peep to the level of ICU setting E: What can happen when surgeon remove the pack? C: Blood pressure can drop, may bleed again, may improve ventilation. BELL (other candidate was asked about transfusion reaction)

10 Years Collection of ANZCA Part II Anaesthetic Viva

MK Yuen & SW Ku

10 Years viva collection 2009

315

ANZCA Anaesthetic Viva May 2008 Day 1 Scenario only


Scenario 1
A 25year old primigravida patient presents to delivery suite at 38 weeks gestation complaining of a headache and difficulty with her vision. Her BP is 170/110 and she has ++++protein in her urine. Soon after arrival she has a seizure. What are your priorities in assessment and resuscitation of this patient? AIMS OF THE VIVA y y y y y Candidate can describe appropriate assessment and resuscitation of a woman presenting with eclampsia. Discuss options for anaesthesia for Emergency Caesarean section Diagnose and appropriately treat pulmonary oedema and coagulopathy Interpret CXR showing pulmonary oedema. Interpret Coagulation studies showing coagulopathy 2ndy to hypofibrinogenemia

Scenario 2
A 2 year old boy was a passenger in a car when he was struck in the head by a flying brick lost from a passing truck. On arrival at your hospital the paramedics report he responds to pain and they have not been able to establish intravenous access. His initial observations are: Heart rate: 120 Blood Pressure: 75 systolic Respiratory Rate: 25 SpO2: 99% He is receiving oxygen by mask via a self inflating bag and has a bandage wrapped around his head. He has a hard cervical collar in place. Outline the initial management of this patient. AIMS OF THE VIVA y Describe appropriate initial assessment and resuscitation

10 Years Collection of ANZCA Part II Anaesthetic Viva

MK Yuen & SW Ku

10 Years viva collection 2009

316

y y y y

Discuss options for IV access when this is difficult Discuss strategies to control raised intracranial pressure Describe a safe technique of anaesthesia for decompressive craniotomy Discuss the DD and management of intraoperative hypoxia

Scenario 3
You are called by your dentist colleague to the emergency department. He has asked you to assess a 17 year old autistic boy who has a suspected tooth abscess that requires drainage. The boy has a large left submandicular swelling in the left submandibular region and has complained to his parents of localized pain. On arrival to the EC you observe a fearful young man who is curled up in the corner begin comforted by his parents. His vital signs are as follows: HR 120, BP 110/70, T 38.5 What are the important aspects of your pre-anaesthetic assessment of this boy? AIMS OF THE VIVA y y y Describe an appropriate assessment and plan for anaesthesia in this patient. Discuss the management of gaseous induction/difficult intubation in this patient. Describe appropriate diagnosis and management of aspiration.

Scenario 4
A 50 Year old Turkish man with Motor Neurone Disease on nocturnal BIPAP is scheduled to have a PEG (Percutaneous Endoscopic Gastrotomy) tube inserted for nutrition. The nursing staff tells you that the patient does not want ot be resuscitated in the event of life-threatening complications from anaesthesia or surgery. What are the important issues to consider regarding this patients request not to be resuscitated? AIMS OF THE VIVA y Describe an appropriate response and understanding of the issues involved in patients initial request.

10 Years Collection of ANZCA Part II Anaesthetic Viva

MK Yuen & SW Ku

10 Years viva collection 2009

317

y y y

Discuss options for anaesthesia/sedation for patent for PEG insertion Interpret ABGs showing mixed respiratory and metabolic acidosis when patient develops peritonitis. Candidates can discuss issues surrounding further surgery or palliative care for this patient.

Scenario 5
A 70 year old male requires a left lower lobectomy for squamous cell lung cancer. He has ankylosing spondylitis, ceased smoking 25 years ago and is otherwise healthy. One week ago under general anaesthesia for bronchoscopy and mediastinoscopy he was noted to be a Grade 4 laryngoscopy. How would you manage this mans airway for this surgical procedure? AIMS OF THE VIVA y y y Discuss strategies for lung separation in a patient with difficult airway. Trouble shoot problems with ventilation during one-lung ventilation. Describe pain management options in thoracotomy including use of paravertebral catheters.

Scenario 6
A 65 year old male is booked for an elective endoluminal repair of his abdominal aortic aneurysm in the angiography suite. He has had a history of ischaemic heart disease with an episode of cardiac failure 3 years ago; other medical history includes controlled hypertension. His previous GA was 5 years ago for knee arthroscopy. Medications: trandolapril, carvedilol, simvastatin. Nil known allergies Outline your approach to assessing his cardiac risk for this procedure. AIMS OF THE VIVA y y y Describe an appropriate anaesthesia technique for this procedure. Describe and justify their anaesthesia machine check. Describe appropriate dd and management fo severe hypotension due to reaction to contrast.

10 Years Collection of ANZCA Part II Anaesthetic Viva

MK Yuen & SW Ku

10 Years viva collection 2009

318

Scenario 7
A 75 year old man presents to your pre-operative assessment clinic prior to an inguinal hernia repair. He has a smoked a packet of cigarettes a day for the past 50 years. To assess his respiratory system what do you want to know from his history and physical examination? AIMS OF THE VIVA y y y Describe appropriate Preoperative respiratory assessment and interpret respiratory function tests. Discuss options for regional anaesthesia for this patient and demonstrate knowledge of the innervation of the inguinal region. Describe the DD and management of postoperative delirium (due to alcohol withdrawal).

Scenario 8
A 55 year old male scheduled for open radical prostatectomy is seen in the pre-operative clinic one week prior to operation. He had a laminectomy for lumbar radiculopathy three years prior and is on long term oxycodone for chronic back pain. He is otherwise well. Discuss your discussion of the procedure he asks you if hes likely to need a blood transfusion. How will you respond to his enquiry? AIMS OF THE VIVA y y y Discuss hazards of blood transfusion and Blood conservation strategies to reduce allogenic blood transfusion in this patient. Describe dd and management of intra-operative hypotension (due to air embolism). Discuss appropriate pain management options for this patient.

10 Years Collection of ANZCA Part II Anaesthetic Viva

MK Yuen & SW Ku

10 Years viva collection 2009

319

ANZCA Anaesthetic Viva May 2008 Day 2 Dr. Yvonne Li, PWH
Scenario 1
The obstetricians call you from their antenatal clinic. They would like you to review a woman aged 33 years with a BMI of 38 who is 36 weeks gestation as they are concerned about her. She says she has become very short of breath recently and she has marked peripheral oedema. How will you evaluated her dyspnoea and set about making a diagnosis? AIMS OF THE VIVA y y y y Discuss differential diagnosis of initial presentation and evaluation of dyspnoea in pregnancy. Describe principles of treatment of heart failure Describe safe plan for anaesthesia including regional vs general anaesthesia for LSCS, monitoring and postpartum care. Interpret basic echo data

E: How would you assess her. E: Hx- medical, antenatal, drug, allergy. Exam BP, CVS, resp exam. E: Describe to me how you would do a CVS exam. E: Her BP 100/70, RR 22, cold periphery, raised JVP, Left displaced heart apex, soft diastolic murmur, bilateral basal crep, edema up to mid calf. Plt count 105. Afebrile. E: Given ECG, CXR, Echo photo. E: Next day, she presents for Em C/S. What is your plan of management? C: Reassess, consent, iv, premed. Prepare OT, difficult intubation. Plan for GA, with a-line,CVP. Dx of peripartum cardiomyopathy. E: why GA?

10 Years Collection of ANZCA Part II Anaesthetic Viva

MK Yuen & SW Ku

10 Years viva collection 2009

320

Scenario 2
The hospital Emergency Department requests your assistance with an 18 month boy who is en route to hospital having been run over by a car in the driveway. Ambulance staff comminciate that the patient is conscious, shocked with a bruised and distended abdomen. You are the lead are giver. Give an overview of your preparation for the childs arrival in the Emergency Department. AIMS OF THE VIVA y y y y Describe appropriate initial assessment and resuscitation. Discuss transfusion strategies for massive blood loss in this children. Discuss the principles of conservative vs operative management of blunt trauma in children Discuss DD and management fo hypoxia due to pulmonary contusion during laparotomy.

E: Preparation for AED? C: activate trauma call, alert OT, inform Paedi ICU. Staff with universal precaution. Warm AED, set IV fluid. Use Browslow chart. Mx according ATLS and PALS guideline. E: BB can cry, BP lowish. Disteneded abd. E: stabilized, no USG a/v. Surgeon wants CT, allow? E: OK, now in OT for laparotomy. What is your ventilation strategies, for suspected blunt lung trauma?

Scenario 3
A 35 year old obese male (BMI of 35) presents to the Emergency Department with severe stridor. He has had a sore throat with dyspagia for 2 days. You are called down urgently to review him. What possible diagnoses will you consider as you approach the Emergency Department? AIMS OF THE VIVA y y y Discuss the differential diagnosis of stridor in this patient. Discuss options to secure airway. Recognise and manage hypoxia and difficulty with ventilation from misplaced tracheostomy tube.

10 Years Collection of ANZCA Part II Anaesthetic Viva

MK Yuen & SW Ku

10 Years viva collection 2009

321

E: What are you differential diagnosis: C: For the presenting symptoms of stridor, ie upper airway obstruction, my ddx will include obstruction from inside airway (FB), of the airway (acute epiglottis, tracheal stenosis), outside airway ( thyroid, thymus, LN, Ca lung with SVCO, Ca larynx); also can be central cause, eg drug, with decrease in consciousness. E: Further information of patient? C: History time course, associated sym. Other medical problem. Vital signs. Assess for difficulty in intubation. E: fever, vital stable Say you make the ddx of acute epiglottis. How would you management. C: Set monitor. Set iv. 100% oxygen. At patients best comfortable position. Inform ENT surgeon. Inform OT. Inform ICU. Prepare OT for difficult intubation and emergency tracheostomy. Escort patient to OT. Consent for airway management. E: How would you intubate at OT C: Monitoring, secure iv. Skilled assistance. Difficult intubation trolley. ENT surgeon scrubed for tracheostomy. Gas down with 100% oxygen and sevoflurane. DL, lignocaine spray, intubate with #7 ETT, confirm ETT position and secure the ETT. Sedated the patient. E: How long do you think would need to gas down the patient? E: would you use Nitrous oxide? E: anything to do? C: iv antibiotic, eg cefuroxime 1.5g

Scenario 4
A male anaesthetic trainee in his 3rd year of training (ATY1) is rostered to be with you on your routine gynaecology list. You meet the trainee the night before the list and discuss the management of the booked cases. The first case is a previously well 66-year old woman undergoing laparoscopy for pelvic pain. The other two cases have also been seen in clinic and are previously well patients undergoing diagnostic laparoscopies for infertility. What are you expectations of a trainee at this level of training during this operative list? AIMS OF THE VIVA y y y Describe the objectives of in training assessment. Describe an appropriate plan for assessment of trainees at this level. Recognise the signs of possible drug abuse in a colleague and describe appropriate short-term strategy for dealing with the trainee.

10 Years Collection of ANZCA Part II Anaesthetic Viva

MK Yuen & SW Ku

10 Years viva collection 2009

322

E: what is you expectation from the trainee? C: He has completed Part I exam, should be competent with anaes related pharmacology and physiology. He has been working for >2 yrs, should have reasonable medical knowledge, would be able to handle simple GA Gynae list, would have basic airway management skill E: What level of supervision you think he needs? What are the level of supervisions? E: ok..you start the 1st case together. After iv induction, he cannot intubate with DL. What would you do? C: I would make sure he can ventilate, by putting in oral airway, better position to open up airway, keep 100% oxygen. E: OK , he can ventilate well. C: I would help him better position patient in stiffing position, change laryngoscopy McCoy blade, prepare for bougie and suction ready, let him try second time. E: After OT, patient in RR, complains pain. E: Found out your trainee not given opioid in OT that checked. Suspected trainee drug abuse. What is your plan of mx. C: Search for more evidence. Talk to his friends. Notify department head. call for meeting. E: Who to involve in meeting? E: In meeting, trainee runs out of room, what to do? C: try to find him back. E: How? C: call him by phone, involve hospital security. Ask his friend (accompany person) to find him. E: He is out of hospital, cant be reached by phone, not at home. Will you call the police?

Scenario 5
You are asked to review a 31 year old woman in the Pre-admission clinic. She has myasthenia gravis and is booked to have a thymectomy via a median sternotomy. What are your major concerns regarding this patient? AIMS OF THE VIVA y y y Describe appropriate peri-operative management of myasthenic patient Interpret and trouble shoot changes in CO2 traces Discuss pain management options following sternotomy.

E: Assessment?

10 Years Collection of ANZCA Part II Anaesthetic Viva

MK Yuen & SW Ku

10 Years viva collection 2009

323

E: Rx: pyridostigmine 90mg q6h, prednisolone 5mg daily. E: Criteria for postop ventilation

Scenario 6
You are contacted by the nurse-in-charge of the Cardiac Investigation unit, the morning of your elective list in the catheterization laboratory. Due to the breakdown of the X-ray equipment in the regular cath lab, your list has been transferred to another fluoroscopy suite which has satisfactory cardiac investigation equipment. You have never been to this location. Your first case is: Mr P.A. 35M ELECTROPHYSIOLOGY STUDY +/RADIOFREQUENCY ABLATION\ How would you normally approach anaesthestising in an unfamiliar location? AIMS OF THE VIVA y y y y Describe the minimum standards required for providing anaesthsia services to the EPS lab Interpret ECG showing WPW Discuss options for anaesthesia/sedation for EPS studies Discuss DD and management of profound hypotension due to tamponade.

( shown photo of cardiac cath unit ) E: prepare the place E: Why need an anaes machine? Nurse said we have infusion pumps. E: shown ECG C: WPW E: features? E: Anaes plan? 2hrs procedure. E: While inducing VT, BP 90/50, what to do? E Now BP 60/40.

10 Years Collection of ANZCA Part II Anaesthetic Viva

MK Yuen & SW Ku

10 Years viva collection 2009

324

Scenario 7
You are doing a locum for a private group. There is an addition to tomorrows list that has been emailed to you at 16.00. A 60 year old woman presents for an elective left ankle arthrodesis. She has had rheumatoid arthritis for 20 years. Her persent medications include prednisone 20mg/day, long acting morphine 60mg/day, methotrexate and the COX-2 inhibitor, etorcoxid. The preoperative questionnaire indicates she has some neck pain and back pain, with non-specific intermittent pain down one leg. She has indicated she wants the operation performed under some sort of local block. What further information do you need from your preoperative phone call the night before? AIMS OF THE VIVA y y y Discuss the options for regional anaesthesia in this patient and describe the innervation of the ankle. Discuss the implications of Rheumatoid arthritis and long-term steroid therapy for anaesthesia/surgery Discuss appropriate perioperative pain management in patient on long term opioids.

E: Assessment. C: Assess for RA, arthritic and non-arthritic involvement. Other medical problem, prev anaes Hx. Pain control/Hx. E: She wants RA, choices? E: Describe popliteal nerve block. Shown USG picture of popliteal fossa, name? E: the night after OT, Nurse in ward call you patient now in tachycardia, raised BP, sweating. What to do? C: ensure vitals stable. Go back to hospital to see patient. Check for surgical complications. Ensure resume usual morphine. E: Patient actually not taking usually morphine as she is vomiting after OT. C: convert usually oral morphine to morphine infusion (calculate dose, ie ~1mg/hr), add multimodial analgesia (eg panadol, continue COX2)

10 Years Collection of ANZCA Part II Anaesthetic Viva

MK Yuen & SW Ku

10 Years viva collection 2009

325

Scenario 8
A 60 year old man with long standing diabetes managed on insulin presents with acute right iliac fossa pain. He is distressed, febrile, sweating and has signs of peritonism. You are asked to anaesthetise him for a laparotomy. What are the implications of diabetes for this patient? AIMS OF THE VIVA y y y Discuss the preoperative assessment of a diabetic patient Correctly interpret ABGs showing ketoacidosis and describe appropriate perioperative management of fluids and electrolytes in this patient. Describe the DD and Ix and management of post-operative cognitive dysfunction (due to stroke)

10 Years Collection of ANZCA Part II Anaesthetic Viva

MK Yuen & SW Ku

10 Years viva collection 2009

326

ANZCA Anaesthetic Viva October 2008 Day 1 Scenario only


Scenario 1
A 60 year old male is booked for an elective multi-level L2-S1 posterior fusion and instrumentation. This is the only case on an all-day list and the booking form has requested specific metalwork (pedicle screws and rods), image intensifier, cell-saver and post-operative high dependency bed. Past medical history includes hypertension and chronic back pain, secondary to degenerative spinal stenosis. He has significant radicular pain down one leg, which restricts his walking to 15 metres. He has a Body Mass Index of 25 and recent echocardiography was normal. Current medications include: Slow-release morphine 100 mg BD Gabapentin Oxycodone Atenolol 900 mg TDS 10-20 mg Q4h PRN 100 mg mane

What specific principles would you use when managing his pain perioperatively? AIMS OF THE VIVA y y y Discuss an appropriate plan for pain management in the opioid tolerant patient having major surgery Diagnose and manage hypotension secondary to a large concealed haemorrhage from unrecognised aortic laceration occurring during surgery. Interpret coagulation tests and manage coagulopathy.

Scenario 2
34 year old male is brought in from an industrial explosion to the Emergency Department. He has flash burns to his face, torso and arms. He is short of breath and hypoxic on admission. What are your priorities in the initial resuscitation?

10 Years Collection of ANZCA Part II Anaesthetic Viva

MK Yuen & SW Ku

10 Years viva collection 2009

327

AIMS OF THE VIVA y y y Prioritise initial management of burns/trauma including airway management Describe management of ventilation with increasing hypoxia. Discuss the indications and complications of escharotomy, and the management of severe hyperkalemia including ECG interpretation and cardiac arrest management

Scenario 3
Six (6) hours after an uneventful tonsillectomy in an obese eight (8) year old boy, you are called by the treating surgeon to take the child back to theatre for control of bleeding. The childs weight is fifty (50) kilograms. Describe the initial management of this child AIMS OF THE VIVA y y y Discuss options for IV access and fluid resuscitation of this patient Discuss a safe plan for induction of Anaesthesia. Manage difficulties in patient ventilation during procedure due to endotracheal tube blocked with blood, ? Boyle Davis gag.

Scenario 4
A 21 year old year old primigravida at 32 week gestation presents to the delivery suite with BP 150/95, lower abdominal pain and vaginal blood loss of 500ml. Outline your initial assessment AIMS OF THE VIVA y y y Manage acutely cocaine toxic patient for Caesarean section for placental abruption Diagnose and manage intraoperative ST segment changes and VT Describe neonatal resuscitation

10 Years Collection of ANZCA Part II Anaesthetic Viva

MK Yuen & SW Ku

10 Years viva collection 2009

328

Scenario 5
You are the on call anaesthetist at a large rural regional hospital also covering ICU for the weekend. You are called in to help manage a 48 year old woman who has collapsed at home after complaining of a severe headache. She has no relevant past history. On admission to hospital her BP is 180/100 and she has a right hemi paresis, opens her eyes to painful stimuli, makes incomprehensible sounds, and flexes abnormally to painful stimuli. What is her Glasgow Coma Scale (GCS) score, and what is its significance? AIMS OF THE VIVA y y y Describe initial resuscitation and management of patient with raised ICP. Discuss the diagnosis of brain death Discuss the anaesthetic management of patient for organ donation

Scenario 6
A 65 year old woman with symptoms of heart failure presents for left mastectomy for proven carcinoma of the breast. The surgeon is concerned about the severity of the cardiac failure. From the history, how will you assess the severity of her cardiac disease prior to anaesthesia? AIMS OF THE VIVA y y y Understand the pharmacological and mechanical (biventricular pacing) management of severe heart failure and ensure optimisation preoperatively Interpret relevant ECG abnormalities Discuss Analgesia for mastectomy and management of chronic postsurgical pain.

Scenario 7
You are a specialist at a tertiary hospital and are allocated to provide sedation during a CT guided biopsy of a mediastinal mass in a 50 year old woman who has been investigated for new onset cough and suspicious CXR. She was previously well, apart from chronic anxiety disorder.

10 Years Collection of ANZCA Part II Anaesthetic Viva

MK Yuen & SW Ku

10 Years viva collection 2009

329

Weight 60 kg. Height 1.65m Medications: Diazepam 5mg daily and as required. No known allergies Uneventful GA for hysterectomy in past Explain the most important features of your preoperative assessment. AIMS OF THE VIVA y y Conduct appropriate preoperative assessment of a patient with a mediastinal mass, including interpretation of relevant investigations. Manage life threatening complications due to haemorrhage and airway compromise in radiology suite and transfer to OT

Scenario 8
A 60 year old woman with hepatic cirrhosis from chronic hepatitis B presents for repair of fractured neck of femur. She has Grade 2 encephalopathy with lethargy, some confusion and a flapping tremor (asterixis). Mild ascites is present but she does not appear jaundiced. What in the history and examination would you focus on in planning her anaesthetic? AIMS OF THE VIVA y y y Assess hepatic disease and optimise management preoperatively Discuss risks and benefits of regional anaesthesia for surgery for fractured neck of femur Manage the patient who fails to wake after general anaesthesia

10 Years Collection of ANZCA Part II Anaesthetic Viva

MK Yuen & SW Ku

10 Years viva collection 2009

330

ANZCA Anaesthetic Viva October 2008 Day 2 Scenario only


Scenario 1
A 70-year-old man with carcinoma of the prostate and lumbar spine metastases has suffered with back pain for one month. This pain has been managed with oral oxycodone 15 mg 4-hourly. He presents with a six hour history of bilateral leg weakness. You are requested to provide anaesthesia for urgent surgery to relieve compression of his spinal cord. Outline the features of your anaesthesia technique that will ensure optimal spinal cord perfusion in this man. AIMS OF THE VIVA y y y Discuss principles of maintaining spinal cord perfusion in this patient Diagnose and manage anaphylaxis while in the prone position. Including appropriate investigation. Discuss management of postoperative neuropathic pain.

Scenario 2
A 20 year old known Haemophiliac presents to the emergency department following a hotel brawl, during which he was kicked whilst on the ground. He is known to be Hepatitis B and C positive and has inhibitors to factor VIII. He has a painful left chest and a tender abdomen. He has a GCS of 15, a BP of 80/40 mmHg and a pulse rate of 120/min following 2 litres of Hartmanns solution. His SaO2 is 92% on 15 litres/min O2 via mask. You have been called to the Emergency Department to review the patient. Please comment on the chest x-ray. What other investigations would you like to see? AIMS OF THE VIVA y Describe initial assessment and resuscitation of this patient including appropriate

10 Years Collection of ANZCA Part II Anaesthetic Viva

MK Yuen & SW Ku

10 Years viva collection 2009

331

management of his coagulopathy and interpretation of CXR y y Manage pulmonary haemorrhage with bleeding into the airway. Discuss management of a needle stick injury occurring to trainee during resuscitation

Scenario 3
An eight week old baby boy who weighs 4kg presents with a history of vomiting. The baby has an obstructed inguinal hernia that is not reducible. The baby has been scheduled for emergency surgery to repair the obstructed hernia. How would you assess the hydration in this baby? AIMS OF THE VIVA y y y Assess and resuscitate an infant with dehydration with an appropriate fluid regimen including interpretation and correction of abnormal electrolytes. Induce anaesthesia for infant for emergency surgery. Manage post extubation laryngospasm

Scenario 4
You are seeing patients at the high risk obstetric anaesthetic clinic. A 29 year old morbidly obese patient (BMI 42) presents with a strong family history of malignant hyperthermia. She is now at 34 weeks. She is very anxious about pain relief during labour Explain how you would assess her MH risk? AIMS OF THE VIVA y y y Advise a patient at risk of MH about analgesic options for labour and delivery. Give phone advice to trainee about safe plan for GA in this patient. Manage failed intubation in this patient.

10 Years Collection of ANZCA Part II Anaesthetic Viva

MK Yuen & SW Ku

10 Years viva collection 2009

332

Scenario 5
You are asked to assess a patient who has presented to the emergency department with acute difficulty swallowing. He is a 70 year old man who has been found to have a supraglottic lesion in the larynx. He has been scheduled by the head and neck surgeon to have a panendoscopy and biopsy on your list tomorrow, with a view to laryngectomy in a few days. He has a history of ischaemic heart disease and has had two coronary artery stents inserted 12 months ago. He has recently stopped smoking after many years of heavy use. He has controlled hypertension, type 2 diabetes mellitus which is diet controlled and he is treated for high cholesterol. Medications: Amlodipine Felodipine Atorvastatin Clopidogrel Aspirin Salbutamol inhaler prn What are the specific issues associated with coronary artery stenting in patients undergoing non-cardiac surgery? AIMS OF THE VIVA y y y Identify and discuss the relevant issues in this patient presenting for microlaryngoscopy and describe an appropriate anaesthetic technique. Management of Airway problems in recovery. Management of perioperative myocardial ischemia in this patient.

Scenario 6
You are the anaesthetist rostered on for the emergency theatre in your hospital. A 35 year old man is booked for emergency laparoscopy/ laparotomy for appendicitis. During your pre-anaesthesia consultation, he tells you that two of his siblings have suffered sudden cardiac death in their thirties. He has never had chest pain or breathlessness and has never smoked. On questioning he describes two episodes of syncope during exercise, the last of which occurred six months previously. He weighs 70kg, is in severe pain, is febrile at 38.5C, tachycardic at 120bpm, is sweaty and

10 Years Collection of ANZCA Part II Anaesthetic Viva

MK Yuen & SW Ku

10 Years viva collection 2009

333

looks unwell. He has been vomiting and is clinically dehydrated. An ECG taken several hours ago is shown below. What is your assessment of this patients family history and episodes of collapse? Can you explain briefly what causes Long Q-T Syndrome? AIMS OF THE VIVA y y y Understand the relevant pharmacology and clinical significance of long QT syndrome for anaesthesia including interpretation of relevant ECG abnormalities. Plan safe general anaesthesia in this man. Discuss the prevention and management of dental damage.

Scenario 7
You have been asked to anaesthetise a 60 year old man with bronchial carcinoma. The surgeon plans to use a rigid bronchoscope to laser parts of the tumour which has extended into the trachea causing a degree of tracheal obstruction. He has requested not to be resuscitated in the event of cardiac arrest. Discuss your preoperative assessment of this patient? AIMS OF THE VIVA y y y Discuss the implications of NFR orders for anaesthesia and surgery. Describe a safe approach to anaesthesia for laser bronchoscopy. Diagnose and treat tension pneumothorax in recovery

Scenario 8
A 76 year old woman with severe rheumatoid arthritis presents with neck pain and occasional abnormal sensation in her left hand. Describe the X-ray and CT findings? AIMS OF THE VIVA y Identify the key issues in patients with severe RA for sedation/anaesthesia

10 Years Collection of ANZCA Part II Anaesthetic Viva

MK Yuen & SW Ku

10 Years viva collection 2009

334

y y

Manage resuscitation in the MRI unit Diagnose and manage (including postoperative follow up) iatrogenic drug swap.

10 Years Collection of ANZCA Part II Anaesthetic Viva

MK Yuen & SW Ku

10 Years viva collection 2009

335

ANZCA Anaesthetic Viva May 2009 Day 1 Dr. Jonty Golshevsky, Royal Children Hospital, Melbourne
Scenario 1
You are asked to review a pregnant woman at 30 weeks gestation. She was a passenger in a motor vehicle accident and has been admitted to the emergency department of the general hospital in which you work. She is complaining of severe abdominal pain. You are aware that the hospital does not have an obstetric service. Describe your initial assessment of this patient? AIMS OF THE VIVA y y y y y Apply EMST principles to the pregnant patient and assessment abdominal pain in the pregnancy Manage abdominal haemorrhage at Caesarean Section Manage obstetric (and neonatal) services with limited resources How do you assess volume in pregnant patient? Differences? Issues? How to resolve?

no idea about any of this. Fetal heart y y y y y y y y y y Why? What looking for? What is bradycardia and what is tachycardia? Shown results DIC. How would you manage? How to induce for laparotomy? Drug doses How to manage massive haemorrhage Asked how ABG would help me after I mentioned it Role of cell saver in pregnancy? Issues? Role of factor 7

I said embolism, they wanted more I think BELL

10 Years Collection of ANZCA Part II Anaesthetic Viva

MK Yuen & SW Ku

10 Years viva collection 2009

336

Scenario 2
A 40 year old woman with a history of ovarian cancer treated with oophorectomy and chemotherapy presents to the emergency department of your hospital with increasing shortness of breath. A Chest X-ray was obtained soon after admission. Please comment on the Chest X-ray. AIMS OF THE VIVA y y y Understand the general implications of cancer and its treatment for anaesthesia Interpret relevant ECGs Diagnose and manage acute postoperative pericardial tamponade

CXR classic APO with massive heart. y y ddx Given ECG

It had widespread ST elevation? pericarditis or pericardial something? Added pericardial effusion to ddx y y y y y y y y y y y y y Side effect of chemo relevant to anaesthetizing her? What other Investigations? Signs of tamponade on echo? How to manage? Who? What monitoring? How many ECG leads? CVC? All goes well called to ICU hypotensive, hypoxaemic. Ddx? Given ECG SVT or rapid AF how to differentiate? Asked to time it with a bit of paper What Ix you want? Echo, electrolytes Return to OR how would you induce? Drug doses.. She had paclitaxel and cisplatin

I said drain under ultrasound I said someone else, not me.

I said yes, should have said no.

irregular AF

BELL (other candidates mentioned there was blocked pericardial drain in ICU causing the hypotension) BELL

10 Years Collection of ANZCA Part II Anaesthetic Viva

MK Yuen & SW Ku

10 Years viva collection 2009

337

Scenario 3
You are called to the Emergency Department (ED) to see a 5 year old boy who fell off his trampoline one hour after dinner. He sustained a fracture of his right tibia and fibula with marked deformity. The orthopaedic surgeons would like to take him to theatre as soon as possible. He weighs 30kg and his mother denies any significant past medical history. He is lying in the ED crying, complaining that is leg is really sore, refusing to allow anyone to get anywhere near him. The ED staff have had several attempts at IV access and failed. How would you assess and treat his pain? AIMS OF THE VIVA y y y y y y y y y y y y y y y y y y y y y y Manage induction of anaesthesia in an un-fasted child with difficult IV access Manage postoperative pain including increasing analgesic requirements Diagnose and manage a compartment syndrome in the presence of strong analgesia Cant get drip in. Options How to assess pain in a 5 year old? Why? Why not VAS? What age? Evidence for Wong-Baker. Shown picture of a fat kid what do you think of this picture? implications for anaesthesia? Lean body mass vs ideal body mass what do they mean? How diagnose OSA in a 5 year old? No IV how to induce? Details Why assistant needed? Cricoid Pressure? why might that be a bad idea? Called for post op delirium. Ddx? Management if post anaesthetic delirium Called to ward severe pain. Ddx? Given drug chart 450mg paracetamol 4-6/24 and 1mg IM morphine 4/24 prn. What do you think of this chart? Wanted you to basically represcribe everything in RCH doses What would you do morphine infusion How to make it up? Numbers? Problem with morph and OSA monitoring requested?

I said oral, bad idea. IM doses and drugs. Wong baker faces

fat kid

10 Years Collection of ANZCA Part II Anaesthetic Viva

MK Yuen & SW Ku

10 Years viva collection 2009

338

BELL

Scenario 4
A 50-year-old man is undergoing a dacryostorhinostomy (DCR) under GA. Initial intubation was difficult. Prior to prepping and draping the surgeon adds adrenaline to a local anaesthetic solution and administers 20mls before the scrub nurse alerts him to the hight concentration (1:10,000) of adrenaline he has used. He has already infiltrated 2mg of adrenaline. The patient rapidly develops a tachycardia with marked ST depression and hypertension. The heart rate is 170/min and the blood pressure 210/110Hg both are rising How would you manage this situation? y y y y AIMS OF THE VIVA Manage severe intra-operative hypertension, tachycardia, ST changes and VT Manage the dislodged airway intra-operatively Manage unprofessional and dangerous behaviour in a colleague

I said phentolamine bad. y y y y y y y y y y y y y y y y y Long chat about alpha and beta receptors Then went on to GTN, propofol etc. What investigations Patient goes into VT, management How to do DCR what dial up, what buttons to press, synchronous? ETT and IV get pulled out by surgeon trips over them. Management Cant reintubate. Management. How will change position help? Pillows? Flexion vs extension etc. How to do it details What to connect onto it 3 way tap etc. how much wall oxygen? Where to go post op? How to manage? Airway? Surgeon angry with nurse for stuffing up how to deal with it Surgeon wants nurse to lie about adrenaline. How to deal with Refuses to go to a meeting what to do What to tell patient Would you tell lawyer

Then all good

Blah blah then surgical airway

10 Years Collection of ANZCA Part II Anaesthetic Viva

MK Yuen & SW Ku

10 Years viva collection 2009

339

Has patient come to any harm?

All rubbish and vague no idea what they wanted me to do. BELL

Scenario 5
You are asked to review a 60 year old woman in pre-assessment clinic. She has chronic respiratory disease and is scheduled for a primary total hip joint replacement. Which elements of your history and examination are important in assessing the severity of her lung disease? AIMS OF THE VIVA y y y y y y y y y y y y y Interpret relevant arterial blood gases and chest X-rays Manage severe CAL perio-operatively Manage severe hypoxia and hypotension during femoral/acetabular cememt application How to assess function? Patient cant do anything because of hip pain Features interested in preop? What investigations? Given CXR classic COPD Given ABG resp acidosis How to prognosticate based on symptoms and investigations (no RFTs) How to anaesthetize.? Why? What sedation would you use? How about analgesia?

Symptoms, treatment etc

I said spinal and sedation

I said PCA, then was invited to change it to something better then said fascia iliaca catheter he preferred that. y y y y y y y y Intraop: hypotension, hypoxia, bradycardia ddx and management cement implantation syndrome?. Management Fluids, vasopressors adrenaline and doses Another ABG: mixed metabolic resp acidosis why? Then what to do with patient? Shown CXR ARDS Shown another CXR IJV CVC, and ?right upper lobe collapse or weird pericardial

10 Years Collection of ANZCA Part II Anaesthetic Viva

MK Yuen & SW Ku

10 Years viva collection 2009

340

collection or something? No idea. BELL (other candidates were asked about the benefit of stop smoking. They commented the CXR with IJV CVC has right upper lobe collapse. They was shown one more blood gas (metabolic acidosis) after this CXR)

Scenario 6
You are to anaesthetise a 35 year old man for craniotomy and removal of frontal lobe tumour the following day. He was seen in the preoperative clinic the week before. According to his notes, problems identified at that visit were: y y y y y y BMI 56 HT, on verapamil Type 2 diabetes, on insulin Hypothyroidism, on thyroxine ? difficult intubation On dexamethasone

He has been admitted that day through the Emergency Department for increasing headache. When you arrive on the ward for your preoperative consultation, the nurse tells you the patient has become more confused and agitated in the last hour. What is your response? AIMS OF THE VIVA y y y y y y y y y y y y Manage induction of anaesthesia in the morbidly obese uncooperative patient Manage increased intracranial pressure pre- and/or intra-opeartively Diagnose and manage failure to wake postoperatively What is your approach? List differential? Was oxycontin overdose with CO2 retention and worsening ICP? Why does hypercapnia cause increased ICP What happens first increased flow or increased volume? Monroe Kelly mechanism etc Would you order CT? What is GCS how is it calculated? How would you induce? Shown picture of patient beard, MP IV, fat neck.

(completely missed by me several times)

(examiner fell asleep)

10 Years Collection of ANZCA Part II Anaesthetic Viva

MK Yuen & SW Ku

10 Years viva collection 2009

341

Talk about awake FOB y y y y y y y y Patient was uncooperative He didnt like it Asked for drugs and doses Asked difference between propofol and remi for brain surgery propofol vs volatiles tight brain intraoperative what can you do? Doesnt wake up ddx? Ventilatory strategies in icu? Suggested gas induction Then gave IV thio sux alfentanil (all part 1 stuff)

(didnt get to a diagnosis) No idea. BELL

Scenario 7
An 84-year-old man has been admitted with ischaemic pain in his left foot at rest. The vascular surgeons have asked for an anaesthetic assessment with a view to taking him to theatre for a femoral-popliteal bypass. He has non-insulin dependent diabetes mellitus and a history of congestive cardiac failure. What information do you regard as important prior to anaesthetising this man? AIMS OF THE VIVA y y y y y y y y y Interpret relevant ECGs for heart failure and ischaemic heart disease Discuss the advantages and disadvantages of regional anaesthesia versus general anaesthesia for peripheral vascular surgery Manage postoperative confusion Features on assessment? Discuss assessment of CCF severity, history, exercise, investigation How to judge functional status? What drugs would he be on? Biventricular pacing What are the indications? Echo: EF 38% anteroseptal hypokinesis. Would you procede?

I said yes, he has rest pain.

10 Years Collection of ANZCA Part II Anaesthetic Viva

MK Yuen & SW Ku

10 Years viva collection 2009

342

y y y Yes. y y y y y y y

Shown ECG old anterior infarct What other investigations? Would you procede? What would you discuss regards the risk? Advantages of regional over general for this Evidence for benefits of regional? You are seeing the patient 3 months later. He complains of forgetfulness etc How would you manage? Patient worried about next operation and want anaesthetist to guarantee no POCD again. What can you offer? What drugs would you give intraoperative to minimize chances? What wouldnt be given?

Blood loss? DVT? POCD? POCD discussion

no idea sounded like there is a drug that you should give BELL

Scenario 8
A 16 old girl has a fracture and dislocation of the head of the humerus. The surgeron is concerned about vascular compromise to the humeral heard. Urgent open reduction and internal fixation is planned. She has anorexia nervosa and severe bulimia. How would you assess this girl before anaesthesia? AIMS OF THE VIVA y y y y y y y y y y y Interpret and manage relevant electrolyte abnormalities Manage difficulties in obtaining consent for surgery Discuss regional anaesthesia in this case including relevant anatomy How to assess? Features of bulimia and anorexia Complications Cardiac manifestations? long QT, cardiomyopathy? Electrolyte changes ECG changes of hypokalaemia Old ABG: metabolic alkalosis, chronic. Why? Symptoms of hypokalaemia? What are they? Would she be symptomatic at 2.2? ABG now, on admission: Normal everything, just elevated bicarb and wide anion gap.

mixed metab acidosis (lactic acidosis from hypovolaemia) and metabolic alkalosis from

10 Years Collection of ANZCA Part II Anaesthetic Viva

MK Yuen & SW Ku

10 Years viva collection 2009

343

vomiting and diuretic abuse y y y y y y y y y y y y y y y y How would you resuscitate? What fluid to use? Hb 9.5, would you transfuse How much do you think she would weigh? What would you discuss with her? She refuses anaesthetic + procedure can she? What would you do? How to anaesthetize? Talk about anatomy of brachial plexus at that level Shown ultrasound picture small and crap label things (had arrows pointing to the plexus) Complications of interscalene What happens if inject local into carotid or vertebral artery? How much local needed to make them fit? How much would you use? How make it up? Technique (ultrasound) in plane vs out of plane, what needle. Incidence of phrenic nerve palsy? Incidence of horners syndrome?

I said she couldnt they didnt like it dont know what answer is Went on to discuss interscalene

BELL

10 Years Collection of ANZCA Part II Anaesthetic Viva

MK Yuen & SW Ku

10 Years viva collection 2009

344

ANZCA Anaesthetic Viva May 2009 Day 2 Scenario only


Scenario 1
You are the anaesthetic consultant on duty for the maternity services at your hospital (a tertiary referral multi disciplinary centre). A 30 year old pregnant (gravida 3 para 2) woman was admitted to the labour ward over night, with threatened pre-term labour. She is currently at 28 weeks gestation, and required caesarean sections for both deliveries in the past. You have been asked to see her as although the contractions have settled, a recent ultrasound is strongly suggestive of a placenta praevia. The patients history documents that the patient is a Jehovahs Witness. What issues would you like to explore in this case? AIMS OF THE VIVA y y y Negotiate a peri-operative plan with a Jehovahs Witness patient Manage severe bleeding at Caesarean section requiring hysterectomy Manage neonatal resuscitation

Scenario 2
A 20 year old male is booked for video assisted thoracoscopic (VAT) pleurodesis of his right lung for a recurrent pneumothorax. Can you comment on the pre-operative CXR? AIMS OF THE VIVA y y y Interpret a preoperative chest X ray with a pneumothorax Diagnose and manage hypoxia during one lung ventilation Manage postoperative pain

10 Years Collection of ANZCA Part II Anaesthetic Viva

MK Yuen & SW Ku

10 Years viva collection 2009

345

Scenario 3
An 8 year old girl with newly diagnosed acute lymphoblastic leukaemia (ALL) presents for insertion of a Hickman catheter (tunnelled surgical central line). She has no other significant medical history and no allergies. She weights 30 kg. Yesterday she had an uneventful anaesthetic for bone marrow aspiration and lumbar puncture. What are the potential problems in anaesthetising a child with acute lymphoblastic leukaemia (ALL)? AIMS OF THE VIVA y y y Understand the implications of cancer and its treatment for anaesthesia Diagnose and manage an intro-operative air embolus Manage suspected awareness in a child

Scenario 4
It is 8am in the morning. A general surgical colleague wants to book a case for 2pm (six (6) hours from now). The patient is a 40-year-old male for removal of mid oesophageal food bolus. The patient has told the surgeon that his brother died 10 years ago from malignant hyperthermia during a general anaesthetic. How do you assess if this patient is at risk of malignant hyperthermia? AIMS OF THE VIVA Assess and manage a potential MH risk patient Assess and manage the difficult airway Manage accidental intra-operative airway removal by the surgeon

Scenario 5
20-year-old male is brought into the Emergency Department after falling from his surfboard and hitting his head on a sandbar. He was pulled from the water by a friend, who reports that he was coherent when first brought to shore and that he was complaining of neck pain. He has a hard collar in place and is on oxygen by facemask. He is mumbling in a confused manner. He is restless and not obeying commands. He opens his eyes when you speak to him. He is localising to painful stimuli. SpO2 95%

10 Years Collection of ANZCA Part II Anaesthetic Viva

MK Yuen & SW Ku

10 Years viva collection 2009

346

Pulse rate 88/min BP 120/65mmHg What is his Glasgow Coma Score (GCS)? AIMS OF THE VIVA y y y Manage severe trauma (EMST) Manage a seizure Diagnose and manage acute spinal cord injury including spinal shock

Scenario 6
You are to anaesthetise a 50 year-old man for coiling of an anterior communicating cerebral aneurysm. He has been admitted today through the Emergency Department for increasing headache, and drowsiness. A CT of the brain confirms a subarachnoid haemorrhage. What elements of the patients history, examination and investigations would help you to assess the severity of brain patho-physiology? AIMS OF THE VIVA y y y Manage SAH peri-operatively in the context of coiling in the radiology suite Diagnose and manage a ruptured aneurysm during coiling Diagnose and manage VT/VF with a faulty defibrillator

Scenario 7
A 50 year old female patient with chronic renal failure presents for revision of an arterio-venous fistula under regional anaesthesia. She is anuric, has non insulin dependent diabetes, hypertension, and is on haemodialysis. She was last dialysed two days ago for one hour What electrolyte disturbances would you expect in this patient? AIMS OF THE VIVA y y y Interpret and explain electrolyte and acid-base abnormalities in chronic renal failure Manage intra-operative myocardial ischaemia Discuss regional anaesthesia for this case including relevant anatomy

10 Years Collection of ANZCA Part II Anaesthetic Viva

MK Yuen & SW Ku

10 Years viva collection 2009

347

Scenario 8
A 50 year old man presents to a regional hospital with a 24 hour history of increasing upper abdominal pain. He is noted to be distressed, sweating, dyspneic and jaundiced. The Resident calls you to the Emergency Department for assistance in the managing this man, after obtaining these blood results: ABGs: pH 7.19 (7.35 7.45) pCO2 pO2 HCO3 %Satn diagnosis? AIMS OF THE VIVA y y y Interpret investigations (especially arterial blood gases) relevant to severe sepsis Manage severe sepsis in the peri-operative period Manage difficulties with ventilation intra-operatively 38 87 14 95 (36 45) (85 110) (21 28) (>95)

Base XS -14 (-3 - +3) How do you interpret these ABGs? What other investigations would help you to make a

10 Years Collection of ANZCA Part II Anaesthetic Viva

MK Yuen & SW Ku

10 Years viva collection 2009

348

ANZCA Anaesthetic Viva September 2009 Day 1 Dr Alex Wan, PWH & Dr Dhugal Tam, NDH, Syndey
Some scenarios described in the examination report were a simplified version of the scenarios during examination.

Scenario 1
Background: A 60 year old male for carotid endarterectomy Aims of the viva: y y y Compare the risks and benefits of regional versus general anaesthesia Describe a regional technique (including relevant anatomy) for this procedure Diagnose (including ECG) and manage intraoperative myocardial ishcaemia

The detail scenario reported by candidate should be: 60 year old gentleman. History of TIA. Carotid Doppler shows right carotid artery stenosis of 90%. A CEA is planned. Hypertension BP 150/90 Smoker. Stopped 1 week ago He has heard that the procedure can be done awake. What are the anaesthetic options for this procedure and what are the advantages and disadvantages of each? E: what are the anaesthetic options for this procedure? C: This procedure can be done by either a Regional technique involving Deep and/or Superficial cervical plexus block or a GA. E: What the advantages and disadvantages of GA? C: GA Pros: Airway controlled. Better operating conditions. Patient comfort. Cons: Lack of reliable cerebral function monitor, loss of autoregulation, increase shunt rate, haemodynamic response to intubation and extubation, difficult assessing neurological status post-op. E: and Regional ? C: Regional pros: Awake patient can monitor cerebral function, no loss of autoregulation, reduce shunt rate, post-op pain, shorter hospital stay. Cons: complication of block, emergency control of airway more difficult, patient discomfort. E: Describe how would you do a superficial cervical plexus block? C: Consent, IV access, Monitoring including 5 lead ECG and A-line. Landmarks Posterior

10 Years Collection of ANZCA Part II Anaesthetic Viva

MK Yuen & SW Ku

10 Years viva collection 2009

349

border of SCM at level of cricoid. 22G Needle pierce cervical fascia and inject 15ml LA. E: (shows diagram of superficial cervical plexus). What are the branches of the cervical plexus? C: Lesser occipital, greater auricular, transverse cervical, supraclavicular E: Which technique is better GA or Regional? C: Recent GALA trial shows no difference in 30 day mortality and stroke rate between regional and GA. E: How will you assess cerebral function with a GA? C: Stump pressure, EEG, TCD, NIRS, Jugular bulb oximetry, SSEP, MEP (had to name them all!) E: What are the disadvantages of shunting? C: Kinking, embolism, intimal flap, and concerns about long term stenosis rate. E: OK, you do a cervical plexus block (superficial), how would you actually monitor cerebral function? C: Speech talk to patient, Cognitive get patient to count backwards from 100, motor ask patient to squeeze toy in contralateral hand. E: Operation proceeds. During incision you notice blood pressure becomes 190/110 and pulse 110. What would you do? C: Verify reading, inform surgeon and stop surgery. Support ABC. Enquire about pain because block may be inadequate. E: How would you treat it? C: I would give a beta blocker. Esmolol 0.5mg/kg followed by infusion. E: What is your target blood pressure? C: I would aim for a pressure of baseline+20% during cross clamp. E: the surgeon has clamped the internal carotid. 15 min later patient becomes increasingly agitated. What would you do? C: I would review the vital signs checking SpO2, BP, and Pulse, support the ABC if necessary. Inform the surgeon. My primary concern is cerebral ischaemia. I would speak to the patient and ask him about pain, discomfort, and assess his cerebral function E: there is no evidence of cerebral ischaemia. What will you do? C: I would start a dexmedetomidine infusion for sedation. E: ok, the operation finishes. In the recovery you perform a 12 lead ECG because the patient complains of chest pain. (Examiner shows me 12 lead ECG with antero-lateral ST Elevation) C: The patient has an anterolateral MI. I would assess the patient calling for additional help. I would support the ABC by giving high flow oxygen via non-rebreathing mask. Increase 02 delivery by oxygen, GTN, transfuse if anaemic. Reduce O2 consumption by treating tachycardia, hypertension, and hypothermia. I would start a beta blocker metoprolol 1mg IV. I would then consult ICU and look for and treat complications such as arrhythmia and

10 Years Collection of ANZCA Part II Anaesthetic Viva

MK Yuen & SW Ku

10 Years viva collection 2009

350

pulmonary oedema. BELL

Scenario 2
Background: A 25 year old man needs open reduction and internal fixation of a compound fractured tibia and fibula; he is on naltrexone 50mg/day for alcohol dependence Aims of the viva: y y y Understand the implications of naltrexone use in the perioperative period Manage severe pain not responsive to initial treatment Describe a regional technique (including relevant anatomy) that may assist with patient management E: so what are your concerns? C: Patient concerns regarding alcoholic liver disease, complications of cirrhosis, infectious status, concurrent abuse of other illicit substances. Anaesthetic concerns trauma patient with inadequate fasting, ensure ATLS done, coagulopathy, naltrexone causes opioid resistance therefore difficult with pain management. Surgical concerns regarding urgency of operation, any presence of compartment syndrome. E: What are the signs that the patient may have liver cirrhosis? C: Liver flap, Dupuytrens, spider naevi, loss of axillary hair, ascites, ankle swelling E: Patient has none of those. Would you wait for fasting for this patient? C: I would discuss with surgeon and if possible fast for 6 hours E: What are the disadvantages of waiting? C: Because of compound fracture there is a risk of infection E: Surgeon wants to be ASAP, theatre is available, will you insist on fasting patient for 6 hours? C: Yes I would E: (Looks rather annoyed). Will fasting for 6 hours really reduce the risk of aspiration? C: There are no studies to suggest it would E: tell me in the real world would you really wait? C: ok, in that case I would proceed. (Examiner lets me off finally). E: What is your technique? C: I would choose a regional technique to reduce the risk of aspiration. But I would check the clotting profile to ensure there is no coagulopathy due to liver disease E: So you would wait until the clotting profile is back before proceeding? C: Yes, it only takes 1-2 hours. E: (Looks annoyed x2). Dont try to second guess the question. Will you really wait? and how

10 Years Collection of ANZCA Part II Anaesthetic Viva

MK Yuen & SW Ku

10 Years viva collection 2009

351

likely is it that patient without signs of liver disease will have deranged clotting? C: Not very likely. So I would proceed. E: OK, the patient refuses a regional and just wants to be asleep. C: I would explain the options, as long as patient understands the risk of GA including risk of aspiration then I would perform a RSI with cricoids pressure. E: The case proceeds uneventfully, in the recovery the patient complains of a lot of pain. How will you deal with this situation? C: I will attend to the patient and take a history and examination with regards to his pain, after ensuring that the patients vital signs are stable. I would exclude the presence of compartment syndrome. E: What are the signs of compartment syndrome? C: No palpable pulses, unable to move toes, swelling of the leg, and white in colour? E: How can you measure compartmental pressure? C: By using a manometer. Pressures of greater than 30mmHg can lead to necrosis of muscles. E: ok, patient has none of those. How would you manage the pain. The nurse has already given 10mg of morphine? C: I would use a multimodal technique. Paracetamol 1g, Ketarolac 30mg, tramadol 100mg, and consider a regional technique. E: Well come to regional techniques later. Anything pharmacologically you can think of? C: Gabapentin can be used for acute post-surgical pain, with studies suggesting a large single dose of 1200mg. E: Anything else? C: (in restrospect examiner was probably asking about Clonidine) E: ok, what regional technique would you choose? C: I would put in a lumbar epidural. Other options include a sciatic and femoral nerve block combined. E: Fair enough, an epidural would be a good option. Lets say you decide to put in a sciatic nerve block. Can you tell me the anatomy of the sciatic nerve? C: L4-S4 root. From lumbosacral plexus on piriformis muscle, exits pelvis via greater sciatic foramen, descends superficially between greater trochanter and ischial tuberosity and divides into posterior tibial nerve and CPN above the popliteal fossa. E: How would you perform a sciatic nerve block? C: I would use the classical approach of Lebat. Position patient with lateral leaning forwards, with leg flex(standard technique) E: what nerve stimulator settings would you use? C: 1.5mA initially, then dial down until stimulation achieved between 0.2-0.5mA E: and the response? C: Any ankle dorsiflexion and extension is acceptable.

10 Years Collection of ANZCA Part II Anaesthetic Viva

MK Yuen & SW Ku

10 Years viva collection 2009

352

E: What are the disadvantages of the sciatic nerve block? C: Will miss medial aspects of the lower leg supplied by the femoral nerve. E: So how will you block the femoral nerve? C: Just lateral to the femoral artery at the inguinal area BELL

Scenario 3
Background: A woman in labour ward has had a seizure. Post treatment she is unrousable; magnesium infusion has inadvertently been run through as a bolus. Aims of the viva: y y y Diagnose and manage seizure during labour Manage cardiac arrest during pregnancy Diagnose and manage magnesium toxicity

The scenario in examination only provided information about patient having a seizure in labour ward. The rest was given during the progress of viva. E: So how will you manage this situation? C: This is a crisis situation. I would get additional help. Inform the obstetricians. Support the ABC. Airway with triple maneuver and oropharyngeal airway. Assistant to apply cricoids pressure. Breathing high flow oxygen via NRM, Circulation left tilt and establish IV access. Terminate seizure by 0.1mg/kg of Midazolam. Once seizure terminated then move patient to recovery position. Continue stabilization and ask obstetrician to assess fetal status. E: What are the likely causes? C: Obstetric related eclampsia/AFE/PE. Non-obstetric related preexisting epilepsy/hypoglycaemia/intracranial events (vascular and tumours)/concurrent drug abuse. E: How will you determine the cause of the seizure? C: Hx/Examination/Ix. Review the antenatal records for the presence of hypertension after 20 weeks pregnancy and proteinuria which suggests pre-eclampsia. Review past hospital admissions and medical history for epilepsy/DM. Speak to partner about any suspected drug abuse. Examine for any pupil abnormalities. Ix routine bloods and electrolytes/clotting/LFT/glucose/ABG/Drug screen, may need CT head. E: OK, on reviewing the notes you notice that patient had BP of 150/100 after 20 weeks of pregnancy with proteinuria. C: This is eclampsia. E: what subsequent management would you initiate?

10 Years Collection of ANZCA Part II Anaesthetic Viva

MK Yuen & SW Ku

10 Years viva collection 2009

353

C: The principle is stabilization and delivery of fetus remains the definitive treatment. Blood pressure Hydralazine 5-10mg bolus is first line at our instituition E: what BP would you aim at, lets say patients current BP is 190/100 C: I would aim for 150/90 E: ok, carry on C: Seizure prophylaxis - Start magnesium 4g bolus over 10 min. Followed by 1g/hr and if patient fits a further 2g bolus can be given. Monitor levels with target 2-3.5mmol/l. Test patellar reflexes (lost at 5mmol/l) Fluid management risk of pulmonary oedema due to leaky capillaries. Titrate fluid to keep UO at least 0.5ml/kg/hr. Fetal monitoring need to assess CTG for fetal well-being and urgency of delivery E: What anaesthetic technique would you use for C-section? C: Providing patient is not coagulopathic and platelet counts are OK, I would do a single shot spinal. This is because the eclamptic pregnant patient will likely have a difficult airway, and avoid BP response to intubation. E: Disadvantages of SA? C: CI if low platelet count and deranged clotting. Emergency control of airway more difficult if patient fits, post-op pain management, previous concerns regarding SA causing precipitous drop in BP due to contracted intravascular state in pre-eclampsia but not proven in studies. . E: Do you know why? C: Probably related to presence of endothelial activators from the disease process, and the fact the baby will more likely have IUGR thereby causing less ACC. E: OK, on transfer to theatre. The patient becomes unresponsive and you cannot feel a pulse. What would you do in that case? (the rest was about Magnesium toxicity/perimortem section/risk management) BELL

Scenario 4
Background: A 2 year old brought in to the emergency department after near drowning Aims of the viva: y y y Manage basic resuscitation in a 2 year old child Diagnose and manage resistant VF Diagnose and manage persistent hypoxia in this setting (including ABG interpretation)

The detail scenario reported by candidate should be: A 2 year old boy is pulled from a lake. He is brought into A&E by the paramedics. You are the

10 Years Collection of ANZCA Part II Anaesthetic Viva

MK Yuen & SW Ku

10 Years viva collection 2009

354

most senior medical person in a rural hospital receiving this boy. What are your priorities? E: So how will you manage this case? C: I would assess and resuscitate according to ATLS starting with primary survey. Airway with cervical spine control, breathing, circulation(regurgitated my notes) In addition, rural hospital may not have paed ICU services and patient may need to be transferred to a tertiary centre E: So what are the differences between adult and paediatric in terms of trauma management? C: Airway I would use a non-cuffed size 4.5mm tube, circulation IV access may be difficult especially if this patient is likely to be hypothermic, may require intra-osseous access(looking blank) E: ok, you mentioned hypothermia as a complication of this case, the patients temp is 29, how will you treat this patient? C: Active and passive warming. Passive covering up patient, warm fluids, forced air warming, radiant heater. Active intra-peritoneal irrigation. As I have no experience in performing this I would seek advice from paediatric ICU about how to do this. E: Any other active means? C: irrigation of bladder E: ok, what target? C: Dont want to be too aggressive I would aim 35 (guess) E: How fast? C: ..Not sure, maybe 0.5 degrees per hour E: Good guess (examiner ticks on mark sheet) E: What are the disadvantages to warming too aggressively? C: Vasodilatation would drop the blood pressure, precipitate arrthymias, electrolyte abnormalities. E: OK, while rewarming is instituted the nurses tell you that she cant feel the femoral pulses. What would you do? C: Cardiac arrest situation. Try to feel the pulse myself, if none get crash trolley, declare cardiac arrest. I would get one person to start chest compressions with ratio of 15:2 breaths. Attach defibrillator and assess the rhythm. Adrenaline 10mcg/kg every 3 min. Secure airway if not already done. If VF or pulseless VT give shock initially of 2J/Kg followed immediately by chest compressions. Next shock will be 4J/Kg. If PEA/asystole continue chest compressions, treat reversible causes 4H and Ts. E: This is the initial rhythm (shows ECG strip) C: This is VF. So I would shock this patient initially with 2 J/kg followed by immediate CPR. E: What could be the cause of this? C: Hypothermia, electrolytes hypoCa, hypoMag, toxins, hypoxia

10 Years Collection of ANZCA Part II Anaesthetic Viva

MK Yuen & SW Ku

10 Years viva collection 2009

355

E: Is VF common in children with hypoxia C: emno E: what rhythm do you get in hypoxia? C: Asystole E: ok, anything else you would give? C: Amiodarone 5mg/kg E: The patient is still hypothermic would you keep on shocking the patient until the patient is warm? C: Hypothermic patients will have shock refractory VF. So my priority is warming up the patient and continue chest compressions. E: So how many times will you shock? C: 5 times and then continue chest compressions and warming measures (complete guess). Rest of viva about how drowning leads to ARDS and briefly about ventilation strategies for ARDS. I thought this was a difficult viva. Drowning, and resuscitation in a hypothermic child. BELL

Scenario 5
Background: A 50 year old female with a retrosternal thyroid for resection with symptoms suggesting significant airway obstruction. During surgery the drapes catch alight and a scrub team member inadvertently spreads the flames Aims of the viva: y y y Recognise significant tracheal narrowing on imaging (CXR and/or CT) Form a plan for airway management Manage an intraoperative fire

The scenario in examination provided information about goitre only. The part about the flames was given during the progress of viva. E: How would you assess to see if she has airway obstruction from history and physical examination? C: History:

Hoarseness(vocalcordpalsysecondarytorecurrentlaryngealnervepalsy) Localanatomicaldistortion(dysphagia) Respiratorysymptoms(dyspnea,orthopnea) Historyofdifficultintubationinthepast

P/E:

10 Years Collection of ANZCA Part II Anaesthetic Viva

MK Yuen & SW Ku

10 Years viva collection 2009

356

Excluderetrosternalextensionbyfeelingthelowerborderofthethyroid gland Feelforthetrachealdeviation LookforsignsofSVCO(Pambertonsign) Clinicaltestsinclude(mouthopening,MPscore,Neckextension,TMD) Neckcircumference

E: This patient does have orthopnea and needs to sit up at 30 degrees during asleep. What investigation would you like to have? C: I would like to have the following Investigation:

Flexiblelaryngoscopytoassessairwayedema(SVCO)andvocalcord palsy(RLNpalsy) CXRtolookfortrachealdeviationandruleinretrosternalinvolvement CTneckandthoraxtoassessthecrosssectionofthetracheaand relationshipwiththegoiter. Alsotoassesstheextensivenessof retrosternalextension.

E: (Shows a CXR), describe the abnormalities.

C:Trachealdeviationtotherightduetoaretrosternalgoiter.
E: Any other abnormalities?

C:Cannotidentifyotherabnormalities.
E: (Shows a CT neck and thorax), describe the abnormalities.

C:Retrosternalgoiterthatextendstothesuperiormediastinumtogetherwith trachealdeviationtotherightside.
E: How would you anaesthetize this patient?

C:Myconcerninthispatientinclude: y Difficultairwaywithretrosternalextensionofgoiter y IwouldmaintainpatientsspontaneousrespirationbeforeIsecurethe airway MychoicesincludeAwakefibreopticintubationwithairwaytopicalizationfollowed bygeneralanaesthesiaorinhalationalinduction. Sinceairwayobstructioncan occurevenwithinhalationalinduction,myfirstchoiceisawakefibreopticintubation.


E: Before we move onto awake fibreoptic intubation, tell me what tube you would use, and what size you are going to use?

C:Iwoulduseanarmoredreinforcedtubewithassortedsize.Inalady,mychoice initiallywouldbesize7.However,inviewofanarrowedairway,Iwouldbeginwitha size6.5tube.


E: Why a reinforced tube?

C:Itisnonkinkable,andthetubewouldbepositionedtoreachtheforeheadand connecttothecathetermountthatconnectstothebreathingcircuitontopofthe
10 Years Collection of ANZCA Part II Anaesthetic Viva MK Yuen & SW Ku

10 Years viva collection 2009

357

head. Itismoreresistanttocollapse,soanycollapseoftheairwayaftermusclerelaxation wouldbebettercounteractedbytheuseofreinforcedtubethanaPVCtube.


E: You dont have a fibreoptic bronchoscope, its broken, how would you secure the airway?

C:Difficultintubationtrolley,ENTsurgeonstandby,goodpreoxygenation,andmy firstchoiceistoattemptawakeintubationunderlocalanaesthesia,thisissaferthan inhalationalinductionbutisveryuncomfortable.


E: How else could you secure the airway?

C:Inhalationalinductionwith100%O2/Sevoflurane.Graduallyintroducethe sevofluranefromalowconcentration,andassessthepatientcontinuouslyforthe depthofanaesthesia,meanwhileconstantlypayingattentiontopatientsvitalsigns.


E: How do you fix the tube?

C:Bymeansoftapingandensurethatthetubeisinthedesiredpositionbeforeallow surgerytobegin.
E: You have secured the airway, and how are you going to place the ETT at the optimal place without a FOB?

C:Clinically,Iwoulddeliberatelyachieveendobronchialintubation,whichusually wouldplacetheETTintotheRightmainbronchus.IwouldauscultateuntilIhear bilateralairentry,whichsignifiesthatthetubeisatthecarina,andIwouldwithdraw theETTabout1cmabovethecarina,asthegoiterwouldleadtoairwaycollapseifI positionthetipofETTproximaltotheobstruction.


E: Good. The operation went smoothly, but suddenly the scrub nurse is screaming and there is fire on the drapes. What is your management?

C:Thisisacrisissituationwithafireintheoperatingtheatre! Iwouldcallforhelpimmediately Iwouldattendtothepatientimmediatelyandprotectthepatientby maintainingairwaybreathingandcirculation. Iwouldalsoprotectthe staffsbyaskingthemtostandawayfromthefire. Iwouldremovethe drapesandextinguishitwithIVfluidfromthewarmcupboard. Iwouldthenassesstheextensivenessofinjuryinthispatient,toseeifit involvestheairway,neck,andthorax.
E: Why would there be fire?

C:Source:Diathermy Fuel:Drapeswithalcoholpreps Oxidizingagent:Oxygeninair.


E: How do you prevent that from happening again?

C:Ensurethattheskinpreparationisdryandthedrapesarenotsoakedwithalcohol.
E: What would be your subsequent management?

10 Years Collection of ANZCA Part II Anaesthetic Viva

MK Yuen & SW Ku

10 Years viva collection 2009

358

C:Iwouldmaintainanaesthesiaandallowthesurgerytocontinueifthepatientis stable. Iwoulddocumenttheincidenceandfillinthecriticalincidentform. IwouldprepareforfuturequalityassurancebyM&Mmeeting.


E: The nurse screamed, do you think you need to attend to her?

C:Iwouldtalktothenurseandassessiftherewasanytraumatoherphysicallyas wellaspsychologically.Iwouldarrangecounselingwithherandlettheseniornurses andmyselftocloselymonitorherconcerns.


E: After the operation, how would you counsel the relatives?

C:Thisisapotentialmedicolegalsituation. Iwouldpreparetheinformationthatoccurredintraoperativley. Iwouldensurethepatientisstable


BELL

Scenario 6
Background: Asked to provide anaesthesia for ECT at a new hospital (stand alone psychiatric hospital). The first patient has an implanted defibrillator in situ Aims of the viva: y y y Assess a new hospital with a view to providing anaesthesia services Manage the patient with an AICD perioperatively Manage the patient who fails to wake after a prolonged seizure

The information about implanted defibrillator was provided in the middle of scenario. E: What are the facilities that you would consider to be placed in this ECT suite?

C:IwouldsetuptheECTsuiteaccordingtotheANZCAcollegeguideline. (Imodifiedabittofacilitatemypresentation,butforcompletenessplease refertointerimreviewT12008) Fortheanaesthesiaequipment,Iwouldhavethefollowingchecklist. y Oxygensourcefrompipeline/walloutletandoxygencylinderat correctpressureandconnectionwithoutanyleakage y Oxygendeliverydeviceswhichincludevariableperformancedevice suchasnasalcannulaandHudsonmaskorfixedperformancedevice suchasVenturimasks. y AirwaydevicesincludingDL/ETT/LMA/Guedel/NPairwayswith assortedsizes
10 Years Collection of ANZCA Part II Anaesthetic Viva MK Yuen & SW Ku

10 Years viva collection 2009

359

y y y y y y

Equipmentstoventilatethepatientincludingmanualresuscitator, ventilator IVinfusiondevices,syringesandneedles,IVfluids Drugsforanaesthesiaandresuscitation Functioningwallsuctionwithanalternativesuctionsystem MeasurementdevicesforNIBP/SpO2/ECGandacardiacmonitorthat displaysandmeasuresthevitalsfromtimetotime Devicesthatshouldbestandingbyincludedefibrillator

E: What about the staffing?

C:Eachfacilitymustdesignate: y Oneormorespecialistanaesthetiststoadviceonthechoiceand maintenanceofanaesthesiaequipment y Oneormoreofitsnursingortechnicalstafftoberesponsibleforthe organizationofcleaning,maintenanceandservicingofanaesthesia equipment. y Thereshouldbeanassistantfortheanaesthetistforpositioningofthe patientaswellastoensureproperfunctioningandservicingofall equipment. Fortheproceduralist,whowouldbethepsychiatristinourlocality, shouldhaveskillsforadvancedlifesupport. Inourlocality,patientsarecaredbyanaesthetistwhoprovide anaesthesiaduringECT

y y

E: How do you maintain the standards as far as facilities are concerned?

C:Thereshouldberegularsterilizing,cleaning,andhousekeepingroutinesforthe careofequipment Documentedservicingoftheanaesthesiadeliverysystemandmedicalgasequipment byanappropriateorganizationmustbecarriedoutatintervalsrecommendedby manufacturer. Aspecialistshouldbeinchargetoadviceonthechoiceandmaintenanceof anaesthesiaequipment.


E: What about standards in the recovery room? E: You now have a middle aged lady coming in for a ECT. She is known to have a AICD implanted a few years ago. What would you do?

C:MyconcernispotentialhemodynamicdisturbancessecondarytoECTinapatient withAICD Iwouldstartwithmypreoperativeassessment,focusingonthepatientaswellasthe AICD. Fromthehistory,physicalexamandinvestigation,Iwouldidentifythe


10 Years Collection of ANZCA Part II Anaesthetic Viva MK Yuen & SW Ku

10 Years viva collection 2009

360

y y

y y y y y y

IndicationforAICD,exactmanufacturedate,modeof pacing/defibrillation Currentcardiacstatus,anyactivecardiacproblemssuchasunstable angina,MI;decompensatedheartfailure;significantarrhythmias includingMorbitztypeII,completeheartblock,symptomaticbradyor tachyarrhythmias;significantvalvularheartlesions Anyshocksexperiencedbythepatientsincetheimplant? Ifyes,how frequent Anyhistoryofsyncope Anyantiarrhythmicstakeninadditiontothe antipsychotic/antidepressants Exercisetolerance Comorbidities ForP/E: x CVS:ruleoutheartfailure/arrhythmia x Resp x Airway Forinvestigation x CXRtolookforplacementofleads

x ECGtolookforpacemakerdependency x ElectrolytestooptimizeKorMgifabnormal IwouldconsultcardiologistfortheiropinionregardingthepresenceofAICDinthis patient.


E: What would you want the cardiologist to do?

C:Provideexpertmanagementin y Interrogationandadviceonwhetherdefibrillationmodeneedstobe switchedoff.Ifthedefibmodeneedstobeswitchedoff,itshouldonly bedoneintheoperatingtheatre,wheremonitoringandresuscitation canbeachievedimmediatelyifnecessary y Echocardiogramtolookatthecardiacfunction


E: How would you perform the anaesthetic?

C:Myplanistoadequatelyfastthepatientexceptusualmedication Consentforminorifpatientisnotcompetent OnthedayofOT,Iwouldpreparethetheatre,equipmentanddrugs,togetherwitha defibrillatortrolleystandby. Iwoulddrawuptheresuscitationdrugsincludingtheantiarrhythmicagents. IwouldtheninsertanarteriallineunderLA


E: Why would you want to insert an arterial line with a 3 minute procedure?

10 Years Collection of ANZCA Part II Anaesthetic Viva

MK Yuen & SW Ku

10 Years viva collection 2009

361

C:ThisisbecausethepatientmightdeveloparrhythmiasecondarytoECTasaresult ofinitialparasympatheticandsubsequentsympatheticsurge.Arrhythmiamay precipitatehemodynamicinstability.


E: What information can you obtain from Arterial line? BELL

Scenario 7
Background: A 20 year old male suffers severe blunt chest trauma Aims of the viva: y y y Resuscitation the trauma patient (EMST principles) and manage a large haemopneumothorax Diagnose and manage TRALI (including ABG interpretation) Manage difficulties with ventilation in this setting

The scenario provided to candidate during examination should be: 20 year old mechanic trapped underneath a vehicle for one hour before being brought in by paramedics. During the transfer, the heart rate 120 bpm, blood pressure 80/40mmHg. E: How would you manage the case on arrival?

C:IwouldpreparetomanagethispatientaccordingtotheATLSalgorithm. Universalprecautionwithtriageofthispatient Warmtheemergencydepartment ContactthebloodbankforOnegativeblood Beginmyassessmentwithprimarysurveyfollowedbysecondarysurvey


E: Wont you want to talk to the paramedics?

C:Yes,Iwouldaskwhathashappenedatthesceneandwhathashappenedduring thetransfer.Inoticedthepatientwashypotensionwithtachycardia,Iwouldaskwhat managementwasgiven.Havingsaidthat,myaimistofocusonthepatientwith primarysurveywithoutdelay.


E: So how would you assess this patient?

C:IwouldaskmyassistantstoattachmonitorsforECG/NIBP/SpO2. Iwouldassesshisairwaywithcervicalspineimmobilizationwithaneckcollar Talktothepatientandlistenforstridor/hoarseness,removeforeignbodyor excessivesputuminthethroatbygentlesuctioning.


E: He is conscious and talking, why dont you jump into history taking now?

10 Years Collection of ANZCA Part II Anaesthetic Viva

MK Yuen & SW Ku

10 Years viva collection 2009

362

C:Iwouldcompletemyprimarysurveybeforeobtainhistoryfromthispatient.
Q: ok

C:Forbreathingandcirculation,Iwouldgivehighflowoxygenthrough nonrebreathingmask. Iwouldthenassessforanycyanosis,respiratoryrate,trachealdeviation,flailchest, hyperresonsancenoteandabnormalbreathsoundstoruleoutlifethreateningchest problems,namelytensionpneumothorax,openpneumothorax,hemothorax,flail chest.


E: He has decreased air-entry on the left with tracheal deviation to the left.

C:Iwouldpercussontherighttolookforhyperresonancenotetoruleintension pneumothorax.
E: Hyper-resonance note on the left.

C:Tensionpneumothorax,needlethoracostomywith16Gangiocathtomidclavicular line2ndintercostalspace. FollowedbychestdraininsertionwithconnectiontoUWSD.


E: What would you do next?

C:Reviewairwayandbreathing,ifimprovedthenmoveontoassesscirculationwith hemorrhagiccontrol.Iwouldinsert2largeboreIV16G.
E: Why dont you insert larger angiocatheters?

C:Inpatientwithhypovolaemia,theymayhaveperipheralshutdown,ideallyIwould liketoinserta14GbutincaseIcouldntIwouldinserta16G.
E: Whats next?

C:CheckNIBP/pulse GiveNormalsalinefullrate1Landassessresponse CheckbloodforCBC/ABG/electrolytes/clotting/T&S/glucose Lookforexternalsourcesofbleeding Lookforoccultbleedingsourcesbyexcludingdistendedabdomen/pelvis#/longbone fracture


E: Patients BP is 80/40mmHg, pulse 120 bpm.

C:Iwouldcontinueresuscitationandlookforunderlyingcause Lookforabdominaldistensionandpalpateforanytenderness
E: The abdomen is distended, what would you do?

C:IwouldruleoutintraabdominalbleedingbyperformingaFASTscan.
E: What to look for in a FAST scan?

C:Freefluidinhepatorenal/splenorenalfossa/PouchofDouglas/pericardium.
E: There is free fluid, would you do a CT abdomen?

C:No,becauseofhemodynamicinstability. Iwouldconsultgeneralsurgeonforimmediateassessmentandalertthetheatrefor
10 Years Collection of ANZCA Part II Anaesthetic Viva MK Yuen & SW Ku

10 Years viva collection 2009

363

potentialemergencylaparotomy BELL

Scenario 8
Background: A 30 year male with a Fontan circulation from previous surgery for congenital heart disease presents for an appendiectomy Aims of the viva: y y y Understand the Fontan circulation and the principles of haemodynamic management in the perioperative period Diagnose and manage an obstruction to the expiratory limb of the circle leading to a pneumothorax Discuss an iatrogenic complication with the patient and family

10 Years Collection of ANZCA Part II Anaesthetic Viva

MK Yuen & SW Ku

10 Years viva collection 2009

364

ANZCA Anaesthetic Viva September 2009 Day 2 Scenario only


Scenario 1
Background: A 60 year old male for endoluminal aortic aneurysm repair Aims of the viva: y y y Understand the risks and benefits of neuraxial blockade for postoperative analgesia in patients to receive intraoperative anticoagulation Recognise (ECG) and manage slow AF in the immediate preoperative period Recognise (ECG) and manage intraoperative rapid AF

Scenario 2
Background: A 50 year old man for acromioplasty and rotator cuff repair in a day surgery unit. He is already on oxycodone and refuses any regional blockade Aims of the viva: y y y Form a perioperative pain management plan Manage severe pain in the recovery ward Manage the transfer of a heavily sedated postoperative patient to another hospital

Scenario 3
Background: Labour ward call for analgesia in a woman with prolonged ruptured membranes. On examination she is tachycardic and febrile Aims of the viva: y y y Assess the febrile pregnant woman Discuss epidural analgesia in the presence of fever Manage sepsis during Caesarean section

10 Years Collection of ANZCA Part II Anaesthetic Viva

MK Yuen & SW Ku

10 Years viva collection 2009

365

Scenario 4
Background: A 4 year old on the ward with pneumonia requires drainage of a large left pleural effusion under ultrasound guidance Aims of the viva: y y y Assess a medically unwell child Manage severe oxygen desaturation on the general ward (including ABG interpretation) Manage the interhospital transfer to a paediatric interventional radiologist

Scenario 5
Background: A large, uncooperative, developementally delayed 20 year old male presents for restorative dental work. He has a grade 4 Mallampati view Aims of the viva: y y y Plan the airway management in an uncooperative patient Manage a total intraoperative power failure (blackout) Manage emergence delirium in this setting

Scenario 6
Background: A 70 year old male with lung cancer and previous right penumonectomy presents for craniotomy for removal of an isolated metastasis. Aims of the viva: y y y Discuss ethical issues surrounding a decision to operate in the face of a poor prognosis Conduct safe neuroanaesthesia in the presence of significant comorbidities Manage postoperative acute pulmonary oedema

Scenario 7
Background: A motor vehicle accident victim with major abdominal and pelvic injuries. Aims of the viva: y y y Manage multiple trauma with severe bleeding Diagnose (and interpret appropriate lab results) and manage severe intraoperative coagulopathy Diagnose and manage postoperative hepatic dysfunction

10 Years Collection of ANZCA Part II Anaesthetic Viva

MK Yuen & SW Ku

10 Years viva collection 2009

366

Scenario 8
Background: A 40 year old male in cardiac catheter laboratory for urgent coronary stent placement develops cardiogenic shock Aims of the viva: y y y Manage acute cardiogenic shock Diagnose (ECG) and manage complete heart block including the appropriate settings for temporary pacing Diagnose and manage an obstructed expiratory valve on the self inflating bag during transfer to the intensive care unit

10 Years Collection of ANZCA Part II Anaesthetic Viva

MK Yuen & SW Ku

10 Years viva collection 2009

367

Appendix 1
April 1994 FANZCA Dr. Phoebe Mainland

The candidates are divided into groups of four, to rotate through the four stations of each block. Meanwhile other groups of four doing the same, ie there are five examiners asking the same question at one time, and each may add their own direction to the stem, although there is a guide to the evolution of the scenario. Of course the examiners are all different; some seemed a little bored as no doubt they were. Because of the guide to the questions they all must ask, the task sometimes was a tick the box once the answers given exercise. Some viva sessions were observed, by one observer. The observer usually sat on the same side of the table as the examiner, ie in view. In a few there was just one examiner and me. For the medicals, one sees the patient in one room and is then taken to another room to present the case and answer questions. For both of my cases the examiners were not back in the patients room when the bell for me to go in and start asking the history. I just went ahead and asked what is your problem type of introduction. There didnt seem to be a problem with this. Practice vivas I did in Sydney just before the exam were taken by a successful candidate from the previous exam. He introduced the case such as This is Mr. X who was admitted to hospital with increasing shortness of breath. Would please ask him relevant questions and perform an examination as you think indicated? He said if one went off the track the examiners would interrupt and direct you back to the important areas. The examiners also tell you when you are half way through the viva. One medical I had to take the BP myself, another they said dont bother, her BP is: I suggest going for the cuff. I had taken my watch off for all the vivas, and didnt have it for the medical. Need watch AND stethoscope (some people turned up without one of these!) The guy taking us for the practice sessions said pack-years smoking was a favorite concept. On these trail sessions I was reminded how in HK we dont see the IHD cases that fill the wards in Australia. Generally the examiners were friendly and encouraging. The questions were mainly straightforward, almost too obvious, and sometimes one wonders how can one fail, or at least the feeling that it is hard to score extra points. Questions were usually broad enough but with some indication of direction so that I often found the chance to answer the question but to then go on to talk about what might be a controversial issue, or expand with an explanation. Also I found it not a problem to say I dont know when this was the case. The questions were very

10 Years Collection of ANZCA Part II Anaesthetic Viva

MK Yuen & SW Ku

10 Years viva collection 2009

368

clinical and applied. Having the exam over two days is a strain as at the end of day one of course you cant feel that its all finished. I went the pub with a friend to chat, which ws a good thing to do! In a way though it does help to reduce the build up of having it all pending on your performance in a very short time, and since day one went OK, I approached day 2 as just 2 more hours to go and its all over almost I dont care, just lets get it over; ie less panic etc. I havent spoken to anyone who failed, so I cant give any ideas as to with what they catch up people. Hope this helpful!

10 Years Collection of ANZCA Part II Anaesthetic Viva

MK Yuen & SW Ku

10 Years viva collection 2009

369

Appendix 2

ANZCA Medical Viva May 1994 Day 1 Medical Viva 1


Introduction - 36 yo with a complex PMHx of Lupus and CRF, he also has a heart problem. Can you evaluate his cardiac problem. Hx - unsure of the cause of his heart problem, aware of murmur, no RHD, hypertensive, exertional dyspnoea, ischaemic chest pain - Ix by angiogram = leaky aortic valve Ex - BP 120/50, palpable carotid systolic bruil, water hammer pulse, displaced AB, palpable Ao systolic thrill, soft S2, loud systolic ejection murmur radiating to carotid, loud diastolic murmur at LSE Presentation - brief summary of history examination, conclusion = mixed aortic valve isease, assessment of severity of AS and AI, clinical decision of the more haemodynamically significant lesion y y How would you investigate this? - ECG, CXR, Echo P Asked to comment on CXR.

Medical Viva 2
Introduction - 35 year old woman presents for thyroidectomy History - euthyroid, NIDDM, weight loss. Examination - general approach - airway, skin and hair, eye signs, neck signs, RS and CVS, BP - had to choose obesity cuff forgot to examine tendon reflexes when asked is there anything else you might like to do? Presentation - obese woman with NIDDM, thyoid goitre and clinically euthyroid, potential anaesthtic problems are y y y y y Signs of hyper/hypothryroidism? Investigation of thyroid mass - FNA, VIS, Nuclear Scan, TFTs Comment on these TFTs - why is the TSH low? Management of diabetes peri-operatively? How would you anaesthetise her for thyroidectomy?

10 Years Collection of ANZCA Part II Anaesthetic Viva

MK Yuen & SW Ku

10 Years viva collection 2009

370

Causes of post-operative stridor and management

Investigation
PFTs Respiratory difficulties in a patient 4 weeks after a CHI FEV 1.0/ FVC = 35 % y y y y y y how do you differentiate poor cooperation from obstructive picture? flow-volume curve - typical fixed extra-thoracic obstruction differential causes - likely cause = sub-glottic stenosis how would this be managed surgically? how would you provide an anaesthetic for laser surgery of the trachea? how do you prevent airway fires?

Biochemistry Na 136, K 3.2, HCO3 28, Urea 10, Creat. 10, ALP 67, ALT 2100, AST 2800, Bili. 64, GGT 124 y y y Comment on the pathological causes hepatocellular jaundice possible and most likely causes what are likely causes of the sodium and potassium changes

10 Years Collection of ANZCA Part II Anaesthetic Viva

MK Yuen & SW Ku

10 Years viva collection 2009

371

ANZCA Medical Viva May 1994 Day 2 Medical Viva 1


male ?60s, pulmonary fibrosis, ischaemic heart disease ?cags/angioplasty, pacemaker, iddm, renal impairment. stone, arm stump (home made bomb as a child), CVA (not picked) y y cause of dyspnoea, CXR to help periop. management of his diabetes - how much insulin, would you add K to infusion

Medical Viva 2
female ?50s, palpitations, dyspnoea, investigated with cardiac cath - "leaking valve", assessment for replacement said mitral regurgitation; probably mixed mitral disease y y y y y y any problems with her medications, which include Ca channel blocker and beta blocker complications of AF medication for management of these complications what INR would you aim for in anticoagulation what would be contraindication to mitral valve replacement cause of mitral valve disease, which is likely cause in this lady

Investigations
y y y y y y y wide complex tachycardia - what is the rhythm flow volume loops of patient who has bad prolonged intubation laser surgery to tracheal stenosis - ? take stent out first explain how to set up/principles of ice bottles patient with ccf - comment on these renal function tests and liver function tests, explanation of why they are abnormal cxr bilateral bullous lung disease, patient to have stapling of bullae - problems of this anaesthetic, how to minimize, do worst side first trauma CXR- pneumotbx with icc x 2 incomplete inflation of lung

10 Years Collection of ANZCA Part II Anaesthetic Viva

MK Yuen & SW Ku

10 Years viva collection 2009

372

ANZCA Medical Viva September 1994 Day 1 Medical Viva 1


A 60 year old lady with 5 years history of palpitation, told to have fibrillation, started after a viral flu. Told by doctor to have viral infection of the heart. She is now on sotalol, digoxin and lasix. She has no S/S of heart failure. Also had history of stroke 1.5 year ago causing L hemiparesis. Now on warfarin as well. P/E: scar on L chest, no other abnormality.(scar - pacemaker) To Physcian - P P: OK, after examining the patient, can you give us a summary of the patient? C: A 60 year old lady with viral infection of the heart 5 years ago complicated by atrial fibrillation and thromboembolic stroke. Now on warfarin, digoxin, sotalol and pacemaker. P: Why you said the patient has infection of the heart? C: The patient was told by her doctor to have viral infection of the heart. P: What can it be? C: Myocarditis or pericarditis. P: Patient is on pacemaker, is it common for patient with atrial fibrillation to have pacemaker? C: No, it may be due to the presence of sudden tachycardia and then bradycardia result in Strokes Adams attacks. P: What disease can have this tachy-brady-arrthymia? C: The sick sinus syndrome. P: What is sick sinus syndrome? C: It is a disease of the sinus causing abnormal firing. P: What is the cause? C: Degeneration and IHD. P: Assume 5 years ago, you are the first one to see the patient, what will be your assessment? C: I will ask for the history, do a P/E and order some investigations. For the history, I will look for any syndromes of heart failure, effort tolerance and Strokes-Adams attacks. For P/E, look for abnormal HS, murmur and signs of heart failure. P: Any murmur do you hear? C: No. P: What cardiac lesion are you looking? C: Mitral stenosis. P: Can this patient have MS? C: Yes, there are clinically not detectable MS. P: What investigation will you do? C: CXR - heart size, calcifications, signs of pulmonary odema, ECG, Echocardiogram -

10 Years Collection of ANZCA Part II Anaesthetic Viva

MK Yuen & SW Ku

10 Years viva collection 2009

373

structural valve lesion. P: What blood test will you do? C: Thyroid function test. P: Can this patient have thyrotoxicosis? C: Patient do not give any history of thyrotoxicosis and there is no eye signs, no tremor, no sweating and no goiter. P: If the patient is clinically euthyroid, will you still do the TFT? C: Yes, there may be clinical euthyroid but chemical euthyroid. To Anaesthetist - A A: Supposed this patient is going to have a emergency laparotomy within 2 hours, what will be your considerations? C: The problems of this patient are: 1. AF on digoxin, sotalol and other cardiac problems 2. Pacemaker 3. History of stroke 4. Warfarin 5. Electrolyte disturbance A: What about the pacemaker? . C: I will assess the integrity and function of the pacemaker, type of pacemaker, battery life, programmability, any Strokes--Adams attack and whether a magnet is usefully to turn it to fix mode. A: What is the use of the magnet? C: To switch the pacemaker to non-demand mode so that when the pacemaker rhythm is disturbed by the electrocaulty, it can still function. A: What precautions to take for using diathermy? C: The ground plate to be as far as possible; use short brusit of lowest energy for haemostasis and use bipolar if possible. P: What about the warfarin? C: Stop warfarin, measure PT use Vit K and FFP to bring it to normal P: Vit K or FFP ? C: Vit K need at least one day, so I will give both Vit K and FFP. P: What is your aim of PT? C: About 1.5. P: How about the arrthymia, the patient now has a tachycardia of 140/min? C: I will check the digoxin level and the K level. The tachycardia may be due to digoxin toxicity as a result of increased level of digoxin or hypoK P: The digoxin level and K are normal the BP is normal, what will you do? C: I will give amidodarone to treat the AF.

10 Years Collection of ANZCA Part II Anaesthetic Viva

MK Yuen & SW Ku

10 Years viva collection 2009

374

P: What dose? C: 5 mg/kg and then follow by an infusion. P: What is the S/E of amidodarone? C: Hypothyroidism and hyperthyroidism

Investigation
Lung function test showing a 25 year old man having : normal FVC, FEV1, TLC decrease in DLCO to 20% predicated. He is admitted to the hospital because of SOB. E: What is the abnormality? C: The only abnormality is decrease in DLCO to 20 % of normal. It may be due to perfusion defect. E: Yes, what is the cause of the perfusion defect? C: It can be due to ARDS. E: Is the condition comparable to ARDS? C: No. E: So, it is not ARDS, what can it be? C: Decrease perfusion due to multiple emboli. E: The patient is only 25, is it likely? C: No. E: What else? C: Patients with emphysema also have decrease DLCO, but this patient has no obstructive element. E: It is not emphysema, what else? C: ............. E: OK. let's look at the blood gas ? pH 7.4 PCO2 4.1 PO2 6.5 BE -1.5 Venous admixture 25 %

10 Years Collection of ANZCA Part II Anaesthetic Viva

MK Yuen & SW Ku

10 Years viva collection 2009

375

ANZCA Medical Viva September 1994 Day 2 Medical Viva 1


60 years old woman with history of asthma since 20 years old diagnosed to have myelofibrosis 10 years ago on hydroxyurea history & P/E of Haematological system y problems of patient

Resp. problems Hepatosplenomegaly y y CXR - pul. fibrosis pattern lung function test Dx & diff Rx

Medical Viva 2
30 years old lady Eisenmenger syndrome - History & P/E of cardiovascular system. y Problems

-systolic -cyanotic heart disease y y y y CBP - Hb: l9 Echo l arge ASD & VSD pre-op problems & intra-op problems How to induce patient?

Investigation
CXR 70 years old patient for TURP

10 Years Collection of ANZCA Part II Anaesthetic Viva

MK Yuen & SW Ku

10 Years viva collection 2009

376

Dx aortic stenosis

M.V. 14L/min, FiO2 0.5 pH - 7.25 pCO2 - 50mmHg pO2 80mmHg BE -10 Na 125 K 4.5 y Dx and Diff Rx

ECG Acute anterior wall infarction y y time of MI delay op or not (elective) why?

supraventricular tachycardia y Dx and Diff. Rx

lung function test: FEV1 FVC FEV1/FVC DLCO FEV1/F1V1 <1 y Dx

10 Years Collection of ANZCA Part II Anaesthetic Viva

MK Yuen & SW Ku

10 Years viva collection 2009

377

ANZCA Medical Viva May 1995 Day 1 Medical Viva


E: Mr. B has shortness of breath on exertion for 8 year ago. He is going to have an operation one week later. Please assess him. C: I know you have difficulty in breathing 8 years ago, did you see any doctor at that time? P: yes. C: What did the doctor say? Did he say it is a heart problem or a lung problem? P: A heart problem. C: Apart from SOB on exercise, do you have chest pain, palpitation, syncopy? P: No. C: How many pillows do you need during sleep? P: Only one. C: Have you ever wake up in the middle of night while you are sleeping? P: No. C: Over these year, have you been admitted to the hospital for operation or investigation? P: I have a cardiac catheterization 6 years ago. C: What was the result of the cardiac cath? P: A leaky valve. C: Did you have rheumatic fever before? P: Yes, when I was young. C: I would like to do a physical exam on you. C: Did you have any infection in your heart valve and what drugs are you taking? Phy: This are the drugs the patient currently taking. Phy: Can you tell us your finding C: This 65 year old gentlemen had rheumatic fever in childhood, 8 years ago he experience dyspnoea on exertion, no other cardiac symptoms associated. Cardiac catheterization show a valvular problem. He did not has any history of thromboembolic events and infective endocarditis. On physical examination, this gentlemen is not tachypnea, not cyanosis, not pale, the peripheral pulse is 85 beats per min. irregularly irregular signifying atrial fibrillation. The jugular venous pressure is 2 cm raised but there is no prominent v wave. The apex beat is deviated to 2 cm lateral to the mid-clavicular line. There is no parasternal heave. A systolic thrill is palpable at the apex. I can not hear the first heart sound, the second heart sounds are normal, there is a pansystolic murmur heard over the apex with radiation to the axilla, a mid

10 Years Collection of ANZCA Part II Anaesthetic Viva

MK Yuen & SW Ku

10 Years viva collection 2009

378

diastolic murmur heard over the apex. No other murmur heard over other areas in the precordium. There is no ankle edema and no basal crep. In conclusion, this gentlemen is suffering from mitral stenosis and mitral regurgitation caused by chronic rheumatic heart disease complicated by atrial fibrillation but he is currently not in heart failure. Phy: Do you surprise that this man is not on warfarin? (drugs: digoxin, ftusemide, slow K, renetec, omeprazole) C: Yes, I expect a man with mitral valve disease and atrial fibrillation put on warfarin. Phy: do you know omeprazole. C: oh, yes, this man had gastric ulcer, this is a contraindication to anticoagulation. Phy: What premedication do you give to this patient? C: The most important drug is antibiotic cover to prevent infective endocarditis. Phy: what antibiotic? C: it depends on what operation Phy: laminectomy. C: There is no standard regime for orthopaedic surgery, but in my hospital we treat them as high risk for infective endocarditis and give 2 gm ampicillin and 1.5mg/kg gentamicin with max. dose of 80mg 1 hour before operation ivi, and repeat 8 hours later, if the patient's renal function is impaired may consider to omit the second dose of gentamicin. Phy: sure. To anaesthetist. A: How do you anaesthetise this man? C: I would optimise the patient's condition before operation. Check electrolyte, digoxin level, complete blood picture, renal and liver function test. CXR, ECG. I would crossmatch enough blood give antibiotic cover to patient, continue all the cardiac drugs on day of operation. I monitor the patient's ECG pulse oximetry, CVP, urine output. I induce the patient with thiopentone, fentanyl and vecuromum. A: After induction the heart rate run up to 140. atrial fibrillation BP 70 C: I would correct hypoxia, hypercarbia, check electrolytes, increase lVF. A: what drug do you use. C: Amiodarone 5mg/kg. A: Any other drug? C: Well, I can control the ventricular response by beta blocker., however I am reluctant to give long acting one especially the BP of this patient is a bit low, I try 10 mg esmolol. A: Do you consider cardioversion? C: No, chronic AF is resistant to cardioversion, even it can be converted to sinus rhythm., there is a risk of thromboembolism.

10 Years Collection of ANZCA Part II Anaesthetic Viva

MK Yuen & SW Ku

10 Years viva collection 2009

379

A: Why there is thromboembolism? C: The LA can be dilated and thrombus can be formed when atrium regain contractilty the thrombus will dislodge and A: Yes, you never cardiovert chronic AF? C: Elective electrical cardioversion require anticoagulant 3 weeks before the procedure and continue 4 weeks after the cardioversion. In fact an echocardiogram is required to see whether there is thrombus formation in the LA. A: You anticoagulate all or only those with thrombus. C: All.

10 Years Collection of ANZCA Part II Anaesthetic Viva

MK Yuen & SW Ku

10 Years viva collection 2009

380

ANZCA Medical Viva May 1995 Day 2 Medical Viva 1


Marfans Syndrome patient not allowed to tell you the diagnosis Assess the patient from the cardiac point of view directed to CVS and Chest examination. y y Discussion regarding the medications Investigations - CXR, ECG

Medical Viva 2
Alpha 1 anti-trypsin deficiency History given - please assess respiratory status y y y CXR - bullous emphysema, recent infection, norma cardiac size, ?Pulmonary HT ECG PFTs - obstructive pattern with no reversibility, reduced DLCO

Medical Viva 3
Cystic Fibrosis no details given

Medical Viva 4
Congenital Heart Disease patient not allowed to give diagnosis, never had cardiac surgery presentation y y causes of severe cyanotic heart disease CXR - big heart, pulmonary HT

10 Years Collection of ANZCA Part II Anaesthetic Viva

MK Yuen & SW Ku

10 Years viva collection 2009

381

Medical Viva 5
Scleroderma asked to assess the respiratory system y y y main discussion on presence of pulmonary hypertension signs and symptons, ECG, CXR then discussion on risks of anaesthesia for this patient

difficult intubation reflux and aspiration Resp. function and CVS function general condition

Investigation
Chest X-Ray Pneumothorax, Pleural Effusion, Globular Heart y y y y describe the X-Ray what possible causes? trauma, COPD, BPF, which pneumonia preop. preparation? where would you site the UWSCD? if trauma what would you look for and why?

large superior mediastinal mass y y y what are the causes? - what is the most likely? what are your pre-anaesthetic investigations? patient develops post-op respiratory distress - causes?

ABGs pH 7.14 pO295 pCO253 HCO3 17 BE -10 ventilated VE 16 I/min FiO2 0.5 T37 y y describe voicing thoughts put it together and list the possible causes

ECG rapid atrial tachycardia and inferior infarction diagnosis for WPW treatment for WPW & SVT

10 Years Collection of ANZCA Part II Anaesthetic Viva

MK Yuen & SW Ku

10 Years viva collection 2009

382

PFTs patient is 10 weeks following de-cannulation of tracheostomy flow-volume loop showing fixed extra-thoracic obstruction discussion around intrathoracic obstructions - what would it look like and why?

10 Years Collection of ANZCA Part II Anaesthetic Viva

MK Yuen & SW Ku

10 Years viva collection 2009

383

ANZCA Medical Viva September 1995 Day 1 Medical Viva 1


History: 55 year old lady with 15 year history of polyarthritis and bloody diarrhoea. Told she had lupus. Previous cervical and lumbar fusion. Easy bleeding. Medications include prednisone P/E: Skin changes of chronic steroid use. Slightly limited neck movements. Heart and chest normal y y y y reasons for medications steroid replacement therapy management of airway regional vs GA for knee replacement

Medical Viva 2
History: 65 year old lady with 2 years history of paroxysmal palpitations lasting a day or two. 4 x cardioversion. No previous history of significant heart disease. Medications include sotalol and warfarin P/E: Sinus rhythm rate 60. Mid diastolic murmur loudest at apex and loud pulmonary second sound. No heart failure. Investigation shown: 1st ECG of sinus rhythm and p mitrale 2nd ECG of atrial fibrillation y y y y y further Ix including cardiac echo reasons for medications natural progress of the disease management of periop rapid AF antibiotic prophylaxis for surgery

Investigation
ECG: recent non Q infero-Iateral myocardial infarction. y Discussed duration

10 Years Collection of ANZCA Part II Anaesthetic Viva

MK Yuen & SW Ku

10 Years viva collection 2009

384

CXR: intubated, left central line, right tension pneumothorax with deviated trachea and mediastinium. Post sternotomy wires seem. Chest drain in situ.

(the following CXR, cardiac output data interpretation and lung function test are separate questions in one candidate but a bundle of question for another)

Young man involved in major car accident: y y CXR: NGT in left chest (ruptured diaphragm), multiple rib fractures, chest drain Discussed methods of airway management. Management if suspect upper airway injury.

After few days of intensive care, patient came for operation. Shown Swan Ganz readings: CO 2, CI 0.6, SV 30, SVRI 7500, MAP 80, HR 120, PCWP 30, CVP 30. E: What is the clinical problem E: How would you mamage this patient if he were booked for emergency OT C: Optimise his cardiovascular status with inotrope, vasodilation E: What inotrope? C: dopamine cardiac dose 5 -10 mcg/kg/min, consider dobutamine. E: What is the difference between the two? C: dopamine acts as dopaminergic, beta, and alpha receptor at high dose. Dobutamine has more selective beta 1 action, more inotropic effect thant chronotropic action, less tachycardia. It can however produce vasodilation with its beta 2 effect. E: What vasodilatior? E: Nitroprusside vs nitroglycerine. Shown improved readings. HR 130, MAP 78, SVRI 1500, CVP 15, PCWP 20, CO 6 Few months later patient presented with persistent stridor. Shown flow volume loop of fixed obstruction. Repeated lung function test after resection of stenotic segment. Shown restrictive pattern, no obstructive element.

10 Years Collection of ANZCA Part II Anaesthetic Viva

MK Yuen & SW Ku

10 Years viva collection 2009

385

ANZCA Medical Viva May 1996 Day 2 Investigation


X-ray Dx: features of mitral stenosis cardiomegaly, double cardiac shadow, upper lobe diversion of the lung field. E: How to differentiate the enlargement of different heart chambers? Dx: same patient with operation done + R. pleural effusion. E: How to differentiate collapse and consolidation? flexion + extension view of cervical spine + open mouth view. C: Increase atlanto-axial distance, fracture of C3 body, separation of lateral mass of C1. Diagnosis: Jefferson's fracture.

Hx: 55 years old man with history of cardiac failure, now present with SOB. I was shown the Lab result: Liver function test + electrolyte result C: Impaired liver function + hypokalaemia. Together with history, suggestive of cardiac cirrhosis.

ECG: 12-lead ECG C: acute transmural infarction over the anterior chest leads + old inferior MI.

Lung function test: I was shown the flow volume loop FIV 50/ FEV 50 = 0.63 C: Fixed intrathoraclc obstruction of upper aIrway. E: Why fixed obstruction? E: What is the Ddx? Another Lung function test: After treatment with FIV 50/ FEV 50 C: Improved after treatment, but with residual obstruction.

10 Years Collection of ANZCA Part II Anaesthetic Viva

MK Yuen & SW Ku

10 Years viva collection 2009

386

ANZCA Medical Viva September 1997 Day 1 Medical Viva 1


Man in 50s with syncope, dyspnoea and angina chest pain on exertion and of recent onset with no past history of IHD on examination, ejection systolic murmur over left sternal edge. Shown ECG with AV conduction block most likely dignosis hypertrophic obstructive cardiomyopathy

Medical Viva 2
Man in 30s with history of congenital heart diagnosed in childhood. No previous cardiac surgery. Good exercise tolerance. No cyanosis. On examination, has collapsing pulse and systolic murmur over pulmonary area probably patient ductus arteriosis

10 Years Collection of ANZCA Part II Anaesthetic Viva

MK Yuen & SW Ku

10 Years viva collection 2009

387

ANZCA Medical Viva May 2000 Medical Viva 1


Hx given: patient with Chest pain and SOB Hx from patient: cardiac valvular lesion found on angiogram to investigate chest pain Angiogram with normal vessels and decided no benefit with CABG. Palpitations. No syncope. Exercise tolerance 1 set stairs stops due to SOB. Dx: Aortic Stenosis y y y y What Hx and Sx suggest a severe lesion? What qualities of murmur would suggest severe AS? Asked what Ix do you want? Shown CXR asked if any evidence of LA enlargement.

Medical Viva 2
Hx given: SOB & Ex Tolerance in 48yr old man Hx of being febrile given by examiners Hx from patient: cough for 7 years. Normally 3 tbsps per day. Now approx cup daily. Greenish yellow. Hx intermittent infective exacerbation. Life long non smoker. Excellent exercise tolerance still but noted can do less than usual recently. Recent travel to UK Recurrent sinusitis Sx x 2 RLL signs Dx = bronchiectasis y y y Asked what Ix you would like to see Shown CXR RLL changes Asked about potential significance of Overseas travel and Recurrent sinusitis

10 Years Collection of ANZCA Part II Anaesthetic Viva

MK Yuen & SW Ku

10 Years viva collection 2009

388

Investigation
Male in recovery post CABG with bleeding y y What Ix would you like to see? You give list then asked what abnormalities would be looking for on each test? FBC Hb and Platelets Pl Aggregation tests: dysfunctional due to CPB INR and aPTT FDPs =? DIC ACT = residual heparinisation y y y y y What is a normal ACT? When would you normally check the ACT in the OT? What would you want the ACT to be if you give heparin prior to bypass? If ACT > 150 secs, what would you do then? protamine. when to give platelets and FFP

Given ACT = 120 secs

Right pneumonectomy who is 5/7 post op Noted increased SOB and difficulty breathing through day y Ix wanted: CXR right side of chest half filled with fluid Left lung has increased vascular markings and upper lobe re-distribution Dx = fluid overload y y ECG rapid AF and inferolateral ischemia What is your management? Haemodynamically unstable amiodarone Told that give amiodarone and shown another ECG which shows SR and essentially normal. 2/7 later patient coughs up blood stained sputum plug and becomes acutely SOB. y y y y y What do you think might have happened? What is the DDx? How would you assess this patient? How would you manage this patient? ABC, ICC What Ix would you want prior to OT? Bronchopleural fistula/disruption of pneumonectomy stump.

Mx = haemodynamically unstable - DCS

10 Years Collection of ANZCA Part II Anaesthetic Viva

MK Yuen & SW Ku

10 Years viva collection 2009

389

bronchopneumonia - microorganism in HIV patients

respiratory function test - severe obstructive airway disease

draw flow volume loop for variable intrathoracic obstruction

10 Years Collection of ANZCA Part II Anaesthetic Viva

MK Yuen & SW Ku

10 Years viva collection 2009

390

ANZCA Medical Viva May 2001 Day 1 Medical Viva 1


Lung Trarnsplant

Medical Viva 2
Carcinoid syndrome

Investigation
Alcoholic patient with vomiting for 3 days ABG pH 7.42 CO2 42 O2 83 HCO3 26 y DDx of Increased Anion gap Acidosis

Imaging y y y y CXR trauma with # 1st ribs Collapsed left lung Re-expansion pulmonary oedema CT 9 years with Cerebral oedema

10 Years Collection of ANZCA Part II Anaesthetic Viva

MK Yuen & SW Ku

10 Years viva collection 2009

391

ANZCA Medical Viva May 2001 Day 2 Medical Viva 1


MI with emergency CABG done, subsequent HF not respond to medical treatment. Post-Heart transplant. (a post-operative CXR with CVP inserted into left neck and right pneumothorax was shown)

Medical Viva 2
COAD on nocturnal O2 supplement (a lung function test was shown)

Investigation
RTA victim with operation done, developed polyuria and required intubation, electrolyte normal, osmolarity not suggest DI. y What is the cause of polyiuria? fluid in about 5 litre during past 12 hours, said to be fluid overload

Imaging y y y y CXR: coarction of aorta CXR: hiatus hernia (PA + lateral) CT thorax: pneumothorax (# ribs, haemothorax, mediastinal shift by other candidate) CXR: R middle and lower lobe collapse (PA + lateral)

lung function test: obstructive lung

ECG y y y ECG: RBBB + LAD + old anterior MI ECG: atrial flutter + variable block ECG: 2:1 AV block

10 Years Collection of ANZCA Part II Anaesthetic Viva

MK Yuen & SW Ku

10 Years viva collection 2009

392

ANZCA Medical Viva May 2002 Day 1 Medical Viva 1


History provided: Carcinoid tumour for 7 years. Concentrate on GI system. y y y What is your finding? Ddx of hepatomegaly LFT with hypertriglyceride and elevated urate, ALP, GTP

Medical Viva 2
History provided: COAD with single lung transplant. Concentrate on Respiratory system. y y y What is your finding? CXR of patient (I am not able to give diagnosis to make examiner satisfied) Interpretation of blood gas (type 2 respiratory failure), would this CO2 give bounding pulse?

Investigation
ECG (Examiner: all patient was admitted for operation and asymptomatic) AF variable block E: why it is not atrial fibrillation WPW E: What is the diagnosis? E: show me the delta wave? E: would you give any drug before operation? E: what is the treatment for WPW? E: does it work? CHB + RBBB E: what is the diagnosis? E: what would you do before operation?

10 Years Collection of ANZCA Part II Anaesthetic Viva

MK Yuen & SW Ku

10 Years viva collection 2009

393

ECG showed ST elevation III, avF, V2 to V5. E: what is the diagnosis? C: inferior MI E: why inferior? C: because there is ST elevation in inferior lead? E: where is inferior lead? C: lead III and avF E: (I cannot remember what exactly he said but to me he didnt agree this diagnosis. He gave me some hint for other diagnosis) C: for the widespread involvement of ST elevation, it may be due to pericarditis ECG (didnt mention whether it is pre- or post-operation) showed definite S1Q3T3, and also poor R wave progression and q wave V1-V3 E: What is the diagnosis? C: There is S1Q3T3. This patient may have pulmonary embolism. E: but this patient is not having any symptom at all C: there is also q wave from V1 V3. He may have old anteroseptal MI. E: is there any condition that patient may not feel the chest pain? C: diabetes neuropathy

Imaging CXR: Hiatus hernia E: what is the ddx? Neonate film post-difficult intubation, showed collapsed RUL and L lung, dilated stomach with NG tube Intubated trauma patient with widen superior mediastinum C: dissection aortic rupture E: what investigation would you do? C: depends on clinical condition. If stable, CT scan. If unstable, TEE. E: what can you do in ICU, there is no TEE. C: transthoracic echo but it is not as sensitive as above. E: this patient has fracture pelvic as well, what operation you would do first? C: I believed these cannot be done at the same time. I would like to do the aortic rupture first, as it is fetal.

10 Years Collection of ANZCA Part II Anaesthetic Viva

MK Yuen & SW Ku

10 Years viva collection 2009

394

Female at her 20s was told she has a congenital murmur that is now disappeared. She is asymptomatic. CXR showed increased CT ratio, bulging R heart border. Definite no rib notching. Increased PA trunk but no pruning. BELL (I want to answer ASD before the bell rang, but some other candidates said it is VSD, but how can a VSD patient with no murmur and asymptomatic)

10 Years Collection of ANZCA Part II Anaesthetic Viva

MK Yuen & SW Ku

10 Years viva collection 2009

395

ANZCA Medical Viva 2003 May Day 2 Medical Viva 1


M/60 liver disease Take a relevant history and perform GI investigation C: I am told that you have liver disease, do you know the cause of liver problem? P: From me drinking C: ok, do you know if there is any other cause, suck as infection, hepatitis? P: No. C: Have you had any blood transfusion? P: No. C: How did you find out about your liver problem? P: I have ulcer on my leg didnt heal.(I cannot remember what he said, but he lead to somewhere he had an investigation done and show 3 liver masses and waiting for liver surgery) C: SO you are waiting for surgery for your liver nodules? P: Yes C: have you suffer from any complication from your liver problems? P: Not that I know of. C: Any thing like bleeding from the gut, easy bruising/bleeding? P: No, I was on aspirin for heart diseases since my heart attack 10 years ago, but it is stop now, I have no problem with easy bleeding. C: How much alcohol do you drink now? P: only a couple of light beer, not all the time C: OK, regarding you heart disease, do you suffer from any chest pain, SOB? P: not anymore C: Can you exercise? P: Yes I play me golf. C: Apart from golf, can you climb stair, how many fight of stair can you climb before you stop? P: C: Do you take any medication? P: I take my antihypertensive. C: Any more investigation done for your heart? P: I had angiogram done when I had my heart attack, it was ok, I have my regular follow up, no more test done since. Then proceed to examination, when I start from hands, the examiner asked me what am I

10 Years Collection of ANZCA Part II Anaesthetic Viva

MK Yuen & SW Ku

10 Years viva collection 2009

396

looking for. C: I check for clubbing, palmar erthema, flapping tremor Spider naevi present and significant numbers Palpate the abodomen, enlarged liver 5 cm below costal margin, I thought I could also feel the tip of the spleen, auscualte for shifting dullness which was mild, BS normal. I was asked what would I do if I have more time, I said BP, Then I was told BP 140/70. I was then asked what else would I like to do, I said look into the sclera for jaundice Discussion: E: Can you summarize the finding C: E: What are the causes of liver failure? C: alcohol, infection, autoimmune, biliary cirrhosis, idiopathic E: How to check severity of liver failure? C: by Childs classificationAlbumin, bil, Encephalothy, nutritional state, ascites, INR E: What else? C: Tranaminase, ALP, GGT E: What does tell you if GGT high C: It indicates alcoholic consumption E: you mention that he has enlarge spleen, what does it indicate? C: portal hypertension E: Is this patient at risk of encephalopathy E: what are other S/S of Portal hypertension? C: Esophageal varies, PR bleeding fro haemorrhoid, prominent veins on abdomen radiating form umbilicus, ascistes, coma encephalopathy.. E: What can ppt liver failure E: How do they present with liver failure E: What is the treatment

Medical Viva 2
M/62, post heart transplant, hx and P/E on cardiovascular system From history, he is a patient post transplant for ?idipathic cardiomyopathy. Post transplant functional status was good, no sign of rejection with repeated investigation incl bx, echo. Has occasional dizziness when he stands up. Was hypertensive 4 months ago, started on atenolol since. Other medication includes cyclosporine, methyl.., steroid, ACEI.

10 Years Collection of ANZCA Part II Anaesthetic Viva

MK Yuen & SW Ku

10 Years viva collection 2009

397

P/E: PR 74/min, scar over L upper chest (scar for AICD prior to transplant) plus sternotomy scar. Normal heart size, 2 HS with no murmur, chest clear. As I finished the examination and washing my hands, I was asked what else would I do as I still have 2 more minutes, I said I would check BP. I did it with sphymo and it was120/70. Discussion E: can you summarize the case? E: What are the sign of rejection? E: Do they usually have chest pain with ischaemia? E: Why do you think he is hypertensive? C: I dont know (it oculd be due to cyclosporine) E: why do you think he has dizziness on standing up? C: Arrhythmia/postural hypotensio E: (show me ECG) can you describe this ECG? C: SR 100/min, 2 P waves, no other abnormalities E: Do you think the heart rate was normal?

Investigation
CXR 1. enlarged hilar with patchy opacity both lower lobes, gave a few ddx, but didnt get the right answer, now I think could be sacrodosis 2. infant, R endobronchial intubation with RUL and L lung collapse, NGT too far in , large gastric bubble 3. kid history of couging in party, First Insp film slight hyperinflation L lung sied, ask for exp film, L lung remained hyperinflated, no F seen, inhalation L main bronchus 4. lateral c-pine of RA patient, atlantoaxial subluxation and osteophytes, unstable spine, asked how to intubate and protect spine 5. ECG: first degree heart block, LAD, bifacicular block, sinus brady, on atenolol. Proceed to OT? 6. Old infarct Q V2-3, poor R progression lat leads,

10 Years Collection of ANZCA Part II Anaesthetic Viva

MK Yuen & SW Ku

10 Years viva collection 2009

398

7. ABG: ph 7.2, pO2 89, PCO2 59, HCO3 26 8. Na 145, K 2.9, HCO3 33, on frusemide, digioxin, slow K, ?proceed to OT. Why alkalosis? What level of K 9. Post op pt: PT/INR/APTT/PLT Normal, Fibrinogen 410 (sl Increased)

10 Years Collection of ANZCA Part II Anaesthetic Viva

MK Yuen & SW Ku

10 Years viva collection 2009

399

ANZCA Medical Viva 2005 May Day 1 Medical Viva 1


65 years old with heart transplant 3 years ago because of viral myocarditis. Examine CVS system. E: Discuss your findings. Given ECG for you to see E: Why this patient in on -blocker?

Medical Viva 2
56 years old having arthritis. Assess her arthritis and airway. Patient has RA with hand, shoulder, foot, C-spine involvement. Suffered from MI and encephalitis 1 year ago. E: Describe your findings. E: How about patients functional status: self-care and disability from joints deformities E: Ask about the detail about joint deformities.

Medical Viva 3
Mrs Jennifer Garden. 56 years old. Complex heart surgery before. History and physical examination regarding cardiovascular system. History: MVR + AVR 4 years ago. Etiology: rheumatic fever at teenage. Symptomatology before surgery: AF with dizziness, heart failure, NYHA 3-4 Cardiac state after surgery: not in heart failure, NYHA 2, controlled AF. E: What will you expect her cardiac status 10 years later and why? C: continue to deteriorate. May need redo surgery. Due to preexisting myocardial dysfunction before surgery. Ongoind stherosclerosis leading to ischaemia from underlying hyperlipidaemia (medication shown she has hyperlipidaemia)

10 Years Collection of ANZCA Part II Anaesthetic Viva

MK Yuen & SW Ku

10 Years viva collection 2009

400

E: If she is going to have knee replacement surgery, do you think shes optimized? C: Yes. E: Choose one investigation that you want? C: Echo E: if echo show some mild regurgitation over both valves, are you surprised? Your response? C: not surprised, rather common, may be due to progressing cardiomegaly. Accept for surgery because regurgitation lesion well tolerated usually. E: What antibiotic prophylaxis? (forgot the rest of questions)

Medical Viva 4
Mr Smithers, 77 years old, having some fait problem. Ask history regarding walking disturbance and do LL neurological examination. Finding: Peripheral neuropathy for 20 years, requiring walking aids. Chronic heavy drinker and smoker. Physical examination consistent with peripheral neuropathy. Examiner showed his medication during discussion include sonium valproate, ischaemic heart disease and COAD mediation. E: What do you think the cause and differential diagnosis? C: Alcoholism. DDx - uraemia, DM, B12 deficiency. E: Investigation? C: nerve conduction study, EMG, blood for RFT, FBG, B12/folate, CBC. E: CBC result WBC 12.8, lymphocyte, slight monocyte. C: may be underlying hematological disease leading to peripheral neuropathy e.g. lymphoma, leukaemia. E: Peripheral blood smear result. C: lymphoproliferative disease. Further investigation by BM trephine and refer hematologist.

Investigation
CXR: 1. Normal CXR what is under/ over penetration, describe heart/ mediastinal border 2. abnormal shadow overlapping mediastinal and heart shadow

10 Years Collection of ANZCA Part II Anaesthetic Viva

MK Yuen & SW Ku

10 Years viva collection 2009

401

ECG: 1. AF + bifascicular block 2. T inversion over inferior leads 3. Acute anterolateral MI + old inferior MI E: How will your treat? C: MONA, SK if no contadindication. E: What other drugs will you consider? C: Heparin, -blocker if patient is not in heart failure. ACEI for cardiac remodeling. Data interpretation 1. Chronic respiratory acidosis with metabolic compensation + polycythaemia likely COAD 2. SIADH picture E: Differential diagnosis C: hypervolaemia, diuretics E: Causes of SIADH 3. PA catheter reading CVP , SVRI , CI normal, LVSWI E: What is differential diagnosis? C: septic shock, anaphylaxis E: How would you treat? C: fluid, inotropes 4. ACT E: what is the mechanism of ACT E: How is it being performed? E: What drug can affect ACT value?

10 Years Collection of ANZCA Part II Anaesthetic Viva

MK Yuen & SW Ku

10 Years viva collection 2009

402

ANZCA Medical Viva 2005 May Day 2 Medical Viva 1


Examine CVS system in this patient M/66, Two times CABG with pacemaker, CHF with worsening exercise tolerance. HT, hyperlipidaemia.

Medical Viva 2
This 34 years old female has connective tissue disease. Please assess the respiratory system. Scleroderma. Graft taken from left back of chest to cover bilateral leg ulcers. E: How to assess lung function if patient cant walk because of painful leg ulcer? E: What are the organs involvement of scleroderma? E: Why do scleroderma patients get chronic chest problem?

Medical Viva 3
Please examine the cardiovascular system. M/84, presented with SOB. P/E: MR murmur (deafness, difficult to take history) E: Diagnosis, severity, causes? E: management of MR? E: pathophysiology of MR?

Medical Viva 4
Please examine his GI system. RA and liver disease. Liver cirrhosis secondary to gold treatment and alcoholism. P/E: jaundice, caput medusa, spleomegaly E: What is the name of the distended vien? E: direction of flow? Why this direction? E: classify the severity of the cirrhosis

10 Years Collection of ANZCA Part II Anaesthetic Viva

MK Yuen & SW Ku

10 Years viva collection 2009

403

E: (given patients clotting result) why check PT and what is the significant? Why check CBP (pancytopenia) and what is the significant? C: Feltys syndrome: RA + splenomegaly + pancytopenia) E: what is the natural history of cirrhosis?

Investigation
X-Ray: 1. lateral CXR which is right hemidiaphragm? Which part of heart corresponds to RV, LV, RA? How to determine degree of penetration? 2. CXR (PA film) RUZ collapsed with raised right hemidiaphragm but no tracheal deviation? ECG: 1. Trifascicular block + ST depression over lateral leads 2. inferior MI with reciprocal ST depression in lateral leads 3. AF but narrow complex bradycardia 50 60/mins TEG: E: tell me difference of TEG from other coagulation profile E: (trace of TEG shown) which correspond to decreased platelet and clotting factors. E: which will the trace be if blood cant clot at all C: straight line Swan: Decreased CO, increased SVRI, increased CVP 22, PAWP 5 E: What is Diagnosis? Not PE, not RHF, not cardiac tamponade. Biochemistry: Normal Na, K, creatinine 0.15, urea 12, glucose 8, pH 7.31, PCO2 21, PO2 100 (room air) decreased HCO3 C: metabolic acidosis E: if patient developed confusion, what are the ddx? C: lactic acidosis, TCA, theophylline, aspirin E: what are the presentation of aspirin overdose? E: why is there fever if aspirin is anti-pyretic

10 Years Collection of ANZCA Part II Anaesthetic Viva

MK Yuen & SW Ku

10 Years viva collection 2009

404

ANZCA Medical Viva May 2006 Day 1 Medical Viva 1


Heart transplant patient y y y y y Endomyocardial Bx Angio Echo Complications of immunosuppressants Patient had evidence of Right heart failure likely cause & Mx

Medical Viva 2
CF Patient Shown bloods, comment Increased ALP/ normal GT causes of isolated increased ALP Increased globulins/ normal albumin causes Discussion about treatment and medications

Investigation
Biochemsitry: y y y CXR y y y ECG y y CHB Inferior & posterior ischaemia Draw on paper appearance of LUL collapse on AP & lat (including how fissures move) RLL consolidation & effusion causes of effusion Post cardiac Surgery Mixed AG acidosis + metabolic alkalosis Alcoholic LFTS - AST / YGT Normal Anion gap acidosis + respiratory compensation

10 Years Collection of ANZCA Part II Anaesthetic Viva

MK Yuen & SW Ku

10 Years viva collection 2009

405

y y

Multifocal atrial tachycardia Type A WPW

10 Years Collection of ANZCA Part II Anaesthetic Viva

MK Yuen & SW Ku

10 Years viva collection 2009

406

ANZCA Medical Viva September 2006 Day 2 Medical Viva 1


Bronchiectasis y y y y y y y What is the symptoms? What is the anaesthetic implications? What is the causes of bronchiectasis? Common organisms in chest infection? What is the treatment? Antibiotic prophylaxis? Complication of bronchiectasis?

Medical Viva 2
Pending Heart re-transplant, fluid retention, valve leak. P/E: ankle edema, elevated JVP, 2/6 PSM at apex radiate to axilla, irregularly irregular pulse. Apex displaced. Sternostomy scar. y y y y Ddx of irregularly irregular pulse? What is grade 2/6 murmur? Clinical sign of pulmonary HT? Causes of MR? Ddx of PSM?

Investigations
Biochemsitry y What is eGFR? Problem of using creatine to estimate GFR?

Pulmonary Function y Draw normal flow-vol loop, flow-vol loop in fixed laryngeal obstructive Ca, flow at maximum expiration CXR y T7 cross section: gross anatomy picture for labeling

10 Years Collection of ANZCA Part II Anaesthetic Viva

MK Yuen & SW Ku

10 Years viva collection 2009

407

CXR: widen mediastinum, angiogram showed descending thoracic aorta rupture + labeling of major vessels.

ECG y y y y complete heart block and pre-op management RBBB + left axis deviation, pre-op management Torsades de pointes due to failed pacing capture first degree heart block, long QT, pre-op management

10 Years Collection of ANZCA Part II Anaesthetic Viva

MK Yuen & SW Ku

10 Years viva collection 2009

408

ANZCA Medical Viva May 2007 Day 1 Medical Viva 1


22 years old female with increased SOB. Please examinate her CVS. (patient refuse to told you the diagnosis. P/E showed mid sternotomy scar due to previous TOF surgery and ? PS murmur, subsequent Echo showed PR) y y y y y Whats TOF? Whats the name of the shunt and how it works? What will the surgeon do in corrective surgery? Whats the anasesthetic implication of TOF Do you know the reason for increase SOB recently?

Medical Viva 2
60 years old male. Increaed SOB. Please examinate his Respiratory system. History: 10 years of SOB. Hsitory of hospital admission because of increased SOB and haemoptyesis. Chronic smoker. Work as a manual worker in steal production factory. P/E showed features of COAD and subsequent lung function test showed severe obstructive pattern. y y Whats your diagnosis How do you differentiate bronchiectasis, fibrosing alveolitis ,asbetossis and COAD?

Medical Viva 3
Patient had heart disease, please take history and physical exam for the patient Patient had HOCM p/w sudden collapse with previous Echo and coronary angiogram y y y y What murmur would you expect and why Echo finding? SAM What drug they usually take How the murmur intensity increased, (Valvulsa)

10 Years Collection of ANZCA Part II Anaesthetic Viva

MK Yuen & SW Ku

10 Years viva collection 2009

409

Medical Viva 4
Please exam the abdominal system Pateint initially presented with ascites and ankle edema, no definite cause from the history and P/E basically normal, stigmata of chronic liver disease. Echo: cirrhotic change y y What is the cause of liver cirrhosis Show some investigation: CBP: low PLT, why? LFT: liver enzyme, albumin and PT ask child grading Alpha fetal protein: why measure this parameter

Investigations
CXR y y ECG y y y y P mitrale inferior MI Mobiz type 2 ?poor R-wave progression Imaging y y y lumbar spine (lateral film): burst fracture of vertebral body If you see this film, what imaging will you do MRI show MRI showing cord compression, What Mx - for surgical decompression whats the causes of poor R-wave progression? loculated pleural effusion and causes draw RLL collapse on PA and lateral view

Lung Function Test y PFR, restrictive pattern

Others y y Whats direct and indirect Coombs test? A diagram contain a heart with supplying blood vessels: name the vessels on the diagram and what area of heart do the vessels supply?

10 Years Collection of ANZCA Part II Anaesthetic Viva

MK Yuen & SW Ku

10 Years viva collection 2009

410

ANZCA Medical Viva May 2007 Day 2 Medical Viva 1


Patient present with one year of dizzy spell and chest tightness. Cardiovascular system examination. C: Rina is a 43 year lady, previously an office executive, presented with 1 year history of dizzy spell and blackout, there is no association with posture like squatting or standing, no vertigo and not proceded with any parasympathic prodromal symptom, 24 hr holter has been done, ICD inserted ; however, dizzy spell persist. She also complain of retrosternal chest tightness, dull in origin with no relation with exercise or any radiation, coronary catheterization has been done and she was told to be normal. She denied any family history of heart disease. Her medication includes: metoprolol, frusemide, PPI, Oxycontin and atorvastatin. She is a NSND On PE: regular narrow pulse, 80/min, no anaemia or raised JVP, a mid stenotomy scar, displaced apex, metallic first heart sound, PSM grade 2/6 radiate to axilla, no basal crepitation or LL edema. In conclusion: the patient is suffering from MVP with MR and MVR done, she is suffering from arrhymia not properly treated with ICD. She is not in heart failure. E: Lets say, the patient has ESM 2 weeks before her operation, what is the ddx of ESM? E: HOCM with ICD and MVP done. E: What is HOCM? E: Genetic background E: Sign and symptoms of HOCM E: Prognosis of HOCM E: Treatment

Medical Viva 2
77 years old man with ESRF. Cardiovascular system examination. C: Mr McDonald is a 77 year old retired sailor. He has suffered from ESRF and complicated with IHD. He has stable angina which is being relieved by SL GTN. He enjoyed good exercise tolarance with more than 1 FOS. He does not have PND or orthopnoea. His HT is under well control. He is currently on EPo, Fe, Caltrate, aspirin, Imdur, Norvasc and Nicardil. He is an ex-heavy smoker for 40 years and a social drinker. PE: Regular pulse 70/min, no anaemia or cyanosis, displaced apex, no surgical scar, PSM grade 2/6 and Mid diastolic rumbling murmur, no heart failure. In conclusion: Mr Mcdonald is suffering from chronic HT and stable IHD with

10 Years Collection of ANZCA Part II Anaesthetic Viva

MK Yuen & SW Ku

10 Years viva collection 2009

411

mixed MR and MS valvular problem, good functional state with no heart failure. E: Grade severity of heart failure

Investigations
Lung function test y y y respiratory function test for bleomycin toxicity Sign and symptom of bleomycin toxicity Flow vol loop: variable extrathoracic obstruction

PA catheter y PAC pressure tracing, detail of CVP wave component

Data Interpretation y y y ECG y y y y neonate ECG: CHB ECG: prolonged QTc, what does QTc mean? ECG: AF ECG: Mobitz type II block How many types of troponin? Uses? Give a lab result of more than normal range, why is there a reference range (<0.05) for troponin, does it mean normal people have elevated troponin? Source of troponin in normal people?

Radiology y y Lateral cervical XR: Acute epiglottitis, How do you manage this patient? Draw on AP & lateral CXR: What a RUL collapse will look like? Why a lateral CXR is not good at diagnosing RUL collapse?

10 Years Collection of ANZCA Part II Anaesthetic Viva

MK Yuen & SW Ku

10 Years viva collection 2009

412

ANZCA Medical Viva April 2009 Medical Viva 1


Mr A is a 39 year old man with a long history of scleroderma. Please take a history and examine his respiratory system.... Pateint: diffuse scleroderma. Severe Reynauds with amputations. Pulmonary fibrosis. No right heart failure. Facial tightening. On warfarin for AF. Ex: severe sclerodactyly and amputations. Pulmonary fibrosis, loud P2. y y y y Do you think his respiratory disease is mild/mod/severe? What can you tell me about his functional status? Here is his ECG when he presented for ankle surgery last week How would you manage this?

Atrial Flutter at 80bpm, 3-4:1 block. He was haemodynamically stable. I said do nothing, and refer to cardiologist for long term management. Probably needs anticoagulation. y y y Why could this man have atrial flutter? How does scleroderma affect the heart? Then shown Chest Xray: cardiomegaly, bilateral interstitial changes.

I thought maybe double right heart border from pulmonary hypertension? Dilated pulmonary arteries from pulm hypertension too. y How is scleroderma relevant to anaesthesia? BELL

Medical Viva 2
Mrs M is a 74 year old lady who is 3 years post heart surgery. Please take a Hx and Ex cardiovascular system. Patient: AVR 3 years ago for unknown disease, and 2 CABG. Denied premorbid symptoms other than fatigue. Currently NYHA 2 dyspnoea. All CVS risk factors present. Ex: sternotomy, obese, soft MR murmur. y y What examinations would you like? Given ECG...

10 Years Collection of ANZCA Part II Anaesthetic Viva

MK Yuen & SW Ku

10 Years viva collection 2009

413

Left Axis, right bundle - left anterior fascicular block. y y y y y y y Given ECHO (preop)... Aortic gradient 58mmHg, Area 0.8. what does this mean? what else of interest in echo? shown post op CXR Asked about grading severity of aortic stenosis How to do it clinically? Then asked about natural history of aortic stenosis - progression and pathology of disease. Asked what rate does AS progress at - in terms of valve area if left untreated. BELL

LV function, other valves, regurg... ETT, PAC, pulmonary oedema, cardiomegaly.

10 Years Collection of ANZCA Part II Anaesthetic Viva

MK Yuen & SW Ku

10 Years viva collection 2009

414

Backward The above were all the past paper I have collected over past few years. I hope you will find this useful for examination. Please remember this collection will not be possible if all the candidates in the past did not write down their scenario, questions and answers. So, please write down the questions you have after your examination. Even one scenario may change the life of other. I would like to see whether I can add back all the delected graphs in the future. If time allowed, I would like to reorganize all medical viva I have collected. Wish the luck is with you during the examination.

10 Years Collection of ANZCA Part II Anaesthetic Viva

MK Yuen & SW Ku

Вам также может понравиться